小升初数学思维训练教程

申明敬告: 本站不保证该用户上传的文档完整性,不预览、不比对内容而直接下载产生的反悔问题本站不予受理。

文档介绍

小升初数学思维训练教程

数学思维训练教程 小升初系统总复习 目 录 目 录 3‎ 第1讲 计算(一) 速算与巧算 4‎ 第2讲 计算(二) 比较大小、估算、定义新运算 20‎ 第3讲 数字谜、数阵图、幻方 35‎ 第4讲 数论(一) 整除、奇偶性、极值问题 53‎ 第5讲 数论(二) 约数倍数、质数合数、分解质因数 66‎ 第6讲 数论(三) 带余除法、同余性质、中国剩余定理 79‎ 第7讲 几何(一) 平面图形 92‎ 第8讲 几何(二) 曲线图形 116‎ 第9讲 几何(三) 立体图形 130‎ 第10讲 典型应用题(一)和差倍、年龄、植树问题 142‎ 第11讲 典型应用题(二)鸡兔同笼、盈亏、平均数问题 151‎ 第12讲 牛吃草问题 161‎ 第13讲 行程(一) 相遇追及(多次)、电车问题 170‎ 第14讲 行程(二) 平均速度、变速度、流水、电梯 188‎ 第15讲 行程(三) 行程中的比例 201‎ 第16讲 分数与百分数 218‎ 第17讲 工程问题 229‎ 第18讲 浓度与经济问题 247‎ 第19讲 方程 257‎ 第20讲 排列组合 270‎ 第21讲 容斥原理 283‎ 第22讲 抽屉原理 297‎ 第23讲 逻辑推理 305‎ 第24讲 统筹与策略 324‎ 第1讲 计算(一) 速算与巧算 一、知识地图 速算与巧算 ‎ 整数计算 ‎ 基本公式 平方、立方公式 ‎ 数列及特殊公式 特殊方法 分数计算 ‎ 拆分与裂项 几个常用拆分分数 循环小数化分数 二、基础知识 ‎(一)整数计算 ‎1、基本公式 (1) 加法交换律:‎ (2) 加法结合律:‎ (3) 减法的性质:‎ (4) 乘法交换律:‎ (5) 乘法结合律:‎ (6) 乘法分配律:‎ (7) 除法的性质:‎ ‎2、平方、立方公式 (1) 完全平方公式:‎ (2) 平方差公式:‎ 仅做了解 (3) 完全立方公式:‎ (4) 立方和公式:‎ (5) 立方差公式:‎ ‎3、数列及特殊公式 (1) 等差数列:‎ A) 通项公式:………………为什么要“n-1”呢?‎ B) 求项数公式:………………为什么要“+1”呢?‎ C) 求和公式:………………为什么要“÷2”呢?‎ 关于这个等差数列,同学们可以联系植树问题的数量关系来看,怎么把植树问题与等差数列联系在一起呢?‎ ‎“在数轴上植树”,这可是带有一定的技术含量的……‎ 如图:‎ 请体会这里数字与“树”对应、公差与“株距间隔”对应。‎ 例如: ‎ a) ‎22这个数是“第七棵树”,要由“第一棵树”加上六个“间隔”得到,算式为: 22=4+(7-1)×3;‎ b) 如果要求这个数列从4到25,一共有多少个数,相当于把4看作第一棵树,问25是第几棵树?‎ 可以思考,从4到25一共有多少个“间隔”,‎ ‎(25-4)÷3=7,‎ 所以应该是“第8棵树”,这里注意到了为什么求项数“加1”了吧?‎ a) 求和公式的来龙去脉,同学们不可不知:‎ 法一:高斯“配对法”。‎ 例如,在计算1+2+3+…+8+9这一串数列的和时,我们可以把第一个数加上最后一个数,第二个数加上倒数第二个数,这样,一直到第四个数加上倒数第四个数,每一对数的和都是10,这里,要注意还有一个“中间数”5,,没有配上对,所以,这组数列9个数的和是10×4+5=45。‎ 法二:借来还去法。‎ 例如,还是计算1+2+3+…+8+9这一串数列吧,如果我再“借”来一串“9+8+7+…+3+2+‎1”‎,‎ ‎ 这么一串数只是把原来的数列颠倒一下顺序,可以知道两串数是相等的。所以,如果我把这两串数的和求出来,是一定要“除以‎2”‎的!‎ 问题在于,本来要求一串数的和,干嘛我还扯上了另一串,这样做好算吗?答案正在这个地方,就是因为再有这么一串倒过来的数,好算不得了——“变异为同”了!‎ 如图:‎ ‎ ‎ ‎ 所以,可以得出,10×9÷2=45‎ ‎ 回头再看,这里的10可以用(1+9)为代表,则得:‎ ‎ (1+9)×9÷2=45‎ ‎ 再推广开去,对于其他等差数列,都有这么一个公式:‎ ‎ 和=(首项+末项)×项数÷2‎ (2) 等比数列:‎ (3) a) b) (2) (3) ‎(n≤9) ‎ (4) (5) 这一类的数不妨称之为“重码数”,关键于把一个循环节的“个位”的“1”作为记数单位,结合位值原则,我们可以得到上述结果。‎ ‎4、特殊方法 (1) 凑整法:利用运算公式和运算律(如交换律、结合律、分配律)将一些数凑成整一或整十整百再计算。‎ (2) 换元法:将一些数或一个式子记为某个字母,如a,b,c…… 达到化繁为简的目的。‎ ‎(二)分数计算 ‎1、拆分与裂项 (1) (2) ‎ ‎ (3) (4) ‎ ‎ ‎2、几个常用拆分分数 ‎ ‎ ‎ ‎ ‎ ‎ ‎ ‎ ‎ ‎ ‎ … …‎ ‎3、循环小数化分数 ‎ ‎ ‎ ‎ 请聪明的你,来比较1与0.99999999……的大小?‎ 你可能已经知道:0.9999999……=1‎ 也就是:=1,可是这是为什么呢?‎ 铺垫: ‎ ‎ == ‎ ‎ ==‎ ‎ ==‎ ‎ == ==‎ ‎ ‎ 以此题为例推导:‎ 设 为A,那么‎100A=‎ ‎10000A‎=‎ 所以:‎10000A-100A=1234-12‎ ‎9900A‎=1234-12‎ 注意:循环小数化分数,分母中9的个数与其循环节的位数对应,0的个数与小数点后不循环的位数对应。分子是不循环部分连上第一个循环节组成的多位数与不循环部分组成的多位数相减所得到的差。‎ 三:经典透析 ‎【例1】:(☆☆☆) ‎ ‎ 审题要点:‎ 1) 看题目中的数,聪明的你是否发现了什么秘密?‎ 对了,每一个数都有一个小秘密:‎ ‎ ‎ ‎ ‎ ‎ … ‎ 2) 发现了秘密就赶紧动手吧!‎ 详解过程:‎ ‎ ‎ 专家点评:‎ ‎<恍然大悟>‎ 原来平方差公式还可以这么用!‎ ‎ 这道题目不是很难,关键是要学会“凑整”的思路!‎ ‎【例2】(☆☆☆) ‎ ‎ 审题要点:‎ 1) 好大的数啊!别怕,肯定有绝招。‎ 2) 哈哈,终于发现了数之间的小秘密。 ‎ ‎ 详解过程:‎ 专家点评:‎ ‎ 做这道题目,你会发现,奥数的很多题目,不仅仅是记公式就能解决的,很多时候需要你对公式进行消化吸收,达到灵活应用才能在用时得心应手。‎ ‎【例3】(☆☆☆☆) ‎ ‎<友情提示>‎ ①;‎ ②凑整;‎ ③提取公因数;‎ ‎④“借来还去”思想。‎ 审题要点:‎ 1) 这题看着很熟悉→联想平方求和公式 2) 可是起始的数不是?‎ 没关系,缺什么补什么!‎ 详解过程:‎ ‎<友情提示>‎ ①提取公因数的两大特征:‎ ‎ 一是要有“公因数”,“疑似”公因数也不错,我们可以借助下面两招对它加工。‎ ‎ 二是要有互补数。‎ ②‎ ③‎ 专家点评:‎ 很多题目不能就题论题,你必须要在熟练应用公式的前提下,做适当的变换,这道题目就是一个很好的例子。‎ ‎【例4】(☆☆☆☆) ‎ 审题要点:‎ ‎1)“‎73”‎好像是关键。‎ ‎2)如果可以提取73,那不是很简单?‎ 试试吧!‎ 详解过程:‎ ‎ ‎ 专家点评:此处利用了分拆法,将730分拆为73×10,153.3分拆为73×2.1,目的都是为了构造出“公因数”73。此种构造方法很常用,你学会了吗?‎ ‎【例5】(☆☆☆☆)‎ ‎ ‎ 审题要点:‎ 1) 分母很特别哦:‎ 2) ‎ ‎ 3) 详解过程:‎ 原式=‎ ‎=‎ ‎=‎ ‎=‎ 专家点评:‎ 这道题目稍微有点难度,需要先归纳分母的通项,然后利用裂项进行解题,所以同学们应该在记住公式的同时做适当的综合应用。‎ ‎【例6】(☆☆☆☆)‎ 审题要点:‎ 1) 分数相加,分子不相等,似乎不能裂项;‎ 2) 如果做一下变换呢?‎ ‎ … ‎ 试试吧。‎ 详解过程:‎ 原式=‎ ‎=‎ ‎=‎ ‎=‎ ‎=‎ ‎=‎ 专家点评:‎ 这道题目的解题关键在于对裂项的熟练应用。题目本身并不是很难,但是需要同学认真仔细。‎ ‎【例7】(☆☆☆☆)= ‎ 审题要点:‎ ‎ 1)既然题目这样出了,说明绝大部分项能够裂项约掉!试验可知:‎ ‎,(这两个利用辗转相除法),能够约掉37,看来确实可以裂项。‎ 详解过程:‎ 观察到5,37,101以及约去的最大公约数17和65都是偶数的平方+1,所以立刻猜测最后的约分后等于,原式等于。‎ 专家点评:这是一道比较难的计算题,很多人认为只有到了初中,学了因式分解才有可能做出来。但是,小学生如果能够有“找规律”的思维,也是完全可以得出答案的。本题解题的关键在于“试算观察法”与“辗转相除法”的综合运用,你学会了么?‎ ‎【例8】(☆☆☆☆)‎ ‎ ‎ 审题要点:‎ ‎ 1)看到这么庞大的算式,应该想到要换元;‎ ‎ 2)换元时注意要整个括号作为一个整体代换;‎ ‎ 3)不妨设 ‎ ‎ 详解过程:‎ ‎ 原式 ‎ =‎ ‎ =‎ ‎ =‎ ‎ =9‎ 专家点评:‎ ‎ “换元”法在庞大的数学计算中经常用到,数学题目很少是需要你对一个复杂的式子进行每一步的计算,一般都有简便算法,这些需要你平时多积累。利用换元法解题时有两种可能性:一,换元的未知数最后都消去,可直接得出答案。二,换元的未知数不能完全消去,那么就应该将原数或原式重新代入计算,此时的代入计算将很简单,如本题中最后(a-b)须换回原来式子计算得。‎ ‎【例9】(☆☆☆☆) ‎ 审题要点:‎ ‎1):有循环小数的计算,首先要进行分数转换。‎ ‎2):每个数都是混循环小数,应该怎样化成分数?‎ 详解过程:‎ ‎ 原式=+++…+‎ ‎ =++…+‎ ‎ =‎ ‎ =‎ ‎ =‎ 专家点评:‎ ‎ 循环小数化分数,你学会了么?这是个很重要的知识,在比较大小和计算过程中经常用到。‎ ‎ 另外,如果对循环小数的性质很熟悉的话,知道=1,则可观察到:‎ ‎ ‎ ‎ ‎ ‎ 还有一个,所以总和为 ‎ ‎ ‎ 。‎ ‎ 经验证,。‎ 四、拓展训练 1. ‎=‎ ‎[初级点拨] 这道题目不难,关键是考察对公式的应用 ‎;‎ ‎[深度提示] 注意哦,分母中1与3,3与5都是要差2,所以在裂项时,括号外面要乘以;‎ ‎[全解过程] 原式=‎ ‎=‎ ‎=‎ ‎=‎ 1. ‎(-+…-+)×(1-+-+-…+)‎ ‎ -(1-+-+-…+-)×(-+…-)=_______。‎ ‎[初级点拨] 这么庞大的式子,换元毫无疑问,但是要找好,到底换什么哦。‎ ‎[深度提示] 换元时,可以设,;‎ ‎[全解过程] 设,‎ ‎ 原式= ‎ ‎=++‎ ‎=‎ ‎=‎ 2. ‎=________。‎ ‎[初级点拨] 类似于例题2;‎ ‎[深度提示] 运用平方差公式,你会了么?‎ ‎[全解过程] 原式 3. ‎=________。‎ ‎[初级点拨] 这题比较简单,利用;‎ ‎[深度提示] 这道题目很简单,主要就是公式应用的问题;‎ ‎[全解过程] 原式= 2×(-+-+…+-)‎ ‎ = ‎ ‎=‎ 1. ‎=________。‎ ‎[初级点拨] 直接利用公式;‎ ‎[深度提示] 公式的直接应用,但是要注意,分母拆开后,差值是3;‎ ‎[全解过程] 原式=×(1-+-+…+-)‎ ‎=‎ ‎=‎ 2. ‎=________。‎ ‎[初级点拨] 带分数在计算过程中,通常有两种处理办法,或者化成整数和分数的和,或者化成假分数,聪明的同学,想想这道题应该怎么处理呢?‎ ‎[深度提示] 拆成你熟悉的形式:‎ ‎…‎ ‎[全解过程] 原式=(1+2+3+…+6)+(++…+)‎ ‎=‎ ‎=‎ ‎=‎ 1. ‎=________。‎ ‎[初级点拨] 类似于上面的第6题,但是要稍微难点,关键也是对带分数的处理,不要犹豫,你想的没错,写出来试试。‎ ‎[深度提示] 将带分数拆成整数和分数的和;‎ ‎[全解过程] 原式=(3-)-(3+)+(5-)+(5+)-(7-)+(7+)-‎ ‎(9-)+(9+)-(1+)‎ ‎ =‎ ‎=‎ ‎=‎ ‎=‎ 2. ‎=________。‎ ‎[初级点拨] 这道题目是典型的利用公式解题,所以公式一定要熟记哦。‎ ‎[深度提示] 记住两个公式即可,翻看前面的基础知识,你需要的都在那里;‎ ‎[全解过程] 原式=‎ ‎=‎ 1. ‎=________。‎ ‎[初级点拨] 提取公因数,但是要先做下变换,看看,怎么变动一下!‎ ‎[深度提示] 1)找题目中的特殊之处。‎ ‎2)如果分母中也是那多好啊!‎ ‎3)变变嘛!‎ ‎[全解过程] 原式==1.‎ 2. ‎=________。‎ ‎[初级点拨] 第一步肯定是要去括号,聪明的你想到了吗?‎ ‎[深度提示] 去括号,提取公因式,两项结合;‎ ‎[全解过程] 原式=‎ ‎=‎ ‎=‎ ‎=‎ 3. ‎________。‎ ‎[初级点拨] 如果在计算中出现循环小数,那毫无疑问要先化为分数。‎ ‎[深度提示] 循环小数化分数的计算。‎ ‎[全解过程] 原式=‎ ‎=‎ ‎=‎ ‎=‎ 第2讲 计算(二) 比较大小、估算、定义新运算 一:知识地图:‎ 比较大小 ‎ 分数的大小比较 ‎ 通分 ‎ 比倒数 ‎ 与1相减比较法 ‎ 经典结论 ‎ 放缩法 ‎ 化成小数比较 ‎ 两个数相除进行比较 ‎ 对于分数的分子分母同时加上 或减去相同的数和原分数进行比较 ‎ 小数的大小比较 ‎ 估算 ‎ 常用方法 ‎ 经典步骤 ‎ 定义新运算 二:基础知识 ‎(一):比较大小 ‎1、分数的大小比较 ‎1)通分:a) 通分母:化成分母相同的分数比较,分子小的分数小;‎ ‎ b) 通分子:化成分子相同的分数比较,分母小的分数大。‎ ‎2)比倒数:倒数大的分数小。‎ ‎3)与1相减比较法:a) 真分数:与1相减,差大的分数小;‎ ‎ b) 假分数:与1相减,差大的分数大。‎ ‎4)经典结论:a) 对于两个真分数,如果分子分母相差相同的数,则分子分母都大的分数比较大;‎ b) 对于两个假分数,如果分子和分母相差相同的数,则分子分母都小的分数比较大。‎ ‎ 对于分数的分子分母同时加上或减去相同的数和原分数进行比较:‎ ‎(,且为非零自然数时)‎ ‎(1)‎ 即“真分数越加越大,越减越小”()如;‎ ‎(2)即“假分数越加越小,越减越大”。‎ ‎5)放缩法。‎ ‎6)化成小数比较:小数比较大小的关键是小数点对齐,从高位比起。切记!‎ ‎7)两个数相除进行比较。如:和,,所以。‎ ‎2、小数的大小比较 常用方法:将小数排成一个竖列,并在它们的末尾添上适当的“0”,使它们都变成小数位数相同的小数,然后比较。‎ ‎(二)估算问题 ‎1、常用方法 ‎1) 放缩法:为求出某数的整数部分,设法放大或缩小,将结果确定在两个接近数之间,从而估算出结果。‎ ‎2)变换结构:将算式变形为便于估算的形式。‎ ‎2、经典步骤 ‎ 估算和式整数部分:a) 令和式结果等于A;‎ ‎ b) 最小的数×个数<A<最大的数×个数;‎ ‎ c) 求A。‎ 对于较简单的题目,使用“最小的数×个数<A<最大的数×个数”就可以确定整数部分。对于较复杂的题目,这会造成放缩幅度过大。如果出现此情况,设法比较原式与(最小的数+最大的数)×个数÷2的大小,以及与(中位数×个数)的大小(总共有偶数个数的时候,“中位数”视为中间两个数的平均数)。‎ ‎(三)定义新运算 这是近年来出现的一种新题型,解题的过程可以归结为经典三步:阅读→理解→应用。‎ 三:经典透析 ‎【例1】(☆☆☆☆)如果,,那么,中较大的数是_________。‎ 审题要点: 1°通分似乎太麻烦了,怎么办呢? ‎ ‎2°大小比较有6个经典方法,这个似乎与“4 经典结论”相仿哦!‎ ‎3°发现了吗?两个分数的分子分母的差值都相等:, ‎ 详解过程:因为a与b都是真分数,且分母与分子差相同,并且<,所以a<b。‎ 专家点评:分数比较问题,方法很多,做题时要注意从不同角度考虑。‎ ‎ 下面再介绍两种解题方法:‎ 方法二:‎ ‎1°a与b都很接近“‎1”‎;‎ ‎2°聪明的你,发现秘密了吧!‎ ‎1-a= , 1-b=。‎ ‎> 且a和b为真分数,‎ ‎ 所以a<b。‎ 方法(三):比倒数。‎ ‎==1, ==1,‎ ‎> 所以a<b。‎ 方法(四):两个数相除进行比较。‎ ‎,‎ 所以:。‎ 同学们开动脑筋看还有没有更多的思路!‎ ‎【例2】(☆☆☆)如果,A与B中哪个数较大?‎ 审题要点:1°快开动脑筋看这个题目适合用哪个方法?‎ ‎2°不妨先试试比倒数。‎ 详解过程:,‎ 专家点评: 同样是分数比大小的问题,你能做出几种解答过程呢?下面给出另外两个解题过程。‎ ‎ 解法二:‎ ‎1°这么庞大的式子,如果能“4经典结论”该多棒啊!‎ ‎2°开动脑筋做个小变换,,‎ 变换后A与B分子分母相差相同的数“3”,‎ A与B都为真分数且A的分子分母都较小,‎ ‎ 所以A‎ 这类题目解题方法:‎ ‎1)‎ ‎2)‎ ‎3)通过适当相加组合为A 1°这类题目关键是对公式的灵 ‎ 活应用 详解过程:‎ 因为 所以 即 则 专家点评:这道题目比较简单,聪明的你做出来了吧!下面再介绍另外一个解法希望对同学们日后解题有帮助。‎ 利用公式:当 那么:‎ ‎ ‎ 可以得到 所以 ‎【例4】(☆☆☆☆)已知除法算式12345678910111213÷‎ ‎31211101987654321,它的计算结果的小数点后的前三位数字分别是________。‎ ‎ ‎ 审题要点: 1°毫无疑问,这是一道估算题。‎ 详解过程:‎ 1) 取除数的前两位:‎ ‎12.345…÷32<原式<12.345…÷31‎ ‎0.385…<原式<0.398…‎ ‎ 在0.385到0.398之间无法确定小数点后三位的准确值,说明放缩范围太大。‎ 2) 再取除数的前三位:‎ ‎123.456…÷313<原式<123.456…÷312‎ ‎0.394…<原式<0.395…‎ 仍无法确定。‎ 3) 取除数的前四位。‎ ‎1234.567…÷3122<原式<1234.567…÷3121‎ ‎0.3954…<原式<0.3955…‎ 所以小数点后三位分别为3,9,5。‎ 专家点评:这道题目稍微有点难,但是只要是估算的问题,都不需要算出最终结果,通过这道题目,你有没有学到什么呢?‎ ‎ 有些书上解这道题的时候是把被除数和除数都放缩:12÷32<原式<13÷31,123÷313<原式<124÷312,1234÷3122<原式<1235÷3121。‎ 但是,对被除数的放缩只会徒增加放缩幅度,而不会简化计算。如果你认为,计算1234÷3122比计算1234.567…÷3122省事,那你一定是在用计算器!‎ ‎【例5】(☆☆☆☆)老师在黑板上写了7个自然数,让小明计算它们的平均数(保留小数点后面两位),小明算出的答数是14.73,老师说:“除最后一位数字外其他都对了”,那么正确的得数应是 。‎ 审题要点:1°估算题目最直接的方法就是放缩。‎ ‎2°这道题的关键点是,只有最后一位是错的,那么分数的取值范围是 14.70~14.79‎ 详解过程:设这7个自然数之和为A(A为整数) ‎ 平均数为 专家点评:关于平均数的估算,是经常会出的题目,所以同学们一定要学会如何去解答这类题目,下面再介绍另外一种解题方法。‎ ‎14.73中前三位14.7是 正确的 ‎ 所以总数肯定大于 14.7‎ 则总数只能是103、104…‎ ‎ 当和等于103时,平均数是14.71‎ ‎ 当和等于104时,平均数是14.86(不符合)‎ 所以七个自然数的和为103,平均分是14.71‎ 事实上一个自然数被7除如果除不尽,那么所得的商的小数部分一定按照“1、4、2、8、5、7”的顺序不断循环,只是循环初始数字不一定相同。观察一下除式:‎ 所以如果对被7除商数特征熟悉的话,一定能马上反应过来这个平均数是。‎ ‎【例6】(☆☆☆☆),哪个更大,为什么?‎ 审题要点:1°这道题比较难,式子比较庞大,我们不妨引入换元的思想。‎ 设a ‎ ‎2°设 详解过程:‎ 很明显 所以 ‎ ‎ ‎ 专家点评:这道题目,可以说是估算里面比较难的题目,需要自己构建新的算式,同学们学会这个解题思路了吗?‎ ‎【例7】(☆☆☆)数 审题要点:1°这道题的难点集中在分母上 ‎2°不妨把分母单独拿出来,设A=‎ ‎3°看明白了吧 详解过程:‎ 原式=‎ ‎ ‎ 所以 即1<原式<1.9‎ 所以数。‎ ‎ 【例8】(☆☆☆)(第二届小学"希望杯"全国数学邀请赛) ‎ 如果,那么 ‎ 。‎ 审题要点:‎ 定义新运算,不妨试试经典三步法:阅读→理解→应用!‎ 详解过程:‎ 原式=‎ ‎ =‎ ‎ =‎ 专家点评:这是一道综合性题目,首先要看明白定义的新运算,其次要学会用裂项法解题,总的来说,题目不难,关键是要认真仔细。‎ ‎ 【例9】(☆☆☆☆)两个用同样材料做成的球A和B,一个实心,一个空心,A的直径为7,重量为22,B的直径为10.6,重量为33.3。问哪一个球是实心球?‎ 审题要点:显然,两个球中单位体积重量大的球是实心球。所以第一步首先要求两个球的体积各是多少!‎ 详解过程:由球的体积公式可知,球的体积与半径的3次方成正比;显然球的体积与直径的3次方成正比。‎ ‎ 即 V1:V2 = 73:10.63‎ 因为,即可知:‎ 所以 ‎ 即A球为实心球 专家点评:比较大小,估算以及定义新运算,都不是只局限于现有的几个数的计算,很多时候是将知识点作了综合,因此同学们在做题过程中,一定要在读懂题的前提下再动笔开始做!‎ 四、拓展训练 1. 在___.‎ ‎[初级点拨] 1)观察题目,找数之间的相似处 ‎ 2)你是否发现了这个小秘密?‎ ‎ ‎ ‎[深度提示] 1)发现了吗?‎ ‎2)因为 ‎ ‎ ‎[全解过程] ‎ 显然,所以题中最小分数为.‎ 1. ‎,求A的整数部分 .‎ ‎[初级提示] 这道题隐藏的知识点是估算,你发现了么?‎ ‎[深度提示] “经典三步法”看行不行?‎ ‎[全解过程] ,适当扩展:,,所以A的整数部分为44。(同学们想想,还有没有别的方法!)‎ 2. 已知:.‎ ‎[初级点拨] 问题的关键在分母,同于“例‎7”‎(但是要小心,有一点小区别哦)‎ ‎[深度提示] 这道题目中,分母按照我们给出的估算方法,似乎不能做到最后,那怎么办呢?‎ ‎ ‎ ‎ ‎ ‎ ‎ ‎ 发现规律了么?‎ ‎[全解过程] 设 ‎ ‎ ‎ ‎ ‎ ‎ ‎ 即 ‎ 不能确定A的整数部分,怎么办?先看看一个例子 ‎ ‎ ‎ ‎ 则 聪明的你从中会发现一个找“最小界线的新规律”,那么让我们回到原题来看看吧!‎ ‎ 即 ‎ ‎ ‎∴A的整数部分为73。‎ 1. 有8个数,如果按从小到大的顺序排列时,第四个数是 ‎[初级点拨] 循环小数化分数。‎ ‎[深度提示] 1)分数小数的大小比较首先要统一形式 ‎ 2)两个未写出的数要确定它们的位置 ‎[全解过程] ‎ 显然 即 因为8个数由小到大排第4个是所以从大到小第4 个是 2. ‎1)试比较。‎ ‎2)如果A=.‎ ‎[初级点拨] 这两道题目可以参考例1和例2‎ ‎[深度提示] 对于两个真分数,如果分子分母相差相同的数,则分子分母都大的分数比较大 ‎[全解过程] 1)‎ ‎ 2)A‎ ‎1)倒数法 ‎2)与1相减法 ‎3)分子分母差值相同时,分子分母大的分数大.‎ ‎ 1.12121=1.121210000‎ ‎ 1.12=1.120000000‎ ‎ 显然 ‎ (2)‎ 1. 如果用max max.‎ ‎[初级点拨] 1)这是一道典型的定义新运算的问题,经典三大步找出来 ‎2)阅读---max,理解---找最大的数,应用吧!‎ ‎[深度提示] max的作用是找较大的数,所以首先要比较两个分数的大小 ‎[全解过程] ,所以max.‎ 2. 假设有一种计算器,它由A,B,C,D四种装置组成。将一个数输入一种装置后会自动输出另一个数,各装置的运算程序如下:‎ ‎ 装置A:将输入的数加上6之后输出;‎ ‎ 装置B:将输入的数除以2后输出;‎ ‎ 装置C:将输入的数减去5之后输出;‎ ‎ 装置D:将输入的数乘以3后输出;‎ 这些装置可以互相连接,如在装置A后接装置B就记做:A→B。例如输入1后,经过A→B输出3.5‎ ‎1)若经过A→B→C→D,输出120。则输入的数是多少?‎ ‎2)若经过B→D→A→C,输出13,则输入的数是多少?‎ ‎[初级点拨] 1)好复杂的过程啊,但是聪明的同学肯定不会被吓倒 ‎2)对啦,利用经典三步分析后,你会发现,这并不是一道难题 ‎3)阅读→A,B,C,D四个装置是关键 理解→明白这四个装置的运算特点 应用→逆向思维 ‎[深度提示] 装置A:将输入的数加上6;‎ ‎ 装置B:将输入的数除以2;‎ ‎ 装置C:将输入的数减去5;‎ ‎ 装置D:将输入的数乘以3;‎ ‎ 确定每个装置的作用后,从后往前计算结果。‎ ‎[全解过程] 1)经A→B→C→D后输出120‎ 按逆向思维后推 ‎ 设最先输入的数为经过A后变为,经过B后变为,经过C后变为,如上图所示 即 ‎ ‎2)类似于1)的解答,聪明的你试试吧。输入的数是8。‎ 1. 有一个算式,左边方框里都是整数,右边答案写出了四舍五入后的近似值:‎ ‎ ‎ 求左边方框里的整数从左至右分别是什么?‎ ‎[初级点拨] 1)这道题目,入手比较难。开动你的小脑筋看能不能发现什么小秘密?‎ ‎ 2)近似值→估算?‎ ‎[深度提示] 看似一道数字谜问题,其实也是估算的问题,既然结果约等于1.16那么原式可以做一个变换(看明白了么)‎ ‎[全解过程] 将原式做一下变化,,三个分数不妨先通分,‎ 即,将式子扩大105倍得 设A= ;A为整数(想想为什么?)‎ ‎ 121.275,A=122‎ 即 通过试验知 所以左边方框里的值依次是1,2,3。‎ 1. 用表示不超过a的最大整数。例如=0.3; ,记请计算的值.‎ ‎[初级点拨] 1)这是一道综合题目,同学们千万不要被多变的符号给吓倒!‎ ‎2)“三步经典”来分析 ‎[深度提示] 阅读→题中的符号有﹛﹜;[ ]; ,‎ 理解→﹛﹜,〔 〕;的意义及表示方法应用吧!‎ ‎[全解过程] ====1.4‎ ‎==1‎ ‎﹛﹜=﹛1.4﹜=0.4‎ ‎[]=[1.4]=1 ‎ ‎﹛﹜=﹛1﹜=0 []=[1]=1 ‎ 第3讲 数字谜、数阵图、幻方 一,知识地图 ‎ 二,基础知识 趣题导引:‎ ‎ 学而思教育的数学兴趣小组每周都要进行小组讨论。有一次小组讨论时,李同学在黑板上画了一个“九宫格”,问其他同学说:“你们能看出这个表格的的数字规律吗?”这时很多同学都说:“这还不简单啊,这是幻方,每行每列和两条对角线的数字和都相等,我自己也会填。”李同学又画了一个幻方,但是里面数字不全,只有三个数字,说:“那你们能把这个表格补充完整,使它成为一个幻方吗?”‎ 这时刚才非常活跃的同学都沉默了,同学们,你们可以补充完整吗?‎ ‎ ‎ ‎ ‎ ‎(一)数字谜 1、 数字谜介绍 数字谜从形式上可以分为横式数字谜与竖式数字谜,从内容上可以分为加减乘除四种数字谜。横式数字谜一般可以转化为竖式数字谜,所以我们这里主要讨论竖式数字谜的一般解题技巧与思路。‎ ‎2、数字谜常用的分析法介绍 解决数字迷问题最重要的就是找到突破口,突破口的寻找是需要一定的技巧,一般来说首先是观察题目中给出数字位置,同时找出涉及这些已知数字的所有相关计算,然后根据各种分析法进行突破,一般的突破顺序是,三位分析(个位分析,高位分析,进位借位分析),另外加入三大技巧(估算技巧(结合数位),奇偶分析技巧,分解质因数技巧)等。而且一般应该先从涉及乘法的地方入手,然后再考察加法或减法的分析(并不完全是这样)。‎ ‎(1)个位数字分析法(加法个位数规律,减法个位数规律,乘法个位数规律)‎ 若题目已知条件里面给出的主要是个位数字,那么我们就要考虑使用个位分析法,这是 大部分数字谜都要用的分析方法。‎ 加减法的个位数字规律比较简单,这里要求重点掌握乘法个位数的规律。‎ A)加法个位数规律举例:‎ 如右图:由a+8的结果个位数为5可推出a=7,十位进位,9+1+b结果个位为7,‎ 可推出b=7,进而推出c=1。‎ B)减法个位数规律举例:‎ 如右图:由a-7的结果个位为9,可推出a=6,且借一位,进而十位数 中9-1-b结果个位数为4,可推出b=4。‎ 注意:当个位数已经推导出来,那么十位数的推理也可以继续使用个位分析法进行推理,后面依次类推,高位使用个位数字分析法时必须同时考虑进位或借位的情况。‎ C)乘法个位数规律归纳:‎ ‎1、当结果为奇数,其中一个乘数也为奇数时,则另一个乘数也为奇数,‎ 且只有一种答案(注意:数字5除外,5和0的规律比较特殊,后面补充)。 ‎ 如右图:由b×7结果个位为1,可推知b=3,利用后面介绍的高位分析法可 继续推出a=1,c=9。‎ ‎2、当结果为偶数,其中一个乘数为奇数时,则另一个乘数为偶数,且只 有一种答案。‎ 如右图:由b×9结果个位数为8,可推知b=2,由后面介绍的高位分析法 可继续推知a=4,c=7。‎ ‎3、当结果为偶数,其中一个乘数也为偶数时,则另一个乘数有两种可能 性,一奇一偶,且相差5。‎ 如右图:由b×6结果个位数为4,可推知b=4或9,当b=4时,进而推出 a=8或9,相对应c=0或6。‎ 当b=9时,进而推出a=8或9,相对应c=3或9。共有四种可能性,再根据 其他条件进行排除。‎ ‎4、当结果为奇数,其中一个乘数为偶数时,另一个乘数无解,因为根据 奇偶性,偶数乘于任何数都不可能等于奇数。‎ 如右图:由b×8结果为7可推知此题无解。‎ ‎5、当结果为5,则其中一个乘数必须为5,另一个为奇数。‎ 当结果为0,则其中一个乘数为5,另一个为偶数,或者一个乘数为0即可。‎ 当一个乘数为5,则结果为0或5,另一个乘数为偶数时,结果为0;另一个乘数为奇数时,结果为5。‎ ‎(2) 高位分析法(主要在乘法中运用)‎ 如右图:由a×7结果为四十几,结合进位考虑,可知a=5,6或7,再根 据其他条件进行排除。‎ ‎(3) 数字估算分析法(最大值与最小值的考量,经常要 结合数位考虑)‎ 如右图:由×4=A,A为三位数,可推知≥25,由 ‎×3=B,B 为二位数,可推知≤33,由×34=C,C为三位数,可推 知≤29,综合考虑可知b=2,a=5,6,7,8或9。再根据其他条件排除。‎ ‎(4) 加减乘法中的进位与借位分析 前面三种分析法都涉及到了进位与借位,这里再次强调进位与借位的重要性,千万不要忽视了(这是同学们非常容易出错的地方。)‎ 加法进位中,加数有n个,则最多向前进n-1,乘法进位中,乘数是n,则最多 向前进n-1。‎ 如右图:由百位可推知十位向百位进2,而个位最多向十位进2,则推知a至 少为9,即就是9,进而继续推知b也只能是9,而c=d=0。‎ 如右图:由a×8等于四十几而个位最多向前进7,得知a只能为5或6,而不能是4。‎ 注意:此处也可以利用数字估算分析法得出50≤≤62,同学们知道为什么吗?‎ ‎(5) 分解质因数分析法 当乘法数字谜中一个积全部已知或者只有一个数字未知而又没有其他办法判断时,‎ 可考虑使用分解质因数。‎ 由×c=206可将206分解质因数,206=2×2×2×2×13,根据两个乘数分别 是一位数和两位数可推知两种可能性:52×4与26×8,又根据×3为两位数 可确定=26。‎ ‎(6) 奇偶性分析(加减乘法)‎ 数字谜中经常可以直接利用奇偶性进行排除选项。(见乘法个位规律里面的第四点。)‎ 复杂数字谜中不能直接确定某一数字时,经常需要使用假设法进行逐一排除,排除的判断一般是通过另外一个数字或者题目中其他条件来进行。如带汉字的数字谜经常需要符合的条件是“相同汉字代表相同数字,不同汉字代表不同数字”等。‎ ‎(二)数阵图的一般解题思路与步骤 数阵图中的数字关系一般比较复杂,同学们注意一定不要一开始就使用尝试的方法,而应该使用一定的技巧求出某些特殊位置的数字后再逐一分组尝试。‎ 由于数阵图中没有填充之前各个数字的位置无法确定,所以从每一个单个数字上无法进行判断,所以我们采用的是整体与个体相结合考虑的方法,即利用所有相关数字和全部相加法进行分析。‎ 一般分为以下几个步骤:(注意:建议先学习完几个相关例题再回来进行总结)‎ ‎1、从整体考虑,将要求满足相等的几个数字和全部相加,一般为n×S的形式。‎ ‎ 2、从个体考虑,分别计算每一个位置数字相加的次数,将比较特殊的(多加或少加几次)位置数字用未知数表示,全部相加,一般为(题目所给全部数字和)×一般位置数字相加次数±(特殊位置数字和)×多加或少加次数 的形式。‎ ‎ 3、根据整体与个体的关系,列出等式即:n×S=(题目所给全部数字和)×一般位置数字相加次数±(特殊位置数字和)×多加或少加次数 ‎4、根据数论知识即整除性确定特殊位置数的取值及相对应的S值。‎ ‎5、根据确定的特殊位置数字及S值进行数字分组及尝试。‎ 三:幻方 ‎8‎ ‎1‎ ‎12‎ ‎11‎ ‎7‎ ‎3‎ ‎2‎ ‎13‎ ‎6‎ 幻方中的各数互不相同,且横行、竖列、对角线上数的和都相等的数的方阵,具有这一性质的3×3的数阵称作三阶幻方,4×4的数阵称作四阶幻方,5×5的称作五阶幻方……‎ 我们这里重点介绍三阶幻方的主要性质,以上图为例,主要有以下几个,希望同学们牢记:‎ ‎1、能组成幻方的数必须为从小到大排列,首尾对应相加均相等且等于中间数两倍的九个数数列。如1,2,3, 6,7,8,11,12,13中1+13=2+12=3+11=6+8=7×2,一般为等差数列(不完全是)。‎ ‎2、幻方的中心数为数列中的中间数,如上一列数中的7必须位于幻方中心。‎ ‎3、幻方中关于中心对称的两数均为数列中首尾相对应的配对,且两数的平均数为中心数。如上列数中的1,13与4,10的平均数均为7。‎ ‎4、幻方中所有相等的和称做幻和,幻方的幻和等于中心数的三倍。如幻和为21,等于中心数7的三倍。‎ ‎ 5、数列中最大与最小数的配对不能出现在幻方中的四角,即只能出现在中间位置,依次可以得知第二大与第二小数的配对只能出现在四角,在构造幻方的过程中如果能够遵循这个规律可以很快地得出答案。‎ ‎ 6、幻方中四角的数等于与它不相邻的两个行列中间数的平均数。如2等于1,3的平均,6等于1,11的平均,12等于11,13的平均,8等于3,13的平均。‎ ‎ 7、具有一个共同数的一行和一列中其他两个数的和相等(同学们能不能知道为什么?)如第一行和第一列中有一个共同数8,则其他两个数1+12=11+2。‎ ‎ 综合利用上面7个幻方性质就可以得出很多幻方的解题思路了。‎ 趣题解析:‎ ‎ 学习完幻方的性质之后,开篇趣题就很好填了吧,首先根据性质7,可知a+6=10+4,a=8,根据性质3,可知:d=(4+8)÷2=6,根据性质4,可知幻和为18,其他就全部根据每行相加等于幻和求出每一项,结果如图所示:‎ ‎ ‎ 三、经典透析 ‎【例1】(☆☆☆)右面残缺算式中只知道三个“‎4”‎,那么补全后它的 乘积是 。‎ 审题要点:‎ 此题为乘法数字谜,由于其中A中4出现在高位,所以利用高位分析法进行突破。‎ 详解过程:‎ 解:1、由c×=A,A百位数为4,可知c=8或9,若c=8,则 c×a必须向前进8,不可能。所以c=9。‎ ‎2、c=9时,a×9至少向前进4,即a×9≥40,知a≥5。‎ ‎3、对a=5,6,7,8,9进行逐一验算,验算的主要方法是通过c 中的4进行,‎ 若a=5,则A=405,f=4,但5×b末位不可能为4,排除。‎ 若a=6,则A=414,f=3,但6×b末位不可能为3,排除。‎ 若a=7,则A=423,f=2,7×b末位为2,则b=6,‎ 所以乘积为3243。‎ 若a=8,则A=432,f=1,但8×b末位不可能为1,排除。‎ 若a=9,则A=441,f=0,但9×b末位不可能为0,(因为乘数不能0开头),排除。‎ 专家点评:‎ 此题是乘法数字谜中比较经典的一个题型,用到的分析法依次包括:高位分析法(步骤1),进位分析法(步骤1),估算分析法(步骤2),个位分析法(步骤3),逐一尝试法(步骤3),及排除法(步骤3),注意寻找数字谜突破口的方法,抓住题目所给的已知数字,从涉及已知数字所有的计算处考虑,首先考虑乘法的关系,因为能够使用的分析法最多。希望同学也可以自己根据做题体会进行总结。‎ ‎【例2】(☆☆☆)已知右面的除法算式中,每个□表示一个数字,‎ 那么被除数应是 。‎ 审题要点:‎ 此题属于数字谜中的复杂题型,题目给出已知数字只有两个,不能直接使用个位分析法与高位分析法,可以结合数位考虑利用数值大小估值的方法进行分析。‎ 详解过程:‎ 解:1、首先比较明显可得出d=0,然后从2个数字的相关计算 进行突破,首先,,由于B只有两位数,所以可估算 推知=10,11或12。‎ ‎2、又,A为3位数,结合=10,11或12可知,c=9‎ 且,将其他各数补充完整即可。‎ 专家点评:‎ ‎ 此题是结合数位进行估算的一个典型数字迷,也利用了循环推理的原理。先是根据数字8×的乘积,利用数位估算确定的可能值,再根据c×的乘积,利用数位估算确定为12。所以在数字迷中,不仅有数字的地方有突破口,没有数字的地方也可能有突破口,一般是利用数位估算,当然也有其他分析方法,例如利用进退位的分析进行突破。‎ ‎【例3】(☆☆☆☆☆)在右面的乘法算式中,每一个□中要填一个数字,‎ 不同的中文字代表不同的数字,请问:“新年”两字代表什么数字? ‎ 审题要点:‎ 此题属于乘法数字谜中较难的题型,由于题目中出现的几个数字都为个位数,所以首先考虑运用个位分析法进行突破。‎ 详解过程:‎ 解:1、A行中末位为1,可推知a×b有四种可能性,1×1,3×7,‎ ‎7×3,9×9,又因为A行中为5位数,所以排除1×1,若a×b为 ‎9×9,则又结合B,C,D行中末位为9,可推出h,f,d为1,与B,‎ C,D都为5位数矛盾,排除。3×7,7×3暂时不能确定。‎ ‎2、假设a=7,b=3,由三个末位数是9,可推知h=f=d=7,这样反复利用 乘法和加法的个位分析法,可推知c=6,e=4,g无法满足。假设排除。‎ ‎3、假设a=3,b=7,由三个末位数为9,可推知h=f=d=3,同步骤2反 复利用乘法与加法个位分析法,可依次推出,而且满 足,所以“新年”代表15。‎ 专家点评:‎ 此题也是乘法数字谜中非常经典的一个题型,综合运用的分析法非常的多,且反复使用,环环相扣,同学可以在练习中好好体会其中的巧妙之处,依次包括:个位分析法(乘法与加法)(步骤1,2,3),结合数位考虑数字大小估算分析法(步骤1),循环推导(步骤2,3)。‎ ‎【例4】(☆☆☆☆)2008年奥运会快要到了,下图是大家都熟悉的奥林匹克的五环标志,你能把1—9分别填入五个圆相互分割的九个部分,并且使每个圆环内的数字之和都相等吗?‎ 审题要点:‎ 此题属于典型的数阵图,应该利用数字和全部相加的方法进行解答,从整体与个体的角度,同时考察应该填入的数字。‎ 详解过程:‎ 解:1、首先由于一共5个圆圈的数字和相等,由于填入的数无法确认,将5个数字和全部相加,由于数字和未知,设为S,则5个圆圈全部数字和为5S。‎ ‎2、考虑全部数字和5S中的各个组成部分:可知a,b,c,‎ h,i只加了一次,而d,e,f,g四个加了2次,即可以看 成1至9全部数字均加了一次,d,e,f,g多加了一次,表示 为(1+2+3+4+5+6+7+8+9)+(d+e+f+g)。‎ ‎3、 整体等于部分之和,列出方程:5S=(1+2+3+4+5+6+7+8+9)+(d+e+f+g),化简为5S=45+(d+e+f+g),d+e+f+g的最小值为1+2+3+4=10,此时S=11,最大值为6+7+8+9=30,此时S=15,一共5种可能填法。‎ ‎4、逐一进行试验:‎ 当S=11时,d,e,f,g为1,2,3,4时,可得出答案为:‎ 当S=12时,经过试验无解。‎ 当S=13时,d+e+f+g=20,取2,4,6,8,得出答案:‎ 当S=14时,d+e+f+g=25,取3,6,7,9,得出答案:‎ 当S=15时,经过试验无解。‎ ‎ ‎ ‎【例5】(☆☆☆)小兔子在森林玩耍,遇到一个画着奇怪图形的树桩,‎ 上面写着:把10至20这11个数分别填入右图的各圆圈内,使每条线段 上3个圆内所填数的和都相等。如果中心圆内填的数相等,那么就视为同 一种填法,请写出所有可能的填法,小兔子发了愁,你能帮它吗?‎ 审题要点:‎ 此题属于数阵图中的经典题型,同学们一定要利用数字和的规律来做,千万不要直接试,否则会浪费很多时间的。‎ 详解过程:‎ 解:1、由于5条线段的和均相等,从整体考虑将其全部相加为5S。‎ ‎2、从个体考虑,除中间数加了5次外,其他数均加了1次,可看作所有数10至20均加了1次,中间数a多加了四次,表示为(10+11+……+20)+‎4a,‎ ‎3、列出等式为5S=(10+11+……+20)+‎4a,化简为5S=165+‎4a,要使等式成立,a必须为5的倍数,得出三种答案,a=10时,S=41,a=15时,S=45,a=20时,S=49。‎ ‎4、将三种答案逐一尝试,得出三种答案分别如图:‎ 专家点评:‎ 此题中个体考虑时中心数字a一共加了5次,在列式当中一定要注意:加上的是多加的次数,即‎4a,而不是‎5a,因为在全部数字里面a已经相加一次。对a进行取值时的原则是使右边总和能够被5整除,确定a值与S值后进行简单的数字分组即可完成。有的题目在数字分组时需要同时满足多个条件,需要综合考虑,请看例6。‎ ‎【例6】(☆☆☆☆☆)右图中有三个正三角形,将1~9填入它们顶点处 的九个○中,要求每个正三角形顶点的三数之和都相等,并且通过四个○‎ 的每条直线上的四数之和也相等。‎ 审题要点:‎ 此题与上题相类似,注意题目要求两组和相等,应该先由一组进行分析满足后,再在此基础上满足另外一组和相等的条件。前面的分析方法一致,后面尝试部分数字分组时需要多考虑几个部分的和相等即可。‎ 详解过程:‎ 解:1、先考虑三角形和相等的部分,由于无重复数字,全部数字相加一次,直接列出等式:3S =(1+2+3+4+5+6+7+8+9),解出:S =15 ‎ 将所有数字分成数字和为15的三组:(1,5,9);(2,6,7);(3,4,8)‎ 任意选择其中一组填入最里面三角形,另外两组则不能随便填,需要再满足另一组和相等。‎ ‎2、考虑三直线和相等。观察发现内部三角形三数相加两次,其他相加一次,列出等式为:‎ ‎3K=(1+2+3+4+5+6+7+8+9)+(a+b+c)‎ 注意到内部三角形三数之和a+b+c已确定为15,代入解出方程得:K =20。‎ 先看有5和9的直线,其他两个数和为20–5-9=6,即需要从其他两组数中分别选取两数和为6,可为4,2。‎ 再看有5和1的直线,其他两个数和为20–‎ ‎5-1=14,即需要从其他两组数中分别选取两数和为14,可为8,6。‎ 最后看有9和1的直线,其他两个数和为20–9-1=10,即需要从其他两组数中分别选取两数和为10,可为3,7。‎ 分别填入即得到答案如下左图。‎ 如果一开始使用其他的分组法,还可得出下右图。‎ 专家点评:‎ ‎1、此题中两组和均需要满足相等,应该首先选择较为简单的一组进行分析,三角形三数和相等由于无重复数字,可以直接求出和为15,对后面另一组分析有用。‎ ‎2、步骤二的等式分析中,利用了第一组的结论三角形三数和为15,a+b+c=15,此处的利用十分巧妙,大家要注意其特点,还有很多题目中有此类使用。‎ ‎3、步骤二中的尝试是在第一组分组的基础上进行,注意直线上其他两个数必须分别来自两组数字中,大家可以想一想其中的原因。‎ ‎4、步骤二中三组数(4,2)(8,6)(3,7)确定后,填入图中时,同时又要考虑步骤一中的分组情况,即须保证(2,6,7),(3,4,8)在同一直线上。综合考虑,需要全面周到。‎ ‎【例7】(☆☆☆)请你将2~10这九个自然数填入图中的空格内每行、每列、每条 对角线上的三数之和相等。‎ 审题要点:‎ 此题属于构造简单幻方题型,不能简单的直接尝试,而应该利用幻方的性质,按一定的顺序和步骤进行逐一填入。‎ 详解过程:‎ A ‎7‎ B ‎2‎ C ‎9‎ D ‎8‎ E 6‎ F ‎4‎ G ‎3‎ H ‎10‎ I ‎5‎ 解:1、首先将2,3,4,5,6,7,8,9,10找出中间数6,并把其他数按首尾顺序配好对即(2,10),(3,9),(4,8),(5,7),根据幻方性质4知道幻和为18。‎ ‎2、根据幻方性质2将中间数填入幻方中心。‎ ‎3、根据幻方性质3和5依次将配对填入。首先(2,10)填入(B,H)(或者(D,F)也可以)。其次(3,9)可填入(G,C)‎ 注意:(3,9)不能填入(C,G),大家知道为什么吗?)下面两对就不能随便填入了。‎ ‎4、根据每行每列三数相加等于幻和18将其他数依次填入。例如第一行中,A+2+9=18推出A=7,其他依次类推。‎ ‎5、时间允许的情况下最好能够完全检验所有行列及对角线是否相加等于幻和。‎ 专家点评:‎ 构造幻方有很多的方法,这里介绍的是一种比较常用的方法,推荐使用,因为其构造的过程 完全依赖于幻方的性质,掌握此构造过程有利于对幻方性质的更深理解。‎ ‎【例8】(☆☆☆☆)在右图的九个方格中填入不大于12且互不相同的九个自然 数(其中已填好一个数),使得任一行、任一列及两条对角线上的三个数之和都 等于21。‎ 审题要点:‎ 此题也是属于幻方的构造,只不过条件更为复杂,但还是万变不离其宗,利用幻方的基本性质按步骤进行构造。‎ 详解过程:‎ 解:1、由幻和为21首先确定数列中间数及幻方中心数为7,进而确定C为9。‎ ‎2、根据数列不大于12的条件将数列按中间数为7依次列出为2,3,4,5,6,7,8,9,10,11,12,发现有五组可能的配对,所以应该排除一组。‎ ‎3、采用假设法,设最大最小数组为(3,11),按照幻方的性质5将填入(B,H),进而确定(A,I)为(C,G)为(9,5),(A,I)为(8,6),但是此时第一行三数之和8+3+9=20不等于幻和21。‎ ‎4、确定最大最小数组为(2,12),填入(B,H),进而根据幻和依次填入其他数组。满足条件。‎ A ‎8‎ B ‎3‎ C ‎9‎ D E 7‎ F G ‎5‎ H ‎11‎ I ‎6‎ A ‎10‎ B ‎2‎ C ‎9‎ D ‎6‎ E 7‎ F ‎8‎ G ‎5‎ H ‎12‎ I ‎4‎ 专家点评:‎ 此题中根据已知无法直接确定数列,而且幻方中有一组配对已经确定,所以只能采用假设法,按照最大最小数组的选择进行假设,很容易得出结论。假设法是奥数里面常用的一种方法,希望同学们在几种情况无法确定的时候一定要采用假设法。‎ ‎【例9】(☆☆☆☆☆)如图所示,在3×3方格表内已填好了两个数19和95,在其余的空格中填上适当的数,可以使得每行、每列以及两条对角线上的三个数之和都相等。(1)求x;(2)如果中间的空格内填入100,试在上一小题的基础上,完成填图。‎ 审题要点:‎ 此题第一问中给出的条件只有两个数字,无法确定中心数,也无法确定幻和,这里需要利用幻方性质中最为复杂的一个即性质6。第二问相对就简单得多了。‎ 详解过程:‎ A B x C D E ‎ F ‎19‎ G ‎95‎ H I 解:1、直接根据幻方性质6:幻方中四角的数等于与它不相邻的两个行列中间数的平均数。可知95即为19与x的平均数,所以可求出x为171。‎ A ‎24‎ B ‎171‎ C ‎105‎ D ‎181‎ E ‎100 ‎ F ‎19‎ G ‎95‎ H ‎29‎ I ‎176‎ ‎2、由中心数为100确定幻和为300,然后由每一行列数相加为幻和依次计算出其他数组。结果如图所示:‎ 专家点评:‎ 本题中使用的幻方性质6比较偏,同学们可以重点记忆,当然此问也可以用方程法直接解出,但是过程比较复杂,这里就不作深入分析,有兴趣的同学可以自己去试试,得出的答案完全一样。第二问属于一般的幻方计算,综合利用幻方性质就可以直接得出结论。‎ 四、拓展训练 1. 下面是一个残缺的乘法算式,只知道其中一个数字“‎8”‎,请你补全,那么这个算式的乘积是 。‎ ‎[初级点拨] 此题与例2相类似,突破口一样,主要利用大小估值的方法进行分析。‎ ‎[深度提示] 由。‎ ‎[全解过程] 。‎ 2. 右面的除法算式(1)中,每个□表示一个数字,那么商数是 。‎ ‎ ‎ ‎[初级点拨] 此题根据已知数字6,7,1,出现的位置可容易判断需要用到的分析法有高位分析法,个位分析法及 根据数位大小估值法。‎ ‎[深度提示] 由a×=A,而A为三位数,根据高位大小估值法可知a=1。 进而可知d=7,继续考虑B中个位数利用个位分析法。‎ ‎[全解过程] 容易看出B个位数为1,由b×=B,根据个位分析法知b=3, 所以商数为13。‎ 1. 在右面的算式中,相同的汉字代表相同的数字,不同的汉字代表不同的数字。那么,“努力力争”四个汉字所代表的四个数字的和是 。‎ ‎[初级点拨] 题目中唯一出现的个位数字1,考虑使用个位分析法突破,注意 不同的汉字代表不同的数字的限制条件进行排除。‎ ‎[深度提示] A个位数为1,结合习与争为不同数,可知只有两种可能性:1,习=3,争=7;2,习=7,争=3。先假设第一种情况,“习”=3,“争”=7 则A中十位数为学=学×7+2(个位数)推知学=3或8,又习=3,所以学=8,这样A百位数8=8×7+5(进位)不成立,排除。继续假设第二种情况。‎ ‎[全解过程] 先确定力为0,再假设习=7,争=3 ,则十位的学=学×3+2(个位数),推知学=4或9,当学=4时百位4=4×3+1(进位)不成立。所以学=9。千位数×3+2不进位,推知数=1或2,当数=2时,a=8,b=1,7×努,个位为1,则努=3,与争=3重复,排除。所以数=1,则a=5,b=4,努=2,“努力力争”=2003。所以“努力力争”四个汉字所代表的数字和为5。‎ 2. 将1~6填入右图的六个○中,使三角形每条边上的三个数之和都等于k,请指出k的取值范围。‎ ‎[初级点拨] 此题属于典型数阵图填空,利用数字和全部相加的方法,找出每一个数字的相加次数,列出等式进行分析取值。‎ ‎[深度提示] 1、把三个要求相等的数字和相加为3k。‎ ‎2、计算每一个位置数字相加的次数,很明显,全部数字相加一次外,角上三个数字多加一次,表示为(1+2+3+4+5+6)+(a+b+c)。‎ ‎3、列出等式3k=(1+2+3+4+5+6)+(a+b+c)进行分析,计算a+b+c的所有可能取值及对应k的取值。‎ ‎[全解过程] a+b+c最小为1+2+3=6,此时k=9,最大为4+5+6=15,此时k=12,那么k可等于10,11,对应a+b+c=9和12,可取1,3,5和2,4,6,经过尝试四种结果如下:‎ k=9 k=10 k=11 k=12‎ 3. ‎1~9分别填入小三角形内(每个小三角形内只填一个数),要求靠近大三角形三条边的每五个数相加和相等。想一想,怎样填这些数才能使五个数的和尽可能大一些?‎ ‎[初级点拨] ‎ 典型数阵图,利用全部数字和相加,从整体和个体同时考虑,列出等式进行分析取值。注意计算清楚数阵图中每一个位置的相加次数。‎ ‎[深度提示] 1、将三个要求相等的数字和相加为3S。‎ ‎2、计算每一位置的相加次数,可知a,b,c只加一次外,其他都相加两次,可以看成全部数字相加二次,再减去(a+b+c),即为(1+2+3+4+5+6+7+8+9)-(a+b+c)。‎ ‎3、列出等式3S=(1+2+3+4+5+6+7+8+9)×2-(a+b+c)=90-(a+b+c)‎ ‎ 要使S尽可能大,则a+b+c可取1+2+3=6,此时S=28。‎ ‎[全解过程] 取a=1,b=2,c=3,再用尝试法完成其他数字:剩下的六个数分成三组,并且每组中两数的和是三个连续自然数,那么可选择:4+8=12;6+7=13;5+9=14。结果如下:‎ ‎ ‎ 1. 海豚是很聪明的动物,它能将1~9填入下图的九个○内,并且使得每个圆周和每条直线上的三数之和都相等,并且7,8,9依次位于小、中、大圆周上,你能做到吗?‎ ‎[初级点拨] 此数阵图中每一个数字都相加两次,根据等式可以直接确定S,然后根据要求将数字分组并实验。‎ ‎[深度提示] 一共六个要求相等的数字和,而每一个数字都相加两次,无特殊数字。列出等式为6S=(1+2+3+4+5+6+7+8+9)×2 解出S=15。将九个数字分为三组,每组三个数字和为15。‎ ‎[全解过程] 数字分组进行尝试:将全部数字分为三组,注意7,8,9必须分在不同组,无唯一分法,例如(951),(843),(762),又观察可知必须从每一组选一个组成数字和为15,可选择为(942),(537)(186),调换顺序可下列两种答案:‎ 2. 如图是一个三阶幻方,那么标有*的方格中所填的数是多少?‎ ‎[初级点拨] 此幻方给出的已知条件比较少,中心数与幻和均未知,但是观察发现 第一行与第一列除共同的数字外只有一个未知,考虑使用幻方性质7解决。‎ ‎[深度提示] 幻方性质7:具有一个共同数的一行和一列中其他两个数的和相等,可知1+G=8+10,所以G为17,再根据幻方性质3可将中心数填出。‎ ‎[全解过程] 幻方性质3:幻方中关于中心对称的两数均为数列中首尾相对应的配对,且两数的平均数为中心数,可知中心数为10与17的平均,即13.5,确定幻和为40.5,再根据第一行中幻和算出A为22.5。‎ A B ‎8‎ C ‎10‎ D ‎1‎ E ‎ F G ‎17‎ H I A ‎22.5‎ B ‎8‎ C ‎10‎ D ‎1‎ E ‎ ‎13.5‎ F G ‎17‎ H I ‎ ‎ 1. 把1,2,3,4,6,9,12,18,36这9个数分别填入3×3方格表的各方格内,使每一行、每一列及两条对角线上的3个数的乘积都是216。求位于正中间的方格中所填的数。‎ A ‎3‎ B ‎36‎ C ‎2‎ D ‎4‎ E ‎ ‎6‎ F ‎9‎ G ‎18‎ H ‎1‎ I ‎12‎ ‎[初级点拨] 此题是属于三阶乘法幻方,可利用加法幻方的性质及步骤类推得出结论进行解答。‎ ‎[深度提示] 加法幻方步骤进行操作:1,确定中间数6,配对数组(1,36),(2,18),(3,12),(4,9),按大小顺序依次填入幻方中对称位置。注意幻方性质5的利用。‎ ‎[全解过程] 中心数为6。我们继续填下去:幻积为216,将配对依次填入:(1, 36)填入中间(H,B),(2,18),填入四角(C,G),其他再全部根据幻积为216计算,依次填入,答案如图:‎ 2. ‎7个圆内填入7个连续自然数,使得每两个相邻圆内所填数的和都等于连线上的已知数,那么标有★的圆内填的数是多少?‎ ‎[初级点拨] 此题属于数阵图的变形,应用数阵图的基本思路,将所有和相加,进行分析。‎ ‎[深度提示] 将所有和相加得14+11+8+12+9+6+10=70,从个体考虑,可看出每个数相加两次,所以七个连续数和为70÷2=35,所以七个连续数为:2,3,4,5,6,7,8。‎ ‎[全解过程] (法1)从最大和或最小和处开始尝试,14只有唯一分解14=8+6假设★为8,尝试发现不能完成,所以★为6,逐一计算完成如图:‎ ‎(法2)本题也可以用方程的方法可以先假设最顶端的圆圈中所填的数为x,那么根据它与右邻的和为14可以得到它右边的圆圈中的数为14-x,如此按顺时针顺序得到各个圆圈的代数表达式:x-3,11-x,1+x,8-x,x-2,所以最顶端的圆圈的左邻中的数是x-2,根据最顶端圆圈与其左邻圆圈中的数和为10可列方程x-2+x=10,得到x=6,其他圆圈中的数也就可以全部求出。‎ 1. 把1.2,3.7,6.5,2.9,4.6分别填在右下图的5个圆圈内,然后在每个方框中填上和它相连的3个圆圈中的数的平均值,再把3个方框中的数平均值填在三角形中。请找出一种填法,使三角形中的数尽可能小。问这个最小的数是多少?‎ ‎[初级点拨] 此题中数量关系不是很明显,也无法从整体上直接求和,应该设出未知数并表示出最后数,就能看出它们的关系,再进行分析。‎ ‎[深度提示] 设个小圆中的数依次为a1、a2、a3、a4、a5,则三个正方形中的数依次为、、,继而求出三角形中的数值为。要使数值最小,a1、a2、a3、a4、a5应该怎么样取值。‎ ‎[全解过程] 很明显,a3中应该填入最小的数1.2,a2、a4中应该填入次大的2.9和3.7,a1、a5中填入4.6和6.5,这样三角数等于3.1。‎ 第4讲 数论(一) 整除、奇偶性、极值问题 一、 知识地图:‎ 二、 基础知识:‎ 数论是研究整数性质的一个数学分支,它历史悠久,而且有着强大的生命力。数论问题叙述简明,“很多数论问题可以从经验中归纳出来,并且仅用三言两语就能向一个行外人解释清楚,但要证明它却远非易事”。因而有人说:“用以发现天才,在初等数学中再也没有比数论更好的课程了。任何学生,如能把当今任何一本数论教材中的习题做出,就应当受到鼓励,并劝他将来从事数学方面的工作。”所以在国内外各级各类的数学竞赛中, 数论显得格外重要。数论研究的是奇数、偶数、素数、合数,这些最简单的数——整数及其内部关系,但是从这些简单的数中诞生了“费马大定理”、“哥德巴赫猜想”和“朗兰兹纲领”这样的难题,它们吸引数学家们花费数十年、甚至整世纪努力研究。‎ 小学数学竞赛和小升初考试的数论问题,常常涉及整数的整除性、质数与合数、约数与倍数、带余除法、奇数与偶数、整数的分解与分拆。‎ ‎(一)整除问题 数的整除 ‎ 数的整除在算术中应用广泛,下面我们从整除的概念、整除的性质及数的整除特征三方面来介绍。‎ ‎ ‎ ‎ 1.整除的概念 ‎ 在整数范围内,两个数相除,余数为零(没有余数)或不为零,两种结果必定有一种成立。如果余数为零,我们就说被除数能被除数整除。如15÷3=5;24÷2=12。‎ ‎ 一般地,如果用字母表示,可以这样说:‎ ‎ 整数a除以整数b(b≠0),除得的商正好是整数而没有余数,我们就说a能被b整除(也可以说b能整除a),记作b︱a。如15能被3整除,记作3︱15;24能被2整除,记作2︱24。‎ 如果数a能被数b(b≠0)整除,a就叫做b的倍数,b就叫做a的约数。像在上面的算式中,因为3︱15,15是3的倍数,3是15的约数。‎ ‎ 由于0÷b=0(b≠0),就是说零能被任何非零整数整除。因此,零是任意一个非零整数的倍数。‎ ‎1是任意一个整数的约数,也就是说,对于整数a,都能保证1︱a成立。‎ 同样,由于a÷a=1(a≠0),也就保证一个非零整数必能整除它本身,也就是a︱a(a≠0)。‎ ‎ ‎ ‎2.整除的性质 ‎ ‎ ‎(1)性质1 如果数a和数b都能被数c整除,那么它们的和或差也能被c整除。即如果c︱a,c︱b,那么c︱(a±b)。在理解这个性质时,我们要注意,反过来是不成立的,即两数的和(a+b)或差(a-b)能被c整除,这两个数不一定能被c整除。如5 ︱(26+24),但526,524。‎ ‎ 再看下面这个问题:2∣12,12∣36。2能否整除36?显然,回答是肯定的。这是因为36是12的倍数,12又是2的倍数,那么36一定是2的倍数。由此我们又可以得出:‎ ‎(2)性质2 如果数a能被数b整除,b又能被数c整除,那么a也能被c整除。即如果b∣a,c∣b,那么c∣a。‎ ‎ ‎ 用同样的方法,我们还可以得出:‎ ‎(3)性质3 如果数a能被数b与数c的积整除,那么a也能被b或c整除。即如果bc∣a,那么b∣a,c∣a。‎ ‎(4)性质4 如果数a能被数b整除,也能被数c整除,且数b和数c互质,那么a一定能被b与c的乘积整除。即如果b∣a,c∣a,且(b,c)=1,那么bc∣a。‎ ‎ 如:如果3∣12,4∣12,且(3,4)=1,那么(3×4) ∣12。‎ ‎(5)性质5 如果数a能被数b整除,那么am也能被bm整除。‎ 如果 b|a,那么bm|am(m为非0整数);‎ ‎(6)性质6 如果数a能被数b整除,且数c能被数d整除,那么ac也能被bd整除。‎ 如果 b|a ,且d|c ,那么bd|ac;‎ ‎ 3.数的整除特征 ‎ ①能被2整除的数的特征:个位数字是0、2、4、6、8的整数。“特征”包含两方面的意义:一方面,个位数字是偶数(包括0)的整数,必能被2整除;另一方面,能被2整除的数,其个位数字只能是偶数(包括0)。下面“特征”含义相似。‎ ‎ ②能被5整除的数的特征:个位是0或5。‎ ‎  ③能被3(或9)整除的数的特征:各个数位数字之和能被3(或9)整除。‎ ‎ 例如:123的各位数字之和是1+2+3=6,因为6能被3整除,所以3∣123。‎ ‎ 再如:126的各位数字之和是1+2+6=9,因为9能被9整除,所以9∣126。‎ 我们以三位数为例来证明被9整除只需看各位数字之和这一性质 假设该三位数为=‎100a+10b+c=(‎99a+9b)+(a+b+c)‎ 很明显第一个括号里的数是9的倍数,因此只要a+b+c,即各位数字之和能被9整除,那么这个三位数abc就能被9整除,反之亦然。推广到任意位数的自然数,该证明方法仍然成立,请大家自己尝试一下。‎ 注意:从这种证明过程中,我们可以进一步得到两个小技巧:‎ ‎(1)“弃九法”。即看各位数字和能否被9整除,只要先把9划去,或者其它的和是9的几个数划去,剩下的数字之和是否是9的倍数,则可以判定这个数能否被9整除。‎ ‎(2)得余数。通过上面的过程,我们可以看出这个数被9除的余数就是在弃9法以后的余数。‎ 类似地,判断能否被3整除或者不能整除时的余数是几,也可以用这种简便方法。‎ ‎  ④能被4(或25)整除的数的特征:末两位数能被4(或25)整除。‎ ‎  例如:1864=1800+64,因为100是4与25的倍数,所以1800是4与25的倍数。又因为4|64,所以1864能被4整除。但因为2564,所以1864不能被25整除。。‎ ‎  ⑤能被8(或125)整除的数的特征:末三位数能被8(或125)整除。‎ ‎  例如:29375=29000+375,因为1000是8与125的倍数,所以29000是8与125的倍数。又因为125|375,所以29375能被125整除。但因为8375,所以829375。‎ ‎  ⑥能被11整除的数的特征:这个整数的奇数位上的数字之和与偶数位上的数字之和的差(大减小)是11的倍数。‎ ‎  例如:判断123456789这九位数能否被11整除?‎ ‎  解:这个数奇数位上的数字之和是9+7+5+3+1=25,偶数位上的数字之和是8+6+4+2=20。因为25—20=5,又因为115,所以11123456789。‎ ‎  再例如:判断13574是否是11的倍数?‎ ‎  解:这个数的奇数位上数字之和与偶数位上数字和的差是:(4+5+1)-(7+3)=0。因为0是任何整数的倍数,所以11|0。因此13574是11的倍数。‎ ‎  ⑦能被7(11或13)整除的数的特征:一个整数的末三位数与末三位以前的数字所组成的数之差(以大减小)能被7(11或13)整除。‎ ‎  例如:判断1059282是否是7的倍数?‎ ‎  解:把1059282分为1059和282两个数。因为1059-282=777,又7|777,所以7|1059282。因此1059282是7的倍数。‎ 再例如:判断3546725能否被13整除?‎ 解:把3546725分为3546和725两个数。因为3546-725=2821。再把2821分为2和821两个数,因为821—2=819,又13|819,所以13|2821,进而13|3546725。‎ 特殊的,六位数是7、11、13的倍数。‎ ‎ [思考]:为什么要从末三位把这个数一分为二呢?‎ 仔细想一想我们会发现7×11×13=1001,正好比1000大1,由此我们可以得到如下证明 我们设一个多位数的末三位是abc,前面部分是x 那么我们要证明的就是这个多位数能否被7,11,13整除决定于abc-x能否被7,11,13整除 该数=1000x+abc=1001x+(abc-x)‎ 由于1001同时是7,11,13的倍数,所以这个多位数能否被7,11,13整除决定于abc-x能否被7,11,13整除。‎ 希望大家能熟练掌握以上判别方法,并理解我们是如何证明的,考试不会考这些证明,但是这种证明的方法在做一些其他数论题目的时候是非常有效的。‎ ‎ 上面介绍了能被2、3、4、5、7、8、9、11、13整除数的特征。那么,怎样判断一个数能否被6、12、15……等整数整除呢?‎ ‎ 显然6=2×3,12=3×4,15=3×5…… 这里,等号右边的两个因数之间没有相同的约数,于是我们可以把6,12,15…… 这类数的整除问题转化为同时能被2和3整除或3和4整除……等简单的问题来做。‎ ‎4.奇数和偶数 ‎  整数可以分成奇数和偶数两大类。能被2整除的数叫做偶数,不能被2整除的数叫做奇数。‎ ‎  偶数通常可以用2k(k为整数)表示,奇数则可以用2k+1(k为整数)表示。‎ ‎  特别注意,因为0能被2整除,所以0是偶数。‎ ‎  ‎ 奇数与偶数有许多的性质 奇数±奇数=偶数 ‎ 奇数±偶数=奇数 ‎ 偶数±偶数=偶数 奇数个奇数的和或差(相加减)为奇数 ‎ 偶数个奇数的和或差(相加减)为偶数 加减法中偶数不改变结果的奇偶性(偶数都可以看作0或没有操作)‎ 加减法中奇数改变结果的奇偶性(奇数都可以看作1)‎ 奇数×奇数=奇数 ‎ 偶数×偶数=偶数 ‎ 奇数×偶数=偶数 奇数×奇数×奇数×奇数×…×奇数×偶数=偶数 a+b与a-b同奇或同偶 奇数的平方=4K+1,偶数的平方=4K 任何一个奇数一定不等于任何一个偶数。‎ ‎5.最值分析(离散)‎ ‎ 在日常生活、工作中,经常会遇到有关最短路线、最短时间、最大面积、最大乘积等问题,这就是在一定条件下的最大值或最小值方面的数学问题。这类问题涉及的知识面广,在生产和生活中有很大的实用价值。最值分析问题是数学中的一个难点和重点,可以融入到认识的题目中去考核,并因为他们的存在使题目变得更有难度。一般而言,最大与最小问题包括数、式、方程(组)中的最大最小问题、统筹方法中教学思想方法的初步应用、几何中的最大最小问题(周长、面积、最短的路线)。‎ 重要结论:两数和一定时,这两数差越小(越接近)乘积越大 整数分拆中的最值问题 在国内外的数学竞赛试题中经常出现与整数分拆有关的最大值或最小值的问题。‎ 例如: 试把14分拆为两个自然数之和,使它们的乘积最大。‎ 解:把14分拆成两个自然数之和,共有7种不同的方式。对每一种分拆计算相应的乘积:‎ ‎  14=1+13,1×13=13;‎ ‎  14=2+12,2×12=24;‎ ‎  14=3+11,3×11=33;‎ ‎  14=4+10,4×10=40;‎ ‎  14=5+9,5×9=45;‎ ‎  14=6+8,6×8=48;‎ ‎  14=7+7,7×7=49。‎ ‎  因此,当把14分拆为两个7之和的时候,乘积(7×7=49)最大。‎ ‎  说明:本例可以推广为一般性结论:“把自然数n≥2分拆为两个自然数a与b(a≥b)之和,使其积a×b取最大值的条件是a=b或a-b=1(a>b)”。事实上,假设a-b=1+m(其中m是一个自然数),显然n=a+b=(a-1)+(b+1),而有(a-1)×(b+1)=a×b+a-b-1=a×b+m>a×b。‎ ‎  ‎ 例如:试把14分拆为3个自然数之和,使它们的乘积最大。‎ 分析: 假设n=a+b+c(a≥b≥c)且a-c>1时,乘积a×b×c不是最大的。换句话说,若n=a+b+c(a≥b≥c),当a、b、c中的任意两数相等或差为1时,乘积a×b×c取最大值。‎ ‎  解:因为14=3×4+2,由分析可知:当a=b=5且c=4时,乘积a×b×c=5×5×4=100为最大值。‎ ‎  说明:本题可以推广为一般结论:把自然数n≥3分拆为3个自然数,什么情况下乘积最大?‎ ‎  下面我们再研究一个难度更大的拆数问题。‎ ‎  问题:给定一个自然数n,把它拆成若干个自然数的和,使它们的积最大。‎ ‎  这个问题与前面研究的两个拆数问题的不同点是:问题中没有规定把n拆成几个自然数的和。这也正是这题的难点,使分拆的种类要增加许多。我们仍旧走实验-观察-归纳结论这条路。先选择较小的自然数5开始实验。并把数据列表以便比较。‎ 你注意到了吗?我们的实验结果是按把5拆分数的个数多少,由多到少的次序进行的。再注意,当被拆数n>3时(这里n=5),为了使拆分数的乘积最大,拆分数中不能有1。因为当n>3,n=1+(n-1)=2+(n-2),且2×(n-2)>1×(n-1)。‎ ‎ 结果:7拆分成2+2+3时。其积12最大。‎ ‎  注意,分拆数中有4时,总可把4再分拆成2与2之和而不改变分拆的乘积。‎ ‎  实验结果4:8拆分成2+3+3时,其积最大。‎ ‎  实验结果5:9拆分成3+3+3时,其积最大。‎ ‎  实验结果6:10拆分成3+3+2+2时,其积最大。‎ ‎  观察分析实验结果,要使拆分数的乘积最大,拆分数都由2与3组成,其形式有三种:‎ ‎  ①自然数=(若干个3的和);‎ ‎  ②自然数=(若干个3的和)+2;‎ ‎  ③自然数=(若干个3的和)+2+2。‎ ‎  因此,我们得到结论:把一个自然数n拆分成若干个自然数的和,只有当这些分拆数由2或3组成,其中2最多为2个时,这些分拆数的乘积最大。(因为2+2+2=3+3,2×2×2<3×3,所以分拆数中2的个数不能多于2个)‎ ‎  ‎ 我们以上采用的“实验-观察-归纳总结”方法,在数学上叫做不完全归纳法。我国著名数学家华罗庚讲过:难处不在于有了公式去证明,而在于没有公式之前怎么去找出公式。不完全归纳法正是人们寻找公式的重要方法之一。但是这种方法得出的结论有时会不正确,所以所得结论还需要严格证明。这一步工作要等到学习了中学的课程才能进行。‎ ‎【例1】(☆☆☆)在865后面补上三个数字,组成一个六位数,使它能分别被3、4、5整除,且使这个数值尽可能的小。‎ ‎[审题要点] 被3、4、5整除的数的特征, 数字和(8+6+5+a+b+c)是3的倍数;末两位数字组成的两位数是4的倍数;末位数字c是0或5;“数值尽可能小”,条件必须同时满足。‎ ‎[详解过程] 设补上数字后的六位数是,因为这个六位数能分别被3、4、5整除,所以它应满足以下三个条件:‎ 第一,数字和(8+6+5+a+b+c)是3的倍数;‎ 第二,末两位数字组成的两位数是4的倍数;‎ ‎  第三,末位数字c是0或5。‎ 由以上条件,4|,且c只能取0或5,‎ 又能被4整除的数的个位数不可能是5, ∴c只能取0,因而b只能取0,2,4,6,8中之一。‎ ‎  又3|,且(8+6+5)除以3余1,∴a+b除以3余2。‎ ‎  为满足题意“数值尽可能小”,只需取a=0,b=2。‎ ‎  ∴要求的六位数是865020。‎ ‎[专家点评] 被3、4、5整除的特征应用非常重要,一定要掌握好。‎ ‎ 在考虑两个数的和能否被n整除,分别看这两个数被n除的余数,这种技巧题中用到,更多时候在解不定方程的时候,我们称之为“除余试值法”。‎ ‎【例2】(☆☆☆)各位数码是0、1或2,且能被225整除的最小自然数是多少?‎ ‎[审题要点] 被合数整除把225分解,分别考虑能被25和9整除特征。‎ ‎[详解过程] 225=25×9,所以要求分别能被25和9整除。要能被25整除,所以最后两位就是00。要能被9整除,所以所有数字的和是9的倍数,为了使得位数尽可能少,只能是4个2和1个1,这样得到1222200。‎ ‎[专家点评] 为了使得位数尽可能少,所以每个数位数字应尽可能大,高位数小。‎ ‎ ‎ 注意:对整除性质 如果b∣a,c∣a,且(b,c)=1,那么bc∣a。‎ ‎ 所以对于考虑一个数是否能被225整除,只能考虑同时被25和9整除,而不能是15和15,或者5和45。请“奥数研究生”们细作体会。‎ ‎【例3】(☆☆☆)一个六位数,它能够被9和11整除。去掉这个六位数的首、尾两个数字,中间的四个数字是1997,那么这个六位数是多少?‎ ‎[审题要点] 被11整除的特征奇位数减偶位数的差 ‎[详解过程] 设这个六位数是,它能够被9整除,所以 能被9整除,因此或10。当时,、,而119970不能被11整除,所以。‎ 又能被11整除,,说明 或,即或,结合,只有符合,此时、,所以这个六位数是219978。‎ 专家点评:分类讨论考察综合能力 ‎ ‎ ‎【例4】(☆☆☆)下面这个199位整数:被13除,余数是多少? ‎ ‎[审题要点] 100100能被13整除,每六位能被13整除的关于周期的规律经常应用于数论的相关分析中。‎ 这一点与1001=7×11×13有关。‎ ‎[详解过程] 100100能被13整除,每六位能被13整除,199÷6=33余1,原数相当于33个100100的循环的10倍再+1,即1001001001…001001001(199位)-1=1001001001…001001000(199位)=1001001…100100100(198位)的10倍,显而易见1001001…100100100(198位)是1001的倍数,所以也是13的倍数,所以原数被13除所得的余数是1。‎ ‎[专家点评] 多位数整除有的同学无从下手,规律无处不在。此题是香港圣公会小学数学奥林匹克试题。‎ ‎【例5】(☆☆☆☆)用1,9,8,8这四个数字能排成几个被11除余8的四位数?‎ ‎[审题要点] 现在要求被11除余8,我们可以这样考虑:这样的数加上3后,就能被11整除了。所以我们得到“一个数被11除余‎8”‎的判定法则:将偶位数字相加得一个和数,再将奇位数字相加再加3,得另一个和数,如果这两个和数之差能被11整除,那么这个数是被11除余8的数;否则就不是。‎ ‎[详解过程] ‎ ‎ 要把1,9,8,8排成一个被11除余8的四位数,可以把这4个数分成两组,每组2个数字。其中一组作为千位和十位数,它们的和记作A;另外一组作为百位和个位数,它们之和加上3记作B。我们要适当分组,使得能被11整除。现在只有下面4种分组法:‎ 偶位 奇位 ‎(1) 1,8 9,8‎ ‎(2) 1,9 8,8‎ ‎(3) 9,8 1,8‎ ‎(4) 8,8 1,9‎ ‎ 经过验证,第(1)种分组法满足前面的要求:‎ ‎ A=1+8=9,B=9+8+3=20,B-A=11能被11除尽。‎ ‎ 但其余三种分组都不满足要求。根据判定法则还可以知道,如果一个数被11除余8,那么在奇位的任意两个数字互换,或者在偶位的任意两个数字互换得到的新数被11除也余8。于是,上面第(1)分组中,1和8任一个可以作为千位数,9和8中任一个可以作为百位数。这样共有4种可能的排法:1988,1889,8918,8819。‎ ‎[专家点评] 难点在于有余数,偶位数字相加的和与奇位数字相加的和之差能被11整除余数的特点互补性 ‎ ‎ ‎【例6】(☆☆☆☆☆)如右图,用一块边长‎18厘米的正方形硬纸片,在四个 角上截去4个相同的小正方形(图中阴影部分),然后 把四边形折合起来,做成一个没有盖的长方体纸盒。‎ 截去的4个相同的小正方形的边长是多少厘米时,长方 体纸盒的容积最大?‎ ‎[审题要点] 和一定,差小积大 ‎[详解过程] 设高为x,则V=(18-2x)(18-2x)·x,18-2x=18-2x=4x,x=3,所以12×12×3=432。‎ ‎[专家点评] V=abh ‎ ‎==2h=6‎ ‎【例7】(☆☆☆)在黑板上写1~2007这2007个自然数,每次任意擦去两个数,然后写上它们的和或差,一直这样重复操作,经过若干次后黑板上只剩下一个数,请问结果是奇数还是偶数?为什么?‎ ‎[审题要点] 加减法中偶数不改变结果的奇偶性(偶数都可以看作0或没有操作)‎ 加减法中奇数改变结果的奇偶性(奇数都可以看作1)‎ ‎[详解过程] 奇数±奇数=偶数 擦去两个奇数写上一个偶数不改变结果的奇偶性 偶数±偶数=偶数 擦去两个偶数写上一个偶数不改变结果的奇偶性 奇数±偶数=奇数 擦去一个奇数一个偶数写上一个奇数相当于只擦去一个偶数还是不改变结果的奇偶性,在1-2007中有1004个奇数,每次都是成对的擦去奇数,所以最后只剩下一个数是偶数。‎ ‎[专家点评] 此题可以转化为1—2007这2007个数之间任意添加号或减号其结果是奇数还是偶数? ‎ ‎【例8】(☆☆☆)用1、2、3、4、5、6、7、8、9这九个数字(每个数字仅用一次)组成一个四位数和一个五位数,使乘积最大:则□□□□□×□□□□应该怎样填?‎ 若将1——9这九个数字,分别填入下面九个□中,使乘积最大:□□□×□□□×□□□‎ ‎[审题要点] 要使得这两个数的乘积最大,就要使得这两个数的差尽可能小 ‎[详解过程] 补数字0组成两个五位数,把0写在某个五位数的末位。96420-87531=8889,所以最大87531×9642=843973902。‎ 遵循“把比较大的数都填在高位上”的原则和“和一定,差小积大”,有941×852×763 。‎ ‎【例9】(☆☆☆☆☆)9999和99!能否表示成为99个连续的奇自然数之和?‎ ‎[审题要点] 奇数个奇数之和为奇数,而奇数永远不会等于偶数 ‎[详解过程]‎ ‎1)9999能。因为9999等于99个9998之和,所以可以直接构造如下:‎ ‎ 9999=(9998-98)+(9998-96)+…+(9998-2)+9998+(9998+2)+…+(9998+96)+(9998+98)。‎ ‎2)99!不能。因为99!为偶数,而99个奇数之和为奇数,所以99!不能表示为99个连续奇数之和。 说明:利用构造法证明存在性问题,只要把满足题设要求的数学对象构造出来就行 专家点评:构造法在数学中非常重要,也往往是难点。反证法是奇偶分析常用的方法。‎ 四、拓展训练 1. 要使能被36整除,而且所得的商最小,那么A、B、C分别是多少?‎ ‎[初级点拨] 分解为互质的几个数的乘积,36=4×9分别考虑 ‎[深度提示] 因为36=4×9,所以能被4整除,从而C只可能是1,3,5,7,9。要使商最小,A、B应尽可能小,先取A=0,又1+5+6+A+B+C=12+B+C=9+3+B+C,所以3+B+C是9的倍数。‎ ‎[全解过程] 因为36=4×9,所以能被4整除,从而C只可能是1,3,5,7,9。要使商最小,A、B应尽可能小,先取A=0,又1+5+6+A+B+C=12+B+C=9+3+B+C,所以3+B+C是9的倍数,B=1,C=5时,取得最小值。本题商的最值分析是亮点和难点, 被合数整除的特征,应分解为互质的几个数的乘积,被4、9整除的特征是必要前提。‎ ‎2、☆☆能被11整除,那么,n的最小值为多少?‎ ‎3、☆☆☆☆求最小的自然数,它的各位数字之和等于56,它的末两位数是56,它本身还能被56所整除。 ‎ ‎4、☆☆☆(2002年南京市少年数学智力冬令营)一个十位数,如果各位上的数字都不相同,那么就称为“十全数”,例如,3 785 942 160就是一个十全数。现已知一个十全数能被1,2,3,…,18整除,并且它的前四位数是4876,那么这个十全数是 。‎ ‎5、已知四十一位数55…55□99…99(其中5和9各20个)能被7整除,那么中间方格内的数字是多少?‎ ‎6、把三位数接连重复写下去,共写1993个,所得的数恰是91的倍数,试求=?‎ ‎7、已知等式1993×□+4×□=6063,其中□都是自然数,试求这两个“□”的和。‎ ‎8、能否找到这么一个数,它加上24,和减去30所得的两个数都是完全平方数?‎ ‎9、桌子上有7个杯子,开口全部向上,现在允许每次同时翻动其中6个,能否经过若干次翻动使得所有杯子杯口全部向下,若可以,请指出最少需要多少次?并给出具体的翻法。若不可以,请说明理由;‎ ‎10、某农场打算用‎60米长的篱笆靠墙围成5个面积大小相等的羊圈(如图所示),‎ ‎ 问:若要求每个羊圈的面积尽可能大,应为多少平方米?‎ a a a a a b b b b b ‎ ‎ ‎ ‎ ‎ ‎ ‎1.初级点拨:分解为互质的几个数的乘积,36=4×9分别考虑 ‎2.初级点拨:能被11整除的特征奇位数减偶位数的差 ‎3.初级点拨:所求的数写成‎100a+56的形式。由于‎100a+56能被56整除,所以a能被14整除,所以.应是14的倍数。‎ ‎4.初级点拨:这个十全数能被10整除,个位数必为0;能被4整除,十位数必为偶数,末两位只能是20。设这个十全数为。‎ ‎5.初级点拨:考虑到555555和999999都是7的倍数,如果原数是能被7整除,那么由□=□且7为奇数,可知□也能被7整除;‎ ‎6.初级点拨:因为91=7×13,且(7,13)=1,所以7|,13|‎ 根据一个数能被7或13整除的特征可知:‎ 原数能被7以及13整除,当且仅当-能被7以及13整除,也就是能被7以及13整除。‎ ‎7.初级点拨:设1993×a+4×b=6063,其中a、b为自然数。由于6063为奇,4×b为偶数,可知1993×a为奇数,从而判定a为奇自然数。‎ ‎8.初级点拨:设这两个完全平方数分别为A、B那么这两个完全平方数的差为54=(A+B)(A-B)。‎ ‎9.初级点拨:采用反证法,假设可以经过若干次翻动,使得所有杯子杯口全部向下,接着 计算所有杯子被翻动的次数之和。‎ ‎10.初级点拨:每个羊圈尽可能大,五个面积相同。‎5a+6b=60‎ ‎1.深度提示:因为36=4×9,所以能被4整除,从而C只可能是1,3,5,7,9。要使商最小,A、B应尽可能小,先取A=0,又1+5+6+A+B+C=12+B+C=9+3+B+C,所以3+B+C是9的倍数。‎ ‎2.深度提示:分析:中奇位数减偶位数的差为(8-2)n+8=6n+8;当n=6时,(6n+8)是11的倍数 ‎3.深度提示:而且a的数字和等于56-5-6=45。具有数字和45的最小偶数是199998,但这个数不能被7整除。接下来数字和为45的偶数是?‎ ‎4.深度提示:由于它能被11整除,必有b+d-(a+c)=10,所以b、d是9和5;a、c是3和1,这个十全数只能是4 876 391 520,4 876 351 920,4 876 193 520,4 876 153 920中的一个。‎ ‎5.深度提示:又□可以表示成+□,说明□也能被7整除,相当于将原数的前后分别去掉555555和999999后整除性不变,依次下去,得到55□99。‎ ‎6.深度提示:因为(7,10)=1,(13,10)=1,所以7|,13|也就是 ‎7|,13|,因此,用一次性质(特征),就去掉了两组;反复使用性质996次,最后转化成:原数能被7以及13整除,当且仅当能被7以及13整除。‎ ‎7.深度提示:由于1993×4=7932>6063,所以□只能取1或3。当a=1时,b=(6063—1993)÷4=4070÷4不是整数;‎ ‎8.深度提示:由于(A+B)和(A-B)的奇偶性质相同,所以(A+B)(A-B)不是4的倍数,就是奇数 ‎9.深度提示:一方面,每个杯子从杯口向上变成杯口向下,需要翻动奇数次,一共有7个杯子,7个奇数之和一定还是奇数,因此所有杯子被翻动的次数之和是奇数;另一方面,每次翻动六个杯子,因此总次数一定是六的倍数,那么就一定是偶数。‎ ‎10.深度提示:要是ab最大,则‎5a×6b=30ab最大,根据和一定,差小积大原则 ‎1.全解过程: 因为36=4×9,所以能被4整除,从而C只可能是1,3,5,7,9。要使商最小,A、B应尽可能小,先取A=0,又1+5+6+A+B+C=12+B+C=9+3+B+C,所以3+B+C是9的倍数,B=1,C=5时,取得最小值。‎ 本题商的最值分析是亮点和难点, 被合数整除的特征,应分解为互质的几个数的乘积,被4、9整除的特征是必要前提。‎ ‎2.全解过程:中奇位数减偶位数的差为(8-2)n+8=6n+8,当n=6时,(6n+8)是11的倍数;所以n的最小值是6。‎ ‎3.全解过程:1:所求的数写成‎100a+56的形式。由于‎100a+56能被56整除,所以a能被14整除,所以应是14的倍数。而且a的数字和等于56-5-6=45。具有数字和45的最小偶数是199998,但这个数不能被7整除。接下来数字和为45的偶数是289998和298998,但前者不能被7除尽,后者能被7整除,所以本题的答数就是29899856。‎ ‎4.全解过程:这个十全数能被10整除,个位数必为0;能被4整除,十位数必为偶数,末两位只能是20。设这个十全数为。由于它能被11整除,必有b+d-(a+c)=10,所以b、d是9和5;a、c是3和1,这个十全数只能是4 876 391 520,4 876 351 920,4 876 193 520,4 876 153 920中的一个。 经检验,它是4 876 391 520。‎ ‎5.全解过程:考虑到555555和999999都是7的倍数,如果原数是能被7整除,那么由□=□且7为奇数,可知□也能被7整除;又□可以表示成+□,说明□也能被7整除,相当于将原数的前后分别去掉555555和999999后整除性不变,依次下去,得到55□99。因此□44是7的倍数,□3是7的倍数,所以得□=6。‎ ‎6.全解过程:因为91=7×13,且(7,13)=1,所以7|,13|‎ 根据一个数能被7或13整除的特征可知,原数能被7以及13整除,当且仅当-能被7以及13整除,也就是能被7以及13整除。因为(7,10)=1,(13,10)=1,所以7|,13|也就是7|,13|,因此,用一次性质(特征),就去掉了两组;反复使用性质996次,最后转化成:原数能被7以及13整除,当且仅当能被7以及13整除,又∵91的倍数中小于1000的只有91×4=364的百位数字是3,∴=364。‎ ‎7.全解过程:1、设1993×a+4×b=6063,其中a、b为自然数。由于6063为奇,4×b为偶数,可知1993×a为奇数,从而判定a为奇自然数;2、由于1993×4=7972>6063,所以□只能取1或3。当a=1时,b=(6063—1993)÷4=4070÷4不是整数; 3、当a=3时,b=21合于题设等式。所以a+b=3+21= 24。‎ ‎8.全解过程:1. 分析:设这两个完全平方数分别为A、B那么这两个完全平方数的差为54=(A+B)(A-B),由于(A+B)和(A-B)的奇偶性质相同,所以(A+B)(A-B)不是4的倍数,就是奇数;54既不是4的倍数,也不是奇数,所以54不可能等于两个平方数的差,所以这样的数找不到。‎ ‎9.全解过程:采用反证法,假设可以经过若干次翻动,使得所有杯子杯口全部向下,下面计算所有杯子被翻动的次数之和:一方面,每个杯子从杯口向上变成杯口向下,需要翻动奇数次,一共有7个杯子,7个奇数之和一定还是奇数,因此所有杯子被翻动的次数之和是奇数;另一方面,每次翻动6个杯子,因此总次数一定是6的倍数,那么就一定是偶数。由于奇数不可能等于偶数,所以我们的假设错误,故不可能经过若干次翻动使得所有杯子杯口全部向下。‎ ‎10.全解过程:1.分析与解答:每个羊圈尽可能大,五个面积相同。‎5a+6b=60 要是ab最大,则‎5a·6b=30ab最大,根据和一定,差小积大原则, ∴‎5a=6b=60÷2=30,所以a=6,b=5,所以ab=6×5=30。‎ 第5讲 数论(二) 约数倍数、质数合数、分解质因数 一、知识地图 二、基础知识 ‎(一)1.质数与合数 一个数除了1和它本身,不再有别的约数,这个数叫做质数(也叫做素数)。‎ 一个数除了1和它本身,还有别的约数,这个数叫做合数。‎ 要特别记住:0和1不是质数,也不是合数。显然,在自然数范围内,最小的质数是2,2也是惟一的偶质数。最小的合数是4。我们可以按照一个数约数的个数,把自然数分成三类:0和1,质数和合数。因此,除0和1以外的自然数,不是质数就是合数。自然数的个数是无限的。早在2000多年前古希腊数学家欧几里德就证明了质数有无限多个。‎ ‎2. 质因数与分解质因数 如果一个质数是某个数的约数,那么就说这个质数是这个数的质因数。‎ 把一个合数用质因数相乘的形式表示出来,叫做分解质因数。例如,12=2×2×3。‎ 常用的是100以内的质数:2、3、5、7、11、13、17、19、23、29、31、37、41、43、47、53、59、61、67、71、73、79、83、89、97,共计25个;其中2是唯一的偶数,5是唯一的个位为5的质数,这也是多年考试的一个重点。‎ 分解质因数往往是解数论题目的突破口,因为这样可以帮助我们分析数字的特征。同学们必须熟练掌握100以内以及其他常用合数的分解质因数。部分特殊数的分解:111=3×37;1001=7×11×13;11111=41×271;10001=73×137;1995=3×5×7×19;1998=2×3×3×3×37;2007=3×3×223;2008=2×2×2×251;2007+2008=4015=5×11×73;10101=3×7×13×37。‎ 注意:从小学奥数要求看,我们对一个数分解质因数,一般根据唯一分解定理,把相同质因子写成指数形式,这对求这个数的约数个数或者所有约数的和来说,很重要。‎ 例如: 120=23×3×5,而不写成:120=2×2×2×3×5。‎ 判断一个数是否为质数的方法:根据定义如果能够找到一个小于的质数(均为整数),使得能够整除,那么就不是质数,所以我们只要拿所有小于的质数去除就可以了;但是这样的计算量很大,对于不太大的,我们可以先找一个大于且接近的平方数,再列出所有小于的质数,用这些质数去除,如没有能够除尽的那么就为质数。例如,149很接近169=13×13,根据整除的性质149不能被2、3、5、7、11整除,所以149是质数。‎ ‎(二)公约数和最大公约数 ‎1. 几个数公有的约数,叫做这几个数的公约数;其中最大的一个,叫做这几个数的最大公约数。‎ 例如:12的约数有:1,2,3,4,6,12;‎ ‎   18的约数有:1,2,3,6,9,18。‎ ‎12和18的公约数有:1,2,3,6。其中6是12和18的最大公约数,记作(12,18)=6。‎ 一般地,如果两个数a、b的最大公约数是d,我们可以简便地记作(a,b)=d。‎ ‎ 对于较小的两个数a、b可用短除法求最大公约数。‎ ‎2. 最大公约数的性质:‎ ‎(1) 两个自然数分别除以它们的最大公约数,所得的商互质。‎ 即若a=a1×(a,b),b=b1×(a,b),则(a1,b1)=1 (2) 两个数的最大公约和最小公倍的乘积等于这两个数的乘积。‎ ‎[a,b]×(a,b)=ab 还有如下推广:‎ 几个数的公约数,都是这几个数的最大公约数的约数。‎ 几个数都乘以一个自然数n,所得的积的最大公约数等于这几个数的最大公约数乘以n。‎ ‎3. 求最大公约数的方法:‎ ‎(1) 分解质因数法:先分解质因数,然后把相同的因数连乘起来。‎ ‎(2) 短除法:先找所有共有的约数,然后相乘。‎ ‎(3) 辗转相除法:每一次都用除数和余数相除,能够整除的那个余数,就是所求的最大公约数。‎ ‎ 注意:什么时候下,适宜用辗转相除法呢?要去求最大公约数的两个数比较大,或者两个数中含有大质数,我们很难通过分解质因数或者短除法解决的时候,辗转相除法就可以大展身手!‎ 用辗转相除法求两个数的最大公约数的步骤如下:‎ 先用小的一个数除大的一个数,得第一个余数;‎ 再用第一个余数除小的一个数,得第二个余数;‎ 又用第二个余数除第一个余数,得第三个余数;‎ 这样逐次用后一个数去除前一个余数,直到余数是0为止。那么,最后一个除数就是所求的最大公约数(如果最后的除数是1,那么原来的两个数是互质数)。‎ 例如:求600和1515的最大公约数。‎ 第一次:用600除1515,商2余315;‎ 第二次:用315除600,商1余285;‎ 第三次:用285除315,商1余30;‎ 第四次:用30除285,商9余15;‎ 第五次:用15除30,商2余0。‎ 所以:1515和600的最大公约数是15。‎ 辗转相除法是求两个数的最大公约数的方法。如果求几个数的最大公约数,可以先求两个数的最大公约数,再求这个最大公约数与第三个数的最大公约数。这样依次下去,直到最后一个数为止。最后所得的一个最大公约数,就是所求的几个数的最大公约数。‎ ‎4. 求一组分数的最大公约数:‎ 先将各个分数化为假分数;求出各个分数的分母的最小公倍数a;求出各个分数的分子的最大公约数b;即为所求。‎ 例如: ‎ ‎5. 求一个数所有的约数的个数:‎ 用分解质因数形式表示:N=a1p1×a2p2×a3p3×a4p4×……×anpn (a1、a2、…、an为合数N的质因数)‎ 所求的约数的个数A=(p1+1)×(p2+1)×(p3+1)×…×(pn+1);‎ 例如:504=23×32×7 ,那么它有约数:(3+1)×(2+1)×(1+1)=24个;‎ 注意:这个公式是怎么来的呢?‎ 我们如果能注意平时多问一些这样的问题,把它弄明白,奥数功力将会大增。比如说,搞清楚这里的问题,如果你2007年参加“华罗庚金杯”数学邀请赛全国总决赛,第一试的第一道题对你来说,只需要30秒解决问题!!‎ 我们不妨先来把“一个数的约数个数公式”作一个剖析:‎ 首先,大家知道一个数的约数的质因子必在这个数里出现,而且约数的质因子的指数必不大于这个数的里这一质因子的指数。例如:‎ A|23×32‎ 这里A是23×32的约数,则A中可能有质因子2或者3,没有其它的质因子,‎ ‎ ‎ 完全平方数有奇数个约数。‎ 互质数 如果两个数的最大公约数是1,那么这两个数叫做互质数。‎ ‎(三)公倍数和最小公倍数 ‎1. 几个数公有的倍数,叫做这几个数的公倍数;其中最小的一个,叫做这几个数的最小公倍数。‎ 例如:12的倍数有:12,24,36,48,60,72,84,…‎ ‎   18的倍数有:18,36,54,72,90,…‎ ‎12和18的公倍数有:36,72,…。其中36是12和18的最小公倍数,记作[12,18]=36。‎ 一般地,如果两个数a、b的最小公倍数是d,我们可以简便地记作[a,b]=d。‎ 与求最大公约数方法相似,也可用短除法求最小公倍数。‎ ‎2. 最小公倍数的有关性质:‎ ‎(1) 两个数的任意公倍数都是它们最小公倍数的倍数。‎ ‎(2) 两个数的最大公约数与最小公倍数的乘积等于这两个数的乘积。‎ ‎(3) 两个数具有倍数关系,则它们的最大公约数是其中较小的数,最小公倍数是较大的数。‎ ‎3. 求最小公倍数的方法:‎ ‎(1) 分解质因数的方法。‎ 例如:231=3×7×11,252=22×32×7,所以[231,252]= 22×32×7×11=2772;‎ ‎(2) 短除法求最小公倍数。‎ 例如: ‎ ‎ [12,18]=2×3×3×2=36;‎ ‎(3)辗转相除法求出最大公约数后,再求出最小公倍数。‎ ‎4. 求一组分数的最小公倍数方法步骤:‎ 先将各个分数化为假分数;求出各个分数分子的最小公倍数a;求出各个分数分母的最大公约数b;‎ 即为所求。‎ 例如:‎ ‎ ‎ ‎(四) 最大公约和最小公倍的关系:‎ ‎1. 利用最大公约数求最小公倍数 ‎ 由于两个数的乘积等于这两个数的最大公约数与最小公倍数的积。这就是说,求两个数的最小公倍数,可以先求出两个数的最大公约数,再用这两个数的最大公约数去除这两个数的积,所得的商就是两个数的最小公倍数。‎ ‎2. 两个数的最小公倍数和最大公约数的和或差也是最大公约的整数倍 ‎(五)完全平方数 ‎1. 平方数问题a2=22,b2=32,c2=52,其中22=4,32=9,52=25,像4、9、25‎ 这样的数,推及一般情况,我们把一个自然数平方所得到的数叫做完全平方数或叫做平方数。如。12=1,22=4,32=9,42=16,…,112=121,122=144,…其中1,4,9,16,…,121,144,…都叫做完全平方数。平方数分解质因数后,它的质因数必定会成对出现。‎ ‎ 大家知道,如果正方形的边长为a,那么正方形的面积为a;如果正方体的棱长为a,那么正方体的体积为a。这里a表示a×a,叫做a的平方。而a表示a×a×a,叫做a的三次方。同样,把a×a×a×a记作a叫做a的四次方。一般地,把n个a相乘之积a×a×…×a记作a,叫做a的n次方,这种求n个相同因数a乘积的运算叫做乘方。在a中,相同的因数a叫做底数,a的乘积中的个数n叫做指数,乘方运算的结果a叫做幂。‎ 如果一个数能写成某个整数a的平方,则叫这个数为完全平方数,简称为平方数。‎ ‎2. 完全平方数的有关性质 性质1:完全平方数的末位数字只可能是0、1、4、5、6、9。‎ 性质2:完全平方数被3、4、5、8、12、16除的余数一定是完全平方数。‎ 性质3:完全平方数的约数一定有奇数个,反之亦然。因为完全平方数的质因数分解中每个质因数出现的次数都是偶数次,所以,约数的个数是若干个奇数的乘积。‎ 性质4:如果一个完全平方数的个位是6,则十位是奇数,反之亦然。‎ 性质5:如果一个完全平方数的个位是0,则末尾连续的0的个数一定是偶数。如果一个完全平方数的个位是5,则其十位一定是2,且其百位一定是0、2、6中的一个。‎ 性质6:如果一个自然数介于两个连续的完全平方数之间,则它不是完全平方数。‎ 性质7:平方差公式:a2-b2=(a+b)(a-b)。‎ 性质8:偶数的完全平方是4的倍数,奇数的完全平方被8除一定余1,任何自然数的平方数不可能被3除余2。‎ ‎3. 完全平方数的判别方法:‎ ‎ (1)两个连续自然数的乘积不是完全平方数。‎ ‎ (2)两个连续自然数的平方数之间不再有平方数。‎ ‎ (3)一个整数如果除以4余2或者除以4余3,那么这个整数肯定不是完全平方数。‎ ‎ (4)一个整数如果除以3余2,那么这个整数肯定不是完全平方数。‎ ‎ (5)完全平方数的个位数字是奇数时,其十位上的数字必为偶数;若个位数字是6时,其十位上的数字必为奇数。‎ 平方差公式:a2-b2=(a+b)(a-b)‎ 三、经典透析 ‎【例1】(☆☆)把26、33、34、35、63、85、91、143分成若干组,要求每组中任意两个数的最大公约数是1,那么至少要分几组?(1992年小学数学奥林匹克竞赛试题)‎ ‎ ‎ 审题要点: 每组中任两个数的最大公约数都是“1”就必须保证每组中的数没有相同的质因数。‎ 详解过程:先把这八个数分解质因数 ‎ 26=2×13  33=3×11   34=2×17   35=5×7‎ ‎  63=32×7  85=5×17   91=7×13   143=11×13‎ ‎ 从中我们可以看出,每一个数都有2个不同的质因数,并且35,63,91中都有质因数7;26,91,143中都有质因数13。显然有相同质因数的三个数不能同在一组。因此至少要分3组才有可能把这两组三个数分开。‎ ‎ 现取26、33、35为一组,34、63、143为一组,85、91为一组。同组的数中,没有相同的质因数,都保证“每两个数的最大公约数都为“‎1”‎。所以只有至少分3组,才能满足题目要求。即至少要分3组。‎ 专家点评:转化为分析各数的质因数,从而找到答案。阐述理由时,一方面讲为什么不能少于3组,另一方面举例证明这3组确实分得出。这样阐述,理由充足,令人信服。‎ ‎【例2】(☆☆☆)已知自然数、满足以下两个性质:⑴ 、不互素;⑵、的最大公约数与最小公倍数之和为35。那么+的最小值是多少?‎ 审题要点:、的最大公约数一定是它们最小公倍数的约数。‎ 详解过程:因为、的最大公约数与最小公倍数的和是35,所以35是两数最大公约数的倍数。它们的最大公约数可能是5和7(因为两数不互质,所以不为1)。如果、的最大公约数是5,则、的最小公倍数是30,此时有=10、=15或=5、=30;如果、地最大公约数是7,则、的最小公倍数是28,此时有=7、=28。‎ 所以+的最小值为。‎ 专家点评:充分应用最大公约数与最小公倍数的关系,并分类讨论多种可能性,考察推理思维的周密性和严谨性,最终找到最小值。 ‎ ‎【例3】(☆☆☆☆)三个连续正整数,中间一个是完全平方数,将这样的连续三个正整数的乘积称为“美妙数”,问所有的“美妙数”的最大公约数是多少?(第九届华杯赛)‎ 审题要点:任意连续3个正整数中必然有一个是3的倍数 任何一个完全平方数要么是4的倍数要么被8除余1,所以美妙数一定也能被4整除。‎ 任何一个完全平方数的末位数字都是0,1,4,5,6,9,无论是哪一个,它们自己加上前后各一个数中必然有一个是0或5,因此美妙数一定也是5的倍数 详解过程: 这样的数有3×4×5 8×9×10 15×16×17 24×25×26……‎ 容易发现它们的最大公约数是3×4×5=60‎ 首先任意连续3个正整数中必然有一个是3的倍数,所以美妙数一定能被3整除;‎ 其次,任何一个完全平方数要么是4的倍数要么被8除余1,所以美妙数一定也能被4整除;‎ 最后,任何一个完全平方数的末位数字都是0,1,4,5,6,9,无论是哪一个,它们自己加上前后各一个数中必然有一个是0或5,因此美妙数一定也是5的倍数。‎ 综上所述,所有美妙数的最大公约数是60。‎ 专家点评:数的规律本身就是一种美……‎ ‎【例4】(☆☆☆)在一根长木棍上,有三种刻度线,第一种刻度线将木棍分成10等份,第二种刻度线把木棍分成12等份,第三种刻度线把木棍分成15等份,如果沿每条刻度线把木棍锯断,木棍总共被锯成多少段?(第二届“华罗庚金杯”赛决赛试题)‎ 审题要点:假如把木棍长看成是1个单位,那么每个等分线将表示一些分数,寻找起来就不大方便。如果把木棍看成是10、12、15的最小公倍数个单位,那么每个等分线将表示的数都是整数,而且重合位置表示的数都是等分线段长度的最小公倍数。这样一来,可利用求最小公倍数的方法计算出重合部分的个数。‎ 详解过程:由于[10,12,15]=60,先把木棍60等分,每一等分作为一个单位,则第一种刻度线相邻每两刻度间占6个单位,第二种刻度线占5个单位,第三种刻度线占4个单位,分点共有9+11+14=34个。‎ ‎ 由于[5,6]=30,故在30单位处二种刻度重合1次;‎ ‎ [4,5]=20,故在20、40单位处二种刻度重合2次;‎ ‎ [4,6]=12,故在12、24、36、48单位处二种刻度重合4次;‎ ‎ ∴共有不重合刻度34-1-2-4=27个,从而分成28段。‎ 即木棍总共锯成28段。‎ 专家点评:木棍锯成的段数,比锯的次数大1。但是锯的次数并不一定是三种刻线的总和。这是因为两种刻度线重合在一起的时候,就少锯了一次。因此,解决本题的着眼点在于计算出有多少两种刻度线或者三种刻度线重叠在一起的位置。‎ ‎【例5】(☆☆☆)从一张长‎2002毫米,宽‎847毫米的长方形纸片上,剪下一个边长尽可能大的正方形,如果剩下的部分不是正方形,那么在剩下的纸片上再剪下一个边长尽可能大的正方形。按照上述过程不断重复,最后剪得的正方形的边长是 毫米。(1991年小学数学奥林匹克决赛试题)‎ 审题要点:‎ ‎ 从图中可知,在长2002毫米、宽847毫米的长方形纸片的一端,依次剪去以宽‎847毫米为边长的正方形2个,再在剩下的长847毫米、宽308毫米的长方形纸片中,剪去以308毫米为边长的正方形2个,然后又从余下的长‎308毫米、宽‎231毫米的长方形纸片中,剪去一个边长‎231毫米的正方形,剩下的长方形纸片长恰好是宽的3倍,也就是最后可以剪得3个边长为77毫米的正方形。由于‎77毫米是‎231毫米、‎308毫米、‎847毫米的约数,所以剪得同样长的、且边长尽可能长的正方形边长就是77毫米。也即本题就是求2002和847的最大公约数。‎ 详解过程:‎ ‎ ‎ ‎ (2002,847)=77,即最后剪得的正方形边长是77毫米。‎ 说明:这种求最大公约数的方法叫辗转相除法。因为 ‎231=77×3 ‎ ‎ 308=231+77=77×3+77=77×4‎ ‎ 847=308×2+231=77×4×2+77×3=77×11‎ ‎ 2002=847×2+308=77×11×2+77×4=77×26‎ ‎ 由于26和11、11和4、4和3都是互质数。‎ ‎ 所以就保证了(2002,847)=(847,308)=(308,231)=(231,77)=77成立。‎ 专家点评: 这样逐步重复地用除法把所要求最大公约数的一对数,换成愈来愈小的另一对数。可见辗转相除法的优点在于它能在较短时间内求出任意两个数的最大公约数。‎ ‎【例6】(☆☆☆)已知一个苹果重千克,一个梨重千克,且苹果和梨的总重量相同,求最少有几个苹果和几个梨?‎ 审题要点:本题实质上就是一个求分数的最小公倍数的问题,这类问题有固定的解法,一般地,对于两个分数,它们的最小公倍数是,这里[a,c]代表a,c的最小公倍数,(b,d)代表b,d的最大公约数。‎ 详解过程:根据上述分析本题实质上是求和的最小公倍数 由上面给出的结论知道这个数是=‎ 所以苹果和梨的总重量都是20/3千克 因此苹果个数是个 ‎ 梨的个数是个 专家点评:还可以这样解:设苹果有x个,梨有y个 所以x=y,推出x︰y=25︰32‎ 故x最小是25,y最小是32。‎ x=×25=‎ 所以总重量是千克。‎ 上面两个方法,一个是直接使用分数最小公倍数的公式(请同学们自己想一想这个公式为什么成立),另一个是列方程,希望大家都能学会。‎ ‎【例7】(☆☆☆)一个数的20倍减1能被153整除,这样的自然数中最小的是多少? 。‎ ‎(祖冲之杯小学数学邀请赛)‎ 审题要点: 一个的自然数的20倍的末位数一定是0,减1后的尾数是9,最后考虑153乘几尾数是9‎ 详解过程:设这样的数为x,则20x-1=‎153a,a是整数,即20x=‎153a+1,因为20x的末位数一定是0,所以a最小取3,从而x最小是23。‎ 专家点评:本题根据个位率考虑问题是小学常用的特点。‎ ‎【例8】(☆☆☆)一个数加上10,减去10都是一个平方数,求这个数。‎ 审题要点:1.B-A=(A+B)(B-A)公式的应用 ‎2.(A+B)与(B-A)奇偶性相同 ‎3.分解 ‎4.和差求解 详解过程:B- A=20, B- A=(A+B)(B-A)=20,可见右边的数也要分成2个数的积,还得考虑同奇偶性,所以只能拆成2×10,这样A+B=10,B-A=2,所以A=4,B=6,所以这个数为A2+10=B2-10=26。‎ 专家点评: 平方差公式的应用是重点(清华附中入学测试题) ‎ ‎【例9】(☆☆☆☆☆)3个质数的平方和是39630,那它们的和是多少?‎ 审题要点:请注意平方数的个位特征,联系奇偶性。‎ 详解过程:39630=奇数+奇数+偶数,所以3个质数中必定有一个是2;由于质数中除了2、5以外,其他的质数都以1、3、7、9结尾,因此它们的平方末尾相应为1、9、9、1,相加后不可能得到6,而5的平方个位还是5,所以其中一个奇数必定为5。‎ ‎39630--=39601=40000-200-199=所以这三个质数分别为2、5、199,它们的和是2+5+199=206。‎ 专家点评:注意:39601=可以分析一下求出的思想。要求记忆20以内的平方数。‎ 四、拓展训练 ‎1、( ),要使这个乘积的最后四位数字都是0,括号里最小应填什么数?‎ ‎2、4200有多少个约数?这些约数的和是多少?‎ ‎3、23个不同的整数的和是4845,问:这23个数的最大公约数可能值达到的最大的值是多少?‎ ‎4、10个非零自然数的和是1001,则它们的最大公约数的最大值是多少?(2002我爱数学少年夏令营)‎ ‎5、有甲、乙、丙3人,甲每分钟行走‎120米,乙每分钟行走‎100米,丙每分钟行走‎70米。如果3个人同时同向,从同地出发,沿周长是‎300米的圆形跑道行走,那么多少分钟之后3人又可以相聚?‎ ‎6、现在有4个自然数,他们的和是1111。如果要求这四个数的公约数尽可能大,那么这4个数的公约数最大可能是 (1989年小学数学奥林匹克初赛试题)‎ ‎7、从1到2005的所有自然数中,有多少个数乘以72后是完全平方数?‎ ‎8、除以10所得的余数为多少?‎ ‎9、2007+2008×444444的个位数为多少?‎ ‎10、一个正整数,加上100后的结果是一个完全平方数,加上168后的结果也是一个完全平方数,那么这个正整数是多少?‎ ‎1.初级点拨:末尾产生四个0,是由于有四个5和四个偶数相乘 ‎ ‎2.初级点拨:4200= ‎ ‎3.初级点拨:设23个互不相同的数为a1k、a2k…a23k。a1k+a2k+…+a23k=4845‎ ‎4.初级点拨:设这10个非零自然数分别是a1,a2,a3,…,a10,它们的最大公约数是a,那么a1,a2,…,a10都是a的倍数 ‎5.初级点拨:不妨先看一道与之相关的题:甲、乙两人同时从A点背向出发,沿‎400米的环形跑道行走,甲每分钟走‎80米,乙每分钟走‎50米,两人至少经过多长时间才能在A点相遇。‎ 分析:甲,乙走一圈分别需要5分钟和8分钟,因此他们要是在A点再次相遇,两人都要走整圈数,所以所需的时间应是5和8的最小公倍数40分钟。‎ ‎6.初级点拨:由题中4个自然数的和是1111,可知这4个自然数的公约数也一定是1111的公约数,这样,把讨论4个数的公约数问题转化为讨论1111的约数问题。‎ 解答本题的关键是利用约数的知识和整数的性质,把讨论4个数的公约数的问题转化为讨论1111的约数的问题。这种转化的方法对解答数学问题时非常有用的。‎ ‎7.初级点拨:因为,所以要想乘以72以后是完全平方数,这个数本身应该是某个完全平方数的2倍。‎ ‎8.初级点拨:求结果除以10的余数即求其个位数。从1到2005这2005个数的个位数字是10个一循环的,而对于一个数的幂方的个位数,我们知道它总是4个一循环的,因此把每个加数的个位数按20个一组,则不同组中对应的数字应该是一样的。‎ ‎9.初级点拨:任何数乘方的尾数都是4个数一周期 ‎10.初级点拨:设加上100后为,加上168后为,那么 ‎1.深度提示:,‎ ‎2.深度提示:4200的约数个数:指数+1后再相乘 ‎ ‎ ‎3.深度提示:(a1+a2+……+a23)最小是276(即取前23个正整数)。‎ ‎4.深度提示:因此1001是a的倍数,‎ a是1001的约数,显然a不能取1001,若a取143,则a1+a2+…+a10至少是1430也不可能 ‎5.深度提示:设在x分钟后3人再次相聚,甲走了120x米,乙走了100x米,丙走了70x米,他们3人之间的路程差均是跑道长度的整数倍。或甲第一次追上乙需:300÷(120-100)=15(分);甲第一次追上丙需:300÷(120-70)=6(分);乙第一次追上丙需:300÷(100-70)=10(分);‎ ‎6.深度提示:1111=101×11,约数只能是1,11,101,1111。显然1111不符合题目要求,而101是可能的,例如取这4个自然数的为101×1,101×2,101×3,101×5。因此这4个自然数的公约数最大可能是101。‎ ‎7.深度提示:因为,‎ ‎8.深度提示:首先计算的个位数字为4。2005个加数中有100组另5个数,100组的个位数是的个位数即0,另外5个数为、、、、,它们和的个位数字是的个位数 3。‎ ‎9.深度提示:7是7、9、3、1循环,因为2007÷4=501……3,所以2007尾数是3,‎ ‎8是8、4、2、6循环,因为2008能被4整除,所以2008尾数是6,‎ ‎4是4、6、4、6循环,因为98÷4=24……2,所以444444尾数是6,‎ ‎10深度提示:即(a+b)×(b-a)=68。因为与的奇偶性相同,所以只可能是,。‎ ‎1.全解过程:末尾产生四个0,是由于有四个5和四个偶数相乘,‎ ‎,括号里的数必须有一个5和两个偶数,最小是。‎ ‎2.全解过程: 4200= ,4200共有个约数,这些约数的和是 ‎3.全解过程:设23个互不相同的数为a1k、a2k…a23k。a1k+a2k+…+a23k=4845,‎ ‎(a1+a2+……+a23)最小是276。4845=3×5×17×19=285×17,k=17‎ ‎4.全解过程:设这10个非零自然数分别是a1,a2,a3,…,a10,它们的最大公约数是a,那么a1,a2,…,a10都是a的倍数,因此1001是a的倍数,‎ a是1001的约数,显然a不能取1001,若a取143,则a1+a2+…+a10至少是1430也不可能,因此a最大是7×13=91‎ ‎5.全解过程:注意与本题的不同:甲、乙两人同时从A点背向出发,沿‎400米的环形跑道行走,甲每分钟走‎80米,乙每分钟走‎50米,两人至少经过多长时间才能在A点相遇。‎ 分析:甲,乙走一圈分别需要5分钟和8分钟,因此他们要是在A点再次相遇,两人都要走整圈数,所以所需的时间应是5和8的最小公倍数40分钟。‎ 回到本题解答:‎ 解法一: 设在x分钟后3人再次相聚,甲走了120x米,乙走了100x米,丙走了70x米,他们3人之间的路程差均是跑道长度的整数倍。即120x-100x,120x-70x,100x-70x均是300的倍数,那么300就是20x,50x,30x的公约数。有(20x,50x,30x)=300,而(20x,50x,30x)=x(20,50,30)=10x,所以x=30。即在30分钟后,3人又可以相聚。‎ 解法二: 甲第一次追上乙需:300÷(120-100)=15(分);甲第一次追上丙需:300÷(120-70)=6(分);乙第一次追上丙需:300÷(100-70)=10(分);由于[15,6,10]=30,所以3人再此相聚需要过30分钟!‎ ‎6.全解过程:由题中4个自然数的和是1111,可知这4个自然数的公约数也一定是1111的公约数,这样,把讨论4个数的公约数问题转化为讨论1111的约数问题。‎ 解答本题的关键是利用约数的知识和整数的性质,把讨论4个数的公约数的问题转化为讨论1111的约数的问题。这种转化的方法对解答数学问题时非常有用的。1111=101×11,约数只能是1,11,101,1111。显然1111不符合题目要求,而101是可能的,例如取这4个自然数的为101×1,101×2,101×3,101×5。因此这4个自然数的公约数最大可能是101。‎ 答:这4个自然数的公约数最大可能是101。‎ ‎7.全解过程:因为,所以要想乘以72以后是完全平方数,这个数本身应该是某个完全平方数的2倍。因为,所以从1到2005中,符合要求的数有31个。‎ ‎8.全解过程:求结果除以10的余数即求其个位数。从1到2005这2005个数的个位数字是10个一循环的,而对于一个数的幂方的个位数,我们知道它总是4个一循环的,因此把每个加数的个位数按20个一组,则不同组中对应的数字应该是一样的。‎ 首先计算的个位数字为4。2005个加数中有100组另5个数,100组的个位数是的个位数即0,另外5个数为、、、、‎ ‎,它们和的个位数字是的个位数 3,所以原式的个位数字是3,即除以10的余数是3‎ ‎9.全解过程:任何数乘方的尾数都是4个数一周期,7是7、9、3、1循环,因为2007÷4=501……3,所以2007尾数是3,‎ ‎8是8、4、2、6循环,因为2008能被4整除,所以2008尾数是6,‎ ‎4是4、6、4、6循环,因为98÷4=……2,所以444444尾数是6,‎ 所以原式的尾数为3+6×6=39,尾数是9。‎ ‎10.全解过程:设加上100后为,加上168后为,那么,即 。因为与的奇偶性相同,所以只可能是,,解得,,因此原数是。‎ 第6讲 数论(三) 带余除法、同余性质、中国剩余定理 一、 知识地图 二、基础知识 ‎(一)余数问题 ‎ 在整数的除法中,只有能整除和不能整除两种情况。当不能整除时,就产生余数,余数问题在小学数学中非常重要。余数问题和整除性问题是有密切关系的,因为只要我们去掉余数那么就能和整除性问题联系在一起了。‎ ‎1. 带余除法 ‎1定义的引入:‎ 带余除法:一般地,如果a是整数,b是整数(b≠0),那么一定有另外两个整数q和r,‎ ‎0≤r<b,使得a=b×q + r 当r=0时,我们称a能被b整除。‎ 当r≠0时,我们称a不能被b整除,r为a除以b的余数,q为a除以b的不完全商(亦简称为商)。用带余数的除式又可以表示为a÷b=q……r, 0≤r<b 例如:给出整数13 ,整数5,那么就存在另外两个数2和3,使得13=5×2+3‎ ‎ 其实也就是表达了13÷5=2…3,这么一个简单的意思。‎ ‎2.和余数相关的一些性质 余数有如下一些重要性质(a,b,c均为自然数)‎ ‎(1)余数小于除数。‎ ‎(2)被除数=除数×商+余数 ‎ 除数=(被除数-余数)÷商;  ‎ ‎ 商=(被除数-余数)÷除数。‎ 这条性质,要与整除性联系,从被除数中减掉余数,那么所得到的差就能够被除数整除了。因为在一些题目中因为余数的存在,不便于我们计算,去掉余数,回到比较熟悉的整除性问题,那么问题就会变得简单了。‎ ‎(3)如果a,b除以c的余数相同,那么a与b的差能被c整除。‎ ‎ 例如,17与11除以3的余数都是2,所以17-11能被3整除。‎ 此时可以把a,b 都分别表示成c的整数倍加余数的形式,既然余数相同,那么做差的话就把彼此的余数减掉了,c的整数倍之间的差剩下的当然也是c的整数倍。然后再举几个例子来验证一下。比如9与34 除以5 的余数都是4,他们的差25就能被5整除。‎ ‎(4)a与b的和除以c的余数,等于a,b分别除以c的余数之和(或这个和除以c的余数)。 例如,23,16除以5的余数分别是3和1,所以(23+16)除以5的余数等于3+1=4。 ‎ ‎ 注意:当余数之和大于除数时,所求余数等于余数之和再除以c的余数。例如,23,19除以5的余数分别是3和4,所以(23+19)除以5的余数等于(3+4)除以5的余数。‎ ‎(5)a与b的乘积除以c的余数,等于a,b分别除以c的余数之积(或这个积除以c的余数)。例如,23,16除以5的余数分别是3和1,所以(23×16)除以5的余数等于3×1=3。 注意:当余数之积大于除数时,所求余数等于余数之积再除以c的余数。例如,23,19除以5的余数分别是3和4,所以(23×19)除以5的余数等于(3×4)除以5的余数。‎ ‎ 性质(4)(5)都可以推广到多个自然数的情形。‎ 对于性质(4)和性质(5),前半部分的解释类似性质(3)的解释。这儿关键要强调的是两个注意的部分,也就是一定要看看所得的余数的和或积是否大于除数,不能和余数要小于除数这条性质产生矛盾。另外关于性质(4)(5)的推广,我们可以举几个数多一些例子来解释,例如针对性(4):求(12+31+58+41)除以5 的余数,那么就可以分别求出这几个数除以5 的余数再求和。那就是2+1+3+1=7 ,而7比5大,所以所求的余数是2。‎ 性质(5)可以推广到求几个多位数的乘积除以某个数的余数,以及一个数的很多次幂除以一个数的余数等等。结合具体的例题我们在扩展,这两条性质是非常值得探讨的。‎ ‎2. 同余定义:若两个整数a,b被自然数m除有相同的余数,那么称a,b对于模m同余,用式子表示为 ‎ a≡b(mod m) (×)‎ ‎ 同余式(×)意味着(我们假设a≥b)‎ a-b=mk,k是整数,即m|(a-b)‎ 于是我们有下面的 若两个数a,b除以同一个数c得到的余数相同,则a,b的差一定能被c整除 ‎  例如:①15≡365(mod7),因为365-15=350=7×50。‎ ‎  ②56≡20(mod9),因为56-20=36=9×4。‎ ‎  ③90≡0(mod10),因为90-0=90=10×9。‎ ‎  由例③我们得到启发,a可被m整除,可用同余式表示为:a≡0(modm)。‎ ‎  例如,表示a是一个偶数,可以写 ‎  a≡0(mod 2)‎ ‎  表示b是一个奇数,可以写 ‎  b≡1(mod 2)‎ ‎  补充定义:若m(a-b),就说a、b对模m不同余,用式子表示是:‎ ‎  ab(modm)‎ ‎  我们书写同余式的方式,使我们想起等式,而事实上,同余式与等式在其性质上相似。同余式有如下一些性质(其中a、b、c、d是整数,而m是自然数)。‎ ‎  性质1:a≡a(mod m),(反身性)‎ ‎  这个性质很显然。因为a-a=0=m·0。‎ ‎  性质2:若a≡b(mod m),那么b≡a(mod m),(对称性)。‎ ‎  性质3:若a≡b(mod m),b≡c(mod m),那么a≡c(mod m),(传递性)。‎ ‎  性质4:若a≡b(mod m),c≡d(mod m),那么a±c≡b±d(mod m),(可加减性)。‎ ‎  性质5:若a≡b(mod m),c≡d(mod m),那么ac≡bd(mod m)(可乘性)。‎ ‎  性质6:若a≡b(mod m),那么an≡bn(mod m),(其中n为自然数)。‎ ‎  性质7:若ac≡bc(mod m),(c,m)=1,那么a≡b(mod m),(记号(c,m)表示c与m的最大公约数)。‎ ‎  注意同余式性质7的条件(c,m)=1,否则像普通等式一样,两边约去,就是错的。‎ ‎  例如6≡10(mod 4),而35(mod 4),因为(2,4)≠1。‎ ‎  请你自己举些例子验证上面的性质。‎ 同余是研究自然数的性质的基本概念,是可除性的符号语言。‎ ‎3. 中国剩余定理 ‎ 趣题:‎ 中国有一本数学名著《孙子算经》也有类似的问题:“今有物,不知其数,三三数之,剩二,五五数之,剩三,七七数之,剩二,问物几何?”‎ 答曰:“二十三”‎ ‎ 此类题又称为“物不知其数”类型,又叫“韩信点兵”,“鬼谷算”,“隔墙算”,“剪管术”,“神奇妙算”等等,题目与解法都载于我国古代重要的数学著作《孙子算经》中。算法口诀诗则载于明朝程大位的《算法统宗》。宋朝的数学家秦九韶把这个问题推广,并把解法称之为“大衍求一术”,这个解法传到西方后,被称为“孙子定理”或“中国剩余定理”。‎ 口诀诗:‎ ‎ 程大位的算法口诀诗,就是解答此类问题的金钥匙,它被世界各国称为中国剩余定理或孙子定理,是我国古代数学的一项辉煌成果。我们就从上述四句诗中来找答案:‎ ‎ “三人同行七十稀”,把除以3所得的余数用70乘。‎ ‎ “五树梅花廿一枝”,把除以5所得的余数用21乘。‎ ‎ “七子团圆正半月”,把除以7所得的余数用15乘。‎ ‎ “除百零五便得知”,把上述三个积加起来,减去105的倍数,所得的差即为所求。‎ 列式为:2×70+3×21+2×15=233,233-105×2=23。‎ 释疑:‎ ‎ 为什么70、21、15、105有如此神奇作用?70、21、15、105是从何而来?‎ 先看70、21、15、105的性质,70除以3余1,被5、7整除,所以‎70a除以3余a,也被5、7整除;‎ ‎ (强调:这里为什么不是其它两个数的积,而是5×7×2=70,因为70除以3余1)‎ ‎21余以5余1,被3、7整除,所以21b除以5余b,也被3、7整除;15除以7余1,被3、5整除,所以‎15c除以7余c,被3、5整除。而105则是3、5、7的最小公倍数。‎ 总之来说:‎70a+21b+‎15c是被3除余a,被5除余b,被7除余c的数,这个数如果大了,还要减去它们的公倍数。‎ 推广:‎ 注意中国剩余定理不一定只能解决3、5、7的“物不知其数”问题。‎ 对于其他的情况,又要乘以哪些类似70、21、15这样的常数,可以依据“余1”这个关键,推算出来:比如3、5、11。‎ 三人同行:5×11=55, 55≡1(mod3);正好合适!‎ 五树梅花:3×11=33, 33≡3(mod5);不合适!‎ ‎ 3×2=6≡1(mod5),33×2=66。‎ 十一国庆:3×5=15, 15≡4(mod11);不合适!‎ ‎ 4×3=12≡1(mod11),15×3=45。‎ 应用:‎ 一个数被3除余2,被5除余1,被11除余3,这个数最小是多少?‎ 分析:(2×55+1×66+3×45)≡146(mod3×5×11=mod165)‎ 方法:‎ 现在我们从中提炼出一种方法,不妨称为“逐级满足法”,请大家仔细体会。‎ 先把题目改动一下:今有物不知数,五五数之余三 ,七七数之余一,九九数之余四,问物几何?‎ ‎(1)先找除以9余4的数:4这个数正好满足“除以9余‎4”‎的特征;‎ ‎(2)再看既满足前面一级的条件,又满足“除以7余‎1”‎的数有:‎ ‎4+9n≡1(mod7)‎ 因为4≡4(mod7),所以9n≡4(mod7);‎ 而9≡2(mod7);‎ 所以要有2n≡4(mod7),‎ 所以n最小是2,‎ ‎4+9×2=22‎ 所以满足第二级“除以7余‎1”‎的数最小为22。‎ ‎(请注意这个数已经满足前面两个条件)‎ ‎(3)再看满足第三个条件“除以5余‎3”‎的数为:‎ ‎ 22+[7,9]m≡3(mod5),‎ ‎ 因为22≡2(mod5),所以要‎63m≡1(mod5);‎ ‎ 因为63≡3(mod5),所以要‎3m≡1(mod5);‎ ‎ 因为3×2≡1(mod5),所以m最小值为2;‎ ‎ 所以22+[7,9]m=22+63×2=148。‎ 总结:‎ 我们在解决类似“物不知其数”题,也就是出现一个数N除以A余a,除以B余b,除以C余c这一类问题的时候,我们有“四大绝招”把余数问题转化为“整除问题”:‎ 绝招一:减同余。‎ 例如a=b,则有A|N-a,且B|N-a,N-a=[A、B]n,而N的最小值就是N=[A,B]+a;‎ 绝招二:加同补。‎ ‎ 例如:A-a=B-b=d;则有N+d=[A,B]n,而N的最小值是N=[A,B]-d;‎ 绝招三:中国剩余定理。‎ 绝招四:逐级满足法。‎ ‎(二)进制与位值原则 整数的几种表示方法 ‎ 记数法是数的记录和表示方法,不同民族、不同历史时期,有着不同的记数方法。在古代人类就用十个指头来计数,从而创造了1、2、3、4、5、6、7、8、9、10这十个数字。后来,又把十个物体放在一起,作为一个较大的单位,在单位数满十个时,又将这十个单位 放在一起作为一个更大的单位……这样就产生了百、千、万……这就是我们目前常用的记数法——十进制记数法(又称十进位值制记数法)。‎ ‎ 同一个数字,由于它在所写的数里的位置不同,所表示的数也不同。也就是说,每一个数字除了本身的值以外,还有一个“位置值”。例如“‎2”‎,写在个位上,就表示2个一,写在十位上,就表示2个十,写在百位上,就表示2个百。这种把数字和数位结合起来表示数的原则,称为写数的位值原则。‎ ‎ 我们通常使用的是十进制计数法,其特点是“满十进一”。就是说,每10个某一单位就组成与它相邻的较高的一个单位。如:10个1,叫做“十”‎,10个十叫做“百”,10个百叫做“千”,等等。写数时,从右端起,第一位是个位,第二位是十位,第三位是百位,第四位是千位,第五位是万位……它们的计数单位分别是1、10、100、1000、10000……(用乘方的形式来表示,计数单位依次为1、10、10、10、10……)这样,用阿拉伯数字和位值原则,可以表示出一切整数。例如:‎ 等号右边的式子叫做十进制数的展开式。‎ ‎ 在位值制记数法中,除了用十进制外,也可以用其他大于1的自然数的进位制。例如,计算机科学中常用的有二进制,八进制等。过去十六两是一斤(十六进制)。常用的还有五进制和十二进制。‎ ‎ 二进制 ‎ 随着科技的进步和发展,早已进入了计算机时代的各行各业,普遍利用计算机提高劳动效率。计算机采用二进制计数法。通俗地说:是“满二进一”。因此,二进制只用两个数字0和1。二进制的计数单位分别是1、2、2、2、2……二进制数也可以写成展开 式的形式。例如,100110在二进制的表示法为 ‎(100110)2=1×25+0×24+0×23+1×22+1×21+0×20‎ 注意:数学上规定20=1。‎ 下表是十进制与二进制、三进制 、八进制、十六进制的位值(计数单位)对比图:‎ 十进制 ‎…‎ ‎105‎ ‎104‎ ‎103‎ ‎102‎ ‎101‎ ‎100‎ 二进制 ‎…‎ ‎25‎ ‎24‎ ‎23‎ ‎22‎ ‎21‎ ‎20‎ 三进制 ‎…‎ ‎35‎ ‎34‎ ‎33‎ ‎32‎ ‎31‎ ‎30‎ 八进制 ‎…‎ ‎85‎ ‎84‎ ‎83‎ ‎82‎ ‎81‎ ‎80‎ 十六进制 ‎…‎ ‎165‎ ‎164‎ ‎163‎ ‎162‎ ‎161‎ ‎160‎ n进制的运算法则是“逢n进一”、“借一当n”。n进制的四则混合运算和十进制一样:先乘除,后加减;同级运算,先左后右;有括号,先算括号里面的。‎ 三、经典透析 ‎【例1】(☆☆)用一个自然数去除另一个整数,商40,余数是16。被除数、除数、商数与余数的和是933,求被除数和除数各是多少?‎ ‎[审题要点] 被除数=除数×商+余数 可以按这个思路来解:从被除数中减掉余数后,被除数就是除数的40倍了,所以可以得到:877-16=(40+1)×除数 ‎[详解过程] ∵被除数=除数×商+余数,‎ ‎  即被除数=除数×40+16。‎ ‎  由题意可知:被除数+除数=933-40-16=877,‎ ‎  ∴(除数×40+16)+除数=877,‎ ‎  ∴除数×41=877-16,‎ ‎  除数=861÷41,‎ ‎  除数=21,‎ ‎  ∴被除数=21×40+16=856。‎ 专家点评:此题就是考察对于性质(2)对于“被除数=除数×商+余数 ” 的理解和应用。转化成整数倍问题后,可以帮助理解相关的性质。(温州市数学全能竞赛)‎ ‎ ‎ ‎【例2】(☆☆☆)已知2008被一些自然数去除,得到的余数都是10。这些自然数共有几个? ‎ 审题要点: 余数问题与约数相结合 详解过程:2008被这样的自然数除余数是10,那么1998就是这些自然数的倍数,换句话说 我们要求1998的约数有几个,但注意到除数比余数大,所以我们要求的是1998的约数中那些大于10的,枚举显然不可取,我们考虑用约数个数公式 ‎1998=2×33×37,d(1998)=(1+1)×(3+1)(1+1)=16‎ 其中小于10的约数有1,2,3,6,9‎ 去掉它们还有11个 因此这样的自然数共有11个 专家点评:注意到除数比余数大,这是第15届迎春杯的试题 ‎ ‎ ‎【例3】(☆☆☆)甲、乙、丙、丁四个旅行团分别有游客69人、85人、93人、97人。现在要把这四个旅行团分别进行分组,使每组是A名游客,以便乘车前往参观游览。已知甲、乙、丙三个旅行团分成每组A人的若干组后,所剩的人数都相同,问丁旅行团分成每组A人的若干组后还剩几人?‎ 审题要点:‎ ‎ 要与整除性联系,从被除数中减掉余数,那么所得到的差就能够被除数整除了。因为在一些题目中因为余数的存在,不便于我们计算,去掉余数,回到比较熟悉的整除性问题,那么问题就会变得简单了。‎ 详解过程:由85-69=16,93-85=8,推出A=8或4或2,所以丁团分成每组A人的若干组后还剩1人。‎ 专家点评:如果a,b除以c的余数相同,那么a与b的差能被c整除。‎ 此题是第四届小数报数学竞赛决赛试题 ‎ ‎ ‎【例4】(☆☆☆)一个小于200的自然数,被7除余2,被8除余3,被9除余1,这个数是多少?‎ 审题要点: 该数加上5以后可被7和8整除,也就是56的倍数 详解过程: 注意到7-2=8-3=5‎ 也就是说该数加上5以后可被7和8整除,也就是56的倍数 因此这个数只可能是56-5, 56×2-5, 56×3-5‎ 经检验发现只有56×3-5=163被9除余1符合要求,因此该数为163‎ 专家点评:余数不同,但余数的补数相同,这类问题经常出现 ‎【例5】(☆☆☆)满足被5除余3,被6除余1,被7除余2的最小自然数是多少?‎ 审题要点: 被6除余1,被7除余2,余数不同,但是余数的补数相同,所以一定是42的倍数减5,尾数是3或8,而8又不可能,然后再考虑尾数42的4倍符合 详解过程: (法1)在不大于[5,6,7]=210的自然数中必定有一个符合条件,且为符合条件的最小自然数。 该数除以5余3,因此个位数字只能是3或8。而它除以6余1,因此它是奇数,则个位数字只能是3。因为它除以7余2,所以可以寻找被7整除且个位数字为1的数。这些数有21,91,161,将它们都加上2,看是否能符合题目条件。此时可知,163是满足题意的最小自然数。‎ ‎(法2) 被6除余1,被7除余2,余数不同,但是余数的补数相同,所以一定是42的倍数减5,尾数是3或8,而8又不可能,然后再考虑尾数42的4倍符合 专家点评:余数补数相同结合尾数问题分析 ‎ ‎ ‎【例6】(☆☆☆☆)有一个自然数,用它分别去除63、90、130都有余数,三个余数的和为25。这三个余数中最大的一个是多少?‎ 审题要点: a与b的和除以c的余数,等于a,b分别除以c的余数之和(或这个和除以c的余数)。 ‎ 详解过程: 63+90+130-25应该能被这个自然数整除,即这个自然数是258的约数。而258=2×3×43。由于6能整除90,而且这个自然数不能大于63。则这个自然数为43。可见余数最大的是63的余数:20。‎ 专家点评:余数性质的典型应用 注意:当余数之和大于除数时,所求余数等于余数之和再除以c的余数。‎ ‎【例7】(☆☆☆)有一串数:1、3、8、22、60、164、448,…其中第一个数是1,第二个数是3,从第三个数起,每个数恰好是前两个数之和的2倍。那么在这串数中,第2000个数除以9的余数是 ‎ 审题要点: a与b的和除以c的余数,等于a,b分别除以c的余数之和(或这个和除以c的余数)。 当余数之和大于除数时,所求余数等于余数之和再除以c的余数。‎ 详解过程: 找规律根据递推关系把这串数除以9的余数列出来如下 ‎1、3、8、4、6、2、7、0、5、1、3,…。‎ 发现恰好每9个一循环 ‎2000被9除余数是2,所以第2000个和第2个是一样的,除以9的余数是3‎ 专家点评:余数性质的应用结合周期问题 ‎【例8】(☆☆☆☆)求乘积418×814×1616除以13所得的余数。‎ 审题要点: 同余的性质5的应用:若a≡b(mod m),c≡d(mod m),那么ac≡bd(mod m)(可乘性)。‎ 详解过程: ∵418≡2(mod13),‎ ‎  814≡8(mod13),1616≡4(mod13),‎ ‎  ∴ 根据同余的性质5可得:‎ ‎  418×814×1616≡2×8×4≡64≡12(mod13)。‎ ‎  答:乘积418×814×1616除以13余数是12。‎ 专家点评:若先求乘积,再求余数,计算量太大。利用同余的性质可以使“大数化小”,减少计算量。‎ ‎ ‎ ‎【例9】(☆☆☆☆)求14389除以7的余数。‎ 审题要点: 同余的性质的应用 详解过程: ‎ 解法一: ∵143≡3(mod7)‎ ‎  ∴14389≡389(mod 7)‎ ‎  ∵89=64+16+8+1‎ ‎  而32≡2(mod 7),‎ ‎  34≡4(mod7),‎ ‎  38≡16≡2(mod 7),‎ ‎  316≡4(mod 7),‎ ‎  332≡16≡2(mod 7),‎ ‎  364≡4(mod 7)。‎ ‎  ∵389≡364·316·38·3≡4×4×2×3≡5(mod 7),‎ ‎  ∴14389≡5(mod 7)。‎ ‎  答:14389除以7的余数是5。‎ ‎  解法二:证得14389≡389(mod 7)后,‎ ‎  36≡32×34≡2×4≡1(mod 7),‎ ‎  ∴384≡(36)14≡1(mod 7)。‎ ‎  ∴389≡384·34·3≡1×4×3≡5(mod 7)。‎ ‎∴14389≡5(mod 7)。‎ 专家点评:同余的性质能使“大数化小”,凡求大数的余数问题首先考虑用同余的性质化大为小。这道题先把底数在同余意义下变小,然后从低次幂入手,重复平方,找找有什么规律。‎ ‎【例10】(☆☆☆)‎ 审题要点: 位值原理 详解过程: 原式:‎1111a+111b+‎11c+d=1370,所以a=1, 则111b+‎11c+d=1370-1111=259 ,‎ 推知b=2;进而推知c=3,d=4所以=1234。‎ 专家点评:根据位值原理分析问题 四、拓展训练 ‎1.被除数、除数、商与余数之和是2143,已知商是33,余数是52,求被除数和除数。‎ ‎2. 判定288和214对于模37是否同余?‎ ‎3. 甲、乙两个代表团乘车去参观,每辆车可乘36人。两代表团坐满若干辆车后,甲代表团余下的11人与乙代表团余下的成员正好又坐满一辆车。参观完,甲代表团的每个成员与乙代表团的每个成员两两合拍一张照片留念。如果每个胶卷可拍36张照片,那么拍完最后一张照片后,相机里的胶卷还可拍几张照片?‎ ‎4(清华附中入学测试题)一个大于1的数去除300,245,210时,得余数为a,a+2,a+5,则这个自然数是多少?‎ ‎5甲、乙、丙三数分别为603,939,393。某数A除甲数所得余数是A除乙数所得余数的2倍,A除乙数所得余数是A除丙数所得余数的2倍。求A等于多少?‎ ‎6.一个大于2的数,除以3余1,除以5余3,除以7余5,问满足条件的最小自然数是____。‎ ‎7. 著名的斐波那契数列是这样的“1、1、2、3、5、8、13、21……”这串数列当中第2008个数除以3所得的余数为多少?‎ ‎8. 已知三个连续自然数,它们都小于2002,其中最小的一个自然数能被13整除,中间的一个自然数能被15整除,最大的一个自然数能被17整除。那么,最小的一个自然数是多少?(第18届迎春杯)‎ ‎9. 有一个三位数,把它的个位数码移到百位上,百位和十位上的数码相应后移一位成了一个新的三位数,原三位数的2倍恰好比新三位数大1,求原来的三位数。‎ ‎10. 有一个三位数是8的倍数,把它的各位数字的顺序颠倒过来所得到的新三位数与原三位数的和恰好是1111。那么原来的三位数是多少?‎ ‎1.初级点拨:分析:此题就是考察对于性质(2)对于“被除数=除数×商+余数 ” 的理解和应用。‎ ‎2.初级点拨:性质(4):若a≡b(mod m),c≡d(mod m),那么a±c≡b±d(mod m),(可加减性)。‎ ‎3.初级点拨:(南京市第二届兴趣杯少年数学邀请赛试题):这道题本身并不是很复杂,关键是要帮助学生去理解题意,把题意转化成常规的余数问题就很简单了。可以这么说:甲数除以36 余11,乙数除以36余25,问甲乙两数的积除以36 余几?用到的知识点还是性质(5)。‎ ‎4.初级点拨:观察余数,发现余数多2和5,做转化余数相同的进行分析 ‎5.初级点拨:设这个数为M,则603÷M=A1…r1‎ ‎ 939÷M=A2…r2‎ ‎ 393÷M=A3…r3 ‎ ‎ r1=2×r2,r2=2×r3,要消去余数r1,r2,r3,我们只能先把余数处理成相同的,再两数相减。‎ ‎6.初级点拨:余数的补数相同 ‎7.初级点拨:将斐波那契数列转换为被3除所得余数的数列 ‎1、1、2、0、2、2、1、0、1、1、2、0……‎ 第九项和第十项连续两个是1,与第一项和第二项的值相同且位置连续,所以斐波那契数列被3除的余数每8个一个周期,‎ ‎8.初级点拨:结合连续自然数相差1的情况 ‎9.初级点拨:设原三位数是,则2(‎100a+10b+c)-(‎100c+‎10a+b)=1‎ ‎10.初级点拨: 位值原理 ‎1.深度提示:法(一)解:通过对题意的理解我们可以得到:‎ ‎ 被除数=除数×商+余数 ‎ =除数×33+52,‎ 又有:被除数=2143-除数-商-余数 ‎ =2143-除数-33-52‎ ‎ =2058-除数,‎ 所以: 除数×33+52=2058-除数 ‎2.深度提示:∵288-214=74=37×2。‎ ‎3.深度提示:因为甲数除以36余11,乙数除以36余25,所以“甲数×乙数”除以36的余数等于11×25除以36的余数。(11×25)÷36=7……23,即拍完了7个卷后最后一个胶卷还要拍23张,还可拍36-23=13(张)。‎ ‎4.深度提示:观察余数,发现余数多2和5,一个大于1的数去除300,243,205时余数相同,这个自然数是多少?‎ ‎5.深度提示:这样我们先把第二个式子乘以2,这样被除数和余数都扩大2倍,同理,第三个式子乘以4。‎ 这样我们可以得到下面的式子: 603÷M=A1…r1‎ ‎ ( 939×2)÷M=‎2A2…(r2×2)‎ ‎ (393×4)÷M=‎4A3…(r3×4)‎ 这样余数就处理成相同的。最后两两相减消去余数,意味着能被M整除。‎ ‎939×2-603=1275,393×4-603=969‎ ‎(1275,969)=51=3×17。‎ ‎603,939,393这三个数有公约数3。‎ ‎6..深度提示:我们发现差都相等,所以把他们都处理成都缺2能被整除 ‎7.深度提示:1、1、2、0、2、2、1、0、1、1、2、0……‎ 第九项和第十项连续两个是1,与第一项和第二项的值相同且位置连续,所以斐波那契数列被3除的余数每8个一个周期,‎ ‎8..深度提示:105+195n=(102+187n)+(3+8n),所以8n+3被17除余16即可,即8n被17除余13,n=8即可满足要求。‎ 因此a=105+195×8=1665,所以最小一个自然数是1664。‎ ‎9.深度提示:化简19(‎10a+b)=‎98c+1,经试验,只有c=6时等式右端能被19整除 ‎10.深度提示: 设原三位数为,则新三位数为,根据位值原理有, ,‎ ‎1.全解过程:(西城区四中小升初选拔试题)‎ ‎(法1)解:通过对题意的理解我们可以得到:‎ ‎ 被除数=除数×商+余数 ‎ =除数×33+52,‎ 又有:被除数=2143-除数-商-余数 ‎ =2143-除数-33-52‎ ‎ =2058-除数,‎ 所以: 除数×33+52=2058-除数,‎ 则除数=(2058-52)÷34=59,被除数=2058-59=1999。‎ ‎(法2)此题也可以按这个思路来解:从被除数中减掉余数52后,被除数就是除数的33倍了,所以可以得到:2143-33-52-52=(33+1)×除数 ‎ 解得除数=59 被除数=33×59+52=1999‎ 转化成整数倍问题后,可以帮助理解相关的性质。‎ ‎2.全解过程:∵288-214=74=37×2。‎ ‎∴288≡214(mod37)。‎ ‎3.全解过程:甲代表团坐满若干辆车后余11人,说明甲代表团的人数(简称甲数)除以36余11;两代表团余下的人正好坐满一辆车,说明乙代表团余36-11=25(人),即乙代表团的人数(简称乙数)除以36余25;甲代表团的每个成员与乙代表团的每个成员两两合拍一张照片,共要拍“甲数×乙数”张照片,因为每个胶卷拍36张,所以最后一个胶卷拍的张数,等于“甲数×乙数”除以36的余数。因为甲数除以36余11,乙数除以36余25,所以“甲数×乙数”除以36的余数等于11×25除以36的余数。(11×25)÷36=7……23,即拍完了7个卷后最后一个胶卷还要拍23张,还可拍36-23=13(张)。‎ ‎4.全解过程:300-243=57=19×3 243-205=38=19×2,所以这个自然数为19‎ ‎5.全解过程:设这个数为M,则603÷M=A1…r1‎ ‎ 939÷M=A2…r2‎ ‎ 393÷M=A3…r3 ‎ ‎ r1=2×r2,r2=2×r3,要消去余数r1,r2,r3,我们只能先把把余数处理成相同的,再两数相减。‎ 这样我们先把第二个式子乘以2,这样被除数和余数都扩大2倍,同理,第三个式子乘以4。‎ 这样我们可以得到下面的式子: 603÷M=A1…r1‎ ‎ ( 939×2)÷M=‎2A2…(r2×2)‎ ‎ (393×4)÷M=‎4A3…(r3×4)‎ 这样余数就处理成相同的。最后两两相减消去余数,意味着能被M整除。‎ ‎939×2-603=1275,393×4-603=969‎ ‎(1275,969)=51=3×17。‎ ‎603,939,393这三个数有公约数3。‎ ‎51÷3=17。则A等于17。‎ ‎6.全解过程:我们发现差都相等,所以把他们都处理成都缺2能被整除,这样得[3、5、7]-2=103。‎ ‎7.全解过程:将斐波那契数列转换为被3除所得余数的数列:‎ ‎1、1、2、0、2、2、1、0、1、1、2、0……‎ 第九项和第十项连续两个是1,与第一项和第二项的值相同且位置连续,所以斐波那契数列被3除的余数每8个一个周期。2008÷8=251,所以第2008项被3除所得的余数同第一周期第8个数为0。‎ ‎8.全解过程:假设中间一个是a,那么a被13除余1,被15整除,被17除余16。先满足前2个条件,被15整除且被13除余1。‎ 因为15除以13余2,2×7=14除以13余1,所以该数为15×7=105。‎ 因为加上13×15=195仍然符合前2条,因此该数形如105+195n,我们只需要满足该数被17除余16即可。‎ ‎105+195n=(102+187n)+(3+8n),所以8n+3被17除余16即可,即8n被17除余13,n=8即可满足要求。‎ 因此a=105+195×8=1665,所以最小一个自然数是1664。(首师大实验班选拔考试题) ‎ ‎9.全解过程:设原三位数是,则2(‎100a+10b+c)-(‎100c+‎10a+b)=1,化简19(‎10a+b)=‎98c+1,经试验,只有c=6时等式右端能被19整除,由此推出c=6,a=3,b=1。原数为316。‎ ‎10.全解过程: 设原三位数为,则新三位数为,根据位值原理有, ,又,且为一位数,所以,;原数为8的倍数,则,所以原来的三位数是704。‎ 第7讲 几何(一) 平面图形 一、 知识地图 ‎ ‎ 二、 基础知识 小学奥数的平面几何问题,是以等积变形为主导思想,结合五大模型的变化应用,交织而成。攻克奥数平面几何,一定要从等积变形开始。‎ ‎1、等积变形。‎ 等积变形,它的特点是利用面积相等而进行相互转换,面积相等的两个图形我们就称之为等积形。我们所研究的等积变形,更多的是三角形的等积变形,三角形等积变形的中心思想是等底等高,因为三角形的面积=底×高÷2,所以说等底等高的两个三角形面积相等。另外,等底等高的平行四边形、梯形(梯形等底应理解为两底和相等)的面积也相等。在实际中,我们经常用到的与等积变形相关的性质主要有以下几点:‎ ‎﹙1﹚直线平行于,可知;‎ 反之,如果,则可知直线平行于。‎ ‎(因为平行线间的距离是处处相等的哦!,聪明的你想到了吗?)‎ ‎﹙2﹚两个三角形高相等,面积比等于它们的底之比;‎ 两个三角形底相等,面积比等于它们的高之比;‎ 特别地,我们有 等腰三角形底边上的高线平分三角形面积 三角形一边上的中线平分这个三角形的面积。‎ 平行四边形的对角线平分它的面积 ‎﹙3﹚共边定理:若△和△的公共边所在直线与直线交于,则;‎ ‎﹙4﹚共角定理:在△和△中,若或,则。‎ ‎﹙5﹚过矩形内部的一点引两条直线分别与两组边平行,所分得的四个小矩形,其面积满足:。‎ ‎ ‎ ‎﹙6﹚E为矩形ABCD内部的任意一点,则 ‎;当E落在矩形的某条边上时,也成立。‎ 特别地,(5)(6)两条性质对于平行四边形同样成立。‎ ‎2、五大模型。‎ 我们把学习中经常遇到的问题归纳为五个基本的模型,总的来说,这五个基本模型都是用来解决三角形边与面积之间关系互相转换的问题。让我们一起来感受一下模型的魅力吧!‎ 模型一:在同一三角形中,相应面积与底成正比关系:‎ ‎ 即:两个三角形高相等,面积之比等于对应底边之比。‎ ‎ 或:两个三角形底相等,面积之比等于对应的高之比。‎ ‎ ‎ ‎ S1︰S2 =a︰b ;‎ 拓展: 等分点结论(“鸟头定理”)‎ ‎ 如图,三角形AED占三角形ABC面积的×= ‎ 鸟头定理是对模型一的一个拓展,有兴趣的话,你可以试着证明一下哦!‎ 模型二:任意四边形中的比例关系 (“蝴蝶定理”)‎ ‎①S1︰S2=S4︰S3 或者S1×S3=S2×S4 ‎ ‎②AO︰OC=(S1+S2)︰(S4+S3)‎ 蝴蝶定理为我们提供了解决不规则四边形的面积问题的一个途径。构造模型,一方面我们可以使不规则四边形的面积关系与四边形内的三角形相联系,另一方面,我们也可以得到与面积对应的对角线的比例关系。‎ 模型三:梯形中比例关系(“梯形蝴蝶定理”)‎ ‎①S1︰S3=a2︰b2‎ ‎②S1︰S3︰S2︰S4= a2︰b2︰ab︰ab ; ‎ ‎③S的对应份数为(a+b)2‎ 梯形蝴蝶定理,给我们提供了解决梯形面积与上下底之间关系互相转换的渠道。构造模型,直接应用结论,往往有事半功倍的效果。‎ 模型四:相似三角形性质 ‎_‎ h ‎_‎ h ‎_‎ H ‎_‎ c ‎_‎ b ‎_‎ a ‎_‎ C ‎_‎ B ‎_‎ A ‎_‎ a ‎_‎ c ‎_‎ b ‎_‎ H ‎_‎ C ‎_‎ B ‎_‎ A ‎_‎ S1‎ ‎_‎ S2‎ ‎① ; ‎ ‎②S1︰S2=a2︰A2 ‎ 所谓的相似三角形,就是形状相同,大小不同的三角形,(只要其形状不改变,不论大小怎样改变他们都相似),与相似三角形相关,常用的性质及定理如下:‎ ‎﹙1﹚相似三角形的对应角相等,对应边成比例。‎ ‎﹙2﹚相似三角形的一切对应线段(对应高、对应中线、对应角平分线、外接圆半径、内切圆半径等)的比等于它们的相似比。‎ ‎﹙3﹚相似三角形周长的比等于它们的相似比。‎ ‎﹙4﹚相似三角形面积的比等于它们相似比的平方。 ‎ ‎﹙5﹚特别的,连接三角形两边中点的线段我们叫做三角形的中位线。关于三角形的中位线我们有这样一个结论:‎ 三角形中位线定理:三角形的中位线长等于他所对应的底边长的一半。‎ 对于梯形,我们也有类似的结论。连接梯形两腰得到的线段我们叫做梯形的中位线。‎ 梯形的中位线长等于它上下底边之和的一半。‎ ‎﹙6﹚那么如何判断三角形是不是相似呢?我们一般有三种方法:‎ a:三个角对应相等的三角形相似,(事实上只要有两个角相等就可以了)。 b:有两边对应成比例且其两条边的夹角相等的三角形相似。 c:三边分别对应成比例的三角形相似。‎ 注意:在小学奥数里,最多出现的情况是因为两条平行线而出现相似三角形,如模型四。‎ 相似三角形模型,给我们提供了三角形之间的边与面积关系相互转化的工具。‎ 模型五:燕尾定理 S△ABG:S△AGC=S△BGE:S△EGC=BE:EC;‎ S△BGA:S△BGC=S△AGF:S△FGC=AF:FC;‎ S△AGC:S△BCG=S△ADG:S△DGB=AD:DB;‎ 燕尾定理因为图形类似燕尾而得名,它的特殊性在于,它可以存在于任何一个三角形之中,为三角形中的三角形面积对应底边之间提供互相联系的途径。‎ ‎3、计算过程中连接辅助线的四个原则。‎ 几何作为数形结合的学科,图形的运用往往在解题过程中起到至关重要的作用。在小学阶段的平面几何学习中,我们在运用图形连接辅助线时一般遵循以下四个原则:‎ ﹙1﹚ 把四边形或者多边形变为三角形,例如:‎ ﹙2﹚ 连接等分点,例如:‎ ﹙3﹚ 构造模型,例如:‎ ‎ ‎ ‎﹙4﹚做高线,构造直角三角形 三、经典透析 ‎【例1】(☆☆☆)如下左图。将三角形ABC的BA边延长1倍到D,CB边延长2倍到E,AC边延长3倍到F。如果三角形ABC的面积等于1,那么三角形DEF的面积是_____。‎ 审题要点: 题目中给出的已知条件都是边的倍比关系,其余的条件中只有一个三角形ABC的面积是已知,要想办法使已知条件能够相互关联,使边的倍比关系可以转化为面积之比,可以选择模型一应用。‎ 详解过程:‎ 解:连结AE、BF、CD(如上右图)‎ ‎ 由EB=2BC,得S△ABE=2。‎ 同理可得S△AED=2‎ S△BEF=2×S△CBF =6。‎ ‎ S△CFD =3×S△ACD =3。‎ ‎ 所以 S△DEF= 1+2+3+1+2+6+3=18。‎ 专家点评:这是北京市第一届“迎春杯”刊赛第32题,非常经典。解题过程中通过连接AE、BF、CD,使题目中所给的边的倍比关系可以构造模型一相互关联,再通过共高三角形面积与相应底边之间的对应比例关系求解。‎ ‎【例2】(☆☆☆)设,,,如果三角形的面积为19平方厘米,那么三角形的面积是_________平方厘米。‎ 审题要点:和【例1】类似,题目已知条件中边的倍比关系比较多,可以考虑应用模型一。‎ 解:‎ S△ABC=(++) S△ABC+19‎ ‎∴‎ 专家点评:这是2004年小学数学奥林匹克A卷的,其实竞赛题不一定都是很难,尤其是平面几何部分,但他们十之八九都是很巧妙的,拿这道题来说,图形长得很普通,而题目当中又给了那么多的倍比关系,那我们是不是可以考虑构造模型一呢?整体看,,除了,其余三个我们可以直接用“鸟头定理”。鸟头定理也是本题的一个中心考点。‎ ‎【例3】(☆☆☆)四边形的对角线与交于点(如图)所示。如果三角形的面积等于三角形的面积的,且,,那么的长度是的长度的_________倍。‎ 审题要点:在本题中四边形ABCD为任意四边形,且出现S△ABD:S△BCD=1:3。联想模型二蝴蝶定理结论。‎ 详解过程:‎ 解法一:‎ ‎∴‎ ‎∴‎ 解法二:‎ ‎∵‎ ‎∴‎ ‎∴‎ ‎∴‎ ‎∴‎ ‎∴‎ 专家点评:本题是2003北京市第十九届小学生“迎春杯”数学竞赛的试题。在本题中,三角形和三角形的面积之比如何转化是关键。方法一直接应用模型二蝴蝶定理的结论,而我们也可以不应用蝴蝶定理,那么观察题目中给出的已知条件是面积的关系,转化为边的关系,我们需要一个中介,于是做垂直于H,于,面积比转化为高之比。再应用模型一的结论:三角形高相同,则面积之比等于底边之比,得出AO=CO。‎ ‎【例4】:(☆☆☆☆)如下图所示,AE︰EC=1︰2,CD︰DB=1︰4,BF︰FA=1︰3,‎ 三角形ABC的面积等于1,那么四边形AFHG的面积是__________。 ‎ 审题要点:四边形AFHG的面积可以看作是三角形ABC的面积减去三角形BEC的面积再分别减去三角形BFH和三角形AGE的面积得到的。如何把三角形边的倍比关系和要求的面积相联系,是这道题的重点问题。‎ ‎ 详解过程:‎ 以下各图为了强调相关部分,暂去掉另外线条。‎ 解: 如下图所示,我们分别求出BFH、AGE的面积问题也就解决。‎ 如上图,我们设BFH=x,则AFH=3x;设AHE=y,则CEH=2y;‎ 于是有ABE=4x+y=‎ ‎ ACF=3y+3x=‎ 有,则9x=,所以x=;‎ 如下图,我们设AEG=a,则CEG=‎2a;‎ 设CDG=b,则BDG=4b; ‎ 于是有ACD=‎3a+b=‎ ‎ BCE=‎2a+5b=‎ 有,则‎13a=,所以a=;‎ 这样,AFHG=ABE-BFH-AEG=--=。‎ 专家点评: 求四边形,可由三角形的面积减去三角形的面积,再分别减去三角形BFH和三角形AGE的面积。而三角形的面积可从三角形面积与底边的比例关系得到,于是问题转化为如何求及。与可由二元一次方程组分别解得。‎ ‎ ‎ 解法二:‎ BH:HE=S△BFC:S△EFC=︰(×)=1︰2 ‎ 所以S△BFH=S△ABE×(×)=×(×)= ‎ 同理: ‎ AG︰GD=S△ABE︰S△BDE=︰(×)=5︰8‎ 所以,S△AGE=S△ADC×(×)=×(×)=‎ ‎ AG︰AD=5︰(5+8)=5︰13‎ 所以,‎ S四边形AFHG=S△ABE-S△BFH-S△AEG ‎=--‎ ‎=‎ 专家点评: 本题解法二应用的考点比较多,基本解题思路和解法一差不多,都是由S△FHG= S△ABE- S△BFH- S△AEG得出,而解法二首先应用蝴蝶定理,先求线段BH与HE的比例关系,再利用鸟头定理解出及,最后求出S四边形AFHG。比解法一略显简洁,而且计算上也比较方便。‎ 注意考点: 鸟头定理和蝴蝶定理的应用 ‎【例5】(☆☆☆)设正方形的面积为1,下图中E、F分别为AB、BD的中点,GC=FC。求阴影部分面积。‎ 审题要点:阴影部分为三角形,知道底边为正方形边长的一半,只要求出高,便可解出面积。‎ 解: 作FH垂直BC于H;GI垂直BC于I 根据相似三角形定理 CG︰CF=CI︰CH=1︰3‎ ‎ 又∵CH=HB ‎ ‎∴CI︰CB=1︰6即BI︰CB=(6-1)︰6=5︰6‎ S△BGE=××=。‎ 专家点评:本题考查模型四,利用三角形相似的性质,求出三角形对应边的比例关系及长度,从而确定阴影部分的面积。‎ ‎【例6】(☆☆☆☆)ABCD是平行四边形,面积为72平方厘米,E、F分别为AB,BC的中点,则图中阴影部分的面积为__平方厘米。‎ 审题要点:题目中出现E、F分别为边的中点, 可以考虑应用中位线定理。‎ 解:设G、H分别为AD、DC的中点,‎ 连接GH、EF、BD。‎ 可得 S△AED=S平行四边形ABCD ‎ 对角线BD被EF、AC、GH平均分成四段,‎ DO︰ED= BD︰ BD=2︰3‎ OE︰ED=(ED-OD)︰ED=(3-2)︰3=1︰3‎ 所以 S△AE0=×S平行四边形ABCD=××72=6‎ S△ADO= 2×S△AEO=12。‎ 同理可得S△CFM=6,S△CDM=12。‎ 所以 S△ABC- S△AEO- S△CFM=24‎ 于是 阴影部分的面积=24+12+12=48‎ 专家点评: 这道题是2000年小学数学奥林匹克竞赛A卷中的一道题。连接EF,BD,根据模型4以及三角形的中位线定理,判断出O,M分别是其所在线段的三等分点,由此求出S△AEO及S△CFM,最后得出阴影部分的面积。‎ 注意:本题应用了三角形的中位线定理以及平行线的相关性质。‎ ‎【例7】(☆☆☆)如图,矩形ABCD被分成9个小矩形,其中5个小矩形的面积如图所示,矩形ABCD的面积为__。‎ 审题要点:矩形被分割成9个小矩形,马上可以联想到矩形等积变形的两个重要结论。‎ 解:矩形PFMD中,矩形OHND的面积等于2×4÷3=8/3‎ 矩形ABCD中,矩形IBLH的面积等于(1+2)×(16+4)÷(8/3)=45/2‎ 所以 矩形ABCD的面积=1+2+4+16+(8/3)+(45/2)=289/6‎ 专家点评:本题是南京市第三届兴趣杯的原题,难度不大,主要是考察对矩形等积变形两个重要结论之一: “过矩形内部的一点引两条直线分别与两组边平行,所分得的四个小矩形,其面积满足:。”的应用。先求出矩形OHND的面积,再求出矩形IBLA的面积,而矩形ABCD的面积由矩形OHND和矩形IBLA以及题目中所给的其他4个已知矩形的面积和求得。‎ 读者可以自行通过求各边比例方法进行验证,进一步加深对定理的理解。‎ ‎【例8】(☆☆☆)如图,在梯形ABCD中,AB与CD平行,且CD=2AB,‎ 点E、F分别是AD和BC的中点,已知阴影四边形EMFN的面积是54‎ 平方厘米,则梯形ABCD的面积是 平方厘米。‎ 审题要点:阴影部分的面积可以分解为两个三角形的面积之和,而E、F又是梯形两腰的中点,连接EF,对上下两个梯形分别应用蝴蝶定理。‎ 解法一:如图,设上底为a,则下底为‎2a,梯形的高为h,‎ 连接EF,则EF=(a+‎2a)=a;‎ 所以 AB︰EF=a︰ a=2︰3,EF︰DC=a︰‎2a=3︰4;‎ 所以 h1=×h=h;‎ ‎ h2=×h=h;‎ ‎ 阴影部分=S△EFM+S△EFN=×a×h+×a×h=ah 即ah=54,ah=140‎ 梯形ABCD的面积=×(1+2)ah=ah=×140=210(平方厘米)‎ 专家点评:阴影部分可以看为两个同底三角形的面积之和,根据梯形的面积公式,求出两个三角形的高和底,进一步求出梯形面积,思考方法很简单,但要注意计算的准确性。‎ 解法二:如图,设上底为a,则下底为‎2a,梯形的高为h,‎ 连接EF,则EF=(a+‎2a)=a;‎ 所以 AB︰EF=a︰ a=2︰3,EF︰DC=a︰‎2a=3︰4;‎ 所以 h1=×h=h;‎ ‎ h1=×h=h;‎ 所以S△EFM︰S△EFN= h1 ︰h1=h︰h=7︰5‎ 根据梯形中的面积关系,得下图。‎ 因为9x︰9y=x︰y=7︰5‎ 且x+y=54÷9=6(平方厘米)‎ 所以x=6×=3.5(平方厘米),y=6-3.5=2.5(平方厘米);‎ 所以梯形ABCD的面积=3.5×25+2.5×49=210(平方厘米)。‎ 专家点评:连接EF以后,我们也可以把它看成是两个梯形叠放在在一起,应用模型三梯形蝴蝶定理,可以确定各个小的三角形之中的比例关系,应用比例即可求出梯形ABCD面积。‎ 注意:应用梯形蝴蝶定理时注意比的运算。‎ ‎【例9】(☆☆☆)如图,在平行四边形ABCD中,BE=EC,CF=2FD。‎ 求阴影面积与空白面积的比。‎ 审题要点:题目中阴影部分不规则,但是有边的倍比关系,BE=EC,CF=2FD可以考虑将边的倍比关系转化为为面积之间的关系。‎ 解法:连接CG,CH,AC交BD于O,设S△BEG=a,‎ 根据燕尾定理S△BEG=S△EGC=S△ABG=S△AGC ‎ S△DHF=S△CFH=S△AHD=S△ACH ‎ 又因为S△AGC=S△ACH 所以S△BEG=3S△DHF ‎ S△AGO=S△CGO=S△ABG ‎ S△AOH=S△HOC=S△AHD 所以S□ABCD=4S△ABO=4×(a+‎2a)=‎‎12a 阴影面积:S△BEG+ S△AGH+ S△DFH=a+‎2.5a+‎0.5a=‎‎4a 空白面积:‎12a-4a=‎8a ‎ 所以阴影面积与空白面积的比‎4a︰‎8a=1︰2‎ 另解:设S△BEG=a,则S△ECG=S△GCO=S△AGO=a, S△ABG=‎2a;‎ 设S△HFD=b,则S△HFC=2b,‎ 设S△HCO=x,则S△AHO=S△HCO=x ‎==‎ 专家点评: 连接CG,CA,CH,构造模型五,应用燕尾定理,分别求出三个阴影三角形面积,再求出平行四边形ABCD的面积,用四边形面积减去三个阴影三角形面积即为空白面积。亦可得到阴影面积与空白部分的面积之比。‎ 注意:本题考点:燕尾定理的应用。‎ 拓展训练: ‎ ‎1、(宁波小学数学竞赛1999),如图所示,已知三角形中,,,,连结、BZ和,三条线段分别交于,,。若(面积是‎1平方米,那么阴影的面积是多少平方米?‎ 初级提示: 连接AM2,BM3,CM1。‎ 深度点拨: 设、的面积分别为,,,‎ 分别解出,,‎ 全解过程: 连结,,。‎ 设、、的面积分别为,,,‎ 得 ‎ ‎ 所以有 ‎ 同理有 ‎ ‎ ‎ ‎ ‎ ‎ AEB=+=+4=‎ ‎=+=3+3=‎ ‎ ‎ ‎ ∴阴影部分面积为 ‎ ‎ ‎2、如图,四边形的面积是‎66平方米,,‎ ‎,,,求四边形的面积。‎ 初级提示:连接DB、AC,构造模型一。‎ 深度点拨:找出四边形ABCD与四边形EFGH的面积关系。‎ 全解过程: 连接。设 ‎∵,‎ ‎∴,‎ ‎ 又∵,‎ ‎∴,‎ ‎ 同理,‎ ‎ ∴‎ ‎ 连接AC,同理 ‎∴,‎ ‎ (平方米)。‎ ‎ ‎ ‎3、如图,在梯形ABCD中,AD︰BE=4︰3,BE︰EC=2︰3,且△BOE 的面积比△AOD的面积小10平方厘米。梯形ABCD的面积是 平方 厘米。‎ 初级提示:应用模型一求出三角形ABD的面积 深度点拨:求出三角形BCD的面积 全解过程: AD︰BE︰EC=8︰6︰9,‎ ‎ ‎ ‎ -=-=10,‎ ‎ =10,=40。‎ ‎ ‎ ‎ ‎ ‎ ‎ ‎ ‎ ‎4、如图,在一个边长为6正方形中,放入一个边长为2的正方形,‎ 保持与原长正形的边平行,现在分别连接大正方形的一个顶点与小正方形 的两个顶点,形成了图中的阴影图形,那么阴影部分的面积为 。‎ ‎ ‎ 初级提示:将小正方形的四个顶点分别与大正方形的四个顶点连接 深度点拨:应用梯形蝴蝶定理求出空白部分面积 全解过程:‎ 解法一:设任意一个梯形(如图),上底为a,下底为b,则阴影 部分的面积可以表示为 S1、S2、S3的和,而S3︰S4=S1︰S2=(S1+S3)︰(S2+S4)=a︰b,同理 S1︰S3=S2︰S4=a︰b,所以:S1︰S2︰S3︰S4=a2︰ab︰ab︰b2,所以阴影 部分的面积等于。‎ 连接两个正方形的对应顶点,则可以得到四个梯形,运用这条结论,‎ 每个梯形中阴影部分的面积都占到了,所以阴影 部分面积是两个正方形之间的面积的,阴影部分的面积为 ‎,‎ 解法二:取特殊值,使得两个正方形中心相重合,由上右图可知,‎ A、B、C、D均为相邻两格点的中点,则图中四个空白处的三角形的高为 ‎1.5,因此空白处的总面积为 ‎,阴影部分的面积是。‎ ‎5、如图所示,三角形BDF、三角形CEF、三角形BCF的面积分别是2、3、4,问四边形ADFE的面积是多少?‎ ‎ ‎ 初级提示:连接AF,构造模型一 深度点拨:应用三角形面积之比等于底边之比求出三角形AFD和三角形AFE的面积 全解过程:设S△AFD=a,S△AFE=b ‎ ‎2a=3+b ‎ 4b=3(2+a)‎ ‎ a= b=‎ ‎ S四边形ADFE=a+b= ‎ ‎6、如图,在△ABC中,延长BD=AB,CE=BC,‎ F是AC的中点,若△ABC的面积是2,则△DEF的面积是多少?‎ 初级提示:连接CD,构造模型一 深度点拨:S△DCF=S△DCA=2‎ ‎ S△FCE=S△BCF=‎ ‎ S△DEC=S△DCB=1‎ 全解过程: ‎ 解法一:S△DCF=S△DCA=2‎ ‎ S△FCE=S△BCF=‎ ‎ S△DEC=S△DCB=1‎ ‎ S△DEF=S△DCF+S△FCE+S△DEC=‎ 解法二:本题还可以用共角定理“当两个三角形有一个角相等或互补时,‎ 这两个三角形的面积比等于夹这个角的两边长度的乘积比”。 ‎ ‎∵在△ABC和△CFE中,∠ACB与∠FCE互补,‎ ‎∴‎ 又;‎ ‎∴‎ 同理可得: ‎ ‎∴‎ ‎7.如图,长方形ABCD中,E为AD中点,AF与BE、‎ BD分别交于G、H,已知AH=‎5cm,HF=‎3cm,求AG。‎ 初级提示:三角形AHB和三角形DHF相似 深度点拨:作OE垂直AD,交AF于O 全解过程:根据三角形相似的性质 ‎ AB︰DF=AH︰HF=5︰3‎ 又因为E为AD中点 OE︰DF=1︰2‎ 所以AB︰OE=10︰3‎ ‎ AG︰GO=10︰3‎ 所以AG=AO=‎ ‎8.在边长为1的正方形ABCD中,BE=2EC,DF=2FC;‎ 求四边形ABGD的面积。‎ 初级提示:连接EF、BD 深度点拨:应用梯形蝴蝶定理 全解过程:等腰梯形四部分面积比为1︰3︰3︰9‎ 所以等腰梯形的面积=‎ 所以 得 ‎9、如图,正方形ABCD面积为1,M是AD边上的中点,求图中阴影部分的面积。‎ ‎ ‎ 初级提示: 构造梯形蝴蝶定理。‎ 深度点拨:S△AMG: S△AGB: S△MCG: S△GCB=1︰2︰2︰4‎ 全解过程: ‎ ‎∵梯形AMCB中各个三角形面积比 1︰2︰2︰4‎ ‎ ∴阴影面积占梯形面积(2+2)/(1+2+2+4)= ‎ ‎∴‎ 本题还可有其他解法(如下)‎ 解法二:连结、,设与交于,。‎ ‎ ∵‎ ‎ ‎ ‎ ∴‎ ‎ 又∵‎ ‎ ∴==x ‎ ∴‎ ‎ ∴‎ ‎ 得,又,所以,。‎ ‎ ∴。‎ 解法三:做 ‎ 则,,,‎ ‎ 连接 ‎ ∵‎ ‎ ∴‎ ‎ 又∵‎ ‎ ∴‎ ‎ ∴‎ 解法四:∵与等底等高 ‎ ∴‎ ‎ ∴‎ ‎ 作,‎ ‎ 设 ‎ ‎ ‎ ‎ ‎ ∴‎ ‎ ‎ 解法五: ∵‎ ‎ ‎ ‎ ∴‎ ‎ ∴‎ ‎ ∵‎ ‎∴==‎ ‎∴+=+=‎ ‎10:(07年仁华学校试题)已知四边形ABCD,CHFG为正方形,S甲︰S乙=1︰8,a与b是两个正方形的边长,求a︰b=?‎ ‎ ‎ 初级提示:连接EO,AF,应用燕尾定理。‎ 深度点拨:做OM⊥AE,ON⊥EF,‎ 全解过程:‎ 如图,根据燕尾定理:‎ S△AOF︰ S△AOE=b︰a (1),‎ ‎ S△AOF︰ S△FO E=a︰b (2)‎ 所以 S△AOE ︰ S△FOE=a2︰b2‎ ‎ 作OM⊥AE,ON⊥EF,‎ ‎ ∵AE=EF,‎ ‎ ∴OM︰ON= a2︰b2‎ ‎ ∴S△AOD︰ S△HOF=a3︰b3=1︰8‎ ‎ ∴a︰b=1︰2‎ 第8讲 几何(二) 曲线图形 一、 知识地图 二、 基础知识 小学数学当中,我们学习了一些简单的几何图形,充分掌握这些图形的性质特点及周长和面积的计算方法是我们解决奥数平面几何问题的重要前提。‎ ‎﹙1﹚组合图形的面积 在求解组合图形的面积时,中心思想只有一个:把不规则的变为规则的,把不可求的变为可以求的,把不熟悉的变为我们熟悉的。在小学奥数的几何问题中,这个思想不单单可以在求组合图形面积的时候应用,求解立体图形的表面积和体积问题时候一样也是解决问题的法宝,甚至可以说是全部小学奥数几何问题的思想精髓。‎ 在求解组合图形的面积时,我们通常可以通过以下思考方法把图形转化我们所熟知的图形。‎ 1、 加减法 把要求的图形转化为几个规则图形相加或者相减的形式,这种解决图形补问题的方法,称为加减法。‎ 1、 割补法 把要求的图形通过切割再拼补成规则图形,这种方法称为割补法。‎ 2、 旋转平移法。‎ 图形的一部分通过旋转或者平移,正好可以和图形的其他部分拼成规则图形,这种方法称为旋转平移法。‎ 3、 重叠法 要求的组合图形可以看作是几个规则图形的重叠部分,可以应用容斥原理求得图形的面积,这种方法称为重叠法。‎ 4、 比例法 把要求的图形分成几个部分,通过寻找各个部分之间的比例关系求解的方法称为比例法。‎ ‎﹙2﹚图形旋转的问题 在这里,我们主要研究的是平面图形在平面旋转所产生的问题。一般情况下,我们所能遇到的有以下两种问题:‎ ‎1、求图形一边扫过的面积 在遇到这类问题时,我们只要先找到要求的是哪条边扫过的面积,再看这条边是以哪个点为圆心运动,首先你让这条边以这个点为圆心按照题目的要求转动,旋转停止后,这条边旋转所得的面积就是你要求的图形一边扫过的面积。‎ ‎2、求图形扫过的面积 在求图形一边扫过的面积的基础之上,要注意,图形中最长处旋转时所成图形,我们在旋转的图形一边停止旋转时,在相应的位置补上图形的其他部分就可以很容易的找到整个图形扫过的部分。‎ ‎﹙3﹚几个特殊问题 ‎1、活动范围的问题 让我们先来看看下面几个问题:‎ A、假设茫茫的草原上有一个木桩,桩子上用一根‎30米的绳子栓着一只羊,问羊能吃到的草的面积是多大?‎ B、草场的主人因为业务发展,准备建羊圈,但是因为资金短缺,所以只先建了一道墙,于是把羊还是用‎30米的绳子栓在了墙角边,问羊这个时候能吃到草的面积是多大?‎ C、羊圈建成了,羊在平时被栓在羊圈的西北角,羊圈长‎20米,宽‎10米,问羊这个时候能吃到的草的面积是多大?‎ 你注意到了吗?栓着羊的绳子在碰到墙拐角的地方运动的圆心在变化,羊所能吃到草的范围活动的半径也在跟着变化。‎ 那么,我们说看变化,找规律,是解决羊吃草一类问题重要思想。另外,数学源自生活,通过想象生活中的情景,比照数学题,寻找变化的规律也是一种不错的方法。‎ ‎2、滚硬币的问题 请你一起动手来做一做:把两个一角钱的硬币挨放在一起,固定其中一个,把另一个延着其周围滚动。当滚动回到硬币原来的位置时,想一想滚动的那个硬币它自己自转了多少周?‎ 注意观察,滚动的硬币绕着不动的硬币走一周的距离实际上是以两个硬币的半径为半径的一个圆周长,而硬币自转的周长是以自身为半径,前者是后者的几倍,即是硬币自转了几周。‎ 这也是一切硬币滚动类问题的特点。常见的还有齿轮,滑轮等。‎ 经典回顾 ‎【例1】(☆☆☆)图是由正方形和半圆形组成的图形。其中P点为半圆周的中点,Q点为正方形一边的中点。已知正方形的边长为10,那么阴影部分面积是多少?(π取3.14。)‎ 审题要点:整个图形由正方形和半圆组成。P为中点,则PD=PC,要 求阴影部分的面积,可以考虑我们前面讲的几种方法。‎ 解法一:阴影面积=整个面积-空白面积=(正方形ABCD+半圆)—(三角形+梯形)‎ ‎ =(10×10+π×5×5÷2)-[15×5÷2+(5+15)×5÷2]‎ ‎ =51.75‎ 专家点评:阴影面积的“加减法”。因为阴影部分面积不是正规图形,所以通过整个面积减去空白部分面积来求解。过P点向AB作垂线,这样空白部分面积分成上面的三角形和下面的梯形。‎ 解法二: S1=小正方形-圆=5×5-×π×5×5‎ 上面阴影面积=三角形APE-S1=15×5÷2-5×5+×π×5×5‎ 下面阴影面积=三角形QPF-S2=10×5÷2-(5×5-×π×5×5)‎ 所以阴影面积=(15×5÷2-5×5+×π×5×5)+(10×5÷2-5×5+×π×5×5)=51.75‎ 专家点评:面积的“加减法”和“切割法”综合运用,思路出现正方形,出现弧线时,注意两个考点:1.半叶形 2、圆,所以我们可以先把面积补上再减去补上的面积。‎ 解法三: 半叶形S1=圆-小正方形=×π×5×5-×5×5‎ 上面阴影面积=三角形ADP+S1=10×5÷2+×π×5×5-×5×5‎ 下面阴影面积=三角形QPC+S2=5×5÷2+×π×5×5-×5×5‎ 阴影面积=(10×5÷2+×π×5×5-×5×5)+(5×5÷2+×π×5×5-×5×5)=51.75‎ 专家点评:面积的“切割法”出现正方形,出现弧线时,注意两个考点:1.半叶形 2. 圆,这样可以考虑把阴影面积切成几个我们会算的规则图形。这道题是迎春杯真题。‎ ‎【例2】(☆☆☆)如图,ABCG是4×7的长方形,DEFG是2×10的长方形,那么,三角形BCM的面积与三角形DCM的面积之差是多少? ‎ 审题要点:要求两个三角形的面积之差,题目没有给出可以直接求出两个三角形面积的条件,那么我们只能考虑应用差不变原理。‎ 解法一: GC=7,GD=10推出HE=3;BC=4,DE=2‎ 阴影BCM面积-阴影MDE面积=(BCM面积+空白面积)-(MDE面积+空白面积)=三角形BHE面积-长方形CDEH面积=3×6÷2-3×2=3。‎ 专家点评:加减思想的应用,小升初中的常用方法,而找出公共部分是本题的解题关键。公共部分要与两个三角形都可以构成规则可求的图形才可以。‎ 解法二:GC=7,GD=10 知道CD=3;‎ BC=4,DE=2 知道BC︰DE=CM︰DM ‎ 所以CM=2,MD=1。‎ 阴影面积差为:4×2÷2-1×2÷2=3‎ 专家点评:画阴影的两个三角形都是直角三角形,而BC和DE均为已知的,所以关键问题在于求CM和DM。这两条线段之和CD的长是易求的,所以只要知道它们的长度比就可以了,这恰好可以利用平行线BC与DE截成的比例线段求得。另外本题还可以构造如下解法,如图:‎ 解法三:连接BD ‎【例3】(☆☆☆)求右图中阴影部分的面积。(取3)‎ 审题要点:△ABC可以看出为等腰直角三角形。‎ 解法一:我们只用将两个半径为‎10厘米的四分之一圆减去空白的①、②部分面积和即可,其中①、②面积相等。易知①、②部分均是等腰直角三角形,但是①部分的直角边AB的长度未知。单独求①部分面积不易,于是我们将①、②部分平移至一起,如下右图所示,则①、②部分变为一个以AC为直角边的等腰直角三角形,而AC为四分之一圆的半径,所以有AC=10。两个四分之一圆的面积和为150,而①、②部分的面积和为1/2×10×10=50,所以阴影部分的面积为150-50=100(平方厘米)。‎ 解法二:欲求图(1)中阴影部分的面积,可将左半图形绕B 点逆时针方向旋转180°,使A与C重合,从而构成如右图 ‎(2)的样子,此时阴影部分的面积可以看成半圆面积减去中间等腰直角三角形的面积。‎ 专家点评:本题考点 旋转平移法。图形通过旋转,得到阴影部分的面积=半圆的面积-等腰直角三角形的面积。‎ ‎【例4】(☆☆☆)如图,已知三角形GHI是边长为‎26厘米的正三角形,圆O的半径为‎15厘米,∠AOB=∠COD=∠EOF=90°。求阴影部分的面积。‎ 审题要点:题中每一条阴影部分面积可以看做是两个大小弓形的面积之差。‎ 解法: 设J为弧GI的中点,则可知GJIO是菱形,GOJ是正三角形,‎ ‎ 所以,三角形GOI的面积=‎ ‎ 所以大弓形的面积: SGJI ‎ 小弓形的面积:SFJE ‎ ‎ 所以,总阴影面积=(138-64.125)×3=221.625(平方厘米)‎ 专家点评: 本题难度在于判断四边形GJIO为菱形,圆中等长的弧所对的弦也是相等的,所以三角形GOJ为正三角形,其实三个阴影部分选择哪一个作为解题的模型都可以。基本上还是加减思想的应用。‎ 总阴影面积=每块阴影面积×3=(大弓形-小弓形)×3‎ 关键在于大弓形中三角形的面积。‎ 总结:本题考点 加减法。‎ ‎【例5】(☆☆☆)如图,ABCD是一个长为4,宽为3。对角线长为5的正方形,它绕C点按顺时针方向 旋转90,分别求出四边扫过图形的面积。(取3)‎ 审题要点:要求边扫过的面积,只需分别看一边旋转所得图形。‎ 分析:1、容易发现,DC边和BC边旋转后扫过的图形都是以线段 长度为半径的圆的,如右图: ‎ 因此DC边扫过图形的面积为4平方厘米,BC边扫过图形的面积为平方厘米。‎ ‎2、研究AB边的情况。‎ 在整个AB边上,距离C点最近的点是B点,最远的点是A点,因此整条线 段所扫过部分应该介于这两个点所扫过弧线之间,见右图中阴影部分: ‎ ‎ 下面来求这部分的面积。‎ 观察图形可以发现,所求阴影部分的面积实际上是:‎ 扇形ACA,面积+三角形ABC面积-三角形ABC面积-扇形BCB,面积+三角形A,B,C,面积=扇形ACA,面积一扇形BCB,面积 ‎ ;‎ ‎3、研究AD边扫过的图形。‎ 由于在整条线段上距离C点最远的点是A,最近的点是D,所以我们可以画出AD边扫过的图形,如下图阴影部分所示:‎ 用与前面同样的方法可以求出面积为:‎ 专家点评:本题是祖冲之杯竞赛的一道试题。‎ 旋转图形的关键,是先从整体把握一下“变化过程”,即它是通过什么样的基本图形经过怎样的加减次序得到的。先不去考虑具体数据,一定要把思路捋清楚。最后你会发现,所有数据要么直接告诉你,要么就“藏”在那儿,一定会有。‎ ‎ 我们可以作进一步的思考,比如平行四边形的旋转问题、一般三角形的旋转问题等等,此类问题的解决对提高解决几何图形问题的能力是非常有益的。‎ ‎【例6】(☆☆☆)求圆中阴影部分与大圆的面积之比和周长之比。‎ ‎ 审题要点:阴影部分可以看作一个整体,那么大圆由四个阴影部分组成。‎ ‎ 解法:把阴影看作一个特殊图形,而大圆的面积恰好是4个这种特殊图形 ‎ 所以 阴影面积︰大圆面积=1︰4‎ ‎ 设小圆半径为x,则大圆半径为2x ‎ 阴影周长=小圆周长+小圆周长+小圆周长+大圆周长 ‎ =小圆周长+大圆周长 ‎ =×2x+×2×2x ‎ =x ‎ 大圆周长=2×2x=4x ‎ 所以 周长之比=x︰ 4x=7︰8‎ 专家点评:应用图形比例关系求解图形,也是整体考虑问题思想的典型代表。‎ ‎【例7】(☆☆☆)如图,半圆半径=‎40CM,BM=CN=DP=22,每个阴影部分的弧长为半圆弧长的,求阴影部分面积?(=3)‎ ‎ ‎ 审题要点:图中上半部分的三个阴影图形并非真正的扇形,所以不能用扇形面积公式来解,只能应用加减法,把图形分解。那么每个阴影部分面积等于1/3半圆面积减去一大一小两个相似三角形面积。‎ 解法:∵△ABO为等边三角形 又∵∠AMB=120度 ‎∴∠MAE=30度 ∴∠BAM=30度 ‎ ‎∴△BMA为等腰三角形即 根据正三角形性质 得BM=2EM ‎∴BE=22+11=33(cm)‎ 阴影部分面积=3×(×40×40-×20×33-×20×11)‎ ‎ =3×(800-330-110)‎ ‎ =3×360=1080(平方厘米)‎ 专家点评:应用加减法,把图形化为我门常用的图形来解题是这道题的关键所在。另一个难点是如何求出三角形的高,其实M,N,P分别是它们所在正三角形的中心。中心将其所在线段分为两部分的比为1︰2,知道这一性质,便可应用面积公式求出阴影面积。‎ ‎【例8】(☆☆☆)如图,哨所门前的两个正三角形哨台拴了两条狼狗,拴狼狗的铁链子长为‎10米,每个哨台的面积为‎42.5平方米现在要绿化哨所所在地(哨所面积忽略不计,把其看做一点,在其周围‎20米范围内铺上草地)为了防止狼狗践踏,则绿化的实际面积为多大合适?(=3)‎ 审题要点:首先确定两条狼狗的活动范围,利用加减法把活动范围为一个菱形+两个半圆,两个半圆即一个整圆。 实际绿化面积=大圆面积-(菱形+小圆面积+2×哨所面积)‎ 解法:可以看出菱形面积为2倍的哨所面积,菱形面积=2×42.5=85‎ ‎ 实际绿化面积=×20×20-(85+×10×10+2×42.5)‎ ‎ =1200-(85+300+85)‎ ‎ =1200-470=730(平方米)‎ 专家点评:本题属于活动范围题,注意确定狼狗的活动范围为两个5/6圆减去其重合部分,即一个菱形+一个圆,另外哨台也是未绿化部分,注意以上两点本题就不难求解。‎ ‎【例9】(☆☆☆☆)如图,15枚相同的硬币排成一个长方形,一个同样大小的硬币沿着外圈滚动一周,回到起始位置。问:这枚硬币自身转动了多少圈?‎ 审题要点:注意硬币滚动时圆心的轨迹。‎ 解法一:当硬币在长方形的一条边之内滚动一次时,由于三个硬币的圆心构成一个等边三角形,所以这枚硬币的圆心相当于沿着半径为硬币2倍的圆旋转了180º-60º-60º=60º。而硬币上的每一点都是半径等于硬币的圆旋转,所以硬币自身旋转了120º。‎ 当硬币从长方形的一条边滚动到另一条边时,这枚硬币的圆心相当于沿着半径为硬币2倍的圆旋转了360º-60º-60º-90º=150º。而硬币上的每一点都是半径等于硬币的圆旋转,所以硬币自身旋转了300º。‎ 长方形的外圈有12个硬币,其中有4个在角上,其余8个在边上,所以这枚硬币滚动一圈有8次是在长方形的一条边之内滚动,4次是从长方形的一条边滚动到另一条边。‎ ‎120×8+300×4=2160,所以这枚硬币转动了2160º,即自身转动了6圈。‎ 解法二:通过计算圆心轨迹的长度,每走一个2即滚动了一周。‎ 对于同样是12个硬币,所转动的圆心轨迹其实分为两部分,一是在“角”上的转动,一是在“边”上的滚动。‎ ‎ 抓住关键方法:圆心轨迹长度÷2=自身转动圈数。‎ 专家点评:此题来源于小学数学ABC。圆运动的轨迹分两种,一种所谓的“跨圆”运动;另一种所谓的“绕圆”运动。掌握“跨圆”运动一次30+60+30=120度;“绕圆”一次180度。角上4次“绕圆”,边上12次“跨圆”,这样结果便一目了然。‎ 拓展训练 ‎1、如图,四边形是平行四边形,,,,高CH=‎4cm, 、分别以、为半径,弧、分别以、为半径,阴影部分面积是多少平方厘米?‎ 初级提示:‎ ‎ ‎ 深度点拨: =10×4=40() ‎ 全解过程: =-‎ ‎ =(2-)-(-2)‎ ‎ ‎ ‎ ‎ ‎2、下图中,四边形ABCD都是边长为1的正方形,E,F,G,H分别是AB,BC,CD,DA的中点;请计算图中两个阴影图形的面积比。‎ 初级提示: 左图阴影可用加减法 即正方形面积减去4个边上的三角形面积。‎ 深度点拨: 右图阴影面积可先看小正方形里的阴影面积为2个小三角形面积之和,而每个小三角形面积恰好是它所在直角三角形面积的三分之二,只要求出直角三角形面积,阴影面积便不难求出了,直角三角形面积为大正方形面积的十六分之一。‎ 全解过程: 左图阴影面积=正方形面积-4个等腰三角形面积 ‎ =1×1-4×××1‎ ‎ =1-=‎ ‎ 右图阴影面积=8个小三角形面积 ‎ =8×(×××)‎ ‎ =8×=‎ ‎ 所以 左:右=:=3:2‎ ‎ ‎ ‎3、(2004第二届“走进美妙的数学花园”中国青少年数学论坛趣味数学解题技能展示大赛3),如图,在平行四边形中,已知三角形、的面积分别是73、100,求三角形的面积。‎ 初级提示:+=‎ 深度点拨:‎ 全解过程:‎ ‎4、下图中除大圆外,所有的弧线都是半圆,且,图中有上、下两块阴影区域,如果上面的阴影区域面积为100平方厘米,那么下面的阴影域面积为________平方厘米。‎ 初级提示:分析题意本题用割补法。‎ 深度点拨:阴影面积可分为四部分,分别求之。‎ 全解过程:设AB=1,则AC=3,AD=6,AE=10,DE=4,CE=7,BE=9。‎ ‎ 上块阴影面积=(S半圆AE-S半圆AD)+S半圆DE ‎=(1/2×25-1/2×9)+1/2×4‎ ‎=8+2=10‎ ‎ 下块阴影面积=(S半圆AC-S半圆AB)+(S半圆BE-S半圆CE)‎ ‎ =(1/2×9/4-1/2×1/4)+(1/2×81/4-1/2×49/4)‎ ‎=+4=5‎ 因为上块阴影面积=100 所以下块阴影面积=50‎ 5. 如图,∠1=15°,圆的周长为‎62.8厘米,平行四边形的面积为100平方厘米。求阴影部分面积?‎ 初级提示:连接AO,AB,作BC的垂线AD交BC于D 深度点拨:S△ABC=S△ABE 全解过程: ‎ 则∠ACO=∠OAC=15°‎ ‎∴∠ACO=150°, ∠AOB=30°‎ ‎∴=102×3.14×=‎ ‎=5‎ 所以 阴影部分面积=50-(-25)=(平方厘米)‎ ‎6、五环图由内径为‎4cm,外径为‎5 cm的5个圆环组成,其中阴影部分的面积都相等。已知5个圆环盖住的总面积是122.5平方厘米。求每个阴影部分的面积。‎ 初级提示:注意重叠部分。‎ 深度点拨:五个圆环总面积-五环面积=阴影面积 全解过程:5×(5×5-4×4)=45=141.3‎ ‎141.3-122.5=18.8‎ ‎18.8÷5=2.35‎ ‎7、(04年华罗庚金杯数学邀请赛)如右图,一个半径为1厘米的小圆盘沿着一个半径为4厘米的大圆盘外侧做无滑动的滚动,当小圆盘的中心围绕大圆盘中心转动90度后,小圆盘运动过程中扫过的面积是多少平方厘米?(取3)‎ 初级提示:小圆盘运动过程中扫过的面积由两部分组成,即两半圆加扇形环。‎ 深度点拨:扇形面积可由半径为4+2、圆心角为90度的大扇形减去半径为4、圆心角为90度的小扇形。‎ 全解过程:第一部分是半径为‎6厘米、中心角为90度的扇形减去半径为‎4厘米、中心角为90度的扇形,面积为;第二部分是半径为‎1厘米的2个半圆,总面积是3。‎ 所以扫过的面积为15+3=18平方厘米。‎ 8、 有一个边长分别为‎4cm的等边三角形木块。现将三角板沿水平线翻滚,如下图,那么从B点开始到结束所经过的总长度为多少?‎ 初级提示:三角形为等边三角形。‎ 深度点拨:在翻滚过程中,B划过了两条圆弧,每段圆弧的圆心角大小都为120°。‎ 全解过程: (120°+120°) 360°= ‎ ‎2×4×= (cm)‎ ‎9、 如下图所示,直角三角形ABC的斜边AB长为‎10厘米,∠ABC=60,此时BC长‎5厘米。以点B为中心,将△ABC顺时针旋转120,点A,C分别到达点E,D的位置。求AC边扫过的图形即图中阴影部分的面积。(取3)‎ ‎ ‎ 初级提示:如右下图所示,本图为扇形和三角形的组合图形。‎ 深度提示:将图形I移补到图形II的位置,则阴影部分为一圆环的。‎ 全解过程:面积为 (AB一BC)÷3=(10一5)÷3 =75×3÷3=75(平方厘米)。‎ ‎10、如图所示,两条线段相互垂直,全长为‎30厘米。圆紧贴直线从一端滚动到另一端(没有离开也没有滑动)。在圆周上设一个定点P,点P从圆开始滚动时是接触直线的,当圆停止滚动时也接触到直线,而在圆滚动的全部过程中点P是不接触直线的。那么,圆的半径是多少厘米?(设圆周率为3.14,除不尽时,请四舍五入保留小数点后两位。如有多种答案请全部写出)‎ 初级提示:两线段垂点附近,圆不能到达(隐含的在题中的已知条件),即有四分之一的圆未接触线段,圆滚动的实际距离只有圆周长的四分之三或四分之七,利用圆周长公式算出半径。‎ 深度点拨:因为在圆滚动的全部过程中点P是不接触直线的。所以这个圆的运动情况有两种可能。‎ 全解过程: 一种是圆滚动了不足一圈,根据P点的初始位置和终止位置,可知圆滚动了270º。另一种是圆在第一条直线上滚动了将近一圈,在第二条直线上又滚动了将近一圈,根据P点的初始位置和终止位置,可知圆滚动了270º+360º=630º。‎ 因为两条线段共长‎30厘米,所以270º的弧长或者630º的弧长是‎30厘米。‎ ‎30÷÷3.14÷2=6.37(厘米),30÷÷3.14÷2=2.73(厘米),所以圆的半径是6.37厘米或2.73厘米。‎ 两线段垂点附近,圆不能到达,即有四分之一的圆未接触线段,圆滚动的实际距离只有圆周长的四分之三或四分之七,利用圆周长公式算出半径。‎ ‎ ‎ 第9讲 几何(三) 立体图形 一、 知识地图 二、 基础知识 万丈高楼平地起。我们可以这样说:把平面图形从平面拎到空间,让平面图形在空间上产生高度就形成了这一讲我们要研究的立体图形。在现阶段,我们主要研究的立体图形有以下几种:‎ 立体图形 表面积 体积 注:是母线,即从顶点到底面圆上的线段长。‎ ‎ 特别的:关于球体还有这样一个结论:‎ 如果一个球体的直径与一个圆柱的直径与高都相等,那么:‎ 球体的体积等于以球大圆为底球的直径为高的圆柱体积的三分之二;‎ 球体的表面积等于以球大圆为底球的直径为高的圆柱的侧面积;‎ 球体的体积还等于以球大圆为底,球的半径为高的圆锥的体积的4倍。‎ 这个图就是有名的阿基米德圆柱容球。‎ 二、求立体图形的表面积和体积 规则立体图形的表面积和体积我们可以直接应用公式进行计算。‎ 不规则的立体图形的表面积和体积,一方面,我们可以应用和平面图形相同思考的方法来考虑把它转化为规则的立体图形进行计算;而另一方面,我们更注重的是观察图形从规则变为不规则的变化过程,通常这个过程我们需要以图形整体考虑为出发点。‎ 这也就是我们求解此类问题常用方法的思想基础:、‎ 方法一:阳光照面 阳光照面法从图形整体考虑出发,观察图形表面积特点。‎ 方法二:与时俱进 图形的变化,是从整体的变到不变的过程,找到变化的规律,注意图形的变化过程,观察求解,与时俱进,就是解决问题的秘籍宝典。‎ 方法三:面包切片 我们都有这样的经验:一个大的桃李早餐面包,从上向下切一刀,横截面是一个正方形。如果是奶黄夹心面包,则横截面是一个环正方形。同样道理,解题过程中你可以想象,把图形切开,横截面的特点可以帮助我们了解图形内部结构,达到解题的目的。‎ 方法四:借来还去 这里的借来还去可以说是平面几何加减思想的一种变形。可以这样解释,把一部分借来与原来的组成一个规则可以求得图形,再把借来的部分从规则中拿去。借来还去的思想在解决求解不规则立体图形的表面积和体积的问题中经常可以用到。‎ 例如:‎ 三、最短路线和展开图的形状 立体图形的展开图形状总结如下:‎ 对于不规则的立体图形的展开图就要充分发挥我们的想象,用“脱衣服”方法,层层剥离展开。在解决这类问题的时候,要注意培养自己的空间想象能力,必要时可以借用纸片等辅助工具帮助想想理解。‎ 例如:‎ ‎ ‎ 和立体图形的展开图结合最为紧密的是图形侧面的最短路线问题。你需要把握的重要一点是:两点之间永远直线线段最短。‎ 四、染色问题 ‎﹙1﹚奥数的经典问题,重要的是掌握几个关于染色问题的数据,其余的问题需要具体问题具体分析,把握好什么地方染到了颜色,什么地方没有染到颜色是解决此类问题的关键。‎ 对于由n3块小正方体构成的n×n×n正方体,三面涂有红色的有8块,两面涂有红色的有12×(n-2)块,一面涂有红色的有6×(n-2)2块,没有涂色的有(n-2)3块。‎ 例如: 右图是4×5×6正方体,如果将其表面涂成红色,那么其中一面、二面、三面被涂成红色的小正方体各有多少块?‎ 分析:三面涂红色的只有8个顶点处的8个立方体;‎ 两面涂红色的在棱长处,共(4-2)×4+(5-2)×4+(6-2)×4=36块;‎ 一面涂红的表面中间部分:‎ ‎(4-2)×(5-2)×2+(4-2)×(6-2)×2+(5-2)×(6-2)×2=52块。‎ 没涂红色的小方块有:(4-2)×(5-2)×(6-2)=24块。‎ 三面——顶点 二面——棱 ‎ 一面——面 ‎ ‎0面——芯 一句话:“角三 棱二 面唯一。”‎ ‎﹙2﹚欧拉公式 严格的说,欧拉公式和我们这里所讲的染色问题关系不是很密切。但这个公式却是和多面体密切相关的完美公式。‎ 首先请同学们观察下面的几个图形的顶点数,面数和棱数之间的关系:‎ 顶点V 面F 棱E V-E+F=2‎ 正四面体 ‎4‎ ‎4‎ ‎6‎ ‎2‎ 正六面体 ‎8‎ ‎6‎ ‎12‎ ‎2‎ 正八面体 ‎6‎ ‎8‎ ‎12‎ ‎2‎ 正十二面体 ‎20‎ ‎12‎ ‎30‎ ‎2‎ 正二十面体 ‎12‎ ‎20‎ ‎30‎ ‎2‎ 通过观察,我们发现了多面体的顶点数,面数,棱数之间存在着如下的关系:‎ V+F-E=2‎ 那么这个公式也就给我们提供了一种解决染色或者多面体问题的思考方法——分类思考 由欧拉公式,我们可以很自然的想到,在解决如上问题的时候,我们思考问题可以从立体图形的顶点数,面数和棱数的角度出发,分类思考。‎ 经典透析:‎ ‎【例1】(☆☆☆)一个由125个同样的小正方体组成的大正方体,从这个大正方体中抽出若干个小正方体,把大正方体中相对的两面打通,下图就是抽空的状态。右图中剩下的小正方体有多少个?‎ ‎ ‎ 解法一:(用“容斥原理”来解)由正面图形抽出的小正方体有5×5=25个,由侧面图形抽出的小正方体有5×5=25个,由底面图形抽出的小正方体有4×5=20个,正面图形和侧面图形重合抽出的小正方体有1×2+2×1+2×2=8个,正面图形和底面图形重合抽出的小正方体有1×3+2×2=7个,底面图形和侧面图形重合抽出的小正方体有1×2+1×1+2×2=7个,三个面的图形共同重合抽出的小正方体有4个。根据容斥原理,25+25+‎20-8-7‎-7+4=52,所以共抽出了52个小正方体。‎ ‎125-52=73,所以上图中剩下的小正方体有73个。‎ 注意这里的三者共同抽出的小正方体是4个,必须知道是哪4块,这是最让人头疼的事。‎ 但你可以先构造空的两个方向上共同部分的模型,再由第三个方向来穿过“花墙”。‎ 这里,化虚为实的思想方法很重要。‎ 解法二:(用“切片法”来解)‎ 可以从上到下切五层,得:‎ (1) 从上到下五层,如图:‎ ‎ ‎ (2) 或者从右到左五片,如图:‎ ‎ ‎ ‎ 请注意这里的挖空的技巧是:先认一种方向。‎ ‎ 比如:从上到下的每一层,首先都应该有第一层的空四块的情况,即——‎ ‎ 如果挖第二层:第(1)步,把中间这些位置的四块挖走如图:‎ ‎ ‎ ‎ 第(2)步,把从右向左的两块成线地挖走。(请注意挖通的效果就是成线挖去),如图:‎ 第(3)步,把从前向后的一块(请注意跟第二层有关的只是一块!)挖成线!如图:‎ ‎ 总结一下“切片法”:全面打洞(例如本题,五层一样)‎ ‎ 挖块成线(例如本题,在前一次的基层上,一条线一条线地挖)。‎ ‎ 这里体现的思想方法是:化整为零,有序思考!‎ ‎【例2】(☆☆☆)如图,在一个正方体的两对侧面的中心各打通一个长方体的洞,在上下底面的中心打通一个圆柱形的洞。已知正方体边长为‎10厘米,侧面上的洞口是边长为‎4厘米的正方形,上下侧面的洞口是直径为‎4厘米的圆,求此立体图形的表面积和体积。‎ 审题要点:大正方形减去右边图形就是我们要求的体积。‎ ‎ ‎ 解法:外侧表面积为:6×10×10-4×4×4-×22×2=536-8。‎ 内侧表面积为:16×4×3+2×(4×4-×22)+2×2×2×3=192+32-8+24=224+16。‎ 总表面积=224+16+536-8=760+8=785.12(平方厘米)。‎ 计算体积时将挖空部分的立体图形取出,如图,只要求出这个几何体的体积即可。‎ 挖出的几何体体积为:4×4×4×3+4×4×4+2××22×3=192+64+24=256+24。‎ 所求几何体体积为:10×10×10- (256+24)=668.64(立方厘米)。‎ 专家点评:打通部分可看为两个小圆柱,两个小长方形和一个大长方形共五部分组成,这样计算体积非常容易,但在计算表面积时要考虑公共面。 这道题是人大附中分班考试题目。‎ 总结:本题考点 不规则图形的表面积及体积。‎ ‎【例3】(☆☆☆)一个酒瓶里面深‎30cm,底面内直径是‎10cm,瓶里酒深‎15cm。把酒瓶塞紧后使其瓶口向下倒立这时酒深‎25cm。酒瓶的容积是多少?‎ 审题要点:观察前后,酒瓶中酒的总量没变,即瓶中液体体积不变。‎ 解法: 酒的体积:15π× (10/2) ×(10/2)=375π ‎ 瓶中剩余空间的体积(30-25) π×(10/2)×(10/2)=125π ‎ 酒瓶容积:375π+125π=500π=1500(ml)‎ 专家点评:当酒瓶倒过来时 酒深‎25cm,因为酒瓶深‎30cm,这样所剩空间为高‎5cm的圆柱,再加上原来‎15cm高的酒即为酒瓶的容积。‎ 注意:本题考点立体图形的等积变形。‎ ‎【例4】(☆☆☆☆)如图,ABCD是矩形,BC=‎6cm,AB=‎10cm,对角线AC,BD相交0.图中的阴影部分以CD为轴旋转一周,则阴影部分扫出的立体的体积是多少立方厘米?‎ 审题要点:以CD为轴确定阴影部分旋转后的形状。‎ ‎ ‎ 解法:设三角形BCO以CD为轴旋转一周所得到的立体的体积是V,V等于高为‎10厘米,底面半径是‎6厘米的圆锥的体积减去2个高为‎5厘米,底面半径是‎3厘米的圆锥的体积。‎ 即:(立方厘米),‎ 专家点评:这个立体图形可看为两个圆锥削掉上半部然后叠加,但还要减去两个小圆锥,才是阴影部分扫出的立体图形的真实体积。 可以考虑多种方法,比如应用容斥原理或者加减的思想都是不错的选择。。‎ 总结:本题考点 平面图形旋转为立体图形的体积问题。‎ ‎【例5】(☆☆☆)左下图是一个正方体,四边形APQC表示用平面截正方体的截面。请在右下方的展开图中画出四边形APQC的四条边。‎ 审题要点:把空间图形表面的线条画在平面展开图上,只要抓住四边形APQC 四个顶点所在的位置这个关键,再进一步确定四边形的四条边所在的平面就可容易地画出。‎ 解法:(1)考虑到展开图上有六个顶点没有标出,可想象将展开图折成立体形,并在顶点上标出对应的符号,见左下图。‎ ‎  (2)根据四边形所在立体图形上的位置,确定其顶点所在的点和棱,以及四条边所在的平面:‎ ‎  顶点:A—A,C—C,P在EF边上,Q在GF边上。边AC在ABCD面上,AP在ABFE面上,QC在BCGF面上,PQ在EFGH面上。‎ ‎(3)将上面确定的位置标在展开图上,并在对应平面上连线。需要注意的是,立体图上的A,C点在展开图上有三个,B,D点在展开图上有二个,所以在标点连线时必须注意连线所在的平面。连好线的图形如右上图 专家点评:对照立体图形展开图上,线的位置,取点确定。‎ 总结:本题考点展开图的形状。‎ ‎【例6】(☆☆☆☆)一个3×3×3的正方体。用红、黄、蓝三种颜色去染这些小正方形,要求有公共边的正方形染不同的颜色,那么,用红色染的正方形最多有多少个?‎ 审题要点:涂色与图形结合,首先确定染色范围。‎ 解法:一个面最多有5个方格可染成红色(见左下图)。因为染有5个红色方格的面不能相邻,可以相对,所以至多有两个面可以染成5个红色方格。‎ 其余四个面中,每个面的四个角上的方格不能再染成红色,至多能染4个红色方格(见上中图)。因为染有4个红色方格的面也不能相邻,可以相对,所以至多有两个面可以染成4个红色方格。最后剩下两个相对的面,每个面最多可以染2个红色方格(见右上图)。所以,红色方格最多有5×2+4×2+2×2=22(个)。‎ 专家点评:注意,单面最多只能为五个,与其对称的面也可为五个,与其相邻的面最多为四个,相邻面的对称面也为四个,剩下的两个对称面每面最多为2个,总计22个。‎ ‎ ‎ ‎【例7】(☆☆☆☆)将一个棱长为整数的(单位:分米)的长方体6个面都涂上红色,然后把它全部切成棱长为1分米的小正方体。在这些小正方体中,6个面都没有涂红色的有12块,仅有两个面涂红色的有28块,仅有一面涂红色的有____块。原来长方体的体积是____立方分米。‎ 审题要点:芯是本题的关键 从芯入手。‎ 解法: 12被3个整数整拆只有4种情况 ‎ 1×1×12 1×2×6 1×3×4 2×2×3‎ 两面涂红的有28块, 因为正方体长,宽,高都有4条,所以长宽高之和为 ‎284=7 符合条件的 只有2+2+3=7 所以芯为2×2×3的长方体 ‎ 一面涂红的为 (2×2+2×3+2×3) ×2=32(个)‎ 原体积 (2+2) ×(3+2) ×(2+2)=80(立方分米)‎ 专家点评:无色必为芯,根据已知12个芯,确定芯的大小,应用“角三,棱二,面唯一”计算出三面、二面、一面的数量。原体积为芯的长宽高各加2再相乘。 这道题是第八届小学生数学报数学竞赛决赛的题。‎ 注意:染不到颜色的地方只能在里面哦!‎ ‎【例8】(☆☆☆)如下图,用若干块单位正方体积木堆成一个立体,小明正确地画出了这个立体的正视图、俯视图和侧视图,问:所堆的立体的体积至少是多少?‎ 审题要点:整体观察 发挥想象。‎ 解法:本题还原的技巧在于反用“切片法”,根据俯视图,最底层必有这么十一个,这是不能再少的。‎ 第二步,不妨先根据正视图,再在一侧加上7块,‎ 第三步,再根据侧视图,说明另一侧至少要加上一块,‎ 最后,注意“最少”,把“躲”在后面的去掉,即成如图所示。‎ 当然,这里的形状不唯一。‎ 专家点评:以俯视图为标准,三行当中,中间行至少有2块,上行至少6块,下行至少10块,此时才能满足正视图和侧视图。‎ 注意:本题考点 切片法。‎ ‎【例9】(☆☆☆)现有一个棱长为‎1cm的正方体,一个长宽各为‎1cm,高为‎2cm的长方体,三个长宽各为‎1cm,高为‎3cm的长方体。下列图形是把这五个图形合并成某一立体图形时,从上面、前面、侧面所看到的图形。试利用下面三个图形把合并成的立体图形(如例)的样子画出来,并求出其表面积。‎ 审题要点:用阳光照面的方法展开图形。‎ ‎  ‎ 解法:立体图形的形状如下图所示。(此题十分经典)‎ ‎  从上面和下面看到的形状面积都为‎9cm2,共‎18cm2;‎ ‎  从两个侧面看到的形状面积都为‎7cm2,共‎14cm2;‎ ‎  从前面和后面看到的形状面积都为‎6cm2,共‎12cm2;‎ ‎  隐藏着的面积有‎2cm2。‎ 一共有18+16+12+2=48(cm2)。‎ 专家点评:画法可先横后竖。表面积可根据上、下、左、右、前、后分别求,最后再作和。‎ 注意:本题考点是不规则立体图型表面积和空间想象力。‎ 真题实战 ‎1、(2004,第二届走进美妙的数学花园”中国青少年数学论坛趣味数学解题技能展示大赛)将NNN(N是正整数)正方体的一些面涂上颜色以后,再将它切割成111的小正方体。已知至少有一面涂色的小正方体恰好占总数的52%,N是多少?‎ 初级提示:一个正整数×52%=另一个正整数, 那么这个正整数必须能被25整除。‎ 深度点拨:那么N必须能被5整除。‎ 全解过程:当N取最小 N=5 正方体有5×5×5=125个小正方体 涂色的小正方体5×5×5×52%=65(个) ‎ ‎ 不可能被涂色的小正方体 3×3×3=27(个) 27+65小于125 成立 ‎ 当N=2×5=10时 , 正方体有10×10×10=1000个小正方体 涂色的小正方体10×10×10×52%=520(个) ‎ ‎ 不可能被涂色的小正方体 8×8×8=512(个) 512+520大于1000 不成立 ‎ 同理N大于10都不成立 ‎ 所以 N=5‎ ‎2、小红的生日舞会,做了一顶圆锥形帽子,要将帽子涂成红色和蓝色,O点为顶点,BC为底面圆直径‎30cm,A点是OB的下三分之一处,OB=‎30cm,从A点出发,CA之间最短的距离之上涂成红色,下边涂成蓝色。那么小红的帽子有多大地方涂的是蓝色?(=3)‎ 初级提示:底面周长为圆锥展开后 扇形的弧长 深度点拨:蓝色面积=圆锥侧面积-红色面积 全解过程: 底面周长=30×=30×3=90‎ ‎ 侧面展开后扇形所在圆的周长=2××30=180 ‎ ‎ 所以侧面展开图为半圆 ‎ 蓝色面积=×30×30×-×(20+20) ×30‎ ‎ =1350-600=750(平方厘米)‎ ‎3、一个正方形纸盒中恰好能放入一个体积为628立方厘米的圆柱,纸盒的容积有多大?(=3.14)‎ 初级提示:设纸盒棱长为 深度点拨:圆柱体积=‎ 全解过程:整理上边式子得 即为纸盒容积。‎ ‎4、图中的立体图形是由14个棱长为‎5cm的立方体组成的,求这个立体图形的表面积?‎ 初级提示:用透视法观察 上、下两个面的面积相等 深度点拨:4个侧面的每个侧面面积为6个小正方形面积 全解过程: 底面棱长5×3=15 上、下两个面的面积=15×15×2=450‎ ‎ 4个侧面面积=4×6×5×5=600‎ ‎ 总面积=450+600=1050(平方厘米)‎ ‎5、圆柱形的售报亭的高和底面直径相等(如图),开一个边长等于底面半径的正方形售报窗口。问窗口处挖去的圆柱部分的面积占圆柱形侧面积的几分之几?‎ 初级提示:窗口上下的弧长为底面圆周长的六分之一 深度点拨:窗口的高为圆柱的高的二分之一 全解过程: 挖去的圆柱部分的面积占圆柱形侧面积的×=‎ ‎6、(北京市第十二届迎春杯)一个正方体木块,棱长是15。从它的八个顶点处各截去棱长分别是1、2、3、4、5、6、7、8的小正方体。这个木块剩下部分的表面积最少是多少?‎ 初级提示:截去一个小正方体,表面积不变。‎ 深度点拨:只有在截去的小正方体的面相重合时,表面积才会减少。‎ 全解过程:所以要使木块剩下部分的表面积尽可能小,应该在同一条棱的两端各截去棱长7与8的小正方体(如图所示),这时剩下部分的表面积比原正方体的表面积减少最多。‎ 剩下部分的表面积最小是: 15×15×6-7×7×2=1252。想想为什么不是15×15×6-7×7-8×8 ? ‎ ‎7、如下图,一个正方体形状的木块,棱长‎1米,沿水平方向将它锯成3片,每片又锯成4长条,每条又锯成5小块,共得到大大小小的长方体60块。那么,这60块长方体表面积的和是多少平方米?‎ 初级提示:我们知道每切一刀,多出的表面积恰好是原正方体的2个面的面积。‎ 深度点拨:现在一共切了(3-1)+(4-1)+(5-1)=9刀,而原正方体一个面的面积1×1=1(平方米),所以表面积增加了9×2×1=18(平方米)。原来正方体的表面积为6×1=6(平方米)。‎ 全解过程:所以现在的这些小长方体的表积之和为6+18=24(平方米)。‎ ‎8、下图是一个棱长为‎2厘米的正方体,在正方体上表面的正中,向下挖一个棱长为‎1厘米的正方体小洞,接着在小洞的底面正中向下挖一个棱长为1/2厘米的正方形小洞,第三个正方形小洞的挖法和前两个相同,棱长为1/4厘米,那么最后得到的立体图形的表面积是多少平方厘米?‎ 初级提示:俯视图发现上表面积就是大正方体的一个面的面积 深度点拨:表面积为大正方体表面积加上3个小正方体的侧面积 全解过程: 2×2×6+1×1×4+××4+××4‎ ‎ = 24+4+1+‎ ‎ =29.25(平方厘米) ‎ ‎9、(2006年香港数学奥林匹克竞赛)如下图给出了一个立体图形的正视图、左视图和俯视图,图中单位为厘米。‎ 立体图形的体积(    )立方厘米。‎ (A) ‎2 (B)2.5 (C)3 (D)3.5‎ 初级提示:首先确定此图形为“不完整的圆柱”,先求出圆柱体积,再求出缺失的半个小圆柱,最后作差。‎ 深度点拨:如图,从给定的正视图、左视图和俯视图可以看出,该立体图形由一个半径为‎1厘米、高为‎1厘米的圆柱和一个半径为‎1厘米、高为‎2厘米的半圆柱组成。。‎ 全解过程:×1×1×(1+2)-×1×1×2=2‎ 这里的要点在于还原,还原的技巧在于先补全,再细雕刻 ‎10、把一个棱长为‎2CM正方体在同一平面的边的中点用线段连接起来,如图。然后把正方体顶点上的三角锥锯掉,请问最后所得的立体图形的表面积的多少平方厘米?(1.732×1.732=3)‎ 初级提示: 所得立体图形表面为6个正方形和8个等边三角形 深度点拨:勾股定理 等边三角形的高的平方=底边的平方-半个底边的平方=底边的平方 全解过程:6个正方形面积=6×(1×1+1×1)=6×2=12‎ ‎ 等边三角形的高的平方=×2= ‎ ‎ 等边三角形的高的平方×底边的平方=×2=3‎ ‎ 所以等边三角形的高×底边=1.732, 等边三角形的面积=1/2×1.732=0.866‎ ‎ 立体图形的表面积=12+8×0.866=18.928 ‎ 第10讲 典型应用题(一)和差倍、年龄、植树问题 一、 知识地图 典型应用题 二、 基础知识 ‎(一) 和差问题:已知两个数的和及两个数的差,求这两个数。‎ 方法①:(和-差)÷2=较小数,和-较小数=较大数 方法②:(和+差)÷2=较大数,和-较大数=较小数 例如:两个数的和是15,差是5,求这两个数。‎ 方法:(15-5)÷2=5,(15+5)÷2=10。‎ ‎(二) 和倍问题:已知两个数的和及这两个数的倍数关系,求这两个数。‎ ‎ 方法:和÷(倍数+1)=1倍数(较小数)‎ ‎1倍数(较小数)×倍数=几倍数(较大数)‎ 或 和-1倍数(较小数)=几倍数(较大数)‎ 例如:两个数的和为50,大数是小数的4倍,求这两个数。‎ 方法:50÷(4+1)=10 10×4=40‎ ‎(三)差倍问题:已知两个数的差及两个数的倍数关系,求这两个数。‎ 方法:差÷(倍数-1)=1倍数(较小数)‎ ‎1倍数(较小数)×倍数=几倍数(较大数)‎ 或 和-1倍数(较小数)=几倍数(较大数)‎ 例如:两个数的差为80,大数是小数的5倍,求这两个数。‎ 方法:80÷(5-1)=20 20×5=100‎ ‎(四)年龄问题 关键①:年龄差不变 例如:今年爸爸比儿子大30岁,明年爸爸比儿子大几岁?‎ 答:还是30岁,爸爸长1岁,儿子也长1岁。‎ 明年父子年龄差=明年爸爸的年龄-明年儿子的年龄 ‎=(今年爸爸的年龄+1)-(今年儿子的年龄+1)‎ ‎ =今年爸爸的年龄+1-今年儿子的年龄-1‎ ‎ =今年爸爸的年龄-今年儿子的年龄 ‎ =30(岁) ‎ 关键②:年龄的倍数关系是变化的。‎ 例如:今年父亲的年龄是儿子年龄的3倍,明年父亲的年龄还是儿子年龄的3倍吗?‎ 答:不是,设今年儿子10岁,设今年父亲30岁,那么明年儿子11岁,父亲31岁, 31÷11=2…9,不是3倍。‎ ‎(五)植树与方阵问题 一、 不封闭型(直线)植树问题 (1) 直线两端植树: 棵数=段数+1=全长÷株距+1;‎ 全长=株距×(棵数-1);‎ 株距=全长÷(棵数-1);‎ 例如:学而思学校附近有一条‎2000米的公路,在路两边每相隔‎50米种一棵树,两端都种,需要多少棵树?‎ 分析:(2000÷50+1)×2=82(棵)‎ (2) 直线一端植树: 全长=株距×棵数;‎ 棵数=全长÷株距;‎ 株距=全长÷棵数;‎ 例如:小熊家门口有一条小路长‎50米,从门口开始在小路的一旁每隔‎5米栽一棵树,问一共栽了多少棵树?‎ 分析:门口不可能植树,所以这是一个一端种树一端不种的情况,棵树等于段数,所以一共栽树:50÷5=10(棵)。‎ (3) 直线两端都不植树: 棵数=段数-1=全长÷株距-1;‎ 株距=全长÷(棵数+1);‎ 例如:学而思学校两栋教学楼之间有一排白杨树,一共有18棵,每两棵树之间以及树与教学楼的距离都是‎3米,请问这两栋教学楼之间的距离是多少米? ‎ 分析:因为两端就是教学楼,不可能种树,所以教学楼之间一共有19个间隔,所以这两栋教学楼之间的距离是3×‎ ‎19=57(米)。‎ 一、 封闭型(圆、三角形、多边形等)植树问题 棵数=总距离÷棵距; ‎ ‎ 总距离=棵数×棵距;‎ ‎ 棵距=总距离÷棵数。‎ 例如:小同家有一个圆形果园,周长是‎1500米,沿圆周每隔‎6米栽一棵苹果树,每两棵苹果树之间栽一棵桃树,问:果园周围共栽种果树多少棵?‎ 分析:果园一周全长‎1500米,每隔‎6米栽一颗苹果树,说明果园的圆周以‎6米为一段,可以分成1500÷6=250(段),由于是圆形,首尾两棵重合,所以段数等于棵数,苹果树有250棵;每两棵苹果树之间栽种一棵桃树,也就是有250棵桃树,所以,苹果树与桃树一共有:250+250=500(棵)。‎ 3. 方阵问题 在方阵问题中,横的排叫做行,竖的排叫做列,如果行数和列数都相等,则正好排成一个正方形,就是所谓的“方阵”。‎ 例如:某校五年级学生排成一个方阵,最外一层的人数为60人。问方阵外层每边有多少人?这个方阵共有五年级学生多少人?‎ 分析:每边人数=四周人数÷4+1,‎ 方阵最外层每边人数:60÷4+1=16(人),‎ 整个方阵共有学生人数:16×16=256(人)。‎ 经典透析 【例1】 ‎(☆☆☆)一个小数的小数点向右移一位与向左移一位所得两数的和624.18,则原来的小数是多少?‎ ‎[审题要点] 本题属于和倍问题。关键是抓住小数点向左移一位,原数就缩小10倍;小数点向右移一位,原数就扩大10倍。‎ ‎[详解过程] 小数点向右移一位所得数是向左移一位所得数的100倍,有624.18÷(100+1)=6.18,6.18×10=61.8,即原数是61.8。‎ 【例2】 ‎(☆☆☆)某校原来参加室外活动的人数比室内的人数多480人,现在把室内活动的50人改为室外活动,这样室外活动的人数正好是室内人数的5倍,参加室内,室外活动的一共有多少人?‎ ‎[审题要点] 本题属于差倍问题 ‎[详解过程] 为了清晰地反映数量的变化及倍数关系,我们画出线段图如下:‎ 把室内50人调到室外,则室外人数比室内人数多480+50×2=580(人),又因为室外人数是室内人数的5倍,也就是多4倍,所以现在室内人数为580÷(5-1)=145(人),一共有145×(1+5)=870(人)。‎ 【例1】 ‎(☆☆☆)小新用20元钱买了5支圆珠笔和12本练习本,剩下的钱若买一支圆珠笔少4角;若买一本练习本还少6角,问一支圆珠笔的价钱是 。‎ ‎[审题要点] 和倍、差倍问题的综合运用 ‎[详解过程] 练习本和圆珠笔的差价为2角。而20元加上4角能买6只圆珠笔和12本练习本。所以如果用20+0.4+0.2×6=21.6元能买18本练习本,每本的价钱为21.6÷18=1.2元,所以圆珠笔的价钱为1.2-0.2=1元。 ‎ 【例2】 ‎(☆☆☆)四个人年龄之和是87岁,最小的一个12岁,他与最大的人年龄之和比另外两个人年龄之和大7岁,那么这四个人中年龄最大的一个年龄是多少? ‎ ‎[审题要点] 把年龄最小的人与年龄最大的人的年龄之和看成一个数,把另外两个人年龄之和也看成一个数。问题就转化为典型的和差同题。‎ ‎[详解过程] 最小的一个与最大的人年龄之和是:(87+7)÷2=47(岁)。最小的12岁,因此最大的年龄:47-12=35(岁)。‎ 【例3】 甲对乙说:“我在你这么大岁数的时候,你的岁数是我今年岁数的一半。”乙对甲说:“我到你这么大岁数的时候,你的岁数是我今年岁数的2倍减7。”问:甲、乙二人现在各多少岁?‎ ‎[审题要点] 甲乙年龄差不变、从已知条件中看出甲比乙年龄大。‎ ‎[详解过程] 为了清晰地反映等量关系,我们画出线段图如下:‎ ‎ ‎ ‎ 从图中可以得到年龄差是7岁,所以,乙现在年龄: 7×3=21(岁),甲现在年龄: 7×4=28(岁)。‎ 【例4】 老陈有几个儿子,老陈的年龄是儿子们年龄和的4.5倍,而1年前,老陈的年龄是他的几个儿子年龄和的7倍,4年后,老陈的年龄就只有他几个儿子的年龄和的2倍,那么老陈有几个儿子?‎ ‎[审题要点] 借助于方程法解决 ‎[详解过程]‎ ‎ 设老陈有n个儿子,则今年老陈的年龄是儿子们平均年龄的4.5n倍,而1年前老陈的年龄是儿子们平均年龄的7n倍,4年后,老陈的年龄是他的几个儿子的平均年龄的2n倍,由于老陈的年龄与儿子们的平均年龄之差是固定的,所以我们以老陈的年龄与儿子们的平均年龄之差为标准,设为,则今年儿子们平均年龄,4年后儿子们的平均年龄,得到方程式:-=(-),解得。‎ 另解:老陈有n个儿子今年儿子们的年龄和为k岁,则4.5k-1=(k-n)×7 ⑴‎ ‎ 4.5k+4=(k+4 n)×2 ⑵, 解得:k=8,n=3。‎ ‎ ‎ 【例1】 在学而思学校内一条小路的一侧植树,每隔‎5米种一棵,共种了21棵,这条路有多长?后来小路又加长了‎30米,仍然每隔‎5米种一棵树,一共补种了多少棵?‎ ‎[审题要点] 加长部分、求得是补种几棵树 ‎[详解过程] 小路原来的长度:5×(21-1)=100(米), 加长后一侧应种的树的棵数:(100+30)÷5+1=27(棵),应补的棵数:27-21=6(棵)。‎ 【例2】 把50枚黑棋子排列在正五边形的五条边上,每条边上的黑棋子个数相等,且每个角上有一枚。然后在所有相邻的两枚黑棋子间放两枚白棋子。问:每条边上白棋子有多少枚?‎ ‎[审题要点] 每个角上有一枚,求出实际每条边几枚 ‎[详解过程] 一共有50枚棋子,放在5条边上,所以平均每条边上放50÷5=10枚黑棋子,又因为每个角上都有一枚棋子,所以实际上每条边上有10+1=11枚黑棋子。11枚黑棋子之间有10个间隔,所以白棋子数是10×2=20(枚)。‎ 【例3】 一个实心正六边形阵,每条边有16人,那么一共有 人;最外面一层有 人;从外向内数第2层每条边有 人,共 人;最外面三层有 人;每条边增加1人,这一层增加 人;原正六边形方阵再增加一层能增加 人;‎ ‎[审题要点] 找规律 ‎[详解过程] 从内往外,第一层1人,第二层每边2人,共6人;第三层每边3人,共12人;第四层每边4人,共18人;…;第十四层每边14人,共78人;第十五层每边15人,共84人;第十六层每边16人,共90人。原实心正方形阵共有1+6+12+…+90=721人。 ‎ 从外往内数第二层就是从内往外数第十五层,每边15人,共(15-1)×6=84人。‎ 最外面三层有90+84+78=252人。每条边增加1人,这一层增加6人。原六边形方阵再增加一层能增加(17-1)×6=96人。‎ 一、 拓展训练 1. 姐姐做自然科学练习,比妹妹做算术练习多用48分钟,比妹妹做英语练习多用42分钟,妹妹做算术,英语两门练习共用了44分钟,那么妹妹做英语练习用了多少分钟?‎ ‎[初级点拨] 本题属于典型的和差问题,只是“差”没有直接告诉我们,绕了个小弯。‎ ‎[深度提示] 根据条件,做英语的时间大于算术的时间。‎ ‎[全解过程] 因为“姐姐做自然科学练习,比妹妹做算术练习多用48分钟,比妹妹做英语练习多用42分钟”,所以妹妹做英语比算术多用了48-42=6(分钟)。画线段图:‎ ‎(44-6)÷2=19(分钟) 算术 ‎(44+6)÷2=25(分钟) 英语 答:妹做英语用25分钟。‎ 1. 在一次期中考试中,小强的英语成绩和数学成绩之和是194分,他的数学成绩和语文成绩之和是186分,而语文成绩和英语成绩之和是180分,那么,小强的英语、数学和语文成绩到底各是多少?‎ ‎[初级点拨] 由条件知:英语成绩+数学成绩=194,数学成绩+语文成绩=186,语文成绩+英语成绩=180,‎ ‎[深度提示] 将三个式子相加 ‎[全解过程] 由条件知:英语成绩+数学成绩=194,数学成绩+语文成绩=186,语文成绩+英语成绩=180,将三个式子相加,2(英语成绩+数学成绩+语文成绩)=560(分),所以英语成绩+数学成绩+语文成绩=560÷2=280(分),用总成绩减去英语成绩和数学成绩之和就是语文成绩:280-194=86(分),同理,英语成绩:280-186=94(分),数学成绩:280-180=100(分)‎ 2. 某学校计划栽种杨树、柳树和槐树共200棵,当种了一半的杨树和10棵柳树之后,又临时运来了6棵槐树,这时剩下的三种树的棵树恰好相等,问原计划要栽种这三种树各多少棵?‎ ‎[初级点拨] 如果没有栽种之前运走10棵柳树,并且运来6棵槐树,那么树的总数就是:200-10+6=196(棵)。‎ ‎[深度提示] 柳树的数量等于槐树的数量等于杨树数量的一半。‎ ‎[全解过程] 为了清晰地反映数量关系,我们画出线段图如下:‎ 树的总数就是:200-10+6=196(棵),柳树的数量等于槐树的数量等于杨树数量的一半,令杨树的一半为一倍数,即为:195÷(2+1+1)=196÷4=49(棵),所以计划种杨树:49×2=98(棵),柳树:49+10=59(棵),槐树:49-6=43(棵)。‎ 1. 今年爷爷78岁,三个孙子的年龄分别是27岁,23岁,16岁,经过几年后爷爷的年龄等于三个孙子年龄的和? ‎ ‎[初级点拨] 今年爷爷与三个孙子的年龄差 ‎[深度提示] 每过一年三个孙子的年龄和比爷爷的年龄增加几岁 ‎[全解过程] 三个孙子年龄的和为27+23+16=66(岁),爷爷比他们三人的年龄的和多78-66=12(岁),每过一年三个孙子的年龄和比爷爷的年龄多增加3-1=2(岁)。因而,经过12÷2=6(年)后,爷爷的年龄是三个孙子年龄的和。‎ 2. 甲对乙说:“当我的岁数是你现在的岁数的时候,你才5岁。”乙对甲说:“当我的岁数是你现在的岁数的时候,你将50岁。”问:甲、乙二人现在各多少岁?‎ ‎[初级点拨] 年龄差不变 ‎[深度提示] 每一次两人变化的年龄都相等,且是年龄差 ‎[全解过程] 根据题意画出示意图:‎ 因为年龄差是不变的量,甲乙二人的年龄差=(50-5)÷3=15(岁),乙现在的岁数是:15+5=20(岁),甲现在的岁数是:20+15=35(岁)‎ 3. 全家4口人,父亲比母亲大3岁,姐姐比弟弟大2岁。四年前他们全家的年龄和为58岁,而现在是73岁。问:现在各人的年龄是多少?‎ ‎[初级点拨] 年龄差不变 ‎[深度提示] 为什么少了1岁?‎ ‎[全解过程] 73-58=15≠4×4,四个人四年应该增长4×4=16岁,但实际上只增长了15岁,为什么少了1岁呢?因为在4年前,弟弟还没有出生,所以弟弟今年应该是3岁。姐姐是3+2=5岁,父母今年的年龄和是73-3-5=65岁,根据和差问题,就可以得到父亲是(65+3)÷2=34岁,母亲是65-34=31岁。‎ 4. ‎“重阳节”‎ 那天,延龄茶社来了25位老人品茶,他们的年龄恰好是25个连续自然数,两年以后,这25位老人的年龄之和正好是2000,其中年龄最大的老人今年多少岁? ‎ ‎[初级点拨] 求出两年之后25位老人的平均年龄 ‎[深度提示] 奇数个数的平均数是中间数 ‎[全解过程] 两年之后25位老人的平均年龄为2000÷25=80(岁),其中年龄最大的老人为80+12=92(岁),年龄最大的老人今年的岁数为92-2=90(岁)。‎ ‎8.大头儿子和小头爸爸两个人比赛跑楼梯,他们从一层开始比赛,大头儿子到四层时,小头爸爸到三层,如此算来,大头儿子到16层时,小头爸爸跑到了几层?‎ ‎[初级点拨] 不封闭型植树问题 ‎[深度提示] “两端都种树”,间隔数=棵树-1‎ ‎[全解过程] 大头儿子跑了三个楼层间隔,爸爸跑了两个楼层间隔,到16层需要跑15个楼层间隔,所以小头爸爸跑了15÷3×2+1=10+1=11(层)。‎ ‎9.如图是某个小区的街道图,街道将整个小区划分为相同的4块正方形,每个正方形的边长为‎110米,街道的宽为‎10米,现在要在所有的街道两边每隔‎10米栽种一棵树,每个拐角都栽树,求这个小区一共要栽树多少棵?‎ ‎[初级点拨] 分解图形 ‎[深度提示] 每个拐角都栽树 ‎[全解过程] 整个小区种植的树实际上可看成4个边长为‎110米的小正方形和一个边长为10+110+10+110+10=‎250米的正方形。所以一共需要栽树(110×4÷10)×4+(250×4÷10)=276棵树。‎ ‎10.正方形操场四周栽了一圈树,四个角上都栽了树,每两棵树相隔‎5米。甲、乙从一个角上同时出发,向不同的方向走去(如右图),甲的速度是乙的2倍,甲在拐了两个弯之后的第5棵树与乙相遇(把角上的树看作第一棵树)。操场四周栽了多少棵树?‎ ‎[初级点拨] 封闭型植树问题 ‎[深度提示] 时间一定,路程比等于速度比 ‎[全解过程] 甲走了两个边长加上4个间距,乙走了两个边长减去4个间距,所以甲比乙多走了8个间距,而甲的速度是乙的2倍,所以走的路程也是乙的两倍,所以乙走了8个间距,所以一圈一共有8+8×2=24个间距,所以操场一圈一共有24个间距。操场四周一共栽了24棵树。‎ ‎11.北京市国庆节参加游行的总人数有60000人,这些人平均分为25队,每队又以12人为一排列队前进。排与排之间的距离为‎1米,队与队之间的距离是‎4米,游行队伍全长多少米?‎ ‎[初级点拨] 不封闭型植树问题 ‎[深度提示] 相当于已知树的棵数,树间的距离,求树列的全长 ‎[全解过程] 相当于植树问题中已知树的棵数,树间的距离,求树列的全长相当。注意段数比树的株数少1。所以,‎ ‎ (1)每队的人数是:   60000÷25=2400(人)‎ ‎   (2)每队可以分成的排数是:   2400÷12=200(排)‎ ‎   (3)200排的全长米数是:   1×(200-1)=199(米)‎ ‎   (4)25个队的全长米数是:   199×25=4975(米)‎ ‎   (5)25个队之间的距离总米数是:4×(25-1)=96(米)‎ ‎   (6)游行队伍的全长是:   4975+96=5071(米)‎ 第11讲 典型应用题(二)鸡兔同笼、盈亏、平均数问题 一、 知识地图 二、 基础知识 公元855年唐朝,我国举行最早的数学选拔赛,题目如下:‎ 一批强盗在树林里商议怎样瓜分抢来的布匹。若每人分6匹,多5匹;每人分7匹,少8匹,问几个强盗?几匹布?‎ (一) 鸡兔同笼问题 1. 假设全是鸡 例如:鸡兔同笼,头共46,足共128,鸡兔各几只?‎ 分析:假设全是鸡,则有2×46=92(足),而实际上是128足,少了128-92=36(足),为什么少了36足呢?因为我们把一只兔当作一只鸡来算时,就少算了2足,所以有36÷2=18(只)兔被我们当作鸡来算,所以有鸡46-18=28(只)。‎ 2. 假设全是兔 例如:鸡兔同笼,头共46,足共128,鸡兔各几只?‎ 分析:假设全是兔,则有4×46=184(足),而实际上是128足,多了184-128=56(足),为什么多了56足呢?因为我们把一只鸡当作一只兔来算时,就多算了2足,所以有56÷2=28(只)鸡被我们当作兔来算,所以有兔46-28=18(只)。‎ 1. ‎“砍足法”‎ 例如: 鸡兔同笼,头共46,足共128,鸡兔各几只?‎ 分析:假如砍去每只鸡、每只兔一半的足,则鸡就变成了“独脚鸡”,兔就变成了“双脚兔”,则鸡和兔足的总数就由128变成了64,而且有一只兔子,则足的总数就比头的总数多1,所以足的总数64与总头数46的差,就是兔子的只数,即64-46=18(只),则鸡的只数就是46-18=28(只)。 ‎ (一) 盈亏问题 盈亏问题,顾名思义有剩余就叫盈,不够分就叫亏,不同的方法分配物品时,经常会产生这种盈亏现象。盈亏问题的关键是抓住两次分配时盈亏总量的变化,我们把盈亏问题分为三类:“一盈一亏”、“两盈”、“两亏” 。‎ ‎1.“盈亏”型 例如:学而思学校提高班的同学分糖果,如果每人分4粒就多9粒,如果每人分5粒则少6粒,问:有多少位同学分多少粒糖果?‎ 分析:为什么第一次多9粒,而第二次还少6粒呢?因为两次分配数量不一样,第二次分配时不仅把第一次多出来的9粒分了,还要再添6粒才够分,也就是说按第二种分配方案比第一次总共要多分9+6=15(粒),那为什么会有这种变化产生呢?因为第二次比第一次每人多分了5-4=1(粒),那么要分15粒,就需要有15÷1=15(人),共有15×4+9=69(粒)。‎ ‎2.“盈盈”型 明明过生日,同学们给他买蛋糕,如果每人出8元,就多出了8元;每人出7元,就多出了4元。那么有多少个同学?蛋糕的价钱是多少?‎ 分析:为什么第一次多8元,第二次就只多4元了呢?因为两次分配数量不一样,第二次分配时每人少出1元,也就是在第一次分配的基础上给每个人退了1元钱,总共退回了8-4=4(元),所以共有4÷1=4(人),蛋糕价钱是8×4-8=24(元)。‎ ‎3.“亏亏”型 学而思学校新近一批书,将它们分给几位老师,如果每人发10本,还差9本,每人发9本,还差2本,请问有多少老师?多少本书?‎ 分析:为什么第一次差9本,第二次就只差2本了呢?因为两次分配数量不一样,第二次分配时每人少发1本,也就是在第一次分配的基础上从每个人那里拿回了1本书,总共拿回了9-2=7(本)书,所以共有7÷1=7(人),书有7×10-9=61(本)。‎ (二) 平均数问题 ‎(1)平均数=总数÷参与平均的事物个数 平均数增量=总数增量÷参与平均的事物个数 平均数减量=总数减量÷参与平均的事物个数 ‎(2)平均数问题最基本的原理是“移多补少”‎ 几个数的平均数一定比其中最大的一个小且比其中最小的一个大 一、 经典透析 【例1】 从前有座山,山里有个庙,庙里有许多小和尚,两个小和尚用一根扁担一个桶抬水,一个小和尚用一根扁担两个桶挑水,共用了38根扁担和58个桶,那么有多少个小和尚抬水?多少个挑水?‎ ‎[审题要点] 鸡兔同笼问题,假设法 ‎[详解过程] 假设全是抬水,38根扁担应担38个桶,而实际上是58个桶,为什么少了58-38=20(个)桶呢?因为当我们把一个挑水的当作抬水的就会少算2-1=1(个)桶,所以有20÷1=20(人)在挑水,抬水的扁担数是38-20=18(根),抬水的人数是18×2=36人。‎ 专家点评:‎ ‎ 可以结合分析工具矩形图,来看鸡兔同笼问题:‎ ‎ ‎ 左图假设全是抬水: (58-38×1)÷(2-1)=20(根) ……20(人)挑水 ‎ (38 -20)×2=36(人) ……36(人)抬水 右图假设全是挑水: (38×2-58)÷(2-1)=18(根) ……18×2=36(人)抬水 ‎ 38-18=20(根)…… 20(人)挑水 ‎【例2】某旅游点有儿童票、成人票两种规格的门票卖, 儿童票的价格为30元,成人票的价格为40元,如果是团体还可以买平均32元一位的团体票,一个由8个家庭组成的旅游团(每个家庭由两位大人,或两个大人、一个小孩组成)来景点旅游,如果他们买团体票可以比他们各买各的少花120元,问这个旅游团一共有多少人? ‎ ‎[审题要点] 鸡兔同笼问题的变形题 ‎[详解过程] 每个三口之家可以少花30+40+40-32×3=14元,每个二口之家可以少花40+40-64=16元,如果这8个家庭都是三口之家,那么一共少花14×8=112元,所以这8个家庭中有(120-112)÷(16-14)=4个家庭是二口之家,所以这个旅游团一共有4×2+(8-4)×3=20人。‎ 专家点评:‎ ‎ 这道题,首先要考虑的是,怎么理解“少花120元”?跟单位少花情况有关,这里的单位:可以不同家庭为单位,也可以成人与小孩为单位。‎ ‎ 一方面,我们可以对两种家庭的“少花”情况进行计算并比较,可以如题所解;‎ ‎ 另一方面,我们不妨以成人与孩子的“少花”情况进行计算并比较,可以另解如下:‎ ‎ 8个家庭,成人必有16人,则每个成人将“少花”40-32=8元。‎ ‎ 所以应该总共少花 16×8=128(元)‎ ‎ 而实际少花相差 128-120=8(元)‎ ‎ 是因为每个小孩多花了32-30=2(元)‎ ‎ 所以,8÷2=4(人) ……小孩人数 ‎ 16+4=20(人)……旅游团一共人数 ‎ 还有一点值得强调的是,我们在使用假设法的过程中,所采用的比较思想非常重要,在一种证明方法——反证法中,假设法会又一次充当主角。‎ ‎【例3】蜘蛛有8条腿,蜻蜓有6条腿和2对翅膀,蝉有6条腿和1对翅膀。现有蜘蛛、蜻蜓和蝉三种小虫16只,共有110条腿和14对翅膀,每种小虫各有几只?‎ ‎[审题要点] 经典鸡兔同笼问题,用两次假设法 ‎[详解过程] 因为有三种动物,没有办法直接用鸡兔同笼解,所以我们想转化为两种动物就可以直接用了。我们先来看腿,发现蜻蜓和蝉有个共同点——都是6条腿,那我们就把蜻蜓和蝉合并在一起,分为两种动物:一种是6条腿,一种是8条腿。‎ 假设全是6条腿的,共有腿6×16=96(条),而实际上是110条,为什么少了110-96=14(条)腿呢?因为当我们把8条腿的蜘蛛当作6条腿算的,有一只蜘蛛就少算2条腿,所以有蜘蛛14÷2=7(只),所以蜻蜓和蝉有16-7=9(只);‎ 我们再来看翅膀: 假设这9只全是蜻蜓,则应该有9×2=18(对)翅膀,比实际多了18-14=4(对),所以有蝉4÷1=4(只),则蜻蜓9-4=5(只)。‎ 专家点评:‎ ‎ 如果我们感觉这样的算术解法有点烦,不妨看看美丽的方程:‎ ‎ 设:蜘蛛有只,蜻蜓有y只,蝉有z只,得:‎ ‎ ‎ ‎(1)×6:‎ ‎(2)-(4):‎ ‎2=14‎ ‎=7‎ 代入(1)式:‎ y+z=9…(5)‎ ‎(3)-(5):‎ y=5。‎ 代入(5)式:‎ z=4。‎ 很多时候,我们发现清晰的等量关系,一定要用,从而可以减少“算理”的思考量,把这种思考量转嫁给方程演算。‎ 对于方程演算,不需要掌握太多的技巧,就能轻松把握。请参见本书第十九讲《方程》。‎ ‎【例4】‎ 老师给同学们分苹果,每人分10个,就多出8个,每人分11个则正好分完,那么一共有多少名学生?多少个苹果?‎ ‎[审题要点] 盈亏问题 ‎[详解过程] 为什么第一次多8个,第二次不多也不少了呢?因为第二次每人多分了1个,所以有8÷1=8(人),苹果8×10+8=88(个)。‎ 专家点评:请注意体会差量分析的应用。这是两种方案之间的差异,而假设法是实际与假设之间的差异,两者有着异曲同工之妙。‎ ‎【例5】皮皮从家到学校,如果每分钟走‎50米,上课就要迟到3分钟;如果每分钟‎60米,就可以比上课时间提前2分钟到校,那么皮皮家距离学校多远?‎ ‎[审题要点] 需要转化条件的盈亏问题 ‎[详解过程] 根据题意,每分钟走‎50米,迟到3分钟,实际上就是还差50×3=150(米)到校;如果每分钟‎60米,提前2分钟到校,即到校后还可以多走60×2=120(米),第一次与第二次相差150+120=270(米),也就是第二次比第一次多走了‎270米,所以皮皮从家到学校所用时间是270÷(60-50)=27(分钟),皮皮家到学校的距离是50×(27+3)=50×30=1500(米)。‎ 专家点评:两种方案,除了速度差,更要感受到路程差,从而看到,这里的数量关系,竟然就是追及关系。从中体会一下“柳暗花明又一村”的数学美感吧。数学是好玩的!‎ ‎【例6】国庆节快到了,学而思学校的少先队员去摆花盆。如果每人摆5盆花,还有3盆没人摆;如果其中2人各摆4盆,其余的人各摆6盆,这些花盆正好摆完。问有多少少先队员参加摆花盆活动,一共摆多少花盆?‎ ‎  ‎ ‎[审题要点] 需要转化条件的盈亏问题 ‎[详解过程] 我们可以把第二个条件转化为如果每人摆6盆花,还缺4盆,那么就是简单的“一盈一亏”。‎ 人数: [3+(6-4)×2]÷(6-5)=7(人),盆数:5×7+3=38(盆)或6×7-4=38(盆)。‎ 专家点评:‎ ‎ 转化思想似乎有点玄,为什么我一定会想到:“把第二个条件转化为如果每人摆6盆花,还缺4盆”?答案在于,我们应该在大方向上有感觉,这道题“每人摆5盆,还有3盆没人摆;每人摆6盆,还……”,“还”字后面的下文怎么接?接上了,转化成功!‎ ‎ 记住:转化的关键在于我需要什么样的条件!‎ ‎ 现有条件能否转化为我要的条件?‎ ‎【例7】有四个数,每次去掉一个数,将其余三个数求平均数,这样算了四次,得下面四个数:36.4,47.8,46.2,41.6,那么原来四个数的平均数是多少?‎ ‎[审题要点] 平均数问题 ‎[详解过程] 设这四个数分别为A、B、C、D,根据条件则有:‎ 所以 ‎[专家点评] 实际上,本题的情境可以换成“小明语文、数学、英语等几门功课的平均分”,也可以换成“某四个小朋友称体重,每三个人称一次”,数量关系不变。‎ 这里要注意所求问题,不一定最后求平均数,也可能求这四个数各是多少。只要用四数总和与三数之和求差就行。‎ ‎【例8】某次数学竞赛原定一等奖10人,二等奖20人,现在将一等奖中最后4人调整为二等奖,这样得二等奖的学生的平均分提高了1分,得一等奖的学生的平均分提高了3分,那么原来一等奖平均分比二等奖平均分多________分。‎ ‎[审题要点] 平均数增量 ‎[详解过程] 第一眼看这样的图,可能有点不够清楚。别急,我们来慢慢欣 赏!首先从总体来看,矩形横向长度表示人数,竖向长度表示平均分,面积 表示总分。请注意一下:d与e分别表示调整前的一等奖与二等奖的平均分;‎ 而a表示一等奖后4名同学的平均分。b与c表示调整后一等奖与二等奖的平 均分。我们要求的量是de之间的平均分之差!‎ 我们要想一想,为什么这么一调整,一等奖的平均分高上去了,同时二等奖的平均分也高上去了呢?原因在于:前6名的 cd之间的面积移补到一等奖后4名da之间的面积部分了。根据面积相等,长与宽成反比关系,可知:cd之间的高度差︰da之间的高度差=4︰6=2︰3即3︰da之间的平均分之差=2︰3。所以 da之间的平均分之差=4.5(分),也就是说,这是后4名现在从原来的d降了4.5分。同理,后4人ab之间的面积=20人be之间的面积;所以 ab之间的高度差︰be之间的高度差=20︰4=5︰1所以 ab之间的平均分之差︰1=5︰1,ab之间的平均分之差=5(分)‎ 所以de之间的平均分之差为4.5+5+1=10.5(分)‎ ‎[专家点评]对于平均数增量问题,用矩形图,数形结合去分析,应该很舒服!要注意平均数问题最基本的原理是“移多补少”,另外要注意所要移补的是总量,而不是平均量。也就是平均分差量与人数的乘积。这段话请结合上面的图形和分析理解,重要!!‎ ‎ ‎ ‎【例9】设四个不同的正整数构成的数组中,最小的数与其余三数的平均值之和为17,而最大的数与其余三数的平均值之和为29。在满足上述条件的所有数组中,其最大数的最大值是多少?‎ ‎[审题要点] 平均数与最值问题 ‎[详解过程] 设这四个数从大到小依次为a、b、c、d,根据题意有 ‎ 。 ①‎ ‎ ,        ②‎ 用②式减去①式,得 ‎ ,‎ 即a-d=18,a=18+d。‎ 因为b、c 分别至少比d大2和1,由①式得 ‎ 7+2d≤17,‎ ‎ d≤5。‎ 由此得a=18+d≤23。所以a的最大值23,且当a、b、c、d依次为23,7,6,5时符合题意。‎ 专家点评:‎ 这里的所谓平均数,直接应用为表示3个数的总和。这是平均数关系中知道几个数时最常用的思路。‎ 另外,对于不等式的求解,建议大家在理解了方程的恒等关系后,一并了解方程的恒不等关系。‎ 不等式两边同时加上相同的数或者同时减去相同的数,或者同时乘以相同的正整数或者同时除以相同的正整数,其不等关系不变。(原来是什么符号,不用变号)‎ 如果是乘以或者除以一个相同的负数,则符号正好变反。这到初中会常用到。‎ 例如:7+2d≤17,‎ ‎ 两边同减7,得:‎ ‎ 2d≤10,‎ ‎ 两边同除以2,得:‎ ‎ d≤5。‎ ‎ ‎ 一、 拓展训练 1. 鸡、兔共笼,鸡比兔多26只,足数共274只,问鸡、兔各几只?‎ ‎[初级点拨] 鸡兔同笼问题,假设法 ‎[深度提示] 设鸡与兔只数一样多 ‎[全解过程] 设鸡与兔只数一样多:274-2×26=222(只),每一对鸡、兔共有足:2+4=6(只),‎ 鸡兔共有对数(也就是兔子的只数):222÷6=37(对),则鸡有 37+26=63(只)。‎ 2. ‎100个和尚140个馍,大和尚1人分3个馍,小和尚1人分1个馍。问:大、小和尚各有多少人?‎ ‎[初级点拨] 鸡兔同笼问题,假设法 ‎[深度提示] 将大和尚、小和尚分别看作鸡和兔,馍看作腿 ‎[全解过程] 本题即中国古算名题“百僧分馍问题”。如果将大和尚、小和尚分别看作鸡和兔,馍看作腿,那么就成了鸡兔同笼问题,可以用假设法来解。‎ ‎ 假设100人全是大和尚,那么共需馍300个,比实际多300—140=160(个)。现在以小和尚去换大和尚,每换一个总人数不变,而馍就要减少3—1=2(个),因为160÷2=80,故小和尚有80人,大和尚有100—80=20(人)。‎ 3. 有两次自然测验,第一次24道题,答对1题得5分,答错(包含不答)1题倒扣1分;第二次15道题,答对1题8分,答错或不答1题倒扣2分,小明两次测验共答对30道题,但第一次测验得分比第二次测验得分多10分,问小明两次测验各得多少分?‎ ‎[初级点拨] 需要转化的鸡兔同笼问题,找相同点转化 ‎ ‎[深度提示] 如果小明第一次测验24题全对 ‎[全解过程] 如果小明第一次测验24题全对,得5×24=120(分)。那么第二次只做对30-24=6(题)得分是8×‎ ‎6-2×(15-6)=30(分)。两次相差120-30=90(分)。比题目中条件相差10分,多了80分。说明假设的第一次答对题数多了,要减少。第一次答对减少一题,少得5+1=6(分),而第二次答对增加一题不但不倒扣2分,还可得8分,因此增加8+2=10分。两者两差数就可减少6+10=16(分)。(90-10)÷(6+10)=5(题)。因此,第一次答对题数要比假设(全对)减少5题,也就是第一次答对19题,第二次答对30-19=11(题)。第一次得分5×19-1×(24- 19)=90。第二次得分8×11-2×(15-11)=80。‎ 1. 学而思学校提高班的同学去划船。他们算了一下,如果增加一条船,正好每条船坐6人;如果减少一条船,正好每条船坐9人。问:这个班共有多少同学?‎ ‎ ‎ ‎[初级点拨] 盈亏问题,先增加一条船 ‎[深度提示] 先增加一条船,那么正好每条船坐6人。然后去掉两条船,就会余下6×2=12(名)同学。‎ ‎[全解过程] 先增加一条船,那么正好每条船坐6人。然后去掉两条船,就会余下6×2=12(名)同学。改为每条船9人,也就是说,每条船增加9-6=3(人),正好可以把余下的12名同学全部安排上去,所以现在还有12÷3=4(条)船,而全班同学的人数是9×4=36(人)。‎ 2. 学而思学校给参加秋游的同学租了几辆大轿车,若每辆车乘28人则有13名同学上不了车,若每辆车乘32人则还有3个空座。问:有多少名同学?多少辆车?‎ ‎[初级点拨] 需要转化的盈亏问题,“每辆车乘28人则有13名同学上不了车”转化为盈还是亏呢?‎ ‎[深度提示] 已知若每辆车乘28人则有13名同学上不了车,可转化为:每辆车乘28人多出13名同学;若每辆车乘32人则还有3个空座,可转化为:每辆车乘32人少3人。‎ ‎[全解过程] 这种类型的题目要将其中的一个条件转化,使之转化为基本的盈亏问题。‎ 已知若每辆车乘28人则有13名同学上不了车,可转化为:每辆车乘28人多出13名同学;若每辆车乘32人则还有3个空座,可转化为:每辆车乘32人少3人,问有多少名学生多少辆车?所以,车数:(13+3)÷(32-28)=4(辆),学生有:28×4+13=125(人)。‎ 3. 钢笔与圆珠笔每支相差1元2角,小明带的钱买5支钢笔差1元5角,买8支圆珠笔多6角。问小明带了多少钱?‎ ‎[初级点拨] 需要转化的盈亏问题,要么都转换成钢笔,要么都转换成圆珠笔。‎ ‎[深度提示] 都转换成钢笔;买5支钢笔差15角,买8支钢笔差(12×8-6)=90角,这是双亏:分差是8-5=3支,总差是90-15=75角,就是说多买3支,就多差75角;‎ ‎[全解过程] 此题的关键在于条件的转换,要么都转换成钢笔,要么都转换成圆珠笔。‎ ‎(法一)都转换成钢笔;买5支钢笔差15角,买8支钢笔差(12×8-6)=90角,这是双亏:分差是8-5=3支,总差是90-15=75角,就是说多买3支,就多差75角;这样就可求出1支钢笔多少钱;继而求出小明带了多少钱。‎ ‎  钢笔的价钱: [(12×8-6)-15]÷(8-5)=75÷3=25(角)‎ ‎  小明带的钱数:25×5-15=125-15=110(角)=11(元)‎ ‎(法二)都转换成圆珠笔;买5支圆珠笔多12×5-15=45角,买8支圆珠笔多6角。‎ ‎ 圆珠笔的价钱[(12×5-15)-6]÷(8-5)=39÷3=13(角)‎ 小明带的钱数13×8+6=104+6=110(角)=11(元)。‎ 1. 某一筐水果中有苹果和梨若干个。若每次拿出1个苹果和1个梨,则拿到没有苹果时,还剩下50个梨;若每次拿走1个苹果和3个梨,则拿到没有梨时,苹果还剩下50个。那么这筐水果共有 个。‎ ‎[初级点拨] 需要转化的盈亏问题 ‎[深度提示] 若每次拿走1个苹果和3个梨,则拿到没有梨时,苹果还剩下50个。由这个条件可以转化为如果要苹果全部拿走,梨还差50×3=150个,所以梨的个数比苹果多50个,比苹果的3倍少150个。‎ ‎[全解过程] 若每次拿走1个苹果和3个梨,则拿到没有梨时,苹果还剩下50个。由这个条件可以转化为如果要苹果全部拿走,梨还差50×3=150个,所以梨的个数比苹果多50个,比苹果的3倍少150个,所以苹果的两倍是150+50=200个,所以苹果有100个,那么梨的个数是150个,所以苹果和梨的总个数为250个。‎ 2. 从5开始的一串连续的自然数5,6,7,8,…,拿走其中一个数,余下的数的平均数是10.75,那么拿走的数是_______。 ‎ ‎[初级点拨] 平均数问题 ‎[深度提示] 5至17这十三个连续自然数的平均数是11‎ ‎[全解过程] 因为(5+17)÷2=11,所以5至17这十三个连续自然数的平均数是11。还有12个数,拿走的数是(11一10.75)×12+11=14。‎ 3. A、B、C、D、E是五个不同的自然数,从小到大依次排列,它们的平均数是23,前四个数的平均数是21,后四个数的平均数是24,C是偶数,求D是多少? ‎ ‎[初级点拨] 平均数问题与不定方程 ‎[深度提示] A=23×5-24×4=19,‎ ‎ E=23×5-21×4=31,‎ ‎ B+C+D=21×4-19=65。‎ ‎[全解过程] 依题意得 ‎ A=23×5-24×4=19,‎ ‎ E=23×5-21×4=31,‎ ‎ B+C+D=21×4-19=65。‎ 因为>21,所以D应大于21。而A20。又C为偶数,因此若C=22,此时D至少为23。若D=23,此时则B=65-22-23=20。若D>23,则B<19,不符合题意。故D=23。‎ 4. 马小哈同学使用计算器计算2000个数的平均数之后,不小心把所求出的平均数与原先的2000个数混在一起。有趣的是,这2001个数的平均数恰好是2001。原来这2000个数的平均数是多少?‎ ‎[初级点拨] 平均数与方程法 ‎[深度提示] 我们可以设这2000个数的和是S,平均数为 ‎[全解过程] 设2000个数的和是S,平均数为,则,这2001个数的平均数为 ‎ ‎ 第12讲 牛吃草问题 一、知识地图:‎ 二、基础知识:‎ 英国科学家牛顿在他的《普通算术》一书中,有一道关于牛在牧场上吃草的问题,即牛在牧场上吃草,牧场上的草在不断的、均匀的生长。后人把这类问题称为牛吃草问题或叫做“牛顿问题”,类似的还有抽水问题等。我们具体来看一道典型的牛吃草问题:‎ 牧场上长满牧草,每天牧草都匀速生长。这片牧场可供10头牛吃20天,可供15头牛吃10天。供25头牛可吃几天?‎ 分析:要想知道这些草供25头牛可吃几天,必须知道草的总量和每头牛每天吃草的量。然而题目当中并没有告诉我们这样的条件。因此我们可以假设1头牛1天吃1份的草,那么10头牛20天可以吃10×20=200份草。15头牛10天可以吃15×10=150份草,有同学可能会奇怪了,同样都是把牧场的草吃完了,为什么吃草的总量不一样啊?你们明白为什么吗?‎ 聪明的同学可能已经明白了,对,因为每天都会有新的草长出来, ,所以草的总量并不是固定不变的。吃的时间越长,长的草越多,草的总量也就多了。由刚才的计算我们可以看出,吃20天的草的总量比10天要多,原因就在于此。我们来看看下面这幅图:‎ 从上面的图可以看出:草的总量可以分成两部分,一部分是原有的草,还有一部分是新长的草。10头牛20天吃的总草量比15头牛10天吃的总草量多,多出部分相当于10天新生长出的草量。设1头牛1天吃1份草,则10头牛20天比15头牛10天多吃份,则这块牧场每天新长份牧草。‎ 在第一种情况中,20天一共新长了份牧草,而牛一共吃了份,说明原来有牧草份。‎ 因为每天长5份的草,因此我们可以这样考虑,安排5头牛专门吃新长的草,剩下的牛吃原有的草,什么时候才能把草吃完呢?当牛把原有的草吃完的时候,草就不再生长了,也就是把所有的草全都吃完了。‎ ‎25头牛中安排5头牛吃新草,剩下的20头牛去吃原有的草,那么原有牧草可维持5天 ‎,即可供25头牛吃5天。‎ 解答牛吃草问题通常设每头牛每日吃掉的草量为单位“1”,解题关键在于通过对题中条件的分析比较,求出牧场上原有的草量,单位时间生长的草量。我们对于基本的牛吃草问题可以做如下总结,我们称之为"五步法":‎ 1. 求出两个总量。‎ 2. 总量的差÷时间差=每天长草量=安排去吃新草的牛数 3. 每天长草量×天数=总共长出来的草 4. 草的总量-总共长出来的草=原有的草 5. 原有的草÷吃原有草的牛=能吃多少天(或原有的草÷能吃多少天=吃原有草的牛)‎ 当然,牛吃草问题的变化还比较多,因此以上"五步法"只能作为参考,切不可生搬硬套。‎ 上面是从算术方法的角度,提供一种分析问题的思路。‎ 我们应该在解题中时刻把握“牛吃草问题”的核心是:‎ 牛吃草总量=草场原有草量+新长草量 ‎ 这种关系,在实际题目中,一般会出现两种方案,对这两种方案的进行比较,是获得解题思路的捷径。‎ ‎ 这种比较主要看两种方案“总草量”之差,这对应着两种方案的“时间差”。‎ ‎ 具体来看这里的关系:‎ ‎ 牛的头数×吃的天数=草场原有草量+每天长草量×吃的天数 ‎ 由此可知,一般牛吃草问题,首先要把两个关键的量求出来:‎ ‎(1)每天长草量 ‎(2)草场原有草量 请“奥数研究生”们在下面的例题中揣摩这两个量的求解方法。‎ ‎ ‎ 经典透析 ‎【例1】有一牧场,已知养牛27头,6天把草吃尽,养牛23头,9天把草吃尽。如果养牛21头,那么几天能把草吃尽呢?‎ 分析:同学们可以试着用"五步法"来解决一下这道题。注意要求出每天长草量和原有草量。‎ ‎   设1头牛1天吃1份的草,‎ ‎1.求两个总量,27×6=162   23×9=207‎ ‎2.总量的差÷时间差=每天长草量=安排去吃新草的牛数 ‎  (207-162)÷(9-6)=15‎ ‎3.每天长草量×天数=总共长出来的草 ‎   15×6=90‎ ‎4.草的总量-总共长出来的草=原有的草 ‎   162-90=72‎ ‎5.原有的草÷吃原有草的牛=能吃多少天 ‎   72÷(21-15)=12‎ 所以如果养牛21头,那么12天能把草吃尽 点评:对于比较基本的牛吃草问题,五步法还是很好用的。‎ ‎【例2】由于天气逐渐变冷,牧场上的草每天以均匀的速度减少。经计算,牧场上的草可供20头牛吃5天,或可供16头牛吃6天。那么,可供11头牛吃几天?‎ 分析:很显然,这道题和我们上一道题是有区别的,上一题每天的草量在增加,而这道题却是草量每天减少。那么该怎么处理这个问题呢?上一道题我们安排了一部分牛去吃新长的草,那么这道题能不能把每天减少的草想象成是有一些牛来帮忙吃了呢?‎ 设1头牛1天吃1份牧草,则20头牛5天吃掉20×5=100份牧草,16头牛6天吃掉16×6=96份牧草,说明6-5=1天牧场上的牧草减少100-96=4份,我们可以假设有4头牛来帮忙把这部分草给吃了。牧场上的原有草量是:100+4×5=120份。原来有11头牛,现在又有4头牛来帮忙吃,所以可维持120÷(11+4)=8天。‎ 点评:这道题的关键在于要把每天减少的草假设成有若干头牛来帮忙吃,如果理解了这个问题,那么剩下的步骤和最基本的牛吃草问题就一样了,我们也可以用"五步法"来解决。‎ ‎【例3】有一个水池,池底有一个打开的出水口,用5台抽水机20小时可将水抽完,用8台抽水机15小时可将水抽完。如果仅靠出水口出水,那么多长时间能把水漏完?‎ 分析:这道题表面上看好象和牛吃草没有什么关系,但是仔细想想,我们可以把抽水机当作牛,把水当作草,把出水口看成是来帮忙吃草的牛。大家可以试试用"五步法"来解答一下。‎ ‎ 设1台抽水机1小时抽出1单位的水,那么5台抽水机20小时抽出5×20=100单位的水,8台抽水机15小时抽出8×15=120单位的水,说明池底的出水口20-15=5小时漏出120-100=20单位的水,则出水口的出水速度是每小时20÷5=4单位,水池中原有100+4×20=180单位的水,如果仅靠出水口出水,需要180÷4=45小时。‎ 点评:牛吃草问题有一些变例,其中比较典型的就是"抽水问题",我们只需要弄清楚它与牛吃草问题的联系,把里面的关系理顺,还是可以用牛吃草问题很容易的加以解决。‎ ‎【例4】有一牧场,17头牛30天可将草吃完,19头牛则24天可将草吃完。现有牛若干头,吃6天后卖了4头,余下的牛再吃2天便将草吃完,问有牛多少头(草每日匀速生长)?‎ 分析:根据"五步法",我们其实很容易完成前几步的操作。‎ 设1头牛1天吃1份草,则牧草每天的生长量: 份;原有草量:份。‎ 做到这里的时候出现一个问题了,本题的一个变化是牛的数量减少了,那么我们该如何处理呢?我们能不能假设这4头牛没卖?如果不卖,草肯定不够吃了,要保证这4头牛在最后两天有草吃,我们必须增加4×2=8份的草才可以。这样就相当于所有的牛都吃了8天的草,如果能理解这一点,那么剩下的问题就好解决了。‎ 假设牛的数量保持不变,连续吃6+2=8天,共需要牧草240+9×8+4×2=320份,因此有牛320÷8=40头。‎ ‎ ‎ 点评:牛吃草问题的一个变化就是牛的数量的改变,对于牛减少了或者增加了,我们应该假设牛没有减少或增加,相应的增加或减少一部分草的总量,然后就可以按照基本的牛吃草问题来处理了。‎ ‎【例5】一块草地,每天生长的速度相同。现在这片牧草可供16头牛吃20天,或者供80只羊吃12天。如果一头牛一天的吃草量等于4只羊一天的吃草量,那么10头牛与60只羊一起吃可以吃多少天?‎ 分析:这道题又有一个新的变化,不是只有牛了,而是有牛又有羊,表面上看起来很复杂,但是冷静的分析一下,因为题目告诉我们1头牛一天的吃草量等于4只羊一天的吃草量,因此我们可以把4只羊换成1头牛,这样就只剩一种动物了。‎ ‎80只羊可以换成20头牛,60只羊可以换成15头牛,然后就可以用我们的“五步法”来操作了。‎ ‎ 设1头牛1天吃1份牧草,那么16头牛20天一共吃了16×20=320份草,20头牛12天吃了240份草,每天长草量为(320-240)÷(20-12)=10份草,原有的草量为320-10×20=120份草,现在有10+15=25头牛,其中吃原有草的牛有25-10=15头,那么可以吃120÷15=8天。‎ 点评:不论是有几种动物,只要他们之间互相有联系,那么都可以把它们转化成一种动物来操作。‎ ‎【例6】有三块草地,面积分别是‎4公顷、‎8公顷和‎10公顷,草地上的草一样厚,而且长得一样快。第一块草地可供24头牛吃6周,第二块草地可供36头牛吃12周。问:第三块草地可供50头牛吃几周?‎ 分析:之前我们讲的所有的牛吃草问题都是在同一块草地上,也就是说草地的面积是固定不变的。然而这道题却有三块面积不同的草地,该怎么办呢?‎ 虽然三块草地的面积不同,但是我们可以把它变成相同的,方法是分别转化成‎1公顷然后再进行计算。‎ 设1头牛1周吃1份牧草。24头牛6周吃掉24×6=144份,说明每公顷草地6周提供144÷4=36份牧草;36头牛12周吃掉36×12=432份,说明每公顷草地12周提供432÷8=54份牧草。每公顷草地12-6=6周多提供54-36=18份牧草,说明每公顷草地每周的牧草生长量是18÷6=3份,原有草量是36-3×6=18份。‎10公顷草地原有18×10=180份牧草,每周新增3×10=30份,可供50头牛吃180÷(50-30)=9周。‎ 点评:对于面积不同的情况,我们先把它转化成面积相同,通常的做法是将所有的面积都转化成单位面积然后进行计算。‎ ‎【例7】有三块草地,面积分别为‎5公顷、‎15公顷和‎24公顷。草地上的草一样厚,而且长得一样快。第一块草地可供10头牛吃30天,第二块草地可供28头牛吃45天。问:第三块草地可供多少头牛吃80天? ‎ 分析:这道题和上一道题其实是同一种类型的,这里提供几种解法给大家参考一下。‎ ‎(方法一)设1头牛1天吃草量为“1”,摘录条件,将它们转化为如下形式方便分析 ‎10头牛30天吃掉10×30=300份,说明:‎ ‎1公顷牧场30天提供300÷5=60份草:1公顷原有草量+30天1公顷新生草量 ‎28头牛45天吃掉28×45=1260份,说明 ‎1公顷牧场45天提供1260÷15=84份草:1公顷原有草量+45天1公顷新生草量 每公顷牧场45-30=15天多提供84-60=24份草,说明‎1公顷牧场1天的草生长量为24÷15=1.6份, ‎1公顷原有草量=60-1.6×30=12。1天‎24公顷新生草=1.6×24=38.4;‎24公顷原有草=12×24=288‎ 那么80天‎24公顷可提供草: 288+38.4×80=3360;所以共需要牛的头数是:3360÷80=42(头)牛。‎ ‎(方法二)除了按照最小公倍数统计外也可以统计为单位量“1”‎ 原条件: ‎5公顷 10头牛 30天 ‎ ‎ 15公顷 28头牛 45天 可转化为:相当于把 ‎5公顷草地分割成 5块每块一公顷有2头牛来吃,所以吃的时间不变 ‎ 相当于把15公顷草地分割成15块每块一公顷有头牛来吃,所以吃的时间不变 ‎1公顷 2头牛 30天 2×30=60:1公顷原有草量+30天1公顷新生草量 ‎ 1公顷 头牛 45天 ×45=84:1公顷原有草量+45天1公顷新生草量 从上易得:1天‎1公顷新生草量=(84-60)÷(45-30)=1.6;‎1公顷原有草量=60-30×1.6=12;‎ 那么80天‎24公顷可提供草: 12×24+1.6×24×80=3360;所以共需要牛的头数:3360÷80=42(头)。‎ ‎(方法三)现在是3块面积不同的草地,解决这个问题,只需将3块草地的面积统一起来就可以了!‎ ‎[5,15,24]=120 ,设1头牛1天的吃草量为“‎1”‎,摘录条件,将它们转化为如下形式方便分析,‎ 原条件: ‎5公顷 10 头牛 30天 ‎ ‎ 15公顷 28 头牛 45天 可转化为:‎120公顷 240头牛 30天 240×30=7200 :‎120公顷原有草量+30天‎120公顷新生草量 ‎ 120公顷 224头牛 45天 224×45=10080:120公顷原有草量+45天120公顷新生草量 从上易得:1天‎120公顷新生草量=192;‎120公顷原有草量=7200-30×192=1440;‎ 则1天‎24公顷新生草量=192÷5=38.4,‎24 公顷原有草量=1440÷5=288;‎ 那么80天‎24公顷可提供草: 288+38.4×80=3360;所以共需要牛的头数是:3360÷80=42(头)牛。‎ ‎【例8】有甲,乙两块匀速生长的草地,甲草地的面积是乙草地面积的三倍。30头牛12天能吃完甲草地上的草,20头牛4天能吃完乙草地的草。问几头牛10天能同时吃完两块草地上的草?‎ 分析:这道题又有一个变化,两块草地的面积不同,但是没有具体告诉我们面积是多少,只是告诉我们面积的倍数关系。在前面我们讲过,如果有好几种动物,各种动物之间有倍数关系,我们可以转化为同一种动物来计算,那么这道题我们能不能把两块草地转化为一块草地来计算呢?同学们试试就可以发现答案是肯定的,具体操作如下:‎ 设1头牛1天的吃草量为“1”,摘录条件,将它们转化为如下形式方便分析,根据甲的面积是乙的3倍可以将关系(将乙看成1份,则甲就是3份)进行转化。‎ 甲: 30头牛 12天 30×12=360:甲原有草量+12天甲地自然增加的草量 甲转化为:10 头牛 12天 10×12=120:乙原有草量+12天乙地自然增加的草量 ‎ 乙: 20头牛 4天 20×4 = 80:乙原有草量+ 4天乙地自然增加的草量 从上表中可以看出(12-4)=8天乙地长草量为(120-80)=40,即1天乙地长草量为40÷8=5;‎ 乙地的原有草量为:120-5×12=60;则甲、乙两地1天的新生草为:5×(3+1)=20,原有草量为:60×(3+1)=240;‎ ‎10天甲、乙两地共提供青草为:240+20×10=440,需要:440÷10=44(头)牛。‎ 点评:面积有倍数关系和动物的食量有倍数关系本质上是相同的,我们都要把它们转化为单一的面积或动物后再进行计算。‎ ‎【例9】一片草地每天长的草一样多,现有牛、羊、鹅各一只,且羊和鹅吃草的总量正好是牛吃草的总量。如果草地放牧牛和羊,可以吃45天;如果放牧牛和鹅,可吃60天:如果放牧羊和鹅,可吃90天。这片草地放牧牛、羊、鹅,可以供它们吃多少天?‎ 分析:这道题有三种动物,但是不知道每种动物之间的数量关系,因此转化成同一种动物比较困难,这里我们要借助三元一次方程的思想,最终的目的还是要转化为单一动物。‎ 设1头牛1天吃草量为“1”,摘录条件,将它们转化为如下形式方便分析 牛和羊   45天  45天牛和羊吃草量=原有草量+45天新长草量 (1)‎ 牛和鹅   60天  60天牛和鹅吃草量=原有草量+60天新长草量 (2)‎ 鹅和羊(相当于1牛) 90天  90天牛(鹅和羊)吃草量=原有草量+90天新长草量 (3)‎ 由(1)×2-(3)可得: 90天羊吃草量=原有草量 羊每天吃草量=原有草量÷90;‎ 由(3)分析知道:90天鹅吃草量=90天新长草量,鹅每天吃草量=每天新长草量;‎ 将分析的结果带入(2)得:原有草量=60,带入(3)得90天羊吃草量=60 羊每天吃草量=‎ 这样如果牛、羊和鹅一起吃,可以让鹅去吃新生草,牛和羊吃原有草可以吃:60÷(1+)=36(天)。‎ 拓展训练:‎ ‎1.现欲将一池塘水全部抽干,但同时有水匀速流入池塘。若用8台抽水机10天可以抽干;用6台抽水机20天能抽干。问:若要5天抽干水,需多少台同样的抽水机来抽水?‎ ‎2.12头牛28天可以吃完10公亩牧场上全部牧草,21头牛63天可以吃完30公亩牧场上全部牧草。多少头牛126天可以吃完72公亩牧场上全部牧草(每公亩牧场上原有草量相等,且每公亩牧场上每天生长草量相等)?‎ ‎3.画展9点开门,但早就有人排队等候入场了。从第一个观众来到时起,每分钟来的观众人数一样多。如果开3个入场口,则9点9分就不再有人排队了,如果开5个入场口,则9点5分就没有人排队了。那么第一个观众到达的时间是8点几分?‎ ‎4.甲、乙、丙三个仓库,各存放着数量相同的面粉,甲仓库用一台皮带输送机和12个工人,5小时可将甲仓库内面粉搬完;乙仓库用一台皮带输送机和28个工人,3小时可将仓库内面粉搬完;丙仓库现有2台皮带输送机,如果要用2小时把丙仓库内面粉搬完,同时还要多少个工人?(每个工人每小时工效相同,每台皮带输送机每小时工效也相同,另外皮带输送机与工人一起往外搬运面粉)‎ ‎5.有一桶酒,每天都因桶有裂缝而要漏掉等量的酒,现在这桶酒如果给6人喝,4天可喝完;如果由4人喝,5天可喝完。这桶酒每天漏掉的酒可供几人喝一天?如果桶没有裂缝由4个人来喝需要几天喝完?‎ ‎6.某建筑工地开工前运进一批砖,开工后每天运进相同数量的砖,如果派15个工人砌砖墙14天可以把砖运完,如果派20个工人,9天可以把砖用完,现在派若干名工人砌了6天后,调走6名工人,其余工人又工作4天才砌完,问原来有多少工人来砌墙?‎ ‎7.一片匀速生长的牧草,如果让马和牛去吃,15天将草吃尽;如果让马和羊去吃,20天将草吃尽;如果让牛和羊去吃,30天将草吃尽。已知牛和羊每天的吃草量的和等于马每天的吃草量。现在让马、牛、羊一起去吃草,几天可以将这片牧草吃尽?‎ ‎8.东升牧场南面一块‎2000平方米的牧场上长满牧草,牧草每天都在匀速生长,这片牧场可供18头牛吃16天,或者供27头牛吃8天。在东升牧场的西侧有一块‎6000平方米的牧场,可供多少头牛吃6天?‎ ‎9.120头牛28天吃完10公顷牧场上的全部牧草,210头牛63天吃完‎30公顷牧场上的全部牧草,如果每公顷牧场上原有的牧草相等,且每公顷每天新生长的草量相同,那么多少头牛126天可以吃完72公顷牧场上的全部牧草?‎ ‎10.如图,一块正方形的草地被分成完全相等的四块和中间的阴影部分,已知草在各处都是同样速度均匀生长。牧民带着一群牛先在①号草地上吃草,两天之后把①号草地的草吃光。(在这2天内其他草地的草正常生长)之后他让一半牛在②号草地吃草,一半牛在③号草地吃草,6天后又将两个草地的草吃光。然后牧民把的牛放在阴影部分的草地中吃草,另外的牛放在④号草地吃草,结果发现它们同时把草场上的草吃完。那么如果一开始就让这群牛在整块草地上吃草,吃完这些草需要多少时间?‎ 初级点拨:1、这是一道抽水问题,可以用最基本的牛吃草问题的方法来解决。‎ ‎2、这是一道三块草地牛吃草问题,请参照例6的做法。‎ ‎3、这是一道入口问题,试着把它转换成牛吃草问题来思考。‎ ‎4、这道题表面上看起来不是牛吃草问题,其实它是三块草地牛吃草的一个变例。‎ ‎5、这是一道经典的牛吃草的变例。‎ ‎6、注意这道题当中人数发生了变化。‎ ‎7、这是一个多种动物的牛吃草问题,而且还不知道各种动物之间的倍比关系。‎ ‎8、这是一道两块草地上牛吃草的问题,而且直接给出了两块草地的数量。‎ ‎9、这是一道三块草地上牛吃草问题。‎ ‎10、这是一个结合平面图形的牛吃草问题。‎ 深度提示:1、可以使用五步法,注意求出原有草量与每天长草量。‎ ‎2、注意把三块草地转换成1公亩,然后进行处理。‎ ‎3、我们可以把人在增加想象成每分钟都在长草,把入口想象成人。‎ ‎4、我们把甲、乙、丙想象成三块草地,然后参照第2题的做法就可以做出来了。‎ ‎5、注意每天漏掉的酒相当于草在减少。‎ ‎6、我们可以假设人数没有变,那么草的总量应该相应增加。‎ ‎7、可以参照解三元一次方程来处理这道题。‎ ‎8、注意2000平米与6000平米之间的关系。‎ ‎9、参照第2题的解法。‎ ‎10、注意观察平面图形的特征。‎ 全解过程:1、设1台抽水机1天的抽水量为1单位,则池塘每天的进水速度为:(6×20-8×10)÷(20-10)=4单位,池塘中原有水量:6×20-4×20=40单位。若要5天内抽干水,需要抽水机40÷5+4=12台。‎ ‎2、设1头牛1天吃1份牧草,则每公亩牧场上的牧草每天的生长量:(21×63÷30-12×28÷10)÷(63-28)=0.3(份),每公亩牧场上的原有草量:21×63÷30-0.3×63=25.2(份),则72公亩的牧场126天可提供牧草:(25.2+0.3×126)×72=4536(份),可供养4536÷126=36头牛 ‎3、‎ 设一个入口1分钟入场的人数为1份,3个入场口9分钟进入了27份观众,5个入场口5分钟进入了25份观众,说明4分钟来的观众人数是27-25=2份,即每分钟来0.5份。因为9点5分时共来了25份,来25份需要25÷0.5=50分钟,所以第一个观众到达的时间是8点15分。‎ ‎4、 设1个工人1小时搬1份面粉。甲仓库中12个工人5小时搬了份,乙仓库中28个工人3小时搬了份,说明甲仓库的传送机5-3=2小时多输送了84-60=24份面粉,即每小时输送24÷2=12份,仓库中共有面粉份。‎ ‎ 丙仓库中120份面粉需在2小时内搬完,每小时需搬份,因此需要工人名。‎ ‎5、一桶酒相当于原有“草”,喝酒人相当于“牛”,漏掉酒相当于草在减少,设1人1天喝酒量为“‎‎1”‎ ‎ 6人 4天 6×4=24:原有酒-4天自然减少的酒 ‎ 4人 5天 4×5=20:原有酒-5天自然减少的酒 从上面看出:1天减少的酒为(24-20)÷(5-4)=4,可供4人喝一天。‎ 原有酒为:24+4×4=40,由4个人来喝需要:40÷4=10(天)。‎ ‎6、依题意知开工前运进的砖相当于“原有草”开工后每天运进相同的砖相当于“草的生长速度”工人砌砖相当于“牛在吃草”。所以设1名工人1天砌砖数量为“‎1”‎,列表分析得 ‎ 15人 14天 15×14=210 :原有砖的数量+14天运来砖的数量 ‎ 20人 9天 20×9 =180 :原有砖的数量+ 9天运来砖的数量 从上面的表中可以看出(14-9)=5天运来的砖为(210-180)=30,即1天运来的砖为30÷5=6‎ 原有砖的数量为:180-6×9=126;‎ 假设6名工人不走,则能多砌6×4=24份砖,则砖的总数为126+24+6×(6+4)=210‎ 因为是10天工作完,所以有210÷10=21名工人。‎ ‎7、设1匹马1天吃草量为“‎1”‎,摘录条件,将它们转化为如下形式方便分析 马和牛   15天  15天马和牛吃草量=原有草量+15天新长草量 (1)‎ 马和羊   20天  20天马和羊吃草量=原有草量+20天新长草量 (2)‎ 牛和羊(同马) 30天  30马(牛和羊)吃=原有草量+30天新长草量 (3)‎ 由(1)×2-(3)可得: 30天牛吃草量=原有草量 牛每天吃草量=原有草量÷30;‎ 由(3)分析知道:30天羊吃草量=30天新长草量,羊每天吃草量=每天新长草量;‎ 讲分析的结果带入(2)得:原有草量=20,带入(3)30天牛吃草量=20,得牛每天吃草量=‎ 这样如果马、牛和羊一起吃,可以让羊去吃新生草,马和牛吃原有草可以吃:20÷(1+)=12(天)。‎ ‎8、设1头牛1天的吃草量为“‎1”‎,摘录条件,将它们转化为如下形式方便分析 ‎18头牛 16天 18×16=288 :原有草量+16天自然增加的草量 ‎27头牛 8天 27× 8=216 :原有草量+ 8天自然增加的草量 从上看出:‎2000平方米的牧场上16-8=8天生长草量=288-216=72,即1天生长草量=72÷8=9;‎ 那么‎2000平方米的牧场上原有草量:288-16×9=144或216-8×9=144。‎ 则‎6000平方米的牧场1天生长草量=9×(6000÷2000)=27;原有草量:144×(6000÷2000)=432。‎ ‎6天里,西侧草场共提供草432+27×6=594,可以让594÷6=99(头)牛吃6天。‎ ‎9、设1头牛1天吃1份牧草。‎ ‎120头牛28天吃掉120×28=3360份,说明每公顷牧场28天提供3360÷10=336份牧草;‎ ‎210头牛63天吃掉210×63=13230份,说明每公顷牧场63天提供13230÷30=441份牧草;‎ 每公顷牧场63-28=35天多提供441-336=105份牧草,说明每公顷牧场每天的牧草生长量为105÷35=3份,原有草量为336-28×3=252份。‎ 如果是‎72公顷的牧场,原有草量为252×72=18144份,每天新长出3×72=216份,‎ ‎126天共计提供牧草18144+126×216=45360份,可供45360÷126=360头牛吃126天。‎ ‎10、 一群牛,2天,吃了1块+1块2天新长的;一群牛,6天,吃了2块+2块2+6=8天新长的;即3天,吃了1块+1块8天新长的;即1群牛1天吃1块6天新长的;即群牛,1天,吃了1块1天新长的草量。‎ ‎ 又因为,的牛放在阴影部分的草地中吃草,另外的牛放在④号草地吃草,它们同时吃完。所以,‎ ‎③=2阴影部分面积。于是,整个为块地。那么需要群牛吃新长的草,于是=现在。所以需要吃:天。‎ 所以,一开始将一群牛放到整个草地,则需吃30天。‎ 第13讲 行程(一) 相遇追及(多次)、电车问题 一、 知识地图 ‎ 简单相遇追及 ‎ 匀速直线行程 多次相遇追及 ‎ (包括火车过桥)‎ ‎ 发车间隔问题 ‎ ‎ 多次相遇追及 环形线路行程 ‎ ‎(包括钟表问题)‎ 二、 基础知识 在历年“小升初”考试和各类小学奥数竞赛试题中,“行程问题”都占有很大的比重。同时也是小学奥数专题中的难点,“行程问题”经常作为一份试卷中的压轴难题出现,提高解决“行程问题”的能力不仅能帮助在小升初考试和各类数学竞赛中取得优异成绩,还能为今后初中阶段数学、物理学科的学习打下良好的基础。‎ (一) 典型的相遇和追及 所有行程问题是围绕“”这一条基本关系式的展开,比如我们遇到的两大典型行程题相遇问题和追及问题的本质也是这三个量之间的关系,在这里:‎ ‎ ;‎ ‎ ;‎ 这两组关系式中“路程和”或“路程差”实际上对应的是相遇或追及问题中的原始(初始)距离,我们可以通过图示来理解。‎ (一) 多次相遇追及 通过图示介绍直线上的相遇和追及的规律 ‎ 这部分内容涉及以下几个方面:‎ 1 求相遇次数 2 求相遇地点 3 由相遇地点求全程 ‎“线段示意图”和“折线示意图”是解行程问题特别是多次相遇问题的重要方法。‎ 举个例子:假设A、B两地相距‎6000米,甲从A地出发在AB间往返运动,速度为‎6千米/小时,乙从B出发,在AB间往返运动,速度为‎4千米/小时。我们可以依次求出甲、乙每次到达A点或B点的时间。为了说明甲、乙在AB间相遇的规律,我们可以用“折线示意图”来表示。‎ 折线示意图能将整个行程过程比较清晰的呈现出来:例如AD表示的是,甲从A地出发运动到B地的过程,其中D点对应的时间为1小时,表示甲第一次到达B点的时间为1小时,BF表示乙从B地出发到达A地的过程,F点对应的时间为1.5小时,表示乙第一次到达A地的时间为1.5小时,AD与BF相交于C点,对应甲、乙的第一次相遇事件,同样的G点对应是甲、乙的第二次相遇事件。 折线图只能直观的表示出相遇的次数和大致时间和地点,具体的时间和地点还必须通过相遇和追及问题的公式进行计算。‎ 通过计算,我们能得出:甲、乙第一次相遇的时间为6÷(6+4)=0.6(小时),即36分钟。相遇点距离B地0.6×4=2.4(千米),从第一次相遇到第二次相遇,甲、乙行程的路程总和等于两个AB长,所以两次相遇的时间间隔为72分钟。第二次相遇发生的时间为108分钟。‎ 事实上,我们从折线示意图就能看出来,任意两次相邻的相遇事件的时间间隔都是72分钟,而每72分钟,甲乙两人运动的总路程都等于2个AB长,所以我们能得到如下推论:‎ 如果甲、乙是从线段两端出发,那么相邻的两次相遇事件的时间间隔都相等,并且第n次相遇时,他俩行走路程和相当于(2n-1)个线段总长。同样的相邻两次的追及事件(速度快的追上速度慢的)发生的时间间隔都相等。第n次追及时,他俩行走路程差相当于(2n-1)个线段总长。‎ 注意:如果甲、乙在线段的端点碰面,既可以算作相遇事件也可以算作追及事件,例如例子当中的E点,既是甲、乙的第三次相遇,也是甲第一次从后面追上乙。‎ (一) 发车间隔问题 有关公共汽车与行人的问题,主要涉及到这几个量:行人速度、汽车速度、前后相邻汽车间距、汽车发车时间间隔、相遇(追及)事件时间间隔。‎ 这些貌似不相关的数量之间隐含着很多数量关系:‎ 1. 我们首先分析一下公共汽车的发车过程:从一辆汽车发车到下一辆汽车发车,经过一个“汽车发车时间间隔”,所以当下一辆车发车的时候,前一辆车已经开走了“一个汽车发车时间间隔”的时间,这个时间内前一辆车共行驶了“一个汽车发车时间间隔”乘以“汽车速度”,之后两辆车之间的距离保持不变,即距离保持为“相邻汽车间距”,所以我们得到第一条公式:‎ 2. 与公共汽车发车过程类似的,如果行人和汽车相向(反向)行驶,那么从行人遇到第一辆车到遇到第二辆车的过程可以看作一个相遇问题,所以有如下数量关系:‎ ‎ ‎ ‎ 同样的如果行人和汽车同向行驶,则有关系式:‎ ‎ ‎ 一、 经典透析 【例1】 甲、乙、丙三人每分钟分别行‎60米、‎50米和‎40米,甲从B地、乙和丙从A地同时出发相向而行,途中甲遇到乙后15分又遇到丙。求A,B两地的距离。‎ ‎[审题要点]从已知条件中唯一的时间量入手,明确甲、乙、丙之间的距离变化关系,逐步求解。‎ ‎[详解过程]‎ 甲遇到乙后15分钟,甲遇到了丙,所以遇到乙的时候,甲和丙之间的距离为:‎ ‎(60+40)×15=1500(米),‎ 而乙丙之间拉开这么大的距离一共要 ‎1500÷(50-40)=150(分),‎ 即从三人出发到甲与乙相遇一共经过了150分钟,‎ 所以A、B之间的距离为:‎ ‎(60+50)×150=16500(米)。‎ ‎[点评]此题实质上有着三个行程基本问题:两个相遇问题:甲和乙相遇,甲和丙相遇;一个追及问题:丙和乙的追及问题。而且这三个问题之间有着相互的联系,甲和丙的相遇路程就是丙和乙的追及路程,丙和乙的追及时间就是甲和乙的相遇时间。利用这些关系层层推进即可解出答案。‎ 【例2】 甲、乙、丙三车同时从A地沿同一公路开往B地,途中有个骑摩托车的人也在同方向行进,这三辆车分别用7分钟、8分钟、14分钟追上骑摩托车人。已知甲车每分钟行‎1000米,丙车每分钟行‎800米,求乙车的速度是多少?‎ ‎[审题要点]摩托车在各时间点行驶的位置是甲、乙、丙三车行驶距离的度量,所以本题的关键是求出摩托车的速度。‎ ‎[详解过程]‎ 甲与丙行驶7分钟的距离差为:‎ ‎(1000-800)×7=1400(米),‎ 也就是说当甲追上骑摩托车人的时候,丙离骑摩托车人还有‎1400米,丙用了 ‎14-7=7(分)‎ 追上了这‎1400米,所以丙车和骑摩托车人的速度差为:‎ ‎1400÷(14-7)=200(米/分),‎ 骑摩托车人的速度为:‎ ‎800-200=600(米/分),‎ 三辆车与骑摩托车人的初始距离为:‎ ‎(1000-600)×7=2800(米),‎ 乙车追上这‎2800米一共用了8分钟,所以乙车的速度为:‎ ‎2800÷8+600=950(米/分)。‎ ‎[点评]从整体考虑,7分钟的时候摩托车与甲车在同一位置即7×1000=7000(米),14分钟的时候摩托车与丙车在同一位置即14×800=11200(米),所以所以摩托车在7-14分这7分钟内一共行驶了11200-7000=4200(米),所以摩托车的速度为4200÷7=600(米/秒),摩托车在8分钟时的位置为7000+600=7600(米),所以乙车的速度为7600÷8=950(米/分),这种解法比较类似于牛吃草问题。‎ 【例1】 铁路旁一条小路,一列长为‎110米的火车以每小时30千米的速度向南驶去,8点时追上向南行走的一名军人,15秒后离他而去,8点6分迎面遇到一个向北行走的农民,12秒后离开这个农民,问:军人与农民何时相遇?‎ ‎[审题要点]涉及火车的行程问题中,火车的长度当然不能忽略,解题关键是找出15秒(12秒)内,火车行驶和人步行与火车车长之间的数量关系。‎ ‎[详解过程]‎ 分析:火车速度为30×1000÷60=500(米/分)。‎ 要求军人与农民的速度必须先知道知道军人和农民的速度。‎ 由题目条件,从军人被火车头追上到车尾离他而去,一共有15秒,这十五秒可以看作车尾追及军人的时间,所以根据追及公式,火车速度减去军人速度等于 ‎110÷(15÷60)=440(米/分),‎ 所以军人的速度为 ‎500-440==60(米/分),‎ 即‎60米/分,同样的我们还可以求出农民的速度:‎ ‎110÷(12÷60)-500=50(米/分),‎ 即‎50米/分,8点06火车与农民相遇,所以8点时火车头与农民的距离为:‎ ‎(500+50)×6=3300(米),‎ 这么长一段路,军人与农民相遇需要 ‎3300÷(60+50)=30(分)。‎ 此时的时间为8点30分。‎ ‎[点评]1、此题中有着三个基本问题。火车追及军人,火车农民相遇,军人和农民相遇,找到三者之间的关系就是解决题目的关键。‎ ‎2、解决行程问题的关键是三步:‎ a:正确画出示意图;‎ b:把复杂的行程问题分解为每一个基本的相遇或追及问题;‎ c:找到这些相遇或追及问题之间的数量关系,包括路程关系,时间关系与速度关系。‎ 【例1】 一辆卡车和一辆摩托车同时从A、B两地相对开出,两车在途中距A地‎60千米处第一次相遇,然后两车继续前进,卡车到达B地,摩托车到达A地后都立即返回,两车又在途中距B地‎30千米处第二次相遇。A、B两地之间的距离是多少千米?‎ ‎[审题要点]结合两次相遇的时间规律,找出两个相遇点位置和A、B两地距离的关系。‎ ‎[详解过程]‎ 根据题目中所给的条件,可以画出整个行程过程的线段示意图:‎ 由示意图看出卡车从A地出发后行驶了‎60千米时与摩托车相遇,此时卡车和摩托车共同行驶的路程和相当于一个AB距离。而卡车和摩托车第二次相遇的时候,卡车和摩托车共同行驶的路程和相当于三个AB距离。所以如果卡车、摩托车从出发到第一次相遇时所用时间为t的话,那么卡车、摩托车从出发到第二次相遇时所用时间为3t,因此第二次相遇时卡车行驶的距离为:‎ ‎60×3=180(千米)。‎ 这‎180千米等于AB的全程再加上B地到第二个相遇点的距离‎30千米,所以AB的距离为:‎ ‎180-30=150(千米)。‎ ‎[点评] 本题是甘肃省第十四届小学生数学冬令营原题,类似的题目在很多杯赛中出现过。‎ 题目中使用了比例的知识,题目并没有直接求出卡车和摩托车的速度和时间,但使用了两次的比例转换:首先是利用总路程的三倍关系得出时间的三倍关系,然后利用时间的三倍关系得出卡车的路程三倍关系。‎ 【例1】 如下图所示,某单位沿着围墙外面的小路形成一个边长‎300米的正方形。甲、乙两人分别从两个对角处沿逆时针方向同时出发。如果甲每分走‎90米,乙每分走‎70米,那么经过多少时间甲才能看到乙?‎ ‎[审题要点] 当甲看到乙的时候,甲和乙在同一条边上,甲乙两人之间的距离最多有‎300米长。‎ ‎[详解过程]‎ 当甲、乙之间的距离等于‎300米时,即甲追上乙一条边(‎300米)需 ‎300÷(90-70)=15(分),‎ 此时甲走了 ‎90×15÷300=4.5(条)边,‎ 所以甲、乙不在同一条边上,甲看不到乙。但是甲只要再走0.5条边就可以看到乙了,即甲从出发走5条边后可看到乙,共需 ‎(分),‎ 即16分40秒。‎ ‎[点评]解决此类相遇问题或追及问题时,一般是先利用一般的基本问题公式求出答案,但是此时要加入一个判断的过程,不符合要求就在此基础上往后推或往前推,直至得出符合要求的答案。‎ 如果题目要求从甲第一次看到乙到乙从甲视线中消失一共经历多少分钟,应该怎么做呢?‎ 分析:只要甲没有超过乙,乙只要转过一个拐角,甲就看不到乙了, 甲第一次看到乙是分,乙走了条边,乙再走条边,甲便看不到乙了(这里最好检验一下甲到底走到了哪里),所以甲看到乙的时间一共只有(分)。‎ 【例1】 两辆电动小汽车在周长为‎360米的圆形道上不断行驶,甲车每分行驶‎20米。甲、乙两车同时分别从相距‎90米的A、B两点相背而行,相遇后乙车立即返回,甲车不改变方向,当乙车到达B点时,甲车过B点后恰好又回到A点。此时甲车立即返回(乙车过B点继续行驶),再过多少分与乙车相遇?‎ ‎ ‎ ‎[审题要点]分析各个时间段,甲乙两人的行程。 图中C表示甲、乙第一次相遇地点。因为乙从B到C和从C又返回B时所花的时间相等,而整个过程中甲恰好转一圈回到A,所以甲、乙在C点第一次相遇时,甲刚好走了半圈。‎ ‎[详解过程] C点距B点 ‎180-90=90(米)。‎ 而甲从A到C用了 ‎180÷20=9(分),‎ 所以乙每分行驶 ‎90÷9=10(米)。‎ 甲、乙第二次相遇,即分别同时从A,B出发相向而行相遇还需要 ‎90÷(20+10)=3(分钟)。‎ ‎[点评]此题的关键是找出题目中的相等关系,先由乙来回的路程一样得出时间一样,那么甲两段路程的时间也一样,所以路程也一样,然后也可以直接利用路程的比例关系得出甲乙的速度比为2:1,求出乙的速度为10。‎ 【例2】 A、B是公共汽车的两个车站,从A站到B站是上坡路。每天上午8点到11点从A、B两站每隔30分同时相向发出一辆公共汽车。已知从A站到B站单程需105分,从B站到A站单程需80分。问:(1)8:30、9:00从A站发车的司机分别能看到几辆从B站开来的汽车?(2)从A站发车的司机最少能看到几辆从B站开来的汽车?‎ ‎[审题要点]分析各辆车的出发和到达时间,判断两辆车是否相遇,找出各辆车遇到的车辆的出发时间。‎ ‎[详解过程](1)从A站发车的司机看到的车辆包括两类:‎ 一类是他自己发车以前,已经从B站出发但还没到达A站的所有车辆,也就是发车前80分钟内B站所发的所有车辆。‎ 另一类是他发车以后到他抵达B站这段时间内从B站发出的所有车辆,即发车后105分钟内从B站开出的所有车辆。‎ 这就是说在A站车辆出发前80分钟到出发后105分钟之间这185分钟时间区间内,B站发出的所有车辆,该司机都能看到。实际上这185分钟中,只有发车前60分、发车前30分、发车当时、发车后30分、发车后60分、发车后90分,有车辆从B站开出,所以8:30从A站发车的司机能看到8:00到10:00从B站发出的5辆车,而9:00从A站发车的司机能看到8:00到10:30从B站发出的6辆车。‎ ‎(2)11点以后不再有车辆从B站发出,11点发车的司机不可能看到他发车后105分钟内从B站开出的车,所以他只能看到3辆车。‎ ‎[点评]运用“折线示意图”能更好地说明整个行程过程。从“8:‎30”‎引出的线段与其他线段一共有5个端点,所以8:30从A站发出的车一共遇到5辆从B站发出的车,同样的9:00从A站发出的车一共遇到6辆从B站发出的车,11:00从A站发出的车一共遇到3辆从B站发出的车。‎ 【例1】 某人以匀速行走在一条公路上,公路的前后两端每隔相同的时间发一辆公共汽车。他发现每隔15分钟有一辆公共汽车追上他;每隔10分钟有一辆公共汽车迎面驶来擦身而过。问公共汽车每隔多少分钟发车一辆?‎ ‎[审题要点]列出所有涉及到的数量关系,找出发车时间与公共汽车相遇追及间隔时间的关系。‎ ‎[详解过程] 设两车之间相距S,根据公式得,‎ ‎,‎ 那么,‎ 解得,‎ 所以发车间隔(分)‎ ‎[点评]此题中方程有两个未知数,最后解出的是两个速度的倍数关系,由于无法求出速度的具体数值,所以可以将其中一个设为“‎1”‎,这样可以简化方程,方便求解。事实上只要不能求出具体数值而只要得出比例关系时都可以将其中一个看作“‎1”‎,从而简化计算。例如:‎ 另解一:根据每15分钟有一辆公共汽车追上他,那么1分钟汽车追上间隔的,即汽车与人的速度差为;同理根据每隔10分钟有一辆公共汽车迎面驶来擦身而过,那么1分钟汽车和人共行间隔的,即汽车与人的速度和为,最后根据和差问题求出汽车1分钟行间隔的几分之几, ,进而求出发车间隔时间,1÷=12(分)。 此种解题思路实质上是转化为工程问题来解决,工程问题和行程问题在一定程度上是一样的问题,所用方法也很相似,同学可以自己体会。‎ 另解二:反正发车时间和间隔是相等的,这样我们可以假设人先过去,这样每15分钟后面有一辆车追上他,再马上回来时,正好是每10分钟前面有一辆车和他迎面相遇,所以我们假设两地之间走要[15,10]=30分钟,这样过去的时间里有30÷15=2辆车追上他,同理回来的30÷10=3辆车和他迎面相遇,这样在这30+30=60分钟里,总共有2+3=5辆车发出,所以发车间隔为60÷5=12分钟。‎ 【例1】 小峰骑自行车去小宝家聚会的路上注意到,每隔9分钟就有一辆公交车从后方超越小峰,小峰骑车到半路,车坏了,于是只好坐出租车去小宝家,这时小峰又发现出租车也是每隔9分钟超越一辆公交车,已知出租车的速度是小峰骑车速度的5倍,那么如果这三种车辆在行驶过程中都保持匀速,那么公交车站每隔多少分钟发一辆车?‎ ‎[审题要点]列出问题所涉及的所有数量关系,求出各种交通工具的速度比。‎ ‎[详解过程]题目条件涉及到的数量涉及到的数量关系有:‎ 汽车间距=(公交速度-骑车速度)×9分钟;‎ 汽车间距=(出租车速度-公交速度)×9分钟;‎ 所以,公交速度-骑车速度=出租车速度-公交速度;‎ 将上面这条等式变形得到:‎ 公交速度=(骑车速度+出租车速度)÷2=3×骑车速度。‎ 那么:‎ 所以公交车站每隔6分钟发一辆公交车。‎ 一、 拓展训练:‎ 1. 甲、乙、丙三人在学校到体育场的路上练习竞走,甲每分比乙多走‎10米,比丙多走‎31米。上午9点三人同时从学校出发,上午10点甲到达体育场后立即返回学校,在距体育场‎310米处遇到乙。问:(1)从学校到体育场的距离是多少?(2)甲与丙何时相遇(精确到秒)?‎ 2. ‎(2007年希望杯)两条公路成十字交叉,甲从十字路口南‎1200米处向北直行,乙从十字路口处向东直行。甲、乙同时出发10分后,两人与十字路口的距离相等,出发后100分,两人与十字路口的距离再次相等,此时他们距十字路口多少米?‎ 3. ‎(2007年第十二届 “华罗庚金杯”少年数学邀请赛决赛)李云靠窗坐在一列时速‎60千米的火车里,看到一辆有30节车厢的货车迎面驶来,当货车车头经过窗口时,他开始记时,直到最后一节车厢驶过窗口时,所记的时间是18秒。已知货车车厢长‎15.‎‎8米,车厢间距‎1.2米,货车车头长‎10米,问货车行驶的速度是多少?‎ 4. A、B两地相距‎1000米,甲从地、乙从地同时出发,在、两地间往返锻炼。乙跑步每分钟行‎150米,甲步行每分钟行‎60米。在30分钟内,甲、乙两人第几次相遇(含追及)时距B地最近?最近距离是多少?‎ 5. 甲、乙两人在一条长‎100米的直路上来回跑步,甲的速度‎3米/秒,乙的速度‎2米/秒。如果他们同时分别从直路的两端出发,当他们跑了10分钟后,共相遇(不包括追上)多少次?‎ 6. 甲、乙二人分别从A、B两地同时出发,往返跑步。甲每秒跑‎3米,乙每秒跑‎7米。如果他们的第四次相遇点与第五次相遇点的距离是‎150米,求A、B两点间的距离为多少米?‎ 7. A、B两地相距‎24千米,甲和乙两人分别由A、B两地同时相向而行,往返一次,甲比乙早返回原地。途中两人第一次相遇于C点,第二次相遇于点D。CD相距‎6千米,则甲、乙两人的速度比是为多少?‎ 1. 下图中,外圆周长‎40厘米,画阴影部分是个“逗号”,两只蚂蚁分别从A,B同时爬行。甲蚂蚁从A出发,沿“逗号”四周顺时针爬行,每秒爬‎3厘米;乙蚂蚁从B出发,沿外圆圆周顺时针爬行,每秒爬行‎5厘米。两只蚂蚁第一次相遇时,乙蚂蚁共爬行了多少米?‎ 2. 小乐步行去学校的路上注意到每隔4分钟就遇到一辆迎面开来的公交车,到了学校小乐发现自己忘记把一件重要的东西带来了,只好借了同学的自行车以原来步行三倍的速度回家,这时小乐发现每隔12分钟有一辆公交车从后面超过他,如果小乐步行、骑车以及公交车的速度都是匀速的话,那么公交车站发车的时间间隔到底为多少?‎ 3. 从电车总站每隔一定时间开出一辆电车。甲与乙两人在一条街上沿着同一方向步行。甲每分钟步行‎82米,每隔10分钟遇上一辆迎面开来的电车;乙每分钟步行‎60米,每隔10分15秒遇上迎面开来的一辆电车。那么电车总站每隔多少分钟开出一辆电车?‎ 初级点拨:‎ 1. 从出发到甲、乙相遇,甲比乙多走了‎620米,又甲比乙每分多走‎10米,所以从出发到甲、乙相遇共用62分钟。‎ 2. 分析甲乙两人行程路程在问题中提到的两个时间点的数量关系。‎ 3. 可以将该问题看作李云和货车车尾的相遇问题。‎ 4. 画出“折线示意图”,判断哪一次相遇距B地最近。‎ 5. 相遇的总次数与两人行程总和相关。‎ 1. 假设A、B两地相距单位“1”,确定第一次相遇时,甲、乙两人的行程。‎ 2. 因为甲比乙早返回原地,甲的速度比乙快,第二个相遇点D因该比C更靠近A点。由于相关数量未知,首先假设第一次相遇时甲和乙分别行走了x千米和y千米。由两者的数量关系列出第一个方程:‎ x+y=24‎ 3. ‎“逗号”的周长与外圆的周长相等,都是‎40厘米,所以可以假设两只蚂蚁在同一段跑道上,求出相遇点后再作判断。‎ 4. 根据基础知识所给出的公式,写出题目中各个量之间的数量关系,并将已知数代入。‎ 5. 根据基础知识所给出的公式,写出题目中各个量之间的数量关系,并将已知数代入。‎ 深度提示 1. 甲从体育场返回学校只走了62-60=2分钟就遇到了乙,所以甲的速度为310÷2=‎155米/分,学校到体育场的距离为‎155米/分×60分=‎9300米。‎ 2. 乙出发后10分钟两人与十字路口距离相等。因此如果乙从十字路口处出发后往南而不是往东,那么乙将会在10分钟时与从十字路口南‎1200米处出发的甲相遇。‎ ‎ 又因为甲、乙出发后100分两人再次与十字路口距离相等,所以如果乙从十字路口处出发后往北而不是往东,那么乙将会在100分钟时被从十字路口南‎1200米处出发的甲追上。‎ 3. ‎18秒相当于0.005小时,货车总长为 ‎(千米),‎ 由这两个条件可以得出两车的速度和。‎ 4. 甲乙行程的折线示意图如下,图中实现表示甲,虚线表示乙。‎ ‎ ‎ ‎ 由图可知,第3次相遇时距离B地最近。‎ 1. 当两人的行程和分别为‎100米、‎300米、‎500米……时,恰好是他们第1次、第2次、第3次……相遇,‎ 2. 甲乙两人第四次相遇时共行程2×4-1=7,第五次相遇时共行程2×5-1=9。‎ 3. 假设第一次相遇时甲和乙分别行走了x千米和y千米,由假设可以很容易得到第二次相遇时,甲、乙分别行走了3x千米和3y千米。这样我们能用字母表示出两人从返回到相遇分别走了多少路。结合CD距离能得到第二个方程。‎ 4. 乙比甲多爬半圈,即‎20厘米需20÷(5-3)=10(秒),多爬1.5圈需60÷(5-3)=30(秒)。‎ 5. 设公交车的间距为S,根据公式可得关系式:‎ ‎,‎ 类似的关系:‎ ‎;‎ 由两个关系式得到:‎ 等式化简为:‎ 6. 设电车的间距为S,根据公式可得关系式:‎ ‎,‎ 类似可得关系式:‎ ‎,‎ 那么,‎ 代入数值后为:‎ ‎,‎ 全解过程:‎ 1. 从出发到甲、乙相遇,甲比乙多走了‎620米,又甲比乙每分多走‎10米,所以从出发到甲、乙相遇共用62分钟。甲从体育场返回学校只走了62-60=2(分钟)就遇到了乙,所以甲的速度为310÷2=155(米/分),学校到体育场的距离为‎155米/分×60分=9300(米)。丙的速度为155-31=124(米/分),甲和丙相遇需要走两个学校到体育场的路程为:9300×2=18600(米)。‎ 所以相遇时间为:,所以甲与丙在10时6分40秒相遇。‎ 2. 如图所示:‎ 因为甲、乙出发后10分钟两人与十字路口距离相等。所以如果乙从十字路口处出发后往南而不是往东,那么乙将会在10分钟时与从十字路口南‎1200米处出发的甲相遇。由此我们得到甲、乙两人的速度和为:‎ ‎   1200÷10=120(米)。‎ ‎  又因为甲、乙出发后100分两人再次与十字路口距离相等,所以如果乙从十字路口处出发后往北而不是往东,那么乙将会在100分钟时被从十字路口南‎1200米处出发的甲追上。由此我们得到甲、乙两人的速度差为:‎ ‎1200÷100=12(米)。‎ ‎ 由两人的速度和与速度差我们能很容易得到两人的速度。‎ ‎  乙每分钟行 ‎(120-12)÷2=54(米),‎ 出发 100分后距十字路口‎5400米。‎ 3. ‎18秒相当于0.005小时,货车总长为 ‎(千米),‎ 货车车尾速度(货车速度)与李云(火车速度)的速度和为 ‎(千米/小时)‎ 所以货车的速度为104-60=44(千米/小时)。‎ 1. 甲乙的运行图如下,图中实线表示甲,虚线表示乙。‎ ‎ ‎ ‎ 由图可知,第3次相遇时距离地最近。‎ 此时两人共走了 ‎(千米),‎ 用时 ‎(分钟),‎ 相遇地点距离B地 ‎(米)。‎ 2. ‎10分钟,两人共跑了(3+2)×60×10=3000(米),即3000÷100=30个全程。我们知道两人同时从两地相向而行,他们总是在奇数个全程时相遇(不包括追上)1、3、5、7、…、29共15次。‎ 3. 假设A、B两地相距单位“1”,‎ 甲乙两人第四次相遇时共行程2×4-1=7,第五次相遇时共行程2×5-1=9。‎ 第四次相遇时甲走了 ‎,‎ 第五次相遇时甲走了 ‎,‎ 可见两次相遇地点相距,‎ 所以全程AB为(米)。‎ 1. 因为甲比乙早返回原地,甲的速度比乙快,第二个相遇点D应该比C更靠近A点。由于相关数量未知,首先假设第一次相遇时甲和乙分别行走了x千米和y千米。由两者的数量关系列出第一个方程:‎ x+y=24‎ 由假设可以很容易得到第二次相遇时,甲、乙分别行走了3x千米和3y千米。这样我们能用字母表示出两人从返回到相遇分别走了多少路。 例如甲返回时走了3x-(x+y)=2x-y 所以第二个相遇点距B点(2x-y)千米,这段距离比y多‎6千米,所以有:‎ ‎ (2x-y)-y=6‎ 联立两个方程有:能解得x=13.5,y=10.5,所以两人的速度比为9︰7。‎ 2. ‎“逗号”的周长与外圆的周长相等,都是‎40厘米,所以可以假设两只蚂蚁在同一段跑道上,乙比甲多爬半圈,即‎20厘米需20÷(5-3)=10(秒),多爬1.5圈需60÷(5-3)=30(秒),第一次乙比甲多爬‎20厘米时,甲爬了‎30厘米,位于圆内的弧线上,而乙位于外圆周上,两只蚂蚁没有相遇。乙比甲多爬‎60厘米需60÷(5-3)=30(秒),此时两只蚂蚁都在外圆周上,是第一次相遇,乙爬了5×30=150(厘米)。‎ 3. 设公交车的间距为S,根据公式可得关系式:‎ ‎,‎ 类似的关系:‎ ‎;‎ 由两个关系式得到:‎ 等式化简为:‎ 根据公交车发车过程中的数量关系有 ‎,(其中t为发车的时间间隔)‎ 因此有等式:,‎ 将代入得到:‎ ‎(分钟)‎ 1. 设电车的间距为S,根据公式可得关系式:‎ ‎,‎ 类似可得关系式:‎ ‎,‎ 那么,‎ 代入数值后为:‎ ‎,‎ 方程解得(米/分),‎ 代入第一个式子可得 解得:米,‎ 因此发车间隔为9020÷820=11(分钟)。‎ 第14讲 行程(二) 平均速度、变速度、流水、电梯 一、 知识地图 ‎ 变速直线行程(求平均速度)‎ ‎ 流水行船 ‎ 不同参照系的行程 ‎ 自动扶梯 在前一讲中,我们讨论了行程问题中典型的相遇和追及问题以及这类问题的拓展——多次相遇和公交车发车间隔问题,细心的读者应该注意到,前一讲讨论的问题中所涉及人或车辆的运动速度都是不变的,但是在我们的实际生活中,不会出现真正意义上的匀速运动,这时我们就要引入“平均速度”这一概念:‎ 二、 基础知识 (一) 和平均速度有关的题 公式中的“总路程”“总时间”“平均速度”都是相对应的,例如:一辆卡车从A地到B地所花时间为T1,再从B地到C地所花时间为T2,那么这辆卡车在AB之间的平均速度,BC间的平均速度为,而AC间的平均速度为。‎ 注意,平均速度不是多段路程上不同速度的简单平均值,一般有关平均速度的问题,仅给出部分路段行驶速度,而缺省路程或时间的数量条件。根据缺省条件的不同可以将求平均速度的题目分为两类:一类题目缺少路程条件,但给出了时间(或时间比);另一类缺少时间条件,但给出了路程(或路程比)。前一类题目应该设总时间为特定的数值。对于后一类题目,我们通常使用假设总路程为特定数值的方法来解题,但无论用什么方法解答哪一类问题,都应该根据公式,将各个数量化为总路程和总时间的比后再求平均速度。‎ (二) 流水和电梯 ‎ 涉及速度的变化不得不提在不同参照系中速度的变化,典型的问题有“流水行船问题”和“自动扶梯行走问题”。‎ ‎ ‎ 流水问题是研究船在顺水和逆水中船只速度关系问题,流水问题的典型之处在于船在河流中航行时,除了本身的前进速度外,还受到流水的推动或阻滞,所以顺流而下的速度和逆流而上的速度不同,行船速度除了跟船只本身的速度有关外,还受到河流中的流水速度的影响。‎ 逆水船速=静水船速-水流速度;‎ 顺水船速=静水船速+水流速度;‎ 由以上两条关系式结合和差原理,能得到以下两个公式:‎ 静水船速=‎ 水流速度=‎ 除此以外,在流水行船问题中还经常运用到一条性质:河流漂流物体速度=水流速度。‎ 流水行船问题中的相遇与追及问题看似复杂,但所涉及的量仍遵循相遇问题或追及问题中的数量关系,我们举个例子,在一条河流的上下游有甲、乙两个港口,相距‎120千米,一艘游船从甲港出发往下游航行,与此同时一艘快艇从乙港出发往上游航行,已知游船的静水速度为‎15千米/小时,快艇的静水速度为‎25千米/小时,求经过多长时间两船相遇?‎ 这道题只给出了两艘船的静水速度,而要求相遇的时间必须知道船的实际速度。当然,实际速度可以由船的静水速度和水速计算得到,所以我们只要知道流水的速度就能求出两艘船的实际速度,为了研究这个问题,我们将不同水速下的相遇情况列表:‎ 流水速度 游船实际速度 快艇实际速度 两船速度和 相遇时间 ‎0千米/小时 ‎15千米/小时 ‎25千米/小时 ‎40千米/小时 ‎3小时 ‎5千米‎/小时 ‎20千米/小时 ‎20千米/小时 ‎40千米/小时 ‎3小时 ‎10千米/小时 ‎25千米/小时 ‎15千米/小时 ‎40千米/小时 ‎3小时 ‎15千米/小时 ‎30千米/小时 ‎10千米/小时 ‎40千米/小时 ‎3小时 之所以有以上结论,是因为在流水环境中,甲乙两船的速度有如下数量关系。‎ 甲船顺(逆)水速度+乙船逆(顺)水速度 ‎=(甲船速±水速)+(乙船速水速)‎ ‎=甲船静水船速+乙船静水船速。‎ 同样的在追及问题也有类似的数量关系:‎ 甲船顺(逆)水速度-乙船顺(逆)水速度 ‎=(甲船速±水速)-(乙船速±水速)‎ ‎=甲船静水船速-乙船静水船速。‎ 由此我们能总结出一个一般性的结论:水速对相向行驶的两船速度和或同向行驶的两船速度差没有影响,所以水速对于相遇和追及的时间不产生影响。‎ (一) 自动扶梯问题 与流水行船问题类似的有自动扶梯上行走的问题,与行船问题类似的,自动扶梯的速度有以下两条关系式:‎ 顺行速度=正常行走速度+扶梯运行速度 逆行速度=正常行走速度-扶梯运行速度 与流水行船不同的是,自动扶梯上的行走速度有两种度量,一种是“单位时间运动了多少米”,一种是“单位时间走了多少级台阶”,这两种速度看似形同,实则不等,拿流水行程问题作比较,“单位时间运动了多少米”对应的是流水行程问题中的“船只顺(逆)水速度”,而“单位时间走了多少级台阶”对应的是“船只静水速度”,一般奥赛题目涉及自动扶梯的问题中更多的只出现后一种速度,即“单位时间走了多少级台阶”,所以处理资料的时候要非常小心,理清了各种数量关系,自动扶梯上的行程问题会变得非常简单。‎ ‎ 这两种度量统一于流水行船的数量关系:‎ ‎ 顺水行程=顺水速度×时间 顺水行程=(船速+水速)×时间 ‎ 扶梯长度=顺向速度×时间 扶梯长度=(人速+梯速)×时间 ‎ 或者:‎ ‎ 逆水行程=逆水速度×时间 逆水行程=(船速-水速)×时间 ‎ 扶梯长度=逆向速度×时间 扶梯长度=(人速-梯速)×时间 ‎ 结合后一种度量,有以下数量关系式:‎ ‎ 扶梯长度= 人速×顺向时间+梯速×顺向时间 ‎ 或者:‎ ‎ 扶梯长度= 人速×逆向时间-梯速×逆向时间 ‎ 在解决自动扶梯问题时,通常的解法是利用后面两个关系式作为等量关系来列方程,这里最关键的是求出时间比。‎ 一般而言,时间比可以通过所走的路程与速度进行计算。‎ 例如:“甲上楼行走的速度是乙下楼行走速度的3倍,甲上楼走了54级,乙下楼走了36级。”由这一组条件,我们可以求出甲上楼与乙下楼所用的时间的比为:‎ 所以,如果题中再给出“扶梯是从上往下行驶”的话,可以列出方程:‎ ‎ 54-x=36+2x ‎ 这里,其实运用了量化思想,即把比的份数代替数量进行运算。‎ ‎ 实际上,这里如果作一般性的考虑,应该把甲的速度设为‎3a,乙的速度设为a,可以说,这又是一种设而不求的思想在里面。我们容易知道:‎ ‎ ‎ ‎ 正所谓殊途同归!‎ 一、 经典透析 【例1】 有一座桥,过桥需要先上坡,再走一段平路,最后下坡,并且上坡、平路及下坡的路程相等。某人骑自行车过桥时,上坡、走平路和下坡的速度分别为‎4米/秒、‎6米/秒和‎8米/秒,求他过桥的平均速度。‎ ‎[审题要点]要求平均速度必须知道总路程和总时间,在总路程未知的情况下,可以假设总路程,化未知为已知。‎ ‎[详解过程]假设上坡、平路、下坡的长度都是“1个单位”:那么上坡、平路、下坡所花时间依次为:‎ ‎;;。‎ 所花的总时间为:‎ 而总路程为:‎ 所以他过桥的平均速度为:‎ ‎(米/秒)‎ ‎[点评]注意本道题中假设的单位长度可以随意,例如可以假设上坡、平路、下坡的长度为“24个单位”,因为24是4、6、8的最小公倍数,所以计算出来各段时间都是整数,这样更方便于计算。‎ 【例1】 老王开汽车从A到B为平地,车速是‎30千米/时;从B到C为上山路,车速是‎22.‎‎5千米/时;从C到D为下山路,车速是‎36千米/时。已知下山路是上山路的2倍,从A到D全程为‎72千米,老王开车从A到D的平均速度是多少?‎ ‎[审题要点] 涉及到平均速度必须知道总路程和总时间而这道题目中只知道各段路程的速度,所以我们还是要用到假设法。‎ ‎[详解过程]这一次我们假设上山的路程为“180个单位”(180是22.5和36的公倍数)那么下山的路程为“360个单位”。‎ 上山的时间为 CD段所花的时间为:‎ 那么从B到D的总时间为:‎ 所以从B到D的平均速度为:‎ 既然从A到B,从B到D的平均速度都是‎30千米/小时。那么从A到D的平均速度为‎30千米/小时。‎ 点评:1,当几个速度都相等时,那么无论时间是多少 ‎,平均速度都等于这个相等的速度。事实上所有有关平均的概念都有这个性质。例如:a,几个班平均分都一样的情况下,不管人数有多少,总平均分就是这个相等的平均分。b,浓度问题中,两个溶液浓度一样时,无论它们质量有多少,那么混合后的浓度都等于这个相等的浓度。‎ ‎2,路程有倍数关系,所以选择假设路程数进行计算。‎ 【例1】 某司机开车从A城到B城。若按原定速度前进,则可准时到达。当路程走了一半时,司机发现前一半行程中,实际平均速度只达到原定速度的。如果司机想准时到达B城,那么在后一半的行程中,实际平均速度与原定速度的比应是多少?‎ ‎[审题要点]这道题目中只给出了速度比,而没有任何时间、路程等量,所以这道题目中至少应该假设两个量。‎ ‎[详解过程]由于要求速度的比例关系,所以可将原定速度设为13,那么前半路程速度为11,然后和上面题目类似,假设总路程的一半的长度为143,那么原定总时间为143×2÷13=22,‎ 而前半段时间为 ‎143÷11=13,‎ 所以后半段时间为 ‎22-13=9,‎ 后半段速度为 ‎143÷9=,‎ 所以所求比例为:‎ ‎。‎ 【例2】 某船从甲地顺流而下,5天到达乙地;该船从乙地返回甲地用了7天。问水从甲地流到乙地用了多少时间?‎ ‎[审题要点]题目只给出时间条件,而缺少路程或速度条件。要解决此题,求出顺水速度、逆水速度和水速,所以有必要假设路程量。‎ ‎[详解过程]水流的时间=甲乙两地间的距离÷水速,‎ 将甲、乙两地距离看成单位“1”,则顺水每天走全程的,逆水每天走全程的。‎ 水速=(顺水速度一逆水速度)÷2=,所以水从甲地流到乙地需:(天)‎ ‎[点评]这类问题的行程数量关系可以用线段图示来说明:‎ 将船静水中行驶一天的距离看成1大段,将流水一天的行程看作1小段,那么甲乙之间的距离等于5个大段再加上5个小段,也等于7个大段减去7个小段,由此可以得到等量关系:‎ 静水速度×5天+水速×5天=静水速度×7天-水速×7天,‎ 有这条等式能得出:静水速度=6×水速,所以甲乙之间的距离也可以表示成 流水速度×(5+5×6)天,‎ 即水从甲地流到乙地用35天。‎ 【例1】 船往返于上下游的两港之间,顺水而下需要用10小时,逆水而上需要用15小时。由于暴雨后水速增加,该船顺水而行只需9小时,那么逆水而行需要几小时?‎ ‎[审题要点]如果知道上下游两港之间的距离,那么本题中船在顺水、逆水、静水中的速度以及水流的速度都可以求出。所以我们可以首先假设上下两港之间的距离为“‎1”‎个单位。‎ ‎[详解过程]船在静水中的速度是:‎ ‎(单位/小时)。‎ 暴雨后水流的速度是:‎ ‎(单位/小时)。‎ 暴雨后船逆水而上需用的时间为:‎ ‎(小时)。‎ 点评:此题中有一个不变量需要找出,即暴雨前后的船静水速度不变,而水流速度是改变的。有的题目中是抓住水流速度不变,例如两只船在同一条河流中行驶,那么就是同一个水速,而船的静水速度不同。不变量的寻找是解决所有应用题的关键,因为不变量相当于桥梁作用,将各种变量联系起来。‎ 【例2】 江上有甲、乙两码头,相距‎15千米。甲码头在乙码头的上游,一艘货船和一艘游船同时从甲码头和乙码头出发向下游行驶,5小时后货船追上游船。又行驶了1小时,货船上有一物品掉入江中(该物品可以浮在水面上),6分钟后货船上的人发现了,便掉转船头去找,找到时恰好又和游船相遇。则游船在静水中的速度为每小时多少千米?‎ ‎[审题要点]结合本讲基础知识中一般性结论——水速对于相遇和追及的时间不产生影响,对整个行程过程进行分析,找出其中隐含的数量关系。‎ ‎[详解过程]这道题给出的条件比较零碎,我们需要一点一点整理,首先,‎ 两艘船从相距‎15千米的两港出发后5小时,其中一艘船赶上另一艘船。所以有以下数量关系。‎ 货船静水速度-游船静水速度=15÷5=3(千米/小时)‎ 所以,这两艘船的静水速度差为‎3千米/小时,‎ 然后看相遇后一小时,因为两艘船的速度差为‎3千米/小时,所以一小时后两船之间的距离为‎3千米。又过了6分钟,货船与物品之间距离可以表示为:‎ 货船静水速度×6分钟,‎ 因此货船回去找物品所需要的时间为:‎ 货船静水速度×6分钟÷货船静水速度=6分钟 所以从物品掉落到两艘船相遇,共过了12分钟。‎ 即游船追上掉落的物品之间距离花了12分钟,即0.2小时。‎ 游船静水速度×0.2小时=‎3千米,‎ 游船的静水速度为‎15千米/小时。‎ ‎[点评]这是一道清华附中“同方杯”竞赛试题,实际上如果将这道问题放到流水这个参照系中来看问题会变得非常简单,因为以流水为参照物,游船、货船都是以静水速度运动,而物品相当于停留在原地不动,货船六分钟后发现物品丢失,所以返回到物品处也是花了六分钟,那么游船在此12分钟之内行完之前两船一小时之内拉开的距离‎3千米,所以直接求出游船的静水速度:3÷=15(千米/小时)。‎ 【例1】 甲、乙两名选手在一条河中进行划船比赛,赛道是河中央的长方形ABCD,其中AD=‎100米,AB=‎80米,已知水流从左到右,速度为每秒‎1米,甲乙两名选手从A处同时出发,甲沿顺时针方向划行,乙沿逆时针方向划行,已知甲比乙的静水速度每秒快‎1米,(AB、CD边上视为静水),两人第一次相遇在CD边上的P点,4CP=CD,那么在比赛开始的5分钟内,两人一共相遇几次?‎ ‎[审题要点]这道题涉及到两个人在3种水流环境中的共计6种速度,所以整个行程过程非常复杂,但如果各个速度之间的关系已知,那么只要知道其中一个速度就可以求得所有的速度,由此可以列出方程。‎ ‎[详解过程]设乙的速度为x米/秒,根据两人的游程距离和速度,可列出方程:‎ 解得:x=3。所以甲的速度为‎4米/秒。‎ 甲游一圈需要 ‎(秒)‎ 乙游一圈需要 ‎(秒)‎ ‎5分钟内,甲游了3圈还多20秒,乙游了2圈还多秒。‎ 多余的时间不够合游一圈,所以两人合游了5圈。‎ 因为每次两人相遇时,他们的游程之和增加了一圈,所以两人共相遇了5次。‎ ‎[点评]题解中的方程经过化简后能化成我们常见的比例式:,按照解分式方程的步骤要求,最后应该对结果进行检验。‎ 此题中两人的速度也可以不需要列方程而直接用算术方法求出:由于水流速度为1,甲的静水速度比乙大1,所以甲的静水速度等于乙的顺水速度,所以甲在AB和CP段的速度等于乙在AD的速度,而刚好AB+CP=AD=‎100米,所以两段时间一样,那么甲在BC段的时间也和乙在PD段的时间一样,路程比为100:60=5:3,所以速度成正比为5:3,而甲在BC段的顺水速度比乙在PD段的静水速度大2,所以根据按比例分配(差分配)的原理可知甲的顺水速度为5,乙的静水速度为3,继续求出其他各个速度。‎ 【例1】 ‎(2006年南京智力数学冬令营)商场的自动扶梯以匀速由下往上行驶,两个孩子在行驶的扶梯上上下走动,女孩由下往上走,男孩由上往下走,结果女孩走了40级到达楼上,男孩走了80级到达楼下。如果男孩单位时间内走的扶梯级数是女孩的2倍,则当该扶梯静止时,可看到的扶梯梯级有多少级。‎ ‎[审题要点] 关键找出两人上下楼的时间比。‎ ‎[详解过程]因为男孩的速度是女孩的2倍,所以男孩走80级到达楼下与女孩走40级到达楼上所用时间相同,男孩下楼过程中由于自动扶梯上行而多走的路应该等于女孩上楼过程借助自动扶梯少走的路,‎ 男孩比女孩多走的路应等于行程过程中自动扶梯运行的级数的两倍。‎ 因此自动扶梯向上运行了 ‎(80-40)÷2=20(级),‎ 扶梯可见部分有 ‎80-20=60(级)。‎ 点评:因为此题中两人行走的时间即扶梯运行的时间相等,所以男孩多行走的路程等于女孩少行走的路程,所以可以不用列出方程直接利用和差问题公式算出扶梯运行级数,但是当时间不相等时,就应该根据电梯问题中的两个基本关系式,列出方程进行解答,如下题。‎ 【例2】 在商场里,小明从正在向上移动的自动楼梯顶部下120级台阶到达底部,然后从底部上90级台阶回到顶部。自动楼梯从底部到顶部的台阶数是不变的,假设小明单位时间内下的台阶数是他上的台阶数的2倍。则该自动楼梯从底到顶的台阶数为多少?‎ ‎[审题要点] 关键找出上下楼的时间比。‎ ‎[详解过程]小明单位时间内下的台阶数是他上的台阶数的2倍,而小明下楼梯跨了120级,上楼梯跨了90级,所以小明下楼和上楼所花的时间比为:‎ 自动楼梯在相同的时间内运行相同的级数,假设在小明下楼梯过程中,自动楼梯运行了2x级,自动楼梯可见部分为:120-2x,而在小明上楼的过程中,自动楼梯运行了3x级,所以自动楼梯可见部分为:90+3x,由此可列得方程:‎ ‎120-2x=90+3x 解得x=6,‎ 自动楼梯的可见台阶数为。120-6×2=108。‎ ‎ [点评]使用图示可将问题中的数量关系表示出来:将小明上楼期间自动扶梯上行台阶数看作2份,那么小明下楼期间,自动扶梯上行3份,那么5份的台阶数相当于120-90=30份。所以每份的台阶数为6,自动楼梯从底到顶的台阶数为90+6×3=108。‎ 一、 拓展训练 1. 甲、乙两班进行越野行军比赛,甲班以每小时4‎.5千米的速度走了路程的一半,又以每小时5‎.5千米的速度走完了另一半,乙班用一半时间以每小时4‎.5千米的速度行进,另一半时间以每小时5‎.5千米的速度行进。问:甲、乙两个班谁将获胜?‎ 2. 一辆汽车从甲地出发到‎300千米外的乙地去,在一开始的‎120千米内平均速度为‎40千米/小时,要使这辆车从甲地到乙地的平均速度为每小时50千米,那么剩下的路程应该以什么速度行驶?‎ 3. 学校组织春游,同学们下午一点出发,走了一段平坦的路,爬了一座山,然后按原路返回,下午七点回到学校。已知他们的步行速度平地为‎4千米/时,上山为‎3千米/时,下山为‎6千米/时。问:他们一共走了多少路?‎ 4. 从家到学校有两条一样长的路,一条是平路,另一条的一半是上坡路,一半是下坡路。小明上学走两条路所用的时间一样,如果下坡的速度是平路的倍,那么上坡的速度是平路的多少倍。‎ 5. ‎(2007年第五届“走进美妙的数学花园”决赛)今有A、B两个港口,A在B的上游‎60千米处。甲、乙两船分别从A、B两港同时出发,都向上游航行。甲船出发时,有一物品掉落水中,浮在水面,随水流漂往下游。甲船出发航行一段后,调头去追落水的物品。当甲船追上落水物品时,恰好和乙船相遇。已知甲、乙两船在静水中的航行速度相同,且这个速度为水速的6倍。当甲船调头时,甲船已航行多少千米?‎ 1. 两船在静水中速度相同,它们同时自河的两个码头相对开出,3小时后相遇。已知水流速度是‎4千米/小时。求:相遇时甲、乙两船航行的距离相差多少千米?‎ 2. ‎(实验中学阶段测试题)某人畅游长江,逆流而上,在A处丢失一只水壶,他向前又游了20分钟后,才发现丢失了水壶,立即返回追寻,在离A处‎2千米的地方追到,则他返回寻水壶用了多少分钟?‎ 3. 在地铁车站中,从站台到地面有一架向上的自动扶梯。小强乘坐扶梯时,如果每秒向上迈一级台阶,那么他走过20级台阶后到达地面;如果每秒向上迈两级台阶,那么走过30级台阶到达地面。从站台到地面有多少级台阶?。‎ 4. 小淘气乘正在下降的自动扶梯下楼,如果他一级一级的走下去,从扶梯的上端走到下端需要走36级。如果小淘气沿原自动扶梯从下端走到上端(很危险哦,不要效仿!),用下楼时5倍的速度需要走60级才能走到上端。请问这个自动扶梯在静止不动时有多少级?‎ 5. 小志与小刚两个孩子比赛登自动扶梯,他俩攀登自动扶梯上的速度分别为每秒2个台阶和每秒3个台阶,自动扶梯运行后,他俩沿自动扶梯运行方向的相同方向从一楼登上二楼,分别用时28秒和20秒,那么如果小志攀登静止的自动扶梯需要用时多少秒?‎ 初级点拨:‎ 1. 乙班的平均速度为‎5千米/小时。所以只要求出甲班的平均速度,即可判断谁输谁赢。‎ 2. 要求速度,可先求路程和时间,路程为300-120=180(千米)。‎ 3. 由于同学们按原路返回,所以去的时候的上坡,回来时变成下坡,去的时候的下坡回来时就是下坡,因此总路程中下坡的路程和上坡的路程相等。‎ 4. 假设路程和上学时间两个未知数。‎ 5. 首先应该知道水的速度就是物品的速度,船与物品的相对速度(单位时间的距离变化)与船的静水速度相等。而从两船出发到甲船掉头,此外,两船之间无论顺水速度差、静水速度差还是逆水速度差都相等,所以两船之间的距离总是保持‎60千米不变。‎ 1. 要求出相遇时两船航行的距离相差多少,既可以用总路程求差,也可以用速度差乘以时间。‎ 2. 该人丢失水壶后继续逆流而上20分钟,而水壶顺流而下:两者速度和=该人的逆水速度+水速=该人的静水速度-水速+水速=该人的静水速度。‎ 3. 首先求出两种上楼方式所花的时间。‎ 4. 首先上下楼所花时间的时间比。这个时间比等于上下楼过程中自动扶梯运行级数比。‎ 5. 小志和小刚顺向攀登运行的自动扶梯分别都攀登了28×2=56级和20×3=60级,小刚比小志多走了60-56=4级,这4级台阶实际上是小志多走的8秒钟内,自动扶梯“缩”进去的。‎ 深度提示:‎ 1. 设总路程为“1”个单位。则可求出甲班各路段所花的时间。‎ 2. 这辆车的平均速度为‎50千米/小时,所以这辆车路上花的总时间为:‎ ‎300÷50=6(小时)。‎ 3. 设总路程中上坡的路程为“1”个单位。那么下坡的路程也为“1”个单位,上坡所花的时间为,下坡所花的时间为:。‎ 4. 设从家到学校的路程为S,上学时间为T,那么平路上的速度为,那么下坡的速度为,由此可求得下坡时间和上坡时间。‎ 5. 由于甲、乙两船同时碰到物品,所以从甲掉头到两船相遇,两船与物品的距离总是相等的,甲船掉头之时,两船距离物品都是‎30千米。‎ 6. 若考虑将两船的各自航程分别求出的话,需根据:航程=速度×时间,要求出两船的顺水速度或逆水速度,即要求两船(在静水中)的船速。而由已知条件分析,船速无法求出。所以我们只能用速度差乘以时间。而两船的船速相同的情况下,一船顺水,一船逆水,造成速度差的原因是流水速度。‎ 7. 此人发现该丢失水壶时,人与水壶的距离=二者速度和×时间=20×该人的静水速度。该人发现水壶丢失后返回,与水壶一同顺流而下时。二者速度差=该人的静水速度。‎ 8. 每秒迈一级台阶走20级所花时间为20秒,每秒迈二级台阶走30级所花的时间为15秒,而相等时间自动扶梯运行级数相等,自动扶梯运行级数的比等于时间比。‎ 9. 小淘气上楼走60级的时间,下楼只能走60÷5=12(级)。而下楼走了36级,所以下楼用的时间是上楼时间的36÷12=3(倍)。‎ 1. 小志多走的8秒钟内,电梯多运行了4个台阶,因此自动扶梯的运行速度为每秒半个台阶。‎ 全解过程:‎ 1. 显然乙班的平均速度为‎5千米/小时。‎ 设总路程为“1”个单位。‎ 甲班前半段的所花时间为:‎ ‎(单位·时/千米)‎ 后半段所花的时间为:‎ ‎(单位·时/千米)‎ 甲班所花的总时间为:‎ ‎(单位·时/千米)‎ 所以甲班的平均速度为:‎ ‎(千米/小时)‎ 所以乙班的平均速度高于甲班,乙班将获胜。‎ 2. 要求速度,应先求出路程和时间 路程为300-120=180(千米)‎ 这辆车路上花的总时间为:‎ ‎300÷50=6(小时)‎ 而前‎120千米的已经花了 ‎120÷40=3(小时)。‎ 所以剩下的‎180千米的路程只能在3小时内走完。‎ 所以汽车在剩下的路上的平均速度为:‎ ‎180÷3=60(千米/小时)。‎ 3. 设总路程中上坡的路程为“1”个单位。那么下坡的路程也为“1”个单位,上坡所花的时间为,下坡所花的时间为:。‎ 上坡下坡所花的总时间为:‎ 所以在坡路上的平均速度为:‎ 学生们在平路和坡地上的平均速度都等于‎4千米/小时,所以他们整个春游中的平均速度为‎4千米/小时,6个小时中一共行走了6×4=‎24千米/小时。‎ 4. 设从家到学校的路程为S,上学时间为T,那么平路上的速度为,那么下坡的速度为,下坡时间为:‎ ‎,所以上坡所花的时间为:,所以上坡的速度为:。所以上坡速度是平路速度的倍。‎ 1. 首先应该知道水的速度就是物品的速度,船与物品的相对速度(单位时间的距离变化)与船的静水速度相等。而从两船出发到甲船掉头,此外,两船之间无论顺水速度差、静水速度差还是逆水速度差都相等,所以两船之间的距离总是保持‎60千米不变。 ‎ 由于甲、乙两船同时碰到物品,所以从甲掉头到两船相遇,两船与物品的距离总是相等的,甲船掉头之时,两船距离物品都是‎30千米,甲船到物品‎30千米这段距离的产生时间,相当于船在静水中航行‎30千米的时间,在这段时间内,河水流动了30÷6=‎5千米,所以甲掉头时,已经行驶了30-5=‎25千米。‎ 2. 甲船的顺水速度=船速+水速,‎ ‎ 乙船的逆水速度=船速一水速,‎ 故:速度差=(船速+水速)一(船速一水速)=2×水速,‎ 即:每小时甲船比乙船多走2×4=8(千米)。‎ ‎3小时的距离差为3×8=24(千米)。‎ 3. 该人与水壶的距离=二者速度和×时间=20×该人的静水速度。该人发现水壶丢失后返回,与水壶一同顺流而下。二者速度差=该人的静水速度。所以他返回寻水壶用了的时间=20×该人的静水速度÷该人的静水速度=20(分钟)。‎ 4. 每秒迈一级台阶走20级所花时间为20秒,每秒迈二级台阶走30级所花的时间为15秒,设20秒扶梯向上走级,则15秒走了级。由扶梯长度可得20+=30+,解得=40,扶梯长20+40=60(级)。‎ 5. 小淘气上楼走60级的时间,下楼只能走60÷5=12(级)。而下楼走了36级,所以下楼用的时间是上楼时间的 ‎ ‎36÷12=3(倍)。‎ 设小淘气上楼的时间自动扶梯走了x级,根据自动扶梯的级数可列方程:‎ ‎ 36+3x=60-x。‎ 解得x=6(级),自动扶梯有 ‎60-x=54(级)。‎ 6. 小志和小刚顺向攀登运行的自动扶梯时分别都攀登了28×2=56级和20×3=60级,小刚比小志多走了60-56=4级,这4级台阶实际上是小志多走的8秒钟内,自动扶梯“缩”进去的。 小志多走的8秒钟内,电梯多运行了4个台阶,因此自动扶梯的运行速度为每秒半个台阶。那么在小刚登梯的20秒内,自动扶梯也缩了10级,自动扶梯所能见到的部分是60+10=70级,所以,小志攀登静止的自动扶梯分别需要用时70÷2=35秒。‎ 第15讲 行程(三) 行程中的比例 一、 知识地图 ‎ 行程中的比例关系 其他类型 (正反比例运用)‎ ‎ 相遇点变化问题 基础知识 行程问题中有很多经典题目需要用比例关系,由行程问题的基本关系式“路程=速度×时间”即可推导出三条基本比例关系:‎ 速度相同,时间与路程成正比。‎ 事实上在前两讲的行程问题已经涉及第一条比例关系,例如在多次相遇问题中,我们推导任意两次相遇事件发生时间的间隔相等,以及在自动扶梯上的行走问题中,计算电梯运行级数比,都用到了“速度相同,时间比等于路程比”这条比例关系式。‎ 时间相同,速度比与路程成正比。‎ 第二条比例关系在行程问题中运用非常广泛,因为“由时间相同,速度比等于路程比”可以得到很多推论,例如,有这样一个问题:‎ 甲、乙两人原计划同时分别从A、B出发相向而行,并在A、B之间的C点相遇,可是天降暴雨,甲、乙两人只能以原来速度的一半行进,已知甲的原速度大于乙的原速度,问,两人的相遇点在C点的哪一侧?‎ 这个问题,如果用第二条行程比例关系来说明,就非常清楚了,假设甲乙两人原计划的速度分别为‎2a和2b,那么在相遇的相同时间内两人的行程之比为a︰b,而减速后两人实际的速度分别为a和b,那么在相遇的相同时间内两人的实际行程之比为a︰b,原计划和实际的两人行程比也是相等的,所以实际的相遇点还是C点。 ‎ (一) 路程相同,速度比等于时间的反比。‎ 第三条比例关系是行程问题中的难点,通常在题目中以以下两种方式出现:‎ 1. 原计划汽车速度为‎40千米/小时开往目的地,实际汽车的速度为‎50千米 每小时,结果汽车比原计划早到1个小时。求原计划所花时间。‎ ‎ 这类题的算术解法一般有两种:‎ 解法(一):原计划比实际多花的一个小时,本应该多行驶‎40千米,但是实际行驶和原计划行驶的距离是相等的,所以在实际行驶的若干小时内比相同时间原计划速度行驶的距离多‎40千米,而每小时实际行程比原计划行程多‎10千米,所以实际行驶的时间为40÷10=4(小时),原计划所花时间为5小时。‎ 这种解法可以用矩形图来表示,如图:矩形的长表示行程时间,宽表示行程速度,那么面积就表示行程路程,矩形C的长宽已知,所以能求出矩形C的面积为1×40=40,又因为路程相等,矩形A与矩形B的面积和等于矩形C与矩形B的面积和,所以A的面积等于矩形C的面积, 矩形A的长为40÷10=4。‎ 解法(二):原计划行驶速度与实际速度的比为4︰5,那么由于路程相同,所以原计划所花时间和实际所花时间比为实际速度的反比即5︰4,而“汽车比原计划早到1个小时”,由此很容易得出原计划所花时间为1÷(5-4)×5=5(小时)。‎ 1. 原计划汽车到目的地要花5小时,实际汽车比原计划多‎10千米/小时,结果只花了4个小时,求原计划汽车的速度。‎ 同样的这类题算术解法也一般有两种:‎ 解法(一):实际汽车每小时比原计划多‎10千米/小时,四个小时多行‎40千米,原计划少行的这‎40千米,实际上在最后的一个小时内补上了,所以原计划汽车的速度为‎40千米/小时。如图:‎ 解法(二):实际汽车行程时间与原计划时间比为5︰4,由于路程相同所以实际汽车的速度和原计划汽车速度比为原计划时间比的反比4︰5,而汽车速度差已知,所以很容易能得出原计划汽车的速度为‎40千米/小时。‎ 两类题目的第二种解法都使用了行程问题中的第三条比例关系,相对第一种解法而言,运用比例解决行程问题,分析和解答的过程更为自然。‎ 一、 经典透析 【例1】 ‎(☆☆)某人沿公路前进,迎面来了一辆汽车,他问司机:“后面有骑自行车的人吗?”司机回答:“十分钟前我超过一辆骑自行车的人。”这人继续走了十分钟,遇到了这个骑自行车的人,如果自行车的速度是人步行速度的三倍。问汽车速度是人步行速度的多少倍?‎ [审题要点] 由速度比就能得到路程比,求速度比也应该首先知道路程比,关键在于汽车、自行车、行人之间的路程比转化。‎ [详解过程] 如图,自行车的速度是人步行速度的3倍,所以相同时间内自行车行走的路程是人步行路程的3倍,所以从行人与汽车相遇到行人与自行车相遇的十分钟内,自行车与行人所行路程比为3︰1,而这个相遇过程中的总路程实际上就是行人与汽车相遇时汽车与自行车的距离,所以从汽车超过自行车时到汽车与行人相遇,自行车与汽车的行驶路程比为 ‎  3︰(3+3+1)=3︰7‎ 所以自行车的速度和汽车速度的比为3:7,联系自行车的速度是行人速度的3倍,行人与汽车的速度比为:‎ ‎ (3÷3)︰7=1︰7‎ 所以汽车的速度是行人速度的7倍。‎ 【例2】 ‎(☆☆)甲班与乙班学生同时从学校出发去‎15千米外的公园游玩,甲、乙两班的步行的速度都是每小时‎4千米。学校有一辆汽车,它的速度是每小时48千米,这辆汽车恰好能坐一个班的学生。为了使两班学生在最短时间内到达公园,那么甲班学生与乙班学生需要步行的距离是多少千米?‎ [审题要点] 关键是找到步行距离、汽车行驶距离、总路程之间的比例关系。‎ [详解过程] 由于两班速度相同,所以要使时间最少,必须同时出发,同时到达,因此行走的路程要相同,即AD=CB,画图如下:‎ 在某一班行走BC的时间内,车行走的路程就是C—A—B,即CB+BA+AB,这样得出 CB︰(CB+BA+AB)=4︰48=1︰12‎ 该比例式可以化为:‎ CB︰BA=1︰=1︰5.5‎ 所以CB和总路程的比为 ‎1︰(1+5.5+1)=1︰7.5=2︰15‎ CB的长度为 ‎ (千米)‎ 所以每个班步行的距离为‎2千米。‎ 点评:此题的解决主要有两个关键点:‎ ‎1,两个班的行走路程一样。‎ ‎2,找出步行与汽车在相同时间内行走的路程,根据路程与速度成正比的关系得出相应路程的比例关系,最终求出答案。‎ 【例1】 ‎(☆☆☆)甲、乙、丙三个班的学生一起去郊外活动,他们租了一辆大巴,已知学生步行的速度为‎5千米/小时,大巴车的行驶速度为‎55千米/小时,出发地到终点之间的距离为‎8千米,但大巴只够一个班的学生坐,于是他们计划先让甲班的学生坐大巴,乙、丙两班的学生步行,甲班学生搭乘大巴一段路后,下车步行,然后大巴车回头去接乙班学生,并追赶上步行的甲班学生,再回头载上丙班学生后一直驶到终点,如果三个班的学生同时到达,求这些学生到达终点一共所花的时间。‎ [审题要点] 关键是找到步行距离、汽车行驶距离、总路程之间的比例关系。‎ [详解过程] 分析:由于题目条件只涉及速度和总路程,所以如果要求出时间必须首先将速度和路程对应起来,即明确学生或者大巴车的行程路段,因此我们应该画出整个行程过程的线段示意图。‎ 如图所示:虚线为学生步行部分,实线为大巴车行驶路段,由于大巴车的速度是学生的11倍,所以大巴车第一次折返点D到出发点A的距离是乙班学生搭车前步行距离AB的(11+1)÷2=6倍,如果将乙班学生搭车前步行距离AB看作是一份的话,大巴车第一次折返点到出发点的距离AD为6份,大巴车第一次折返点D到接到乙班学生B又行驶了5份距离,‎ 同样的大巴车在B点接到乙班学生到在E点追上甲班学生所走的路程也应该是6份距离,而从E点回来到C点接到丙班的距离为5份,‎ 大巴车从C点到终点F的距离为6份,‎ 这样大巴车一共行驶了6+5+6+5+6=28份距离,而A到F的总距离为6-5+6-5+6=8份,所以大巴车一共行驶了 ‎8÷8×28=28(千米),‎ 所花的总时间为小时。‎ 【例1】 ‎(☆☆☆)小张、小李和小王于某日上午分别步行、骑自行车和开汽车从A地出发沿公路向B地匀速前进。已知小李比小张晚1小时出发,小王比小李晚45分钟出发。他们三人恰在中途某地相遇。若小李比小张早到达B地24分钟,则小王比小张早多少分钟。‎ [审题要点] 由题目可知小张、小李、小王都是以匀速前进,且无论相遇点之前和相遇点之后总行程都相等,所以我们应当使用“路程相同,速度比等于时间的反比”这条比例关系,‎ [详解过程] 步骤一、先看小张和小李的相遇前后的两个追及,相遇前的追及路程为小张行走一小时的路程,相遇后的追及路程为小张行走24分钟的路程,所以追及路程比为 ‎60:24=5:2,‎ 两人速度都不变,所以速度差也不变,所以追及时间比为5:2,所以小李前后行走的时间比也是5:2,即前后两段路程比为5:2。‎ 步骤二、再来看小王和小张的前后两个追及问题:由于前后路程比为5:2,所以小王的行走时间比为5:2,也即是追及时间比为5:2,速度都不变,所以追及路程比为5:2, 而前段追及路程是小张行走60+45=105分钟的路程,所以后段追及路程是小张行走 ‎105÷5×2=42(分钟)‎ 所行走的路程,即小王比小张早42分钟到达。‎ ‎[点评] 本题是2006我爱数学少年夏令营的真题,事实上步骤1,2是刚好相反的分析过程,同学们可以体会其中关系。解这道题如果运用折线示意图,结合基本几何知识,整个行程过程和其中的数量关系即可一目了然,即:‎ 【例2】 ‎(☆☆☆)一辆车从甲地开往乙地,如果把车速提高20%,可以比原定时间提前1小时到达;如果原速行驶 ‎100千米后,再将车速提高30%,也比原定时间提前1小时到达,求甲、乙两地距离。‎ [审题要点] 题目给出的距离信息只有给出‎100千米这一条,我们应当找到驾车行驶‎100千米的总时间。‎ [详解过程] 分析:车速提高20%,那么前后两次的速度比为5︰6,所以两次所用的时间比为6︰5,所花的时间减少1小时,由此可求原计划所花时间为 ‎(小时),‎ 而汽车提速后从甲地到乙地只用5小时,‎ 而这辆车如果提速30%,提速前后的速度比为10︰13,那么这辆车行驶相同距离所花的时间为13︰10,那么如果能将所花时间缩短1小时,则提速后行驶的时间应该为:‎ ‎(小时),‎ 所以原速行驶‎100公里所花的时间为:‎ ‎(小时),‎ 即这辆车原来的速度为:‎ ‎(千米/小时)‎ 甲乙时间的距离为:‎ ‎(千米),‎ ‎ 点评:此题是利用比例解行程问题非常经典题型,事实上题目中给出的条件非常适合用比例法的应用,首先有前后的速度比例关系,其次有时间差。“比例+两者之一或两者和与差”的考题模式是非常常见的。它对应的解题模式是“比例转化+按比例分配(已知两者之一或两者和与差分别求两者)”,希望同学总结体会。‎ 【例1】 ‎(☆☆☆☆)A、B、C三辆汽车以相同的速度同时从甲市开往乙市,开车后1小时A车出了事故,B和C车照常前进。A车停车修理半小时后以原速度的继续前进,B、C两车行至距离甲市‎200千米处B车出了事故,C车照常前进。B车停了半小时后也以原速度的继续前进。结果到达乙市的时间C车比B车早1小时,B车比A车早1小时,求甲、乙两市的距离为多少千米?‎ [审题要点] 题目给出的距离信息只给出‎200千米这一条,所以我们应当求出‎200千米对应的路程比。‎ [详解过程] 如果A车没有停半小时,那么它将比C车晚到1.5小时,因A车后来的速度是C车的,所以A车和C车相同路程行程的时间比为5︰4,即C车每行驶4小时就比A车快1小时,所以C车快了1.5小时,说明C车后来行了 ‎1.5×4=6(小时)。‎ 则从甲市到乙市C车行了 ‎6+1=7(小时)。‎ 同样如果B车没有停半小时,它将比C车晚到0.5小时,而此时B车和C车在相同路段行程的时间比也是5:4,说明C车后来行了 ‎0.5÷(5-4)×4=2(小时),‎ 这段路是甲、乙两市距离的 ‎。‎ 所以B车出事故时,已经行驶了整个路程的。‎ ‎(千米)‎ 甲、乙两市之间的距离为‎280千米。‎ 点评:此题已知条件为速度比例和时间差,也符合“比例+两者之一或两者和与差”的出题模式,所以利用“比例转化+按比例分配(已知两者之一或两者和与差分别求两者)”的解题模式,注意此题中的时间差计算时要扣除停留的时间,注意加减关系。‎ 【例1】 ‎(☆☆☆☆)甲乙两人同时从A、B两地出发,甲每分钟行‎80米,乙每分钟行‎60米,两人在途中C点相遇。如果甲晚出发7分钟,两人将在途中D处相遇,且A、B中点E到C点的距离是到D点距离的2倍。求A、B两地间距离。‎ [审题要点] 甲晚出发7分钟,也就是乙早出发7分钟,先走了60×7=‎420米,两人共同行走的时间也减少了。对应的路程和也发生了变化。‎ [详解过程] 前后两次二人的相遇路程和相差‎420千米,那么前后两次相遇时间相差为 ‎420÷(80+60)=3(分钟),‎ 而本来这三分钟甲能多走 ‎80×3=240(米),‎ 这就说明C点与D点之间的距离为‎240米,由条件“A、B中点E到C点的距离是到D点距离的2倍”可以得到中点到C、D两点之间的距离。不过这里要分两种情况:‎ 情况(一)中点E在C、D之间,那么ED、EC的距离和为‎240米,EC的距离为:‎ ‎240÷(2+1)×2=‎‎160米 也就是说甲乙同时出发后的相遇点距离中点‎160米,即甲比乙多走了‎320米。两人相遇所花的时间为:‎ ‎320÷(80-60)=16(分)。‎ A、B之间的距离为:‎ ‎(80+60)×16=2240(米)。‎ 情况(二)C、D在E点的同一侧,那么ED、EC的距离差为‎240米,EC的距离为:‎ ‎ 240÷(2-1)×2=480(米)‎ 也就是说甲乙同时出发后的相遇点距离中点‎480米,即甲比乙多走了‎960米,两人相遇所花的时间为`:‎ ‎960÷(80-60)=48(分)。‎ A、B之间的距离为:‎ ‎(80+60)×48=6720(米)。‎ 综上所述,A、B两地之间距离为‎2240米或‎6720米。‎ ‎[点评] A、B中点E到C点的距离是到D点距离的2倍,只涉及到距离关系,没有提到位置关系,所以如果这些点在同一条直线的话,不只有一种位置关系,所以本题有两种答案。‎ 点评:总结此题的解题思路为:路程和减少→相遇时间减少→甲行驶时间减少→甲行驶路程减少→相遇点之差→AB路程差→AB相遇时间→AB相遇总路程。‎ 【例1】 ‎(☆☆☆☆)A、B两地间有一座桥,甲、乙两人分别从A、B两地同时出发,3小时后在桥上相遇。如果甲加快速度,每小时多行‎2千米,而乙提前0.5小时出发,则仍旧在桥上相遇。如果甲延迟0.5小时出发,乙每小时少走‎2千米,还会在桥上相遇,则、两地相距多少千米?‎ [审题要点] 三种方式相遇所行的路程都相等,典型的由时间比化速度比的题目,求出了速度再求总路程就简单了。‎ [详解过程] 因为每次相遇的地点都在桥上,所以在这三种情况中,甲每次走的路程都是一样的,同样乙每次走的路程也是一样的。‎ 在第二种情况中,乙速度不变,所以乙到桥上的时间还是3小时,他提前了0.5小时,那么甲到桥上的时间是 ‎3-0.5=2.5(小时),‎ 两次相遇时间比为3:2.5,路程一样,所以甲的速度成反比为2.5:3=5:6,又速度增加‎2千米每小时,所以甲原速为 ‎2÷(6-5)×5=10(千米/小时)。‎ 在第三种情况中,甲速度不变,所以甲到桥上的时间还是3小时,他延迟了0.5小时,那么乙到桥上的时间是 ‎3+0.5=3.5(小时),‎ 与第一种情况相比较,两种相遇时间比为3:3.5,路程一样,所以乙的速度成反比为3.5:3=7:6,又速度减少‎2千米每小时,所以乙原速为 ‎2÷(7-6)×7=14(千米/小时)。‎ 这样就可以求出A 、B两地的距离为 ‎(10+14)×3=72(千米)。‎ 【例2】 ‎(☆☆☆☆☆)甲、乙二人分别从A、B两地同时出发相向而行,5小时 后相遇在C点。如果甲速度不变,乙每小时多行‎4千米,且甲、乙还从A、B两地同时出发相向而行,则相遇点D距C点‎10千米;如果乙速度不变,甲每小时多行‎3千米,且甲、乙还从A、B两地同时出发相向而行,则相遇点E距C点‎5千米。问:甲原来的速度是每小时多少千米?‎ [审题要点] 三种相遇方式两人行程距离,行程时间都不相同,所以应该将其中一项化为相等。‎ [详解过程] 当乙每小时多行‎4千米时,5小时可以多行‎20千米,‎ 所以当两人相遇后继续向前走,5小时的时候甲可以走到C点,乙可以走过了C点‎20千米。‎ 而相遇点D距C点‎10千米,因此两人相遇后各走了‎10千米,所以甲乙二人速度相等,‎ 即原来甲比乙每小时多行‎4千米。‎ 同理,当甲每小时多行‎3千米,则5小时可以多行‎15千米,‎ 所以当两人相遇后继续向前走,5小时的时候乙可以走到C点,甲可以走过了C点‎15千米。‎ 而相遇点E距C点‎5千米,因此两人相遇后甲走了‎10千米,乙走了‎5千米。甲、乙两人的速度比为2:1。‎ 于是题目就化为一道简单的差倍问题。‎ ‎ (4+3)÷(2-1)+4=11(千米/小时),‎ 所以甲原来的速度是每小时11千米。‎ 点评:此题事实上利用了假设法:假设两人相遇后继续相前走,由于时间一样,利用两人前后的路程差与速度成正比得出两个速度关系,然后利用差倍问题或按比例分配得出最后答案。‎ 一、 拓展训练 1. 两个连队同时分别从一个营地出发前往一个目的地进行演习,A连有卡车可以装载正好一个连的人员,为了让两个连队的士兵同时到达目的地,A连士兵坐车出发一定时间后下车让卡车回去接B连的士兵,两营的士兵恰好同时到达目的地,已知营地与目的地之间的距离为‎32千米,士兵行军速度为‎8千米/小时,卡车行驶速度为‎40千米每小时,求两营士兵到达目的地一共要多少时间?‎ 2. 甲班与乙班学生同时从学校出发去公园,甲班步行的速度是每小时‎4千米,乙班步行的速度是每小时3千米。学校有一辆汽车,它的速度是每小时48千米,这辆汽车恰好能坐一个班的学生。为了使两班学生在最短时间内到达公园,那么甲班学生与乙班学生需要步行的距离之比是多少?‎ 3. 一段路程分为上坡、平路、下坡三段,各段路程的长度之比是1∶2∶3,某人走这三段路所用的时间之比是4∶5∶6。已知他上坡时每小时行‎2.5千米,路程全长为‎20千米。此人走完全程需多长时间?‎ 1. 甲火车4分行进的路程等于乙火车 5分行进的路程。乙火车上午8:00从B站开往A站,开出若干分后,甲火车从A站出发开往B站。上午9:00两列火车相遇,相遇的地点离A、B两站的距离的比是15∶16。甲火车从A站发车的时间是几点几分?‎ 2. 甲、乙两人同时分别从A、B两地出发相向而行,他俩相遇后经过5分钟,甲抵达B点,已知甲的速度是乙的速度的2倍,那么甲到达B后还要经过多长时间,乙才能到达A点?‎ 3. 甲骑车自A向B驶去,2小时后,乙步行由A向B走去,乙走出2小时后甲到B,此时乙距B地‎32千米;甲在B休息2小时30分钟又原路返回,经过1小时与一直步行向B走的乙相遇,问此时乙距B地多少千米?‎ 4. 李华每天上学步行5分钟以后,跑步2分钟恰好到校。有一天,他步行了2分钟就开始跑步,结果早到了1分40秒,他跑步的速度是步行速度的多少倍?‎ 5. 如图,在长为‎490米的环形跑道上,A、B两点之间的跑道长‎50米,甲、乙两人同时从A、B两点出发反向奔跑。两人相遇后,乙立刻转身与甲同向奔跑,同时甲把速度提高了25%,乙把速度提高了20%。结果当甲跑到点A时,乙恰好跑到了点B。如果以后甲、乙的速度和方向都不变,那么当甲追上乙时,从一开始算起,甲一共跑了多少米?‎ ‎ ‎ 6. 甲、乙两车分别从A、B两地同时出发相向而行,6小时后相遇在C点。如果乙车速度不变,甲车每小时多行‎5千米,且两车还从A、B两地同时出发相向而行,则相遇地点距C点‎12千米;如果甲车速度不变,乙车每小时多行‎5千米,且两车还从A、B两地同时出发相向而行,则相遇地点距C点‎16千米。乙车原来每小时行多少千米?‎ 7. 甲、乙二人步行远足旅游,甲出发后1小时,乙从同地同路同向出发,步行2小时到达甲于45分钟前曾到过的地方。此后乙每小时多行‎500米,经过3小时追上速度保持不变的甲。甲每小时行多少米?‎ 初级点拨:‎ 1. 确定行车路线并计算这条路线与行军距离比,要注意一个结论:两连步行速度相等,则步行的距离是相等的 1. 本题和上题的不同点在于两班步行速度不一样了。不妨设乙班学生先步行,汽车将甲班学生送至A地后返回,在B处接到乙班学生,最后汽车与乙班学生同时到达公园。‎ 2. 由各个路段的长度之比为1:2:3以及路程全长为‎20千米可得到上坡、平路、下坡的长度分别为:‎ ‎(千米)‎ ‎(千米)‎ ‎(千米)‎ 3. 甲、乙火车的行程比已知,所以甲乙火车相同时间内的行程比也已知。由此可以求得甲火车单独行驶的距离与总路程的比。‎ 4. 由甲、乙的速度比可以得到相遇前的路程比。‎ 5. 甲走完全程需要4小时,乙走5.5小时的路程与甲走1个小时的路程之和也等于一个全程,即甲走3小时的路程和乙走5.5小时的路程相等。‎ 6. 这一天李华跑步比平时多跑的路程等于这天李华步行比平时少走的路程。‎ 7. 相遇后乙的速度提高20%,跑回B点,即来回路程相同,乙速度变化前后的比为5︰6,所以所花时间的比为6:5。设甲在相遇时跑了6单位时间,则相遇后到跑回A点用了5单位时间。‎ 8. 无论是甲每小时多行‎5千米还是乙每小时多行‎5千米,由于速度和不变,所以相遇时间不变,所以通过甲或乙的两次速度差和路程差能求出行程时间。‎ 9. 根据题意,乙加速之前步行2小时的路程等于甲步行2.25小时的路程,由此可求得甲、乙原来的速度之比。‎ 深度提示:‎ 1. 由于卡车的速度为士兵行军速度的5倍,因此卡车折回时已走的路程是B连士兵遇到卡车时已走路程的(5+1)÷2=3倍。从而卡车折回所走的路程是B连士兵遇到卡车时已走路程的2倍。‎ 2. ‎︰=1︰12,︰=1︰16。乙班从C至B时,汽车路线为C-A-B,则两者路程之比为1︰16,不妨设CB=1,则C-A-B=16,CA=(1+16)÷2=8.5,则有CB︰BA=1︰7.5;‎ 3. 根据他上坡的速度可求出上坡所用的时间为:‎ ‎(小时)‎ 4. 从甲火车出发算起,到相遇时两车走的路程之比为5∶4=15∶12,而相遇点距A,B两站的距离的比是15∶16。‎ 5. 因为甲的速度是乙的速度的2倍,所以在相遇前,甲行走的路程是乙行走的路程的2倍,‎ 6. 甲走完全程需要4小时,乙走5.5小时的路程与甲走1个小时的路程之和也等于一个全程,即甲走3小时的路程和乙走5.5小时的路程相等。‎ ‎ 3÷5.5=,所以乙的速度是甲的速度的。‎ 7. 李华平时需要7分钟到校,早到1分40秒,即用了分钟到校,其中步行了2分钟,跑步了分钟。所以跑步2分钟,步行5分钟的距离与步行2分钟,跑步分钟的距离相等。‎ 1. 设甲原来单位时间行程V甲,由题意得:‎ ‎6V甲+5×V甲×(1+25%)=490,得:V甲=40。‎ 从A点到相遇点路程为40×6=240,所以 V乙=(490-50-240)÷6=。‎ 2. 甲每小时多行‎5千米比速度不变时要多行驶12+16=‎28千米。‎ ‎28÷5=5.6(小时),‎ 所以变速后的相遇时间是5.6小时。‎ 3. 根据题意,乙加速之前步行2小时的路程等于甲步行2.25小时的路程,所以甲、乙的速度之比为2︰2.25=8︰9,乙加速之后步行3小时的路程等于甲步行3.75小时的路程,所以加速后甲乙的速度比为3︰3.75=4︰5。‎ 全解过程:‎ 1. 由于卡车的速度为士兵行军速度的5倍,因此卡车折回时已走的路程是B连士兵遇到卡车时已走路程的(5+1)÷2=3倍。从而卡车折回所走的路程是B连士兵遇到卡车时已走路程的2倍。车接到B连士兵后,还要行走3倍B连士兵遇到卡车时已走路程才能追上A连士兵,此时他们已经到达了目的地,因此总路程相当于4倍B连士兵遇到卡车时已走路程,所以B连士兵遇到卡车时已走路程为‎8千米,而卡车的总行程为(3+2+3)×8=‎64千米,这一段路,卡车行驶了64÷40=8/5小时,这也是两营士兵到达目的地所花的时间。‎ 2. 为了确定汽车行进路线,可先假设乙班学生先步行。‎ ‎︰=1︰12,︰=1︰16。乙班从C至B时,汽车路线为C-A-B,则两者路程之比为1︰16,不妨设CB=1,则C-A-B=16,CA=(1+16)÷2=8.5,则有CB︰BA=1︰7.5;类似设AD=1,分析可得AD︰BA=1︰5.5,综合得CB︰BA︰AD=22︰165︰30,说明甲乙两班步行的距离之比是15︰11。若设甲班先步行,结果同上。‎ 3. 由各个路段的长度之比为1︰2︰3以及路程全长为‎20千米可得到上坡、平路、下坡的长度分别为:‎ ‎(千米)‎ ‎(千米)‎ ‎(千米)‎ 根据他上坡的速度可求出上坡所用的时间为:‎ ‎(小时)‎ 最后由此人在各路段所花时间之比得到各路段所花时间,‎ 平路上所花时间为:‎ ‎(小时)‎ 下坡路所花的时间为:‎ ‎(小时)‎ 所花的总时间为:‎ ‎(小时)‎ 1. 甲、乙火车的速度比已知,所以甲乙火车相同时间内的行程比也已知。由此可以求得甲火车单独行驶的距离与总路程的比。 从甲火车出发算起,到相遇时两车走的路程之比为5∶4=15∶12,而相遇点距A,B两站的距离的比是15∶16。‎ 说明甲出发前乙火车所走的路程等于乙火车1个小时所走路程的(16-12)÷16=1/4。也就是说乙比甲先走了一个小时的四分之一。所以甲火车从A站发车的时间是8点15分。‎ 2. 因为甲的速度是乙的速度的2倍,所以在相遇前,甲行走的路程是乙行走的路程的2倍,相遇前乙行走的路程,甲只用5分钟便走完了,所以,在相遇前二人都走了10分钟,相遇前甲走的这一段路让乙来走要用20分钟,所以甲到达B后还要经过20-5=15分钟,乙才能到达A点。‎ 3. 甲走完全程需要4小时,乙走5.5小时的路程与甲走1个小时的路程之和也等于一个全程。‎ 所以甲走3个小时的路程等于乙走5.5小时的路程。‎ ‎(4-1)÷5.5=,所以乙的速度是甲的速度的 甲每小时行全程的,所以乙每小时行全程的 ‎,‎ 乙两个小时行驶,距B地还有全程的。‎ 所以全程为 ‎(千米)‎ 甲的速度为(千米/小时)‎ 所以相遇时乙距离B地‎11千米。‎ 1. 李华平时需要7分钟到校,早到1分40秒,即用了分钟到校,其中步行了2分钟,跑步了分钟。所以跑步2分钟,步行5分钟的距离与步行2分钟,跑步分钟的距离相等。跑步分钟的距离与步行3分钟的距离相等。‎ ‎3÷=2.25,‎ 所以他跑步的速度是步行速度的2.25倍。‎ 2. 相遇后乙的速度提高20%,跑回B点,即来回路程相同,乙速度变化前后的比为5︰6,所以所花时间的比为6:5。设甲在相遇时跑了6单位时间,则相遇后到跑回A点用了5单位时间。设甲原来单位时间行程V甲,由题意得:‎ ‎6V甲+5×V甲×(1+25%)=490,得:V甲=40。‎ 从A点到相遇点路程为40×6=240,所以 V乙=(490-50-240)÷6=。‎ 两人速度变化后,甲的速度为40×(1+25%)=50,‎ 乙的速度为×(1+20%)=40,从相遇点开始,‎ 甲追上乙时,甲比乙多行一圈,所以甲一共跑了 ‎490÷(50-40)×50+240=2690(米)‎ 3. 甲每小时多行‎5千米比速度不变时要多行驶12+16=‎28千米。‎ ‎28÷5=5.6(小时),‎ 所以变速后的相遇时间是5.6小时,‎ 将甲车速不变,乙速加速的情况,与原来的相遇情况作比较。‎ 乙5.6小时应该比原来5.6小时多行 ‎5×5.6=28(千米),‎ 所以原来的相遇情况中,5.6小时乙应该行驶到离C点还有 ‎28-16=12(千米)‎ 这‎12千米应该在6-5.6=0.4小时内行完。‎ 所以乙的速度为 ‎12÷(6-5.6)=30(千米)‎ 1. 据题意,乙加速之前步行2小时的路程等于甲步行2.25小时的路程,所以甲、乙的速度之比为2︰2.25=8︰9,乙加速之后步行3小时的路程等于甲步行3.75小时的路程,所以加速后甲乙的速度比为3︰3.75=4︰5。‎ 所以甲、乙的速度之比为 ‎2︰2.25=8︰9,‎ 乙的速度是甲的速度的1.125倍,加速之后步行3小时的路程等于甲步行3.75小时的路程,所以加速后甲,乙的速度比为 ‎3︰3.75=4︰5,‎ 加速后乙的速度是甲的速度的1.25倍,‎ 所以甲的速度为 ‎500÷(1.25-1.125)=4000(米/时),‎ 所以甲每小时行‎4000米/时。‎ 第16讲 分数与百分数 一、 知识地图 基本类型:‎ 1. 求一个数是另一个数的几分之几(或百分之几);‎ 2. 已知单位“‎1”‎,求它的几分之几(或百分之几);‎ 3. 已知单位“‎1”‎的几分之几(或百分之几),求单位“‎1”‎。‎ 基本题型 1. 画图找对应;‎ 2. 量不变思想解题;‎ 3. 转化“1”;‎ 4. 和工程问题相结合;‎ 5. 结合还原问题的分数应用题;‎ 6. 与行程问题相结合的分数应用题;‎ 7. 与经济有关的分数应用题;‎ 8. 一般复杂应用题。‎ 二、 基础知识 为了提高解答分数应用题的能力,首先要掌握好有关的基础知识,深刻理解分数、分数乘除法的意义。‎ 我们知道分数既有量的意义,又有率的意义。把三个苹果分给四个人每人分得个苹果,也可以说是每人分得一个苹果的。前者在分数前写上了一个具体的单位“个”,表示一个具体的量值。我们就说这是分数量的意义。后者在分数前面加上一个表示单位“1”的量“一个苹果”,表示的是一个数是另一个数的几分之几的倍比关系,我们就称之为率的意义。在解答分数应用题时,要注意找出与具体数量相对应的分率(或百分率),根据“量”与“率”相对应的关系,找到解题线索。这是解答分数应用题的一种重要的解题思路。‎ ‎(一)基本关系:‎ 1. 分率对应的量单位“‎1”‎的量=分率;‎ 2. 单位“‎1”‎的量分率=分率对应的量;‎ 3. 分率对应的量分率=单位“1”的量。‎ ‎(二)解答分数应用题要注意以下几点:‎ ‎1.仔细读题,找出含有倍数关系的句子,确定单位“‎1”‎。‎ ‎2.如果单位“‎1”‎已知,那么找问题中的数量与单位“‎1”‎的对应关系;如果单位“‎1”‎为所求,就找已知数量与单位“‎1”‎的对应关系;如果是“求一个数是另一个数的几分之几”的应用题,就从问题入手,弄清谁和谁比。‎ ‎3.列式时,分率和数量要对应。如果题中有多个单位“‎1”‎,应想法转化为统一的的单位“‎1”‎若是较复杂的分数应用题可通过画线段图来揭示数量之间的相对应的倍数关系。‎ ‎(三)对于分数应用题,最主要的除了注意量率对应的思想方法外,还有两点是小学奥数,尤其是小升初考试中的热点,即:‎ 1. 以不变量为“1”;‎ 2. 转化“1”。‎ 另外,结合份数比的方法来解题,是百分数应用题的常用手法。‎ 三、经典透析 ‎ 【例1】(☆☆‎ ‎)大学图书室内有一书架故事书,借出总数的75%之后,又放上60本,这时架上的书是原来总数的。求现在书架上放着多少本书?‎ 审题要点:借出总数的75%之后,还剩下25%,又放上60本,这时架上的书是原来总数的,这就可以找出60本书相当于故事书总数的几分之几了,问题也就可以求出来了。还可以画线段图找量率对应。‎ 详解过程:(1)60本书相当于故事书总数的几分之几?‎ ‎ -(1-75%)=‎ ‎(2)故事书的总数:‎ ‎60÷=720(本)‎ ‎(3)现在书架上放有故事书多少本?‎ ‎720×=240(本)‎ 答:现在书架上放有故事书240本。‎ 专家点评:本题主要考察量率对应,可以画线段图来帮助解题。‎ 注意:本题中的量率对应还可以根据图用别的方法求。从图中可以看出:故事书的与75%的重叠之出就是60本所对应的分率。这个分率可以用下面的三种方法求出:‎ ‎(1)+75%-1;‎ ‎(2)-(1-75%);‎ ‎(3)75%-(1-);‎ 请你自己想想每种方法的道理。‎ ‎【例2】(☆☆☆)一瓶可乐饮料,一次喝掉一半饮料后,连瓶共重‎700克;如果喝掉饮料的后,连瓶共重‎800克,求瓶子的重量。‎ 审题要点:如下可用文字等式来表示题中的两个已知条件:‎ ‎ 瓶重+饮料重的()=‎‎700克 ‎ 瓶重+饮料重的()=‎‎800克 比较上面两个等式,可以看出‎800克比‎700克多的‎100克就是饮料的()比()多的。找到了量与率的对应,就可以求出饮料重,从而可以求出瓶重。‎ 详解过程:饮料重:(800-700)÷[()-()]=600(克)‎ 瓶重:700-600×=400(克)‎ 答:瓶子的重量为‎400克。‎ 专家点评:可以利用文字等式来观察量与率的对应。:题目中瓶重未知,700和800中都包含未知瓶重,此种情况一般可考虑使用“差额法”来找到其中的对应关系,因为两者都包含相同部分的东西,那么它们的差额是固定的,这是不变量的一个重要来源之一,同学们应该要引起重视。‎ 同理:当两者中一者多一部分数量,另一者少同样多一部分的数量时,那么可以利用“求和法”,确定它们的和是不变的。‎ ‎【例3】(☆☆☆)在希望学校学生阅览室里,女生占全室人数的,后来又进来两名女生,这时女生占全教室人数的。问阅览室里原来有多少人?‎ 审题要点:虽然和都是以全室人数为单位“1”,但后来的全室人数变了,所以这两个分数的单位“1”对应的数量不相同,不能直接进行加减运算。我们要找一个不变的量做单位“1”,把这两个分数进行适当的转化,才能正确地找出分率与数量的对应关系。注意到阅览室里男生人数没变,所以我们就以男生人数为标准。原来女生占全教室人数的,男生占,女生是男生的÷=,现在女生占全教室人数的,男生占,女生是男生的÷=,现在女生比原来多占男生的-=,这个就是2人的对应分率,男生人数可以求了,全室人数也可以求了。‎ 详解过程:2÷[]÷()=36(人)‎ 答:阅览室里原来有36人。‎ 专家点评:在做此题时一定要注意全教室的人数先后发生了变化,两个分率的单位“1”的大小不相等,不能直接相减,要先转化成单位“1”相同的两个分率。一般选择象本题中的不变量“男生人数”为新的单位“1”。此题时将以整体的单位“1”转化为以部分为单位“1”。‎ 选定不变量男生作为“‎1”‎之后,应该将女生的人数转化为男生“1”的分数。转化时也可以利用份数来考虑。原来全班为9份,女生为4份,男生为5份,所以女生占男生的。后来全班19份,女生9份,男生10份,女生是男生的。‎ 当多个分率同时出现的时候,要把它们转化为同单位“1”再进行比较。‎ ‎【例4】(☆☆☆)做一项工程,甲每天的工作效率等于乙、丙二人每天的工作效率的和;丙的工作效率相当于甲、乙每天工作效率和的;如果三人合作只需8天就完成了,那么乙一人单独做需要多少天才能完成?‎ 审题要点:将工程作为单位1,因为三人合作需要8天可以完成工作,因此每天的总工作效率为,根据题意还可以算出甲和丙的工效。‎ 详解过程:因此甲每天的工作效率为,乙、丙二人每天的工作效率之和也为;同时,丙的工作效率相当于甲、乙每天工作效率的,即丙的工作效率为总工作效率的,因此丙的工作效率为,由此可以得出乙的工作效率为,乙单独做要1÷=24(天)‎ 答:乙一人单独做要24天才能完成。‎ 专家点评:本题为分数百分数在工程问题中的应用,一般设工作总量为单位“1”。‎ ‎【例5】(☆☆☆)A、B、C三个桶内都有水,如果把A桶内的水倒入B桶,再把B桶内的水倒入C桶,最后再把C桶内的水倒入A桶,这时各桶内的水都是‎12升,求每个桶内原有水多少升?‎ 审题要点:该题直接计算不好下手,可以采用逆向思维,利用倒推法来解题,最后桶的水都是‎12升,往回推,假设C不倒给A,可以算出这时C和A桶内水的体积,然后再假设B不倒给C,可以算出这时B和C内水的体积,再假设A不倒给B,可以算出这时A和B水的体积。‎ 详解过程:‎ 1. C不倒给A,这时C有水:12÷(1-)=14(升),A有水:12-14×=10(升)‎ 2. B不倒给C,这时B有水:12÷(1-)=16(升),C有水:14-16×=10(升)‎ 3. A不倒给B,这时A有水:10÷(1-)=15(升),B有水:16-15×=11(升)‎ 专家点评:“倒推法“可以使解题过程简化,有时与列表法结合更加一目了然。利用倒推法时,注意分数的单位“‎1”‎是原来的水,所以这里应该用分数除法而不是分数乘法,对应的分率也应该是(1-)而不是(1+)。‎ ‎【例6】(☆☆☆)三种动物赛跑,已知狐狸的速度是兔子的70%,兔子的速度是松鼠的2倍,一分钟内松鼠比狐狸少跑‎16米,那么半分钟内兔子比狐狸多跑多少米?‎ 审题要点:狐狸与松鼠的速度都可与兔子的速度联系起来,因此在该题中把兔子的速度作为单位1,则狐狸的速度为,松鼠的速度为,松鼠在一分钟之内比狐狸少跑‎16米,根据量率对应的关系,可以算出兔子的速度,也可以算出半分钟内兔子比狐狸多跑的距离。‎ 详解过程:兔子的速度为:16÷(-)=80(米/分)‎ 半分钟内兔子比狐狸多跑:80××(1-)=12(米)。‎ 专家点评:本题为分数百分数在行程问题中的应用,关键时找准单位“1”,然后利用量率对应等关系。单位“‎1”‎的选择一般有两个原则:1)不变量为单位“‎1”‎;2)直接联系多的部分为单位“‎1”‎,此题中兔子的速度和其他两个都有直接联系,所以应该为“‎1”‎,这样其他两个分数也好表示。‎ ‎【例7】(☆☆☆)《中华人民共和国个人所得税法》第14条规定中附有下表:‎ 个人所得税税率表 (工资、薪金所得适用)‎ 级别 全月应纳税所得额 税率(%)‎ ‎ 1‎ 不超过500元部分 ‎5‎ ‎2‎ 超过500元至2000元部分 ‎10‎ ‎3‎ 超过2000元至5000元部分 ‎15‎ ‎4‎ 超过5000元至20000元部分 ‎20‎ ‎5‎ 超过20000至40000元部分 ‎25‎ ‎……‎ 目前,上表中“全月应纳税所得额”是从月工资、薪金收入中减去1600元后的余额,它与相应税率的乘积就是应缴的税款数。‎ (1) 李教授今年5月份的工资、薪金收入为3660元,这个月他应缴纳的税款是多少?‎ (2) 顾先生7月份缴纳了1265元个人所得税,这个月顾先生工资、薪金收入是多少元?‎ 审题要点:首先根据不同的收入将纳税收入分成不同档:‎ 第一档(收入500元):纳税500×5%=25(元)‎ 第二档(收入2000元):纳税(2000-500)×10%+25=175(元)‎ 第三档(收入5000元):纳税(5000-2000)×15%+175=625(元)‎ 第四档(收入20000元):纳税(20000-5000)×20%+625=3625(元)‎ 很明显,李教授的工资、薪金收入为3660元,减去1600元后为2060元,很明显是属于第二档和第三档之间的,顾先生缴纳了1265元的个人所得税,那么他是属于第三档和第四档之间的。‎ 详解过程:(1):李教授应缴纳的税款为:(3660-1600-2000)×15%+(2000-500)×10%+500×5%=184(元)‎ 答:这个月李教授应纳税184元。‎ ‎(2):顾先生是属于第三档和第四档之间的,超过1600元的收入应该在5000-20000之间,5000以内那部分应缴纳税款为:(5000-2000)×15%+175=625(元)‎ 所以5000以上部分应缴纳税款:1265-625=600(元)‎ 顾先生5000以上部分收入为:600÷20%=3000(元)‎ 顾先生的收入为:5000+3000+1600=9600(元)‎ 答:顾先生工资、薪金收入是9600元。‎ 专家点评:本题为百分数在经济问题中的应用。‎ ‎【例8】(☆☆☆☆)早上水缸注满了水,白天用去了其中的20%,傍晚又用去‎27升,晚上用去剩下水的10%,最后剩下的水是半水缸多‎1升。问早上注入多少升水?‎ 审题要点:本题涉及到各个量之间的关系比较复杂,我们可以列文字等式来将其联系起来。‎ 白天和傍晚用去水后剩下:1-20%=80%少27(升)‎ 晚上用去水:8%少2.7=(80%少27)×10%=8%少27×10%=8%少2.7(升)‎ 白天、傍晚、晚上总共用去水:20%+8%+(27-2.7)=50%少 1(升)‎ 剩下的过程可以列方程,也可以用算术法。‎ 详解过程:法1:设总共x升。 50%x-1=(20%+8%)x+27-2.7 x=115 法2:50%-(20%+8%)是(27-2.7)+‎1升。 早上水缸的水是 ‎ ‎(27-2.7+1)÷(50%-20%-8%)=115(升)。 答:早上注入水缸中的水是‎115升。‎ 专家点评:本题的关键是在统一单位“1”后,将各个量通过一个等量关系联系起来。‎ ‎【例9】(☆☆☆☆)有三堆棋子,每堆棋子数一样多,并且都只有黑白两色棋子。第一堆的黑子与第二堆的白子一样多,第三堆里的黑子占全部黑子的,把这三堆棋子集中在一起,白子占全部棋子的几分之几?‎ 审题要点:通过“第三堆里的黑子占全部黑子的”这个条件,可知前两堆的黑子数占全部黑子数的。前两堆的黑子数有多少虽然我们也不知道,但可以根据 “第一堆的黑子与第二堆的白子一样多”这个条件,我们可以换个角度思考这个问题。将第一堆里的黑子与第二堆里的白子交换,这时就可以得到第一堆全是白子,第二堆全是黑子,即第二堆的黑子占全部黑子的。又因为每堆棋子数一样多,所以其他两堆棋子数也是全部黑子的。把全部黑子看作单位“1”,所有三堆棋子数是全部黑子的×3=倍。现在再来看白子与黑子的关系,第一堆全是白子,所以是全部黑子的,第三堆中黑子占全部黑子的,所以第三堆中的白子占全部黑子的-=,所以全部白子就占全部黑子的+=。现在分别知道了所有棋子、全部白子与全部黑子的关系,就可以求了。‎ 详解过程:[(1-)+(1--)]÷[(1-)×3]= ‎ 答:白子占全部棋子的。‎ ‎ 专家点评:此题是通过将条件进行转化,变换思维的角度,求出每堆棋子与全部黑子的倍数关系。‎ 四、拓展训练 ‎1、六年级参加作文、数学比赛。参加作文比赛的占参赛人数的,参加数学比赛的占参赛人数的,两项比赛都参加的有12人,这个学校参加比赛的有多少人?‎ 初级点拨:首先要考虑量率对应的关系,可以画线段图来帮助理解,找出与量对应的率。‎ 深度提示:我们设参加比赛的总人数为单位“1”,由图可以看出,比单位“1”多出的那部分(1--)就是两项比赛都参加的人数。‎ 全解过程:这个学校参加比赛的人数为:(人)‎ 答:这个学校参加比赛的有105人。‎ ‎2、学校展示学生绘画,低、中年级科技作品共有120件,中、高年级作品共有168件,又知道低年级作品占高年级作品的,高年级作品有多少件?‎ 初级点拨:首先找准单位“1”,然后再确定可以对应起来的“量”和“率”。‎ 深度提示:可以设高年级为单位“1”,我们还是画线段图来帮助理解,明显可以看出,(168-120)是高年级比低年级多的量,而(1-)是高年级比低年级多的率,这样就找出了对应的“量”和“率”。‎ ‎120‎ ‎168‎ 全解过程:高年级作品有:(168-120)÷(1-)=84(件)‎ 答:高年级作品有84件。‎ ‎3、一瓶酒精,当用去酒精的45%后,连瓶共重‎800克,当用去酒精的55%后,连瓶共重700千克,酒精重多少克?‎ 初级点拨:这不是一道浓度问题,用分数百分数的量率对应方法来考虑,不妨画个线段图试试看。‎ 深度提示:很明显,是以酒精为单位“1”,由线段图可以看出,两次使用酒精后所剩的酒精重量之差这个“量”:(800-700),所对应的“率”为(55%-45%),这样就找出了相对应的量和率。‎ 全解过程:(800-700)÷(55%-45%)=1000(克)‎ 答:酒精重‎1000克。‎ ‎4、甲、乙、丙、丁四人共植树60棵。甲植树的棵数是其余三人的,乙植树的棵数是其余三人的,丙植树的棵数是其余三人的,丁植树多少棵?‎ 初级点拨:本题是工程问题,我们可以利用解决工程问题的一般方法来解决。先找出不变量,工程问题一般设工作总量为单位“1”。‎ 深度提示:设工程总量为单位“1”‎ 甲植的棵数占总工程的:‎ 乙植的棵数占总工程的:‎ 丙植的棵数占总工程的:‎ 这样就可以分别甲、乙、丙植的棵数 全解过程:甲植的棵数为:(棵)‎ 乙植的棵数为:(棵)‎ 丙植的棵数为:(棵)‎ 丁植的棵数为:60-20-15-12=13(棵)‎ 答:丁植树13棵。‎ ‎5、一件工程,甲、乙合做需4小时完成,乙、丙合做需5小时完成,现在先由甲、丙合做2小时后,余下的乙还需6小时完成,乙单独做这件工程需多少小时完成。‎ 初级点拨:根据题意可将条件做适当的转化,在单位“1”不变的情况下,找出与“量”相对应的“率”。‎ 深度提示:将这件工程看作单位1,根据题意可将条件做转化为:甲、乙合做2小时,乙、丙合做2小时后,余下的乙还需(‎6-2-2‎=)2小时完成。甲、乙合做2小时,乙、丙合做2小时后剩余的工程为1-×2-×2=‎ 全解过程:剩余的工程乙需要两个小时完成,所以乙单独完成这件工件需要 ‎1÷(÷2)=20(小时)‎ 答:乙单独做这件工程需要20小时完成。‎ ‎6、滨海市少先队员在城乡学校“手拉手”的活动中,为山区学校捐献了一批图书。按计划把这批书的又6本送给青山小学的;把余下的一部分送给少年宫,送给少年宫的比送给青山小学的3倍还多136本;又把第二次余下的75%又80本送给春苗幼儿园;最后还余下300本,作为山区小学数学竞赛的奖品。问滨海市少先队员一共捐献了多少本图书?(选自北京市第8届迎春杯小学数学竞赛决赛试题)‎ 初级点拨:这是一个量率对应问题,可以尝试运用倒推的思路来解题 深度提示:将总共捐献的图书作为单位1,则给青山小学的图书为总图书的多6本,送给少年宫的图书是送给青山小学的3倍还多136本,即总图书的多18+136本,题意表明,第二次余下的图书的25%为300+80本,因此第二次余下的图书为(300+80)÷25%=1520(本)‎ 全解过程: ‎ 根据题意,送给青山小学和少年宫的图书是总图书的多160,因此总图书为(1520+160) ÷(1-)=2800(本)‎ 答:滨海市少先队员一共捐献了2800本图书。‎ ‎7、超市商店运来264台冰箱,在两天时间里全部卖完,已知第一天卖出的与第二天卖出的相等,求这两天分别卖出多少台冰箱?‎ 初级点拨:这该题需要进行分率转化,得出第一天与第二天的倍数关系才能解题,经常利用比例关系。‎ 深度提示:因为第一天卖出的与第二天卖出的相等,所以第一天×=第二天×,第一天︰第二天=8︰3。‎ 全解过程:根据总冰箱数以及第一天和第二天卖出冰箱的关系,得出每天卖出冰箱的数:‎ 第一天卖出:264×=192(台)‎ 第一天卖出:264×=72(台)‎ 答:第一天卖出192台,第二天卖出72台。‎ ‎8、服装甲的成本是定价的 80%,服装乙的定价是 275元,成本是220元。现在商店把1件服装甲,与2件服装乙配套出售,并且按它们的定价之和的90%作价出售。这样每套可获得利润80元。问:服装甲的成本是多少元?‎ 初级点拨:根据每套服装的利润,可以先得出两件服装乙的利润 深度提示:2件服装乙可获得利润275×2× 90%-220×2=55(元)。因此,1件服装甲获利润80-55=25(元)。‎ 全解过程:将服装甲的成本看作单位1, 服装甲成本是定价的80%,定价是成本的 125%。‎ ‎   因此服装甲的成本为:25÷(125%×90%-1)=200(元)。‎ ‎9、 某校五四年级原有两个班,现在要重新编为三个班,将原一班的与原二班的组成新一班,将原一班的与原二班的组成新二班,余下的30人组成新三班。如果新一班的人数比新二班的人数多10%,问原一班有多少人?‎ 初级点拨:将原来两班总人数看作单位1,求出原来两个班的总人数,再根据新班级的人数比来进行计算 深度提示:原来两班人数总和是:(人)‎ 新一班与新二班人数总和是:-30(人)‎ 新一班人数是:新一班与新二班人数总和(人)‎ 另外,根据(新一班人数)-(新二班人数)=[(原一班人数)-(原二班人数)]× 可求出:(原一班人数)-(原二班人数),那么原一班人数也不难求得。‎ 全解过程:原来两班人数总和是:(人)‎ 新一班与新二班人数总和是:72-30=42(人)‎ 新一班人数是:(人)‎ 新二班人数是:42×=20(人)‎ ‎(新一班人数)-(新二班人数)=[(原一班人数)-(原二班人数)]×=22-20=2(人)‎ ‎(原一班人数)-(原二班人数)=2÷=24(人)‎ 得出:原一班人数=(72+24)÷2=48(人)‎ 答:原一班人数为48人。‎ ‎10、甲、乙、丙、丁四人去买游戏机。甲带的钱是另外三人所带总钱数的一半,乙带的钱是另外三人所带总钱数的,丙所带的钱是另外三人所带总钱数的,丁带910元,四人所带的总钱数是多少元?‎ 初级点拨:大家可以这样来思考,根据甲乙丙三人所带钱数占另外三人的分数得出他们所带钱数占总钱数的分数,从而得出丁所带钱数占总钱数的分数来进行求解。‎ 深度提示:因为甲带的钱是另外三人所带总钱数的一半,设甲带的钱为一份,则其它三人带的钱为两份,因此,甲带的钱占总钱数的;依次类推,乙带的钱占总钱数的;丙带的钱占总钱数的。‎ 由此可以得出丁带的钱占总钱数的1---‎ 由于丁带的钱数为910元,根据对应关系,得出四人带的总钱数 全解过程:四人带的总钱数=910÷(1---)=4200(元)‎ 答:四人带的总钱数为4200元。‎ 第17讲 工程问题 一、知识地图 工程问题的基本数量关系 工程问题分类及解法分析 ‎ 简单的工程问题 ‎ 工程与行程的问题 ‎ 复杂工程问题 ‎ 两人工程问题 ‎ 交替工作问题 ‎ 二、基础知识 在日常生活中,做某件事,制造某种产品,完成某项任务或工程等等,都要涉及到工作总量、工作效率、工作时间这三个量之间的关系。在小学数学中,研究这三个数量之间关系的应用题,我们都叫做“工程问题”。‎ ‎(一)工程问题的基本数量关系 ‎1) ‎ ‎=工作效率 甲工效+乙工效=甲乙合作工效之和 一件工程-已完成的部分=未完成的部分 上面这些数量关系式在题目中给出(或间接给出)工作总量和工作效率的具体数量情况下,进行解题用的。‎ ‎ 2)“‎1”‎的引入 ‎ 如果题目中没有给出工作总量具体的数量,也没有给出工作效率的具体数量,那么我们通常把工作总量看做单位“‎1”‎,工作效率用单位时间内能完成总工作量的几分之一或几分之几来表示。‎ ‎ 我们把工程问题中的工作总量用“‎1”‎表示,工作效率用分率表示,这种方法不妨称为“工程习惯”。‎ ‎ (二)工程问题分类及解法分析 1、 简单的工程问题:利用基本数量关系求解,一定要把分数的意义和工程问题紧密结合起来,这样才能明白在没有准确数据的情况下,工作效率的含义。 ‎ 2、 工程与行程的问题:在解答这类问题时,通常题目中没有直接给出 ‎ ‎ 路程、速度和时间,需要你把它转化成工作总量、工作效率和工作时间来思考。‎ 注意:1)将路程看作“1”‎ ‎ 2)‎ ‎3、 复杂工程问题:这类问题中有的问题具有特殊性与周期性问题有关,有的与实际问题有关,如水管问题。‎ ‎ 水管问题的图表法解答(具体见例1)‎ ‎ 1)如果题目中涉及多个人,例如,甲、乙、丙三人;‎ ‎ 2)题目中可求的工效仅仅只是其中几个人的合工效,如,甲乙合工效,乙丙合工效,甲丙合工效。‎ ‎ 3)这一类题目可以利用图表法 例:‎ 甲 乙 丙 工作效率 ‎√‎ ‎√‎ ‎√‎ ‎√‎ ‎√‎ ‎√‎ 合计 步骤:1)列表 (如图)‎ ‎ 2)在甲、乙、丙……对应的下行内画上“√”。‎ 例如:第一行表示甲、乙合干需三天。‎ ‎3)“合计”中,计算甲“√”个数,乙“√”个数……以及工作效率的和。‎ ‎4)甲、乙、丙“√”个数均为x个,工作效率和为A,则甲、乙、丙……合作工效为 。‎ ‎4、两人工程问题 ‎(1)题目中出现的是两人之间的工作问题;‎ ‎(2)利用经典三步法。‎ ‎ ①设两个人工作效率为x与y,列二元一次方程。‎ ‎ ②求x=?,y=?;‎ ‎ ③代入求解。‎ ‎5、交替工作问题(详见例题)‎ ‎(1)分组,求一组内的合效率,设为A ‎(2)工作总量设为1,的结果取整。‎ ‎(3)剩下的工作具体分配。‎ ‎ ‎ 三、经典透析 ‎ ‎【例1】(☆☆☆)一项工程,甲、乙合作9天完成,甲、丙合作12天完成,乙、丙合作18天完成,甲、乙、丙合作需要几天完成?‎ 经典提示:‎ ‎ 对照基础知识中的介绍的图表法。‎ ‎ 审题要点:‎ ① 工程问题第一步确定三个基本量 ② 题目中只有合作效率 ,很显然利用列表法。‎ ‎ 详解过程:‎ 甲 乙 丙 工作效率 ‎√‎ ‎√‎ ‎√‎ ‎√‎ ‎√‎ ‎√‎ ‎2‎ ‎2‎ ‎2‎ 所以甲、乙、丙的效率和=÷2=,所以三人合作需要工作1÷=8(天)‎ 专家点评:‎ ‎ 这道题目不是很难,关键是对工程问题的图表法的灵活运用,你学会了么?‎ ‎【例2】(☆☆☆☆)一件工作,甲单独作12小时完成,现在甲、乙、丙合作2小时后甲因事外出,剩的工作乙又用了5小时做完,如果这项工作由乙单独做需要几小时?‎ ‎(友情提示)‎ ‎ 两人工程问题 ① 设甲乙的功效为x与y,根据条件列方程 ② 求y=?x ‎ ③ 代入求解 审题要点:‎ ‎①两个人做工程(知道思路了吧 ?)‎ ‎ 详解过程:‎ 设甲乙功效为x,y。‎ ‎ 得 所以乙独做需9小时。‎ 专家点评:这是一道典型的“两人”工程问题,很多题目其实并不是因为难而不会,而是因为该记的公式没有记住。‎ ‎【例3】(☆☆☆) 一项工程,甲独做需12天完成,乙独做需15天完成。要想在10天之内完成,两人至多合作几天,至少合作几天?‎ ‎ 审题要点:‎ ‎1°本题涉及到了最多和最少问题,有一点复杂。实际上仔细想想10天内最多合作几天完成,这个最多不就是从一开始就合作的天数才能最多吗?‎ ‎2°1÷(+)=6(天)<10天。这一问题能不能理解成“两队合作几天才能完成任务”的另一种表达方式呢?‎ ‎3°至少合作几天完成:因为每人单独做的天数都超过10天,所以不可能不合作完成。但显然甲的工效高,这10天甲共能完成×10==,还差1-=的工作量。这些只能让乙来完成,乙需要÷=1(天)。这样可以知道甲单独做10天后,乙再做1天就能完成任务,即可以认为两人合作1天后,甲在独做(10-1)天就能完成任务,这样最符合题目要求。 ‎ 详解过程:‎ ‎1÷(+)=6(天)‎ ‎(1-×10)÷=1(天)‎ 答:要想在10天之内完成,两人至多合作6天,至少合作1天。‎ ‎ 专家点评:‎ 这道题目相对比较难理解,所以做题时,一定要先把题目分析清楚,题目要求 最少的合作时间,因此工效高的人肯定要做比较多的工作,如果能将这些信息 分析好,题目就变得非常简单了。 ‎ ‎【例4】(☆☆☆)加工一批零件,甲、乙合作24天可以完成,现在由甲先做16天,然后乙再做12天,还剩下这批零件的没有完成。已知甲每天比乙多做3个零件。求这批零件共有多少个?‎ 审题要点:‎ ‎1°两人工程问题 ‎ ‎2°经典三步法 ‎ 详解过程:‎ 设甲、乙工作效率分别为x与y,‎ 代入其中一个方程 ‎ ‎ ‎=360个 答:这批零件有360个。‎ ‎ 专家点评:‎ 这道题目很简单,只要把基础知识部分的“两人工程问题”看明白就可以了。‎ ‎【例5】(☆☆☆☆)蓄水池有一条进水管和一条出水管。要灌满一池水,单开进水管需要5小时,排光一池水,单开排水管需三小时。现在池内有半池水,如按进水,排水的顺序,轮流各开一小时,问多少时间后水池的水排完?(精确到分)‎ 审题要点:‎ 1) 看题目,排水工效为,进水工效为。‎ 2) 类似行程中的追击问题。‎ 3) 我们不妨就以行程问题来求解。‎ 详解过程:‎ 因为最后要排完水,所以排列顺序为:进排,进排,进排,进排。‎ 1) 先进1小时,此时,池内的水为 2) 此后的顺序为排,进,排,进,排…… ‎ 即排进各5小时,此时池中水为-5×(-)=‎ 下面开始排,所需时间为 总时间为 专家点评:‎ 交替进水问题,以及交替工作问题,是工程里面的难点,希望同学们能在牢记公式的前提下,灵活应用公式。‎ ‎【例6】(☆☆☆) 小王和小李从甲、乙两地同时相向而行,已知走完全程小王和小李分别需要40分钟和60分钟。出发后5分钟小王发现忘带东西回去取,已知取东西要耽误5分钟,求出发到相遇共需多长时间?‎ 审题要点:‎ 1) 这是一道典型的类工程的行程问题。‎ 2) 可以设出小王与小李的速度为 详解过程:‎ 小王第二次出发与第一次出发相隔十五分钟,因为是相向而行,利用相遇问题解答。‎ ‎ ‎ ‎18+15=33(分)‎ ‎ 答:从出发到相遇共需要33分钟。‎ ‎ 专家点评:‎ 类似于工程问题的行程问题,你会发现除了速度和路程的表示方法改变了,其它的公式和分析方式都没有改变。‎ ‎【例7】(☆☆)小敏周末去少年宫上课,她7点5分出发,当时针与分针第一次重合时她到达少年宫,求路上用了多长时间?‎ ‎ 审题要点:‎ 1) 时钟问题 2) 时钟速度 ‎ 详解过程: ‎ ‎7点时,分针在时针后面35格,利用类行程问题 ‎ ‎ ‎ ‎ 设:7点t分时两针第一次重合,35=(1-)×t,t=(分),则小敏路上用了-5=(分)。‎ ‎ 专家点评:时钟问题里面,一定要记住分针和时针的速度表示方法。这类题目解题时要注意联系路程问题。‎ ‎【例8】(☆☆☆)单独完成一项工作,甲按规定时间可提前2天完成,乙则要超过规定时间3天才能完成,如果甲、乙二人合作两天后,剩下的由乙独做,那么刚好在规定的时间完成,问甲、乙合作需要多少天完成?‎ 审题要点:‎ 1. 这道题目中出现了规定时间,你注意到了吗?‎ 2. 甲、乙的工作问题都与它有关。‎ 3. 怎么办呢?动动脑筋。‎ 详解过程:‎ 1. 设规定时间为t天,则甲(t-2)天完成,乙(t+3)天完成。‎ 2. 甲、乙合作2天,剩下乙独作,正好t天完成,在这个过程中,乙做了t天,甲 ‎ ‎ 干了2天。即乙三天的工作甲2天完成。‎ 所以,整理得2t+6=3t-6,得t=12(天)‎ 甲的工效为 乙的工效为 甲、乙合作需要的时间:1÷(+)=6(天)‎ ‎ 专家点评:‎ 有些同学,可能读了一遍题,就很草率的认定是简单的“两人工程问题”,可是列式子解答时,发现问题并非想象的那么简单。所以,做题的时候不要盲目,把题目分析透了再动手! ‎ ‎【例9】(☆☆☆)某人沿公路匀速行走,他发现公路上的汽车每隔20分就有一辆超过他,每隔12分就有一辆和他相遇。已知公共汽车发车时间间隔相同。运行的速度也相同,问公共汽车每隔多少分发一辆?‎ 审题要点:‎ 发车间隔问题,主要的数量关系等同于相遇关系或者追及关系,关键在于把前后两车间隔的距离在车和人相遇的过程中看作“路程和”,在车超过人的过程中看作“路程差”‎ 所以,把前后两车间隔的距离看作“1”,则有:‎ ‎1=(V车-V人)×20 即V车-V人=‎ ‎1=(V车+V人)×12 即V车+V人=‎ 另外,本题应用了和差关系,请注意。‎ 详解过程:‎ 设车的速度和人的速度分别为V车, V人 ‎ V车+ V人=‎ ‎ V车- V人=‎ ‎ V车=(+)=‎ ‎ 1÷=15(分)‎ 答:公共汽车每隔15分发一辆车。‎ ‎ 专家点评:简单的“发车”问题,只要公式记牢,做题很简单。‎ 四、拓展练习 ‎1、放满一池的水,若同时开1、2、3号阀门,则20分钟可注满,若同时开2、3、4号阀门,则21分可注满,若同时开1、3、4号阀门,则28分钟可以注满;若同时开1、2、4号阀门,30分钟可以完成。‎ 问:(1)如果同时开1、2、3、4号阀门,那么多少分钟可以完成?‎ ‎(2 ) 单开3号阀门多少分钟可以完成? ‎ ‎2、一 项工程第一组单独做需12小时,第二组单独做需18小时,若第一组先做1小时,然后第二组接着做1小时,再由第一组做1小时,…,两组如此交替工作,问完成任务时共用几小时?‎ ‎3、一件工作甲先做6小时,乙接着做12小时可以完成。若甲先做8小时,乙接着做6小时也可以完成,如果甲先做3小时后再由乙接着做,还需要多少小时完成?‎ ‎4、一项工程,甲乙两队合作6天能完成,已知单独做甲完成与乙完成所需时间相等,问单独做甲乙各需多少天?‎ ‎5、甲乙两人植树,单独植完这批树,甲比乙所需时间多,如果两人一起干,完成任务时乙比甲多植36棵,这批树一共多少棵?‎ ‎6、蓄水池有甲,乙,丙三个进水管,甲,乙,丙三个进水管单独灌满一池水依次需要10,12,15小时,上午8点三个管同时开,中间甲管因故关闭,结果到下午2点水池才被放满,问甲管何时被关闭?‎ ‎7、2个师傅和4个徒弟一天可做完一批零件的,8个师傅和10个徒弟一天就能完成任务,如果这批零件全由徒弟一天完成,需要多少个徒弟?‎ ‎8、一个装满了水的水池有一个进水阀及三个口径相同的排水阀,如果同时打开进水阀及一个排水阀,则30分钟能把一池水排空,如果同时打开进水阀和两个排水阀,则10分钟能把水池的水排空,问关闭进水阀并且同时打开三个排水阀,需要几分钟能排空水池的水? ‎ ‎9. 甲、乙、丙三人承包一项工程,发给他们的工资共1800元。三人完成这项工程任务的具体情况是:甲、‎ 乙两人合作6天完成了工程的;因甲有事,由乙、丙合作2天,完成了余下工程的;以后三人合作5天完成了这项工程。按完成工作量的多少来付酬,每人应得多少元?‎ ‎10. 在甲、乙两地之间的公路上,自行车运动员往返骑车,竞走运动员练习竞走。他们同时从甲地出发,竞走运动员走完全程要3小时,自行车运动员骑完全程比竞走运动员少2.5小时。当竞走运动员从甲地走到乙地时,自行车运动员与竞走运动员几次相遇?(包括迎面相遇和从后面追上两种情况)‎ ‎11. 甲、乙两辆汽车从东、西两城相对开出,已知甲车行完全程用10小时,乙车行完全程用15小时,当两车相遇时甲车比乙车多行12千米,问:东西两城相距多少千米?‎ 拓展训练分析与解答 ‎(一)初级点拨:‎ ‎ 1、分析题目出现多个阀门,但每次只开其中3个,什么问题?‎ ‎ 2、交替工程问题。‎ ‎ 3、1)很显然的两人工程问题;‎ ‎ 2)“经典三步”法。‎ ‎ 4、1)“两人”工程问题;‎ ‎ 2)“经典三步”法。‎ ‎ 5、1)分数的转变问题 ‎ ‎2)时间与功效成反比 ‎ 6、1)虽然是3个水管,但是并不像你想得那么难 ‎ 2)中间甲管关闭,但是乙丙照常,发现了么?‎ ‎7、题目读起来,似乎很难,细心的同学会发现,所有的问题都是围绕师傅和徒弟展开的,发现了么?‎ ‎8、进水和排水,肯定存在差值,这道题目毫无疑问是工程问题,怎么能变成我们熟悉的形式呢?‎ ‎9、本题的关键是求出甲乙丙各自的工作效率。‎ ‎10、如果按第一次是在何处追上的,第二次是在何处迎面相遇的……这样考虑实在是太麻烦了。可以从整体上把握,来解决问题。‎ ‎11、将普通的行程问题转化为工程问题来解。关键是如何表示两辆车的速度和相遇时间。‎ ‎ (二)深度提示:‎ ‎1、4个阀门,每次开3个,显然是我们介绍的图表法,你想到了么?‎ ‎ 2、交替工作的问题,我们在基础知识部分介绍了解题方法,但是有一点要注意,即关于“取整”的问题。‎ ‎ 3、“经典三步法”是解决“两人”问题的关键,所以这个方法一定要学会哦。‎ ‎ 4、同样是“两人”工程问题 ‎ 5、时间与工效成反比,甲比乙所需时间多,即甲的时间是乙的倍。‎ ‎ 6、中间甲管关闭,但是乙、丙照常,因此可以将乙、丙合并。‎ ‎7、可以将师傅看作“1人”,徒弟看作“1人”,明显的两人问题。‎ ‎8、进水阀与排水阀各看作“1个人”:“两人问题”‎ ‎9、由甲、乙两人合作6天完成了工程的,可以求出甲、乙两人的工作效率之和。同理可求出乙、丙工作效率之和以及甲、乙、丙三人的工作效率之和,然后可分别求出甲、乙、丙各自的工作效率,再由他们各自的工作天数求出工作量,最后求得他们应得的工资。‎ ‎10、根据题意可知自行车运动员的速度是竞走运动员的6倍,也就是说,当竞走运动员从甲地走到乙地时,自行车运动员已骑了6个全程。除第一个全程外,每骑一个全程必定和竞走运动员遇到一次,这样这个问题不就是解决了吗?‎ ‎11、根据题意可知相遇时间为:(时),那么甲车相遇时行了全程的,乙车行了全程的。甲车多行的12千米正好对应全程的,这样就能求出全程。‎ ‎ (三)全解过程:‎ ‎ 1、列表:‎ ‎1‎ ‎2‎ ‎3‎ ‎4‎ 工作效率 ‎√‎ ‎√‎ ‎√‎ ‎√‎ ‎√‎ ‎√‎ ‎√‎ ‎√‎ ‎√‎ ‎√‎ ‎√‎ ‎√‎ ‎3‎ ‎3‎ ‎3‎ ‎3‎ ‎+++=‎ ‎1,2,3,4号阀门的工效和为÷3=。‎ (1) ‎ 同时开,需18分钟注满。‎ (2) 单开“3”,找表中“3”对应的竖列中的空。‎ ‎“3”号阀门的工效=,‎ 所以单开3号阀门45分钟注满。‎ ‎ 2、‎ 第一组工效为,第二组工效为 取整,所以第一组与第二组先各做7小时,共完成,余下的由第一组完成。‎ 所以总时间为。‎ ‎3、设甲、乙的工作效率为x与y ‎ ‎ 1) ‎ ‎ ‎ ‎ 2) 代入求解 ‎ ‎ ‎4、设甲、乙的工作效率为x与y ‎ ‎ 1) 2) 代入求解 ‎ ‎ 所以甲、乙独做分别需18天,12天。‎ ‎ 5、设甲、乙的工作效率为x与y ‎ ‎ 因为同时合作,所以甲、乙植树的总量比也是3:4,即可以将整个数量分成7份,那么甲植了其中3份的树,而乙植了4份的树。‎ 乙比甲多1份,而又知乙比甲多植36棵 所以总共的棵数(棵)‎ ‎ 6、甲、乙、丙功效为 ‎ 上午8点到下午2点共6个小时 ‎ 甲工作的时间:小时 ‎ 所以甲管9点被关闭。‎ ‎ 7、设师傅与徒弟的工作效率为x与y,‎ ‎ ‎ ‎ ‎ ‎ ‎ 代入求解 所以如果让徒弟第一天完成,需要的人数为:‎ ‎ 8、设进水阀和排水阀的效率分别为x和y ‎ ‎ ‎ ‎ ‎ ‎ 代入求解 ‎ ‎ ‎ ‎ ‎ ‎ ‎ ‎ 答:单开3个排水阀5分钟能排完水池的水。‎ ‎ 9、甲、乙工效和: ; 乙、丙工效和:‎ ‎ 甲、乙、丙三人工效和:[1--(1-)×]÷5=‎ ‎ 甲工效:-= ; 丙工效:-=‎ 乙工效:-=;‎ 甲完成的工作量:×(6+5)= ;‎ 乙完成的工作量:×(6+5+2)=‎ 丙完成的工作量:×(2+5)= ‎ 甲得到的工资:1800×=330(元);乙得到的工资:1800×=910(元)‎ 丙得到的工资:1800-330-910=560(元)‎ 答:甲、乙、丙各得到330元、910元和560元。‎ ‎ 10、1÷3=‎ ‎1÷(3-2.5)=2‎ ‎2÷=6‎ ‎6-1=5(次)‎ 答:自行车运动员与竞走运动员5次相遇。‎ ‎ 11、方法(1):1÷(+)=6 (时)‎ ‎12÷()=60(千米)‎ 方法(2)甲车1小时比乙车1小时多行全程的-=‎ 相遇时间为1÷(+)=6 (时)‎ 所以全程为12÷6÷=60(千米)‎ 答:东西两城相距‎60千米 ‎ 第18讲 浓度与经济问题 一、 知识地图 1) 浓度问题 a) 浓度,溶液配比问题(溶质不变)‎ b) 类浓度问题(溶剂不变)‎ c) 两种不同浓度溶液混合 2) 经济问题 a) 经济问题的典型利用 ‎ i. 求利润 ii. 求成本 iii. 求售价 b) 经济问题的实际应用 ‎ i. 利息问题 ii. 税收问题 iii. 保险问题 iv. 折扣问题 v. 水电费峰谷价问题 二、 基础知识 ‎(一)浓度问题 浓度问题是一种常见的百分数应用题。在日常生活中,“水甜不甜?”等这些问题都是有关浓度的问题。糖水甜的程度是由糖与水两者量的比值所决定的。若水的量一定,则含糖量越多,糖水就越甜。这里的糖就是溶质,水就是溶剂,糖和水和在一起就是溶液,我们把糖与糖水的重量的比值称为糖水的浓度。这三者的关系如下:‎ 溶液的重量=溶质的重量+溶剂的重量 浓度=‎ 溶质重量=溶液重量×浓度 溶液重量=溶质重量÷浓度 溶剂质量=溶液质量-溶质质量=溶液质量×(1-浓度)‎ 浓度问题考察的比较多的是溶液的配比,解题时注意要抓住不变量,常用的一些解题方法有:‎ 1. 计算法:一般为溶质不变,浓度不变等,进行计算;‎ 2. 列方程;抓住不变量,找出等量关系,列方程计算;‎ 3. 十字交叉:适合两种不同浓度的溶液配比问题。‎ (二) 经济问题 经济问题也是一种常见的百分数应用题。商店出售商品,总是期望获得利润。一般情况下,商品从厂家购进的价格称为成本(也叫进价),商家在定价的基础上提高价格出售,就会获得收入,收入比成本高的那部分就是利润,利润与收入之比称之为利润率,利润与成本之比为利润的百分数。利润率通常用百分数来表示。‎ 利润=售价-成本 ‎     售价=成本×(1+利润率)‎ 商店有时为了尽快将商品出售,将商品打折出售来增加销量。打八折就是原价的80%。‎ 售价=原价×折扣 对于利息问题,是人们将钱存入银行,也就是本金,要按照国家规定的利率获得利息。‎ 本金:储蓄的金额。‎ 利率:利息和本金的比。‎ 利息=本金×利率×期数 其它经济问题只要掌握数量关系,如:上交税收=应纳税收入×税率;保险费=保险金额×保险费率×保险期限。方法正确,计算认真准确就行了。‎ 一、 经典透析 ‎【例1】(☆☆☆)现有浓度为16%的糖水‎40千克,要得到含盐20%的糖水,可采用什么方法?‎ 审题要点:原来糖水的浓度为16%,现在要将浓度变为20%,是将浓度变大,通常首先会想到往溶液中再加一下溶质。其实,反过来可用“蒸发”的方法减少水的质量来达到目的。若用加糖的方法,水的质量不变;若用蒸发的方法,糖的质量不变。‎ 详解过程:方法一:采用加糖法,水的质量保持不变 原糖水中含水:40×(1-16%)=33.6(克)也就是现在糖水中也是含水‎33.6克,‎ 现在水的浓度就是(1-20%),‎ 现在糖水的质量为:33.6÷(1-20%)=42(克)‎ 糖水增加的质量就是要加的糖的质量,所以要加糖:42-40=2(克)‎ 方法二:采用蒸发法,糖的质量保持不变 原糖水中含糖:40×16%=6.4(克),即为现在糖水中糖的质量 现在糖水中含糖20%,可求出现在糖水的质量:6.4÷20%=32(克)‎ 所以蒸发水:40-32=8(克)‎ 答:可以加糖‎2克,或者蒸发‎8克水。‎ 专家点评:本题为典型的溶液混合题,只要抓住不变量,将混合前后各个量之间的关系联系起来。有时候利用不同的不变量,会有不同的解法。‎ 注意:大家可以来思考一下如果想降低浓度是否也有两种方法呢?‎ ‎【例2】(☆☆☆)甲种酒精纯酒精含量为72%,乙种酒精纯酒精含量为58%,混合后纯酒精含量为62%,如果每种酒精取的数量比原来多‎15升,混合后纯酒精含量为63.25%,问第一次混合时,甲、乙两种酒精各取了多少升?‎ 审题要点:这道题,我们可以把他看成一道分数百分数问题,首先选取单位“1”,但是注意,两次混合就要选取两次单位“1”,要对应联系起来,我们每次都选取乙为单位“1”‎ 详解过程:第一次混合时甲种酒精用量是乙种酒精用量的分率为:(62%-58%)÷(72%-62%)=;‎ 第二次混合时甲种酒精用量是乙种酒精用量的分率(63.25%-58%)÷(72%-63.25%)= ‎ 根据量率对应的关系:‎ 乙可取15÷[3÷(5-3)–2÷(5-2)]÷(1-)=30(升)‎ 甲可取30×=12(升)。‎ 我们在这里再向大家传授一种求浓度混合配比的妙招,可以采用“十字交叉相减”法,这个方法和杠杆原理很类似,大家想一下,两种浓度不同的溶液混合在一起,混合后的浓度一定在混合前两种溶液的浓度之间,比大的小,比小的大,并且接近质量多的溶液。就好比大家小时候玩的“跷跷板”‎ ‎,如果跷跷板平衡的话,支点一定在两个人中间,并且支点与体重大的那个小朋友接近,我们就以这道题为例,具体解体方法如下:‎ 混合前甲,乙溶液浓度: 甲 乙 交叉相减求差: 62%-58%=4% 72%-62%=10% ‎ 差的比值: 4% : 10%‎ 甲,乙溶液质量的比值: 2 : 5 ‎ 第二次配比也是相同的方法 混合前甲,乙溶液浓度: 甲 乙 ‎ ‎ 交叉相减求差: 63.25%-58%=5.25% 72%-63.25%=8.75% ‎ 差的比值: 5.25% : 8.75%‎ 甲、乙溶液质量的比值: 3 : 5‎ 这样我们可以轻松的得到配比前两种溶液质量的比值,而不用经过烦琐的计算,既节约了时间,又减少了计算错误。剩下的步骤就跟上题一样了。‎ 专家点评:溶液的配比问题可以抓住不变量,溶质不变,利用方程来解决,也可以利用“十字交叉”法来解决。‎ 注意:除了两种溶液配比外,稀释和加溶质也可以用“十字交叉相减”法,如果溶液加水,那么溶液就和0%的溶液来配比,如果单加溶质,就是溶液和100%的溶液来配比。‎ ‎【例3】(☆☆☆)有甲、乙、丙三个容器,容量为1000毫升,甲容器的浓度为40%的糖水400毫升;乙容器有清水400毫升,丙容器中有浓度为20%的糖水400毫升,先把甲,丙两容器中的糖水各一半倒入乙容器搅匀后,再把乙容器中的糖水200毫升倒入甲容器,200毫升倒入丙容器,这时候甲、乙、丙容器中糖水的浓度各是多少?‎ 审题要点:对于涉及到多个变量反复操作的问题,我们采用列表处理的方法 详解过程:列表如下 甲 浓度 溶液 开始 ‎40%‎ ‎400‎ 第一次 ‎40%‎ ‎400-200=200‎ 第二次 ‎200+200=400‎ 乙 浓度 溶液 开始 ‎0‎ ‎400‎ 第一次 ‎400+200+200=800‎ 第二次 ‎15%‎ ‎800-200-200=400‎ 丙 浓度 溶液 开始 ‎20%‎ ‎400‎ 第一次 ‎40%‎ ‎400-200=200‎ 第二次 ‎200+200=400‎ 答:这时甲容器中糖水的浓度是27.5%,乙容器中糖水的浓度是15%,丙容器中糖水的浓度是17.5%。‎ 专家点评:在做有关浓度的应用题时,为了搞清楚溶质质量,溶液质量的变化,尤其是多次变化的,常用列表的方法,使它们之间的关系一目了然。‎ ‎【例4】(☆☆☆)A、B、C三个试管中各盛有‎10g、‎20g、‎30g水,把某种浓度的糖水‎10g倒入A中,混合后取出‎10g倒入B中,再混合后又从B中取出‎10g倒入C中,现在C中糖水的浓度是0.5%,最早倒入A中的糖水浓度是多少?‎ 审题要点:可用倒推法的思想,先求出现在C管中糖的质量,又因为C中原来只有‎30g 水,所以它的糖是从B管取出的‎10g糖水中来的,由此可求出此前B管中糖的质量。而B 管中的糖又是从A管中取出的‎10g糖水中来的,由此可求出A管里‎20g糖水中糖的质量,即所求糖水中的糖。‎ 详解过程:方法1:现在C管中的糖的质量为(30+10)×0.5%=40×0.5%=0.2(g)‎ 则此前B管中糖的质量为:0.2÷10×(20+10)=0.6(g)‎ 则此前A管中糖的质量为0.6÷10×(10+10)=1.2(g),‎ 最早到入A中的糖水浓度是12%。‎ 方法2:设最早倒入A中糖水浓度为x,则A中糖水的浓度为10x÷(10+10)=0.5x,‎ B中糖水的浓度为=x C中糖水的浓度为=x 所以 x=0.5%‎ x=12%。‎ 答:最早倒入A中糖水的浓度为12%。‎ 专家点评:要想在解题过程中分清变化前后谁变了,谁没变,紧紧抓住不变量是突破口。用倒推法能让思路清晰,避免纠缠于中间的变化过程。‎ ‎【例5】(☆☆☆)某商店到水果产地去收购橘子,收购价为每千克1.20元。从产地到商店的距离是‎400千米,运费为每吨货物每运‎1千米收1.50元。如果在运输及销售过程中的损耗是10%,商店要想实现25%的利润率,零售价应是每千克多少元?‎ 审题要点:本题是一道基本的利润问题,其成本包括收购价、运费、损耗三部分。‎ 详解过程:每千克的收购价为1.20元,每千克的运费是1.50×400÷1000=0.60(元)。‎ 因为有10%的损耗,所以每千克的成本为(1.20+0.60)÷(1-10%)=2.00(元)。‎ 商店想每千克实现25%的利润率。‎ 因为:售价=成本×(1+利润率)‎ 所以:售出价=成本×(利润率+1)‎ ‎     =2.00×(25%+1)‎ ‎     =2.50(元),‎ ‎  即零售价应是每千克2.50元。‎ 专家点评:本题的关键是搞清楚成本、利润、售价、利润率这几个量的概念以及它们之间的关系。‎ ‎【例6】(☆☆☆)某商场在迎奥运展销期间,将一批电视机降价出售。如果打九折出售,可盈利215元;如果打八折出售,亏损125元。此电视机的购入价是多少元?‎ 审题要点:第二种方法比第一种多降了定价的20%-10%=10%,而导致第二种方法比第一种少卖了215+125=340元。说明定价的10%就是340元。可以求出定价,也可以求出成本。‎ 详解过程:电视机的定价为:‎ ‎(215+125)÷(20%-10%)=3400(元)‎ 那么该电视机的购入价为:‎ ‎3400×(1-10%)-215=2845(元)‎ 答:此电视机的购入价是2845元。‎ 专家点评:本题为折扣问题,是百分数的典型应用。注意折扣的单位“1”和利润率的单位“1”不同,折扣的单位“1”为原价(定价),利润率的单位“1”为成本,注意区分和转化。‎ ‎【例7】(☆☆☆☆)甲、乙二人原有钱数相同,存入银行,第一年的利息为4%,存入一年后利息降至2%,甲将本钱和利息继续存入银行,而乙将一半本钱投资股市及房地产,获利20%,一年后,甲比乙赚到的钱的一半还少144元,则甲原来有多少元?‎ 审题要点:本题为利息问题,本金×(1+利息×期数)=本息 详解过程:设甲和乙原有钱数都是x。‎ 甲在银行存了两年,第一年利息为4%,钱变成了x(1+4%),接着再存了一年,第二年利息是2%,本息和为x(1+4%)(1+2%),两年赚的钱为:x(1+4%)(1+2%)-x=0.0608x;‎ 乙先将所有的钱在银行存了一年,本息和为x(1+4%),第二年将一半本息接着存入银行,一半本钱投入股市,存入银行的一年后本息和为x(1+4%)(1+2%),投入股市的钱一年后收入为x(1+20%),乙两年赚的钱为:‎ x(1+4%)+x(1+4%)(1+2%)+x(1+20%)-x=0.1504x。‎ 已知甲赚的比乙的一半还少144元,于是得到(144+0.0608x)×2=0.1504 x,‎ 容易解得x=10000元。‎ 答:甲原来有10000元。‎ 专家点评:本题考察的是利息问题和利润问题的综合求解。此题中一般同学在计算本息和时喜欢写成x+x×4%,这种写法不好,最好写成x(1+4%),这样后面的也可以直接写为x(1+4%)(1+2%)了,若写成第一种情况化简起来就会很麻烦。在计算所有增加或者减少分率时都应该这样处理,一般公式为单位“‎1”‎×(1±增加或减少分率)‎ ‎【例8】(☆☆☆)国家规定个人发表文章,出版图书获得稿费的计算方法是:①稿费不高于800元的不纳税;②稿费高于800元又不高于4000元的应缴纳超过800元的那一部分的14%的税 ;③稿费高于4000元的应缴纳全部稿费的11%的税。今得知丁老师获得一笔稿费,并且依法缴纳个人所得税420元,问丁 老师这笔稿费是多少元?又得知马老师获得一笔稿费,并且依法缴纳个人所得税550元,问马老师这笔稿费是多少元?‎ 审题要点:先估计这笔稿费大致有多少元?属于哪个档次?再进行计算 详解过程:第一档的不纳税,第二档的要纳税(4000-800)×14%=448(元)‎ 说明丁老师稿费低于4000元。‎ 丁老师的稿费为:420÷14%+800=3800(元)。‎ 马老师的所得税高于440元,应该用第三档的来计算。‎ 马老师的稿费为:550÷11%=5000(元)。‎ 答:丁老师的稿费为3800元,马老师的稿费为5000元。‎ 专家点评:先估算看属于哪一档的,再进行计算。‎ 注意:这种问题还有类似的,除了个人所得税问题,还有水电费问题等。‎ ‎【例9】(☆☆☆☆)小刚家去年参加了家庭财产保险,保险金额是20000元,每年的保险费是保险金额的0.3%。其家中被盗,丢失了一台彩色电视机和一辆自行车,保险公司赔偿了2940元。已知电视机的价格正好是自行车价格的7倍。如果要购买与原价相同的电视机和自行车,那么加上已交的保险费,小刚家需比原来多花费400元。电视机和自行车原价各多少元?‎ 审题要点:保险问题,计算方法类似于利润问题,但要注意保险费是属于成本。保险费=保险金额×保险费率 详解过程:小刚家的保险金额是20000元,保险费是保险金额的0.3%,那么要交纳的保险费就是20000×0.3%=60(元)。‎ 由于家中被盗,保险公司赔偿了2940元,相当于从保险公司那里得到:2940-60=2880(元)。‎ 而自行车和电视机的价格是:2880+400=3280(元),电视机的价格是自行车的7倍,根据和倍的原理,可以得到自行车的原价是:3280÷(7+1)=410(元)。‎ 电视机的原价是:410×7=2870(元)。‎ 专家点评:保险问题其实和利润问题与利息问题实质相同。‎ 四、拓展训练 ‎1.甲容器有纯酒精‎11升,乙容器有水‎15升。第一次将甲容器中一部分纯酒精倒入乙容器,使酒精和水混合。第二次将乙容器中的一部分混合液倒入甲容器中,这样甲容器中的纯酒精含量为62.5%,乙容器中纯酒精含量是25%。那么,第二次从乙容器倒入甲容器的混合液是多少升? ‎ 初级点拨:本题涉及到两次混合,只要抓住混合后,谁变了谁没变,紧紧抓住不变量。‎ 深度提示:由题意可知,第一次混合后,乙容器中的溶剂没有变,而第二次混合,就是将甲容器里的纯酒精,由100%的浓度稀释到62.5%,而稀释液就是第一次混合后的乙溶液。‎ 全解过程:第一次甲容器倒入一部分酒精到乙容器后,乙容器中的纯酒精含量就是25%。这样第一次从甲容器倒入乙容器的纯酒精是:15÷(1-25%)-15=5(升);‎ 甲容器中还剩下‎6升,乙容器中有‎20升含量为25%的酒精混合液。可以列方程来解,设第二次从乙容器中倒入甲容器x升。‎ ‎(6+x×25%)=62.5% ×(6+x)‎ x=6。‎ 也可以用“十字交叉相减”法来做:‎ 混合前甲、乙溶液浓度 甲 乙 交叉相减求差: 62.5%-25%=37.5% 100%-62.5%=37.5% ‎ 差的比值 37.5% : 37.5%‎ 甲、乙溶液质量的比值 1 : 1 ‎ 答:第二次从乙容器倒入甲容器的混合液是‎6升。‎ ‎2.有甲、乙两个同样的杯子,甲杯中有半杯清水,乙杯中盛满了含50%酒精的液体。先将乙杯的一半倒入甲杯,搅匀后,再将甲杯中酒精溶液的一半倒入乙杯。问这时乙杯中酒精溶液的浓度是多少?‎ 初级点拨:多次混合,用列表法可以使思路比较清晰。‎ 深度提示:紧紧抓住浓度的变化,搞清楚溶质质量、溶液质量的变化,将整个过程联系起来。‎ 全解过程:,所以要搞清楚溶质质量、溶液质量的变化,列表使思路比较清晰。‎ 甲 乙 溶质 溶液 溶质 溶液 开始 ‎0‎ ‎1×50%=‎ ‎1‎ 第一次 ‎×=‎ ‎1‎ ‎×=‎ 第二次 ‎1‎ 所以这时乙杯中的浓度为:‎ ‎。‎ 答:这是乙杯中酒精溶液的浓度是37.5%。‎ ‎3.将‎25g白糖放入空杯中,倒入‎100g白开水充分搅拌后,喝去一半糖水,又加入‎36g白开水,如果要使杯中的糖水和原来一样甜,需要加入多少白糖?‎ 初级点拨:抓住主线,从简单入手,切忌复杂化。‎ 深度提示:要想杯中糖水一样甜,那就说明浓度相同,也就是说明糖和水的比例相同,可以利用浓度相同这个等量关系来列方程,也可以用比例相同这个等量关系来求解。‎ 全解过程:法1)设需要加入xg白糖,则,解得x=9。‎ 法2)设需要加入糖x克,则,解得x=9。抓住主线,只看结果,明显比法一要简单的多。‎ 答:需要‎9克白糖。‎ ‎4。A、B、C三瓶糖水的浓度分别为20%,18%,16%,它们混合后得到‎100g浓度为18.8%的糖水,如果B瓶糖水比C瓶糖水多‎30g,那么A瓶糖水有多少克?‎ 初级点拨:三种溶液混合在一起,混合前溶质的质量和还是等于混合后溶质的质量和。‎ 深度提示:三瓶糖水的浓度都是已知的,并且知道B瓶比C瓶多‎3‎‎0克,可以假设C瓶为x克,那么B瓶为(x+30)克,A瓶糖水为:100-(x+x+30)=70-2x克,利用混合前后溶质相等这个等量关系来解题。‎ 全解过程:设C瓶糖水有x克,则B瓶糖水为x+‎30克,A瓶糖水为100-(x+x+30)=70-2x,‎ ‎(70-2x)×20%+(x+30)×18%+x×16%=100×18.8%,整理得0.06x=0.6,解得x=10,所以A瓶糖水为:70-2×10=50(g)‎ 答:A瓶糖水有‎50克。‎ ‎5.某容器中装有糖水。老师让小强再倒入5%的糖水‎800克,以配成20%的糖水。但小强却错误地倒入了‎800克水,老师发现后说,不要紧,你再将第三种糖水‎400克倒入容器,就可得到20%的糖水了。那么第三种糖水的浓度是百分之几?‎ 初级点拨:抓住主线,多次混合,抓住不变量,找好等量关系,那就是最后浓度不变。‎ 深度提示:老师让小强往容器中倒入5%的糖水‎800克配成20%的糖水,这‎800克糖水中应该含糖800×5%=‎40克,而小强倒入容器里的却是水,没有溶质,这样就少了‎40克糖,而多了‎40克水,这样将第三种糖水倒入容器的时候就应该多倒‎40克糖,少倒‎40克水。‎ 全解过程:第一次少倒糖800×5%=40(克)‎ 第二次为了补上第一次少倒的糖,应该倒入糖400×20%+40=120(克)‎ 所以,第二次倒入糖水浓度为120÷400=30%。‎ 答:第三种糖水的浓度是30%。‎ ‎6.某家商店决定将一批橘子的价格降到原价的70%卖出,这样所得利润就只有原计划的,已知这批橘子的进价是每千克6元6角,原计划可获利润2700元,那么这批橘子共有多少千克?‎ 初级点拨:本题关键是搞清楚利润、成本和售价之间的关系,以及各个量之间的关系。‎ 深度提示:原定价格的(1-70%),也就是少卖的那部分,就相当于少得的那部分利润,也就是原计划利润的(1-),然后找出原定价格、利润、成本之间的关系就可以了。‎ 全解过程:所以原计划的利润相当于原定价格的(1-70%)÷(1-)=45%,所以这批橘子的进价相当于原定价格的1-45%=55%,那么原定的价格就是每千克6.6÷55%=12元,原计划利润就是每千克12-6.6=5.4元,这样就可以求出这批橘子共有2700÷5.4=‎500千克。‎ ‎7.某银行定期储蓄的种类和利率如下图所示,假如你有1000元钱,想存4年,那么你最多可以得到多少元利息。(注意:存款到期后取出后可连本带息一起再存,不考虑利息税。)‎ 存期(年)‎ 年利率 到期利率 ‎1‎ ‎0.1098‎ ‎0.1098‎ ‎2‎ ‎0.1170‎ ‎0.2340‎ ‎3‎ ‎0.1224‎ ‎0.3672‎ 初级点拨:本题考察的是利息问题,只要搞清楚本金,利率和利息之间的关系即可。‎ 深度提示:根据乘法交换律可知,存期排列顺序与利息无关。故只有4种存法:①四次一年期②两次一年期③一次一年期,一次三年期④两次两年期,然后计算看哪种存法所得利息最多。‎ 注意到(1+0.1098)×(1+0.1098)≈1.2317<1+0.2340,即连续存两次一年期的收益小于存一次两年期的收益,而存法1和存法2都有连续存两次1年期,因此1,2两种存法不如4合算。‎ ‎ 全解过程:存法3能获利1.1098×1.3672-1=0.517319,‎ 存法4能获利1.234×1.234-1=0.522756。‎ 故存法4获得最大利,将得到利息1000×0.522756=522.76元。‎ 注意:上面两题关于利息的题还没计算利息税,根据国家法律规定,利息必须缴纳5%的利息率,由银行代扣,实际只能从银行取得95%的利息。考虑纳税的话计算更加复杂些。‎ ‎8.商品甲的成本是定价的80%;商品乙的定价是275元,成本是220元。现在商店把1件商品甲,与2件商品乙配套出售,并且按它们的定价之和的90%作价出售。这样每套可获得利润80元。商品甲的成本是多少元?‎ 初级点拨:最终的售价是打折后的价格。‎ 深度提示:甲的成本是定价的80%,那定价就是成本的1÷80%=125%,1件商品甲与2件商品乙配套捆绑出售,并且都以90%出售,每套所获得的80元利润里包括一件甲的利润和2件乙的利润,然后根据利润关系就可求得甲的成本。‎ 全解过程:甲的利润率为1÷80%×90%-1=12.5%;‎ 每件商品乙获得的利润为:275×90%-220=27.5(元);‎ 每件商品甲获得的利润为:80-27.5×2=25(元);‎ 商品甲的成本为:25÷12.5%=200(元)。‎ 答:商品甲的成本为200元。‎ ‎9.《中华人民共和国个人所得税法》规定,公民全月工资、薪金所得不超过1600元的部分不必纳税,超过1600元的部分为全月应纳税所得额,此项税款按下表分段累进计算。‎ 全月应纳税所得额 税率 不超过500元部分 ‎5%‎ 超过500元至2000元部分 ‎10%‎ 超过2000元至5000元部分 ‎15%‎ ‎……‎ ‎……‎ 某人一月份应交纳税款150元,则他的当月工资薪金所得为多少元?‎ 初级点拨:本题为典型的税收问题,要根据题意确定计算方法和过程。‎ 深度提示:先看所交税款属于哪一挡?第一档最多交:500×5%=25元,第二档最多交(2000-500)×10%+25=175元;第三档……此人所交税款明显属于第二档,可以按第二档税率进行计算。‎ 全解过程:因为25<150<175,所以此人工资属于第二档,500-2000元之内部分应该交税款:150-500×5%=125(元),500元以上工资数额为:125÷10%=1250(元),所以工资总数为:1600+500+1250=3350(元)。‎ 答:他当月工资薪金为3350元。‎ ‎10.某衬衫专卖店经销的男士衬衫,按价格从低到高分为A、B、C、D、E、F、G、H共8个档次,A档次的衬衫每天可卖出120件,每件可获利润50元。每高一个档次,卖出一件可增加利润10元,但是每高一个档次,这种档次的衬衫每天比低一档的衬衫少卖出8件。‎ (1) 在这8个档次的衬衫当中,卖哪个档次的所获得的利润最大?‎ (2) 卖出这种档次的衬衫一天所获得的最大利润是多少?‎ 初级点拨:本题可列表解答。或者运用求最大值的一个结论解答 深度提示:卖第一档的可获得最高利润为:50×120=(40+10×1)×(128-8×1);‎ 卖第二档可得利润为:(40+10×2)×(128-8×2);‎ 卖第三档可得利润为:(40+10×3)×(128-8×3);‎ ‎………。‎ 可得出:卖出第N档可得利润:(40+10N)×(128-8N)。‎ 全解过程:运用求最大值的一个结论解答,卖出第N档衬衫所获得的利润为(40+10N)×(128-8N)=10×(4+N)×8×(16-N)=80×(4+N)×(16-N)因为4+N+16-N=20,所以当4+N=16-N,即N=6时,利润最大。‎ 最大利润为:(40+10×6)×(128-8×6)=8000(元)。‎ 答:(1)卖第6个档次的所获得的利润最大。‎ ‎⑵卖出这种档次的衬衫一天所获得的最大利润是8000元。‎ 第19讲 方程 一、知识地图 今有上禾三秉,中禾二秉,下禾一秉,实三十九斗;上禾二秉,中禾三秉,下禾一秉,实三十四斗;上禾一秉,中禾二秉,下禾三秉,实二十六斗,问上、中、下禾实一秉各几何? ‎ 答曰:上禾一秉九斗四分斗之一,中禾一秉四斗四分斗之一,下禾一秉二斗四分斗之三。‎ ‎——《九章算术》‎ 这是我国历史上一道三元一次方程组的经典名题,具有传统意义的方程概念及解法,由此可见前人在方程领域的研究和造诣。‎ 百鸡问题 今有鸡翁一,值钱五,鸡母一,值钱三,鸡雏三,值钱一。凡百钱买百鸡,问鸡翁母雏各几何?‎ 答曰:鸡翁四,值钱二十,鸡母十八,值钱五十四,鸡雏七十八,值钱二十六;‎ 又答:鸡翁八,值钱四十,鸡母十一,值钱三十三,鸡雏八十一,值钱二十七;‎ 又答:鸡翁十二,值钱六十,鸡母四,值钱十二,鸡雏八十四,值钱二十八。‎ 术曰:鸡翁每增四,鸡母每减七,鸡雏每益三即得。‎ ‎ ——《张丘建算经》‎ 百鸡问题是我国历史上的一道数学名题,百鸡问题标志中国对不定方程理论有了系统研究。秦九韶的大衍求一术将不定方程与同余理论联系起来。我国著名数学家陈景润在1978年所著的《初等数论》中也给出了百鸡问题的解法,实际上就是一个二元一次不定方程。‎ 二、基础知识 ‎ ‎(一)等式的基本性质 (1) 等式:表示相等关系的式子;‎ 如:2+3=5,,…‎ (2) 等式基本性质1:等式两边同时加上同一个数或减去同一个数,等式性质不变;‎ ‎ 即如果A=B,那么A±m=B±m。‎ (3) 等式基本性质2:等式两边同时乘以同一个数或除以同一个不等于零的数,等式性 ‎ 质不变;‎ 即如果A=B,那么Am=Bm或(m、n为两个数,n≠0)。 ‎ ‎(二)一元一次方程 (1) 方程:含有未知数的等式;‎ 如:,,,…‎ (2) 一元一次方程:含有一个未知数,并且未知数的指数是1的方程;‎ 如:,,,…‎ (3) 一元一次方程的解:能使一元一次方程左右两边相等的未知数的值;‎ 如:是方程的解,‎ ‎ 是方程的解,…‎ (4) 解一元一次方程的步骤:‎ 去分母、去括号、移项、合并同类项、化未知数系数为1。‎ 例如:解一元一次方程 ;‎ 解: … 去分母(等式基本性质2)‎ ‎ … 去括号 ‎ ‎ … 移项 ‎ … 化未知数系数为1 ‎ ‎(三)二元一次方程(组)‎ (1) 二元一次方程:含有两个未知数,并且未知数的指数均为1的方程;‎ 如:,,…‎ (2) 二元一次方程组:由两个二元一次方程组成的方程组;‎ 如:,,…‎ (3) 二元一次方程的解:适合于一个二元一次方程的每一对未知数的值,叫作二元一次 ‎ 方程的一个解;‎ 如:是的一个解;也是的一个解;…‎ (1) 二元一次方程组的解:满足方程组中每一个方程的解就是这个方程组的解;‎ 如:是的解; …‎ (2) 解二元一次方程组的基本思想:消元;‎ (3) 解二元一次方程组的方法:代入消元法、加减消元法。‎ 例如:解下列二元一次方程组 解:②式化简得: ③‎ 将③×2得: ④‎ 将①×3得: ⑤‎ ‎④+⑤得:,解得,代入①得:‎ 所以方程组的解为。 ‎ ‎(四)不定方程(组)‎ ‎(1)不定方程(组):未知数个数多于方程个数,且对解有一定限制(比如要求解为正整数等)的方程。‎ ‎(2)古希腊的丢番图早在公元3世纪就开始研究不定方程,因此常称不定方程为丢番图方程。不定方程是数论中最古老的分支之一。中国是研究不定方程最早的国家,公元初的五家共井问题就是一个不定方程组问题。‎ ‎(3)研究不定方程要解决三个问题:①判断何时有解。②有解时决定解的个数。③求出所有的解。‎ ‎(4)几类特殊的不定方程 不定方程是一个内容丰富的课题,许多不定方程的解法有其特殊性。其中几类这样的方程,以及几个有普遍性的方法。‎ 一、余数分析法 将不定方程的解按某个正整数m的余数分类,或者考察方程中的项对某个正整数的余数,再进行分析。‎ 二、因数分析法 任何非零整数的因数个数是有限的,因此,可以对不定方程的解在有限范围内用枚举法确定。‎ 三、不等分析法 利用量的整数性或不等关系,确定出方程解的范围。‎ ‎(五)列方程(组)解应用题 (1) 根据题目已知条件,找出等量关系式;‎ (2) 利用题目已知条件,设出未知数;‎ (3) 根据等量关系式,列出方程(组);‎ (4) 解方程(组);‎ (5) 检验作答。‎ 三、经典透析 【例1】 列一元一次方程解应用题 ‎ (☆☆ 第八届华杯赛决赛试题)用边长相同的正六边形白色皮块、正五边形黑色皮块总计32块,缝制成一个足球。如图所示,每个黑色皮块邻接的都是白色皮块;每个白色皮块相间的与3个黑色皮块及3个白色皮块邻接。问:这个足球上共有多少块白色皮块?‎ 审题要点:先找等量关系式。‎ 详解过程:第一步,先找等量关系式:白色皮块的边中与黑色皮块公用的边数=黑色皮块的边中与白色皮块公用的边数;‎ 第二步,设出未知数:设这个足球上共有x块白色皮块;‎ 第三步,列出方程:;‎ 第四步,解方程:‎ 第五步,检验作答:‎ 检验:当时,左边=;右边=,所以左边=右边,所以是原方程的解;‎ 答:这个足球上共有20块白色皮块。‎ 【例2】 列一元一次方程解应用题 ‎(☆☆ 2001年我爱数学夏令营)如图,在中,是的中点,为上一点,且,已知四边形的面积是35,则的面积是多少? ‎ 审题要点:先找等量关系式。‎ 详解过程:第一步,先找等量关系式:‎ ‎∵BE=AB ‎∴=3‎ ‎∵BD=CD ‎∴=2‎ ‎∴2=3‎ 即等量关系式为;‎ 第二步,设出未知数:设=,则=35-,‎ 第三步,列出方程:‎ ‎=+‎ ‎ =3+‎ ‎ =3-(35-)‎ ‎ =2+35‎ ‎=+‎ ‎ =3+2‎ ‎ =3+2(35-)‎ ‎ =70-‎ 所以 第四步,解方程:‎ ‎∴=150‎ 第五步,检验作答:‎ 检验:当时,左边=;右边=,所以左边=右边,所以是原方程的解;‎ 答:△ABC的面积是150。‎ 【例3】 瓶子里装有浓度为15%的酒精‎1000克。现在又分别倒入‎100克和‎400克的A、B两种酒精,瓶子里的酒精浓度变为14%。已知A种酒精的浓度是B种酒精的2倍,求A种酒精的浓度。‎ 审题要点:A种酒精浓度是B种酒精的2倍。‎ 详解过程:依题意,A种酒精浓度是B种酒精的2倍。设B种酒精浓度为x%,则A种酒精浓度为2x%。A种酒精溶液‎100克,因此100×2x%为‎100克酒精溶液中含纯酒精的克数。B种酒精溶液‎400克,因此400×x%为‎400克酒精溶液中含纯酒精的克数。‎ 解:设B种酒精浓度为x%,则A种酒精的浓度为2x%。‎ ‎     ‎ ‎                150+6x=14×15‎ ‎                   x=10‎ ‎2x%=2×10%=20%。‎ 答:A种酒精的浓度为20%。‎ 【例4】 一群学生进行篮球投篮测验,每人投10次,按每人进球数统计的部分情况如下表:‎ 还知道至少投进3个球的人平均投进6个球,投进不到8个球的人平均投进3个球。问:共有多少人参加测验?‎ 审题要点:投中的总球数既等于进球数不到3个的人的进球数加上至少投进3个球的人的进球数,也等于进球数不到8个的人的进球数加上至少投进8个球的人的进球数。‎ 详解过程:设有x人参加测验。由上表看出,至少投进3个球的有(x-‎7-5-4‎)人,投进不到8个球的有(x-3-4-1)人。投中的总球数,既等于进球数不到3个的人的进球数加上至少投进3个球的人的进球数,‎ ‎  0×7+1×5+2×4+6×(x-‎7-5-4‎)‎ ‎  = 5+8+6×(x-16)‎ ‎  = 6x-83,‎ 也等于进球数不到8个的人的进球数加上至少投进8个球的人的进球数,‎ ‎  3×(x-‎3-4-1‎)+8×3+9×4+10×1,‎ ‎  = 3×(x-8)+24+36+10‎ ‎  = 3x+46。‎ ‎  由此可得方程 ‎  6x-83=3x+46,‎ ‎    3x=129,‎ ‎    x=43(人)。‎ 答:共有43人参加测验。‎ 【例3】 列二元一次方程组解应用题 ‎(☆ 2004走进美妙数学花园试题)甲、乙两件商品的成本共600元,已知甲商品按45%的利润定价,乙商品按40%的利润定价,后来甲打8折出售,乙打9折出售,结果共获利110元。两件商品中,成本较高的那件商品的成本是多少元?‎ 审题要点:甲打8折出售,乙打9折出售,结果共获利110元。‎ 详解过程:设甲商品的成本是元,乙商品的成本是元,列方程组得:‎ 解得:‎ 答:成本较高的那件商品的成本是460元。‎ 【例4】 列二元一次方程组解应用题 ‎(☆☆ 95年圣彼得堡奥林匹克竞赛试题)在H岛上居住着100个人,其中一些人总是说假话,其余人则永远说真话,岛上的每一位居民崇拜三个神之一:太阳神、月亮神和地球神。向岛上的每一位居民提了三个问题:‎ (1) 您崇拜太阳神吗?‎ (2) 您崇拜月亮神吗?‎ (3) 您崇拜地球神吗?‎ 对第一个问题有60人回答:“是”;对第二个问题有40人回答:“是”;对第三个问题有30人回答:“是”。他们中有多少人说的是假话?‎ 审题要点:我们将永远说真话的人称为老实人,把总说假话的人称为骗子。每个老实人都只会对一个问题:“是”。而每个骗子则都对两个问题答:“是”。‎ 详解过程:我们将永远说真话的人称为老实人,把总说假话的人称为骗子。每个老实人都只会对一个问题:“是”。而每个骗子则都对两个问题答:“是”。‎ 设老实人的人数为人,骗子的人数为人,列方程组有:‎ 解得。所以岛上有30个人说的是假话。‎ 【例5】 列不定方程(组)解应用题 百鸡问题译成现代汉语是:公鸡五元一只,母鸡三元一只,小鸡一元三只,用一百元钱正好买了一百只鸡,问:公鸡、母鸡、小鸡各买了多少只?‎ 审题要点:一百元钱正好买了一百只鸡。‎ 详解过程:设公鸡买了只,母鸡买了只,小鸡买了只,列方程组如下:‎ 化简整理:将①×3得: ③‎ 将③-②得:‎ 考虑到是鸡的只数,所以为正整数,解得:‎ ‎ ‎ 再代入②得: ‎ 【例3】 列不定方程解应用题 已知x、y代表两位整数,求方程100x+y=2xy的解。‎ 审题要点:将原方程变形为100x=2xy-y。‎ 详解过程:本题如果变形为,则不易找到y可能的取值,所以变形时应看具体题目而定。这里建议可介绍分离系数法。分离系数法的原则是分离系数较小的未知数。‎ 将原方程变形为100x=2xy-y,解得,要使y是整数,必须是整数,而x为两位整数,即x≥10,故2x-1≥19,即x=13,y=52。‎ 专家点评:但如果用逐个试验法就应该按系数较大的未知数去试,如下题:‎ 求方程17x+8y=158的非负整数解。‎ 分析:由题意有0≤17x≤158,0≤8y≤158,得0≤x≤9,0≤y≤19,可见x情况较少,所以将x=0,1,…,‎ ‎9逐个代入检验知。‎ 【例3】 甲说:“我和乙、丙共有100元钱。”乙说:“如果甲的钱是现有的6倍,我的钱是现有的,丙的钱不变,则我们仍共有100元。”丙说:“我的钱连30元都不到。”问三人各有多少钱?‎ 审题要点:这是三元不定方程组。‎ 详解过程:设甲、乙、丙各有x、y、z元,由题意可列出方程组:,因为z有取值范围不好消去,所以我们可以消y,得,解得,又z<30,所以2z<60,17x>140,因而x=10。所以原方程组的解为,即甲、乙、丙分别有10元、75元、15元。‎ 拓展训练 1. ‎(☆)解下列方程(组):‎ ‎(1); (2);‎ ‎(3)‎ 分析:(1)解:‎ ‎ ‎ ‎(2)解:‎ ‎ ‎ ‎ ‎ ‎ ‎ ‎ ‎ (1) 解:化简原方程组得:‎ ‎②-①得:,解得,代入得 所以原方程组的解为。 ‎ 1. ‎(2006迎春杯集训题)水果店运来的西瓜的个数是白兰瓜的个数的2倍。如果每天卖白兰瓜40个,西瓜50个,若干天后卖完白兰瓜时,西瓜还剩360个。水果店运来的西瓜和白兰瓜共多少个?‎ 初级点拨:抓住卖的天数这一等量关系。‎ 深度提示:设白兰瓜进了x个,则西瓜进了2x个,由天数相等列方程。‎ 全解过程:列方程得:‎ ‎ =480‎ ‎2=960‎ 所以西瓜和白兰瓜共480+960=1440个。‎ 2. ‎(北京市2006年“数学解题能力展示”读者评选活动(迎春杯)高年级组初试第1题)‎ 一个分数约分后是。如果这个分数的分子减去18,分母减去22,可以得到一个新的分数,它等于。那么,约分前的这个分数是 。‎ 初级点拨:设约分前的这个分数是 深度提示:列出方程=‎ 全解过程:=‎ ‎5(‎2a-18)=3(‎3a-22)‎ ‎10a‎-90=‎9a-66‎ a=24‎ 所以=‎ 3. 甲乙两个班的同学去运河公园春游,但只有一辆车接送。甲班的学生坐车从学校出发的同时,乙班学生开始步行。车到途中某处,让甲班学生下车步行,车立即返回接乙班学生上车并直接开往运河公园。两个班的学生步行速度均为每小时‎5千米,汽车载学生行驶的速度是每小时50千米。空车行驶时每小时行‎60千米。问:要使两班学生同时到达运河公园,甲班学生步行了全程的几分之几?‎ 初级点拨:甲、乙两班学生要同时到达运河公园,则这两班学生步行的路程必须相等。‎ 深度提示:利用甲班学生步行所用的时间与乙班学生坐车的时间及空车返回的时间之和相等的关系来列方程 全解过程:如果两班学生各走了x千米,而全程共有s千米时,则行程可用图表示。‎ 可以利用甲班学生步行所用的时间与乙班学生坐车的时间及空车返回的时间之和相等的关系来列方程。,解得:,所以甲班学生步行了全程的。‎ 1. ‎(第10届华杯赛)由于浮力的作用,金放在水里称量和它的重量比较,在水中的“重量”会减少;银放在水里称量和它的重量相比较,在水中的“重量”会减少,某个只含有金银成分的古文物,重‎150克,在水中称量,“重量”是‎141克,则古文物中金占________%。(精确到1%) ‎ ‎ ‎ 初级点拨:设古文物中含金x克,银y克。‎ 深度提示:抓住水中减少的量列方程。‎ 全解过程:列方程得 解得 ‎ 答:古文物中金占84%。‎ 2. 甲乙两人进行400比赛,第1次甲让乙先跑18秒,结果甲落后‎40米。第2次甲让乙先跑‎25米 ‎,结果甲比乙早到7.5秒,跑‎400米,甲、乙各需要多少时间?‎ 初级点拨:列方程根据的是甲、乙跑‎1米所需要的时间。‎ 深度提示:设甲跑‎400米需要秒,乙跑‎400米需要秒。‎ 全解过程:依题意可列方程组:‎ ‎ ‎ 化简得 解得:‎ 1. 甲地有89吨货物要运送到乙地,大卡车的载重量是7吨,小卡车的载重量是4吨,大卡车运一趟耗油‎14升,小卡车运一趟耗油‎9升,运完这批货物至少耗油多少升? ‎ 初级点拨:设大卡车运x趟,小卡车运y趟。‎ 深度提示:列出方程7x+4y=89。‎ 全解过程:7x+4y=89,解得,(3x-1)是4的倍数,且y≥0,得,相应的耗油量分别为14×3+9×17=195(升),14×7+9×10=188(升),14×11+9×3=181(升)。所以至少是‎181升。‎ 2. 某工厂为优秀职工发奖金,一等奖每人1800元,二等奖每人1200元,三等奖每人800元,每种奖都有人领,共有15名优秀职工,他们领的奖金的总数为16000元。获得一、二、三等奖的职工各有多少人?‎ 初级点拨:设一、二、三等奖依次有a,b,c人。‎ 深度提示:利用总额和人数列方程。‎ 全解过程:列方程得 ‎ ‎ ‎ ‎9a+6b+‎4c=80‎ ‎ ‎4a+4b+‎4c=60‎ ‎5a‎+2b=20‎ 根据a,b都是整数,解得:a=2,b=5,c=8。‎ 3. 已知四位数n与其数字和相加的和是1992,求n。‎ 初级点拨:我们知道四位数字的各位数字之和最大是36,因此这个自然数n的前两位是1和9。‎ 深度提示:设这个四位数为 全解过程:依据题意可得:‎ ‎+1+9+a+b=1992‎ ‎1910+‎11a+2b=1992‎ ‎11a‎+2b=82‎ 根据奇偶性,a只能为偶数。‎ 所以a=6,b=8。‎ 即符合条件的四位数是1968。‎ 1. ‎(2007年5月第五届“走进美妙的数学花园”中国青少年数学论坛趣味数学解题技能展示大赛决赛题)‎ 如图,甲、乙两只蜗牛同时从A点出发,甲沿长方形ABCD逆时针爬行,乙沿△AOD逆时针爬行。若AB=10,BC=14,AO=DO=10,且两只蜗牛的速度相同,则当两只蜗牛间的距离第一次达到最大值时,它们所爬过的路程的和为多少?‎ 初级点拨:两只蜗牛的速度相等,所行的总路程也就相等。‎ 深度提示:第一次达到的最大值时,两只蜗牛的位置应该在长方形的对角线上,所以,可能有两种情况。‎ 全解过程:两种情况如下:‎ (1) 在A、C位置:‎ 解:设乙蜗牛经过x个三角形周长,甲蜗牛走过y个长方形的(长+宽)。‎ x(14+20)=y(10+14)‎ ‎34x=24y ‎17x=12y 第一次, 则x=12,y=17‎ 所以它们爬过的总路程=12×34×2=816‎ (2) 在B、D位置:‎ 解:设乙蜗牛经过x个三角形周长,甲蜗牛走过y个长方形的(长+宽)。‎ x(14+20)+20=y(14+10)+10‎ ‎34x+20=24y+10‎ ‎24y-34x=10‎ ‎12y-17x=5‎ 根据y= ‎ x=奇数,=偶数 x=11时,y=16,‎ 第一次最远。此时共走了[11×(14+20)+10]×2=768。‎ ‎⑵的情况与⑴的情况相比,768<816。‎ 答:两只蜗牛的距离第一次达到最大值时,它们所爬过的路程和是816。‎ 第20讲 排列组合 一、 知识地图 1) 加法原理 2) 乘法原理 3) 排列 a) 信号问题 b) 数字问题 c) 坐法问题 d) 照相问题 e) 排队问题 4) 组合 a) 几何计数问题 b) 加乘算式问题 c) 比赛问题 d) 选法问题 二、 基础知识 ‎(一)加法原理:一般地,如果完成一件事有k类方法,第一类方法中有m1种不同做法,第二类方法中有m2种不同做法,…,第k类方法中有mk种不同的做法,则完成这件事共有 N=‎ 种不同的方法。这就是加法原理。‎ ‎  例如:某人从北京到天津,他可以乘火车也可以乘长途汽车,现在知道每天有五次火车从北京到天津,有4趟长途汽车从北京到天津。那么他在一天中去天津能有多少种不同的走法?‎ 解答:分析这个问题发现,此人去天津要么乘火车,要么乘长途汽车,有这两大类走法,并且每种走法都可以直接到达目的地,一步就可以完成任务,可以用加法原理。如果乘火车,有5种走法,如果乘长途汽车,有4种走法。上面的每一种走法都可以从北京到天津,故共有5+4=9种不同的走法。像这样一步可以完成任务,就用加法原理。‎ ‎(二)乘法原理:一般地,如果完成一件事需要n个步骤,其中,做第一步有m1种不同的方法,做第二步有m2种不同的方法,…,做第n步有mn种不同的方法,那么,完成这件事一共有 ‎ N=‎ 种不同的方法。这就是乘法原理。‎ 例如:一个口袋内装有3个小球,另一个口袋内装有8个小球,所有这些小球颜色各不相同。问:从两个口袋内各取一个小球,有多少种不同的取法?‎ 解答:要从两个口袋中各取一个小球,则可看成先从第一个口袋中取一个,再从第二个口袋中取一个,分两步完成,要用乘法原理。共有3×8=24(种)不同的取法。‎ ‎1.加法原理和乘法原理有什么区别?‎ 1) 加法原理:先把方法分类,每一类的方法都能完成这件事。最后把这些方法相加。‎ 2) 乘法原理:先把方法分步,每一步都不能独立完成这件事,但是完成这件事,这些步骤缺一不可。最后把方法数相乘。‎ ‎2.运用两个基本原理时要注意:‎ 1) 抓住两个基本原理的区别,千万不能混。‎ a) 不同类的方法(其中每一个方法都能各自独立地把事情从头到尾做完)数之间做加法,可求得完成事情的不同方法总数。‎ b) 不同步的方法(全程分成几个阶段(步),其中每一个方法都只能完成这件事的一个阶段)数之间做乘法,可求得完成整个事情的不同方法总数。‎ 2) 在研究完成一件工作的不同方法数时,要遵循“不重不漏”的原则。请看一些例子:从若干件产品中抽出几件产品来检验,如果把抽出的产品中至多有2件次品的抽法仅仅分为两类:第一类抽出的产品中有2件次品,第二类抽出的产品中有1件次品,那么这样的分类显然漏掉了抽出的产品中无次品的情况。又如:把能被2、被3或被6整除的数分为三类:第一类为能被2整除的数,第二类为能被3整除的数,第三类为能被6整除的数。这三类数互有重复部分。‎ 3) 在运用乘法原理时,要注意当每个步骤都做完时,这件事也必须完成,而且前面一个步骤中的每一种方法,对于下个步骤不同的方法来说是一样的。‎ ‎(三)排列 一般地,从n个不同元素中取出m个元素(m≤n)排成一列的问题,可以看成是从n个不同元素中取出m个,排在m个不同的位置上的问题,而排列数就是所有可能的排法数量。那么,每个排列共需要m步,而每一步又有若干种不同的方法,排列数可以这样计算:‎ ‎  第一步:先排第一个位置上的元素,可以从n个元素中任选一个,有n种不同的选法;‎ ‎  第二步:排第二个位置上的元素。这时,由于第一个位置已用去了一个元素,只剩下(n-1)个不同的元素可供选择,共有(n-1)种不同的选法;‎ ‎  第三步:排第三个位置上的元素,有(n-2)种不同的选法;‎ ‎  ……‎ ‎  第m步:排第m个位置上的元素。由于前面已经排了(m-1)个位置,用去了(m-1)个元素。这样,第m个位置上只能从剩下的[n-(m-1)]=(n-m+1)个元素中选择,有(n-m+1)种不同的选法。‎ ‎  由乘法原理知,共有:‎ ‎  n(n-1)(n-2)…(n-m+1)‎ 种不同的排法,即:‎ ‎。‎ 这里,m≤n;且等号右边从n开始,后面每个因数比前一个因数小1,共有m个因数相乘。‎ 例如:有五面颜色不同的小旗,任意取出三面排成一行表示一种信号,问:共可以表示多少种不同的信号?‎ 解答:这里五面不同颜色的小旗就是五个不同的元素,三面小旗表示一种信号,就是有三个位置。我们的问题就是要从五个不同的元素中取三个,排在三个位置的问题。由于信号不仅与旗子的颜色有关,而且与不同旗子所在的位置有关,所以是排列问题,其中n=5,m=3。由排列数公式知,共可组成=5×4×3=60种不同的信号。‎ ‎(四)组合 一般地,从n个不同元素中取出m个元素(m≤n)组成一组不计较组内各元素的次序,叫做从n个不同元素中取出m个元素的一个组合。‎ ‎  由组合的定义可以看出,两个组合是否相同,只与这两个组合中的元素有关,而与取到这些元素的先后顺序无关。只有当两个组合中的元素不完全相同时,它们才是不同的组合。‎ ‎  从n个不同元素中取出m个元素(m≤n)的所有组合的个数,叫做从n个不同元素中取出m个不同元素的组合数。记作。‎ 一般地,求从n个不同元素中取出m个元素排成一列的排列数可以分两步求得:‎ ‎  第一步:从n个不同元素中取出m个元素组成一组,共有种方法;‎ ‎  第二步:将每一个组合中的m个元素进行全排列,共有种排法。‎ ‎  故由乘法原理得到:‎ ‎  =·‎ ‎  因此 ‎  ‎ ‎  这就是组合数公式。‎ ‎  ‎ 规定=1,=1。‎ 例如:数学小组有8个人,现在要从这8个人中选出3个人来参加数学竞赛,问有多少种不同的选法?‎ 解答:从8个人里选出3个人参加数学竞赛,只与选出的结果有关,与选的先后顺序无关,所以是组合问题,n=8,m=3。根据组合公式知:,所以有56种不同的选法。‎ 解决排列组合问题的一些解题技巧,这里可以简单的向大家做一个介绍。简单的说是十六字方针“分类相加,分步相乘,有序排列,无序组合。十二个技巧。‎ 1. 优先排列法;‎ 2. 总体淘汰法;‎ 3. 合理分类与准确分步;‎ 4. 相邻问题用捆绑法;‎ 5. 不相邻问题用“插空法”;‎ 6. 顺序固定用“除法”;‎ 7. 分排问题用直接法;‎ 8. 试验法;‎ 9. 探索法;‎ 10. 消序法;‎ 11. 住店法;‎ 12. 对应法。‎ 二、 经典透析 ‎【例1】(☆☆)小明和小王从北京出发先到天津看海,然后再到上海东方明珠塔参观。从北京到天津可以坐火车或者坐公共汽车,坐火车有4种车次,坐公共汽车有3种车次;而从天津到上海可以坐火车,公共汽车,轮船或者飞机,火车有3种,汽车有5种,轮船有4种,飞机有2种。问小明和小王从北京到上海旅游一共有多少种走法?‎ 审题要点:首先看他们完成整个过程需要几个过程,这是判断利用加法原理和乘法原理的依据。很明显整个过程要分两步完成,先从北京到天津,再从天津到上海,应该用乘法原理。‎ 我们再分开来看,先看从北京到天津,无论是坐火车还是汽车都是一步完成,所以要用加法原理,同样的道理,从天津到上海的走法计算也应该用加法原理。‎ 详解过程:从北京到天津走法有:4+3=7种,同样的道理,天津到上海走法有:3+5+4+2=14(种)。‎ 最后,算出从北京到上海的走法有:7×14=98(种)。‎ 专家点评:本题是考察学生对加法乘法原理的理解,只要正确利用加法乘法原理,解这种题应该难度不大。‎ ‎【例2】(☆☆☆)某公园有两个园门,一个东门,一个西门。若从东门入园,有两条道路通向龙凤亭,从龙凤亭有一条道路通向园中园,从园中园又有两条道路通向西门。另外,从东门有一条道路通向游乐场。从游乐场有两条道路通向水上世界,另有一条道路通向园中园。从水上世界有一条道路通向西门,另有一条道路通向小山亭,从小山亭有一条道路通向西门。问若从东门入园,从西门出园一共有多少种不同的走法(不走重复路线)?‎ 审题要点:这个题的已知条件比较复杂。首先让我们将已知条件“梳理”一下:   1.从东门入园,从西门出园;   2.从东门入园后,可以通向两个游览区,龙凤亭与游乐场;   3.从龙凤亭经园中园可达到西门;   4.从游乐场经水上世界可达到西门,或从游乐场经园中园可达到西门;   5.从水上世界经小山亭可达到西门; 根据以上五条可知,从东门入园经龙凤亭经园中园达到西门为一主干线。而东门到龙凤亭有两条不同路线;龙凤亭到园中园只有一条路线;园中园到西门又有两条不同的路线。由乘法原理,这条主干线共有2×1×2=4种不同的走法。再看从东门入园后到游乐场的路线。从东门到游乐场只有一条路,由游乐场分成两种路线,一是经园中园到西门,这条路线由乘法原理可知有1×1×2=2种不同走法;二是经水上世界到西门,从水上世界到西门共有两条路线(由水上世界直接到西门和经小山亭到西门),再由乘法原理可知这条路线有1×2×2=4种不同路线。最后由加法原理计算。从东门入园从西门出园且不走重复路线的走法共有2×1×2+1×1×2+1×2×2=10种。‎ 详解过程:从东门入园从西门出园且不走重复路线的走法共有2×1×2+1×1×2+1×2×2=10(种)。‎ 专家点评:本题主要考察加法乘法原理。先分类利用加法原理,再对每一类进行分步利用乘法原理。‎ 注意:这道题也可用“枚举法”来解。 我们可以先画出一个图,从图上便可以得出正确的答案。‎ 图中A表示东门,B表示西门,C表示龙凤亭,D表示园中园,E表示游乐场,F表示水上世界,G表示小山亭,线表示道路。不同的走法有:即共有10种不同走法。‎ ‎ ‎ ‎ ‎ 如果能够利用加法与乘法原理的题型就没必要用枚举法,因为枚举法比较容易重复和遗漏。‎ ‎【例3】(☆☆☆)由数字0、1、2、3组成三位数,问:‎ ‎  ①可组成多少个不相等的三位数?‎ ‎  ②可组成多少个没有重复数字的三位数?‎ 审题要点:首先确定解题方法,在确定由0、1、2、3组成的三位数的过程中,应该一位一位地去确定。所以,每个问题都可以看成是分三个步骤来完成,最后要用乘法原理来处理。‎ 详解过程:①要求组成不相等的三位数。所以,数字可以重复使用,百位上,不能取0,故有3种不同的取法;十位上,可以在四个数字中任取一个,有4种不同的取法;个位上,也有4种不同的取法,由乘法原理,共可组成3×4×4=48个不相等的三位数。‎ ‎  ②要求组成的三位数中没有重复数字,百位上,不能取0,有3种不同的取法;十位上,由于百位已在1、2、3中取走一个,故只剩下0和其余两个数字,故有3种取法;个位上,由于百位和十位已各取走一个数字,故只能在剩下的两个数字中取,有2种取法,由乘法原理,共有3×3×2=18个没有重复数字的三位数。‎ 专家点评:在解题之前首先确定解题方法,然后各个击破就可以了。运用乘法原理时的分步时,应该先从限制条件最多的地方下手,比如此题中就应该从百位开始取,而不应该先从其他位取。另外特别要注意“是否重复”这个条件,因为这关系着后面每一步的方法数。‎ ‎【例4】(☆☆☆)如下图,A、B、C、D、E五个区域分别用红、黄、蓝、白、黑五种颜色中的某一种染色,要使相邻的区域染不同的颜色,共有多少种不同的染色方法?‎ 审题要点:首先确定解题方法,将染色这一过程分为依次给A、B 、C 、D 、E染色五步,很明显要用乘法原理,现在只要算出各个量就行了。‎ 详解过程:先给A染色,因为有5种颜色,故有5种不同的染色方法;第2步给B染色,因不能与A同色,还剩下4种颜色可选择,故有4种不同的染色方法;第3步给C染色,因为不能与A,B同色,故有3种不同的染色方法;第4步给D染色,因为不能与A,C同色,故有3种不同的染色方法;第5步给E染色,由于不能与A,C,D同色,故只有2种不同的染色方法。根据乘法原理,共有不同的染色方法 ‎5×4×3×3×2=360(种)。‎ 专家点评:染色问题,一般用乘法原理。但要注意限制性条件,从限制条件最多的部分的开始分步,此题中A区域与其他四个区域均有相邻,所以应该先染色,其他依次类推。‎ ‎【例5】(☆☆☆)4名同学到照相馆照相。他们要排成一排,问:共有多少种不同的排法?‎ 审题要点:4个人到照相馆照相,那么4个人要分坐在四个不同的位置上。所以这是一个排列问题。我们的问题就是要从四个不同的元素中取四个,排在四个位置。这时n=4,m=4。‎ 详解过程:由排列数公式知,共有 ‎   种不同的排法。‎ 答:共有24种不同的排法。‎ 专家点评:首先看清楚是排列还是组合,这个是解决排列组合问题的前提,也是必需的条件,区分排列组合后,应该确定元素总数和排列组合元素数,信号旗问题是典型的排列问题。‎ 一般地,对于m=n的情况,排列数公式变为 ‎  ‎ 表示从n个不同元素中取n个元素排成一列所构成排列的排列数。这种n个排列全部取出的排列,叫做n个不同元素的全排列。‎ ‎  全排列式右边是从n开始,后面每一个因数比前一个因数小1,一直乘到1的乘积,记为n!,读做“n的阶乘”,则可以写为:其中n!=n(n-1)(n-2)…3·2·1。‎ 注意:这个问题也可以用乘法原理来做。一般,乘法原理中与顺序有关的问题常常可以用排列数公式做,用排列数公式解决问题时,可避免一步步地分析考虑,使问题简化。‎ ‎【例6】(☆☆☆)从分别写有1、3、5、7、8五张卡片中任取两张,作成一道两个一位数的乘法题,问:‎ ‎  ①有多少个不同的乘积?‎ ‎  ②有多少个不同的乘法算式?‎ 审题要点:①要考虑有多少个不同乘积。由于只要从5张卡片中取两张,就可以得到一个乘积,因为乘法的交换率,有多少个乘积只与所取的卡片有关,而与卡片取出的顺序无关,所以这是一个组合问题。‎ ‎②要考虑有多少个不同的乘法算式,它不仅与两张卡片上的数字有关,而且与取到两张卡片的顺序有关,所以这是一个排列问题。‎ 详解过程:①由组合数公式得到,共有 ‎   个不同的乘积。‎ ‎②由排列数公式,共有=5×4=20种不同的乘法算式。‎ 答:共有10个不同的乘积,20种不同的乘法算式。‎ 专家点评:首先判断是排列还是组合,剩下的就是简单计算了。‎ ‎【例7】(☆☆☆)如下图,问:①下左图中,共有多少条线段?②下右图中,共有多少个角?‎ 审题要点:①在线段AB上共有7个点(包括端点A、B)。注意到,只要在这七个点中选出两个点,就有一条以这两个点为端点的线段,而与选这两个端点的顺序无关,所以,这是一个组合问题。‎ ‎②从O点出发的射线一共有11条,它们是OA, OP1,OP2,OP3,…,OP9,OB。注意到每两条射线可以形成一个角,所以,只要看从11条射线中取两条射线有多少种取法,就有多少个角。显然,是组合问题。‎ 详解过程:①由组合数公式知,共有 ‎  条不同的线段;‎ ‎②共有种不同的取法,所以,可组成个角。‎ 由组合数公式知,共有个不同的角。‎ 专家点评:在几何计数当中也用到了很多排列组合的方法。‎ ‎【例8】(☆☆☆☆)从5幅国画,3幅油画,2幅水彩画中选取两幅不同类型的画布置教室,问有几种选法?‎ 审题要点:首先考虑从国画、油画、水彩画这三种画中选取两幅不同类型的画有三种情况,即可分三类,自然考虑到加法原理。当从国画、油画各选一幅有多少种选法时,利用的乘法原理。由此可知这是一道利用两个原理的综合题。关键是正确把握原理。‎ 详解过程:符合要求的选法可分三类:‎ 设第一类为:国画、油画各一幅,可以想像成,第一步先在5张国画中选1张,第二步再在3张油画中选1张。由乘法原理有 5×3=15种选法。‎ 第二类为:国画、水彩画各一幅,由乘法原理有 5×2=10种选法。‎ 第三类为:油画、水彩画各一幅,由乘法原理有3×2=6种选法。‎ 这三类是各自独立发生互不相干进行的。‎ 因此,依加法原理,选取两幅不同类型的画布置教室的选法有 15+10+6=31种。‎ 答:有31种选法。‎ 专家点评:我们讨论了加法原理、乘法原理、排列、组合等问题。事实上,这些问题是相互联系、不可分割的。有时候做某件事情有几类方法,而每一类方法又要分几个步骤完成。在计算做这件事的方法时,既要用到乘法原理,又要用到加法原理。要正确地解决这些问题,就一定要熟练地掌握两个原理和排列、组合的内容,并熟悉它们所解决问题的类型特点。‎ ‎【例9】(☆☆☆☆)国家举行足球赛,共15个队参加。比赛时,先分成两个组,第一组8个队,第二组7个队。各组都进行单循环赛(即每个队要同本组的其他各队比赛一场)。然后再由各组的前两名共4个队进行单循环赛,决出冠亚军。问:①共需比赛多少场?②如果实行主客场制(即A、B两个队比赛时,既要在A队所在的城市比赛一场,也要在B队所在的城市比赛一场),共需比赛多少场?‎ 审题要点:①实行单循环赛,比赛的所有场次包括三类:第一组中比赛的场次,第二组中比赛的场次,决赛时比赛的场次。总的场次计算要用加法原理。‎ ‎②由于是实行主客场制,每两个队之间要比赛两场,比赛场次是①中的2倍。‎ 另外,由于主客场制不仅与参赛的队有关,而且与比赛所在的城市(即与顺序)有关。还可以用排列的知识来解决。‎ 详解过程:①第一组中8个队,每两队比赛一场,8个队里选两个队,是组合问题,所以共比赛场;第二组中7个队,每两队比赛一场,所以共比赛场;决赛中4个队,每两队比赛一场,所以共比赛场。‎ 实行单循环赛共比赛 ‎  ‎ ‎②第一组共比赛场,第二组共比赛场,决赛时共比赛场。‎ 实行主客场制,共需比赛2×()=110(场)。‎ 或解为:=8×7+7×6+4×3=56+42+12=110(场)。‎ 答:共需比赛55场,如实行主客场制要比赛110场。‎ 专家点评:比赛问题是常见的排列组合问题,一般需要综合求解。‎ 四、拓展训练 ‎1.如下图,从甲地到乙地有4条路可走,从乙地到丙地有2条路可走,从甲地到丙地有3条路可走。那么,从甲地到丙地共有多少种走法?‎ 初级点拨: 分析题意,从甲地到丙地,先看是用加法原理还是乘法原理,判断好方法,然后简单计算就可以了。‎ 深度提示:从甲地到丙地共有两大类不同的走法,用加法原理。‎ ‎  第一类,由甲地途经乙地到丙地。这时,要分两步走,第一步从甲地到乙地,有4种走法;第二步从乙地到丙地共2种走法,所以要用乘法原理,这时共有4×2种不同的走法。‎ ‎  第二类,由甲地直接到丙地,由条件知,有3种不同的走法。‎ 全解过程:由加法原理知,由甲地到丙地共有:4×2+3=11(种)不同的走法。‎ 答:从甲地到丙地有11种不同的走法。‎ ‎2.一个口袋内装有3个小球,另一个口袋内装有8个小球,所有这些小球颜色各不相同。问:①从两个口袋内任取一个小球,有多少种不同的取法?‎ ‎②从两个口袋内各取一个小球,有多少种不同的取法?‎ 初级点拨:先弄清楚用加法原理还是乘法原理,先看有几大类,再看分几步。本题应注意加法原理和乘法原理的区别及使用范围的不同,乘法原理中,做完一件事要分成若干个步骤,一步接一步地去做才能完成这件事;加法原理中,做完一件事可以有几类方法,每一类方法中的一种做法都可以完成这件事。往往有许多事情是有几大类方法来做的,而每一类方法又要由几步来完成,这就要熟悉加法原理和乘法原理的内容,综合使用这两个原理。‎ 深度提示:①从两个口袋中只需取一个小球,则这个小球要么从第一个口袋中取,要么从第二个口袋中取,共有两大类方法。所以是加法原理的问题。‎ ‎②要从两个口袋中各取一个小球,则可看成先从第一个口袋中取一个,再从第二个口袋中取一个,分两步完成,是乘法原理的问题。‎ 全解过程:①从两个口袋中任取一个小球共有3+8=11(种),不同的取法。‎ ‎②从两个口袋中各取一个小球共有3×8=24(种)不同的取法。‎ 答:从两个口袋任取一球有11种不同的取法,从两个口袋各取一球有24种不同的取法。‎ ‎3.右图中共有16个方格,要把A、B、C、D四个不同的棋子放在方格里,并使每行每列只能出现一个棋子。问:共有多少种不同的放法?‎ ‎ ‎ 初级点拨:由于四个棋子要一个一个地放入方格内。故可看成是分四步完成这件事。要用乘法原理。‎ 深度提示:第一步放棋子A,A可以放在16个方格中的任意一个中,故有16种不同的放法;第二步放棋子B,由于A已放定,那么放A的那一行和一列中的其他方格内也不能放B,故还剩下9个方格可以放B,B有9种放法;第三步放C,再去掉B所在的行和列的方格,还剩下四个方格可以放C,C有4种放法;最后一步放D,再去掉C所在的行和列的方格,只剩下一个方格可以放D,D有1种放法,本题要由乘法原理解决。‎ 全解过程:由乘法原理,共有 ‎  16×9×4×1=576(种)不同的放法。‎ 答:共有576种不同的方法。‎ ‎4.如下图,要从A点沿线段走到B,要求每一步都是向右、向上或者向斜上方。问有多少种不同的走法?‎ 初级点拨:注意到,从A到B要一直向右、向上,那么,经过右图中C、D、E、F四点中的某一点的路线一定不再经过其他的点。也就是说从A到B点的路线共分为四类,它们是分别经过C、D、E、F的路线。‎ 深度提示:第一类,经过C的路线,分为两步,从A到C再从C到B,从A到C有2条路可走,从C到B也有两条路可走,由乘法原理,从A经C到B共有2×2=4条不同的路线。‎ ‎  第二类,经过D点的路线,分为两步,从A到D有4条路,从D到B有4条路,由乘法原理,从A经D到B共有4×4=16种不同的走法。‎ ‎  第三类,经过E点的路线,分为两步,从A到E再从E到B,观察发现。各有一条路。所以,从A经E到B共有1种走法。‎ ‎  第四类,经过F点的路线,从A经F到B只有一种走法。最后由加法原理即可求解。‎ 全解过程::如上右图,从A到B共有下面的走法:‎ ‎  从A经C到B共有2×2=4种走法;‎ ‎  从A经D到B共有4×4=16种走法;‎ ‎  从A经E到B共有1种走法;‎ ‎  从A经F到B共有1种走法。所以,从A到B共有:4+16+1+1=22种不同的走法。‎ 答:有22种不同的走法。‎ ‎5.某班要在42名同学中选出3名同学去参加夏令营,问共有多少种选法?如果在42人中选3人站成一排,有多少种站法?‎ 初级点拨:首先根据不同情况分,看清楚是用排列还是组合,然后再根据排列组合公式进行求解。‎ 深度提示:要在42人中选3人去参加夏令营,那么,所有的选法只与选出的同学有关,而与三名同学被选出的顺序无关。所以,共有种不同的选法。要在42人中选出3人站成一排,那么,所有的站法不仅与选出的同学有关,而且与三名同学被选出的顺序有关。所以,共有种不同的站法。‎ 全解过程:由组合数公式,共有 ‎  种不同的选法。‎ 由排列数公式,共有=42×41×40=68880种不同的站法。‎ 答:有11480种不同的选法,有68880种不同的站法。‎ ‎6. 从8人的数学兴趣小组中选2人 ‎(1)分别担任正副组长,有多少种不同的选法?‎ ‎(2)一起参加一次数学竞赛,有多少种不同的选法?‎ 初级点拨:注意分清排列问题和组合问题。‎ 深度提示:(1)选出正副组长,有正副之分,也就是从8人中选2人后,要进一步确认正副组长,因此是个排列问题;‎ ‎(2)选人参加数学竞赛没有顺序,因此是个组合问题。‎ 全解过程:(1)利用排列公式,共有=8×7=56种选法;‎ ‎(2)利用组合公式,共有==28种选法。‎ 答:分别担任正副组长,有56种不同的选法;一起参加一次数学竞赛,有28种不同的选法。‎ ‎7.在一个圆周上有10个点,以这些点为端点或顶点,可以画出多少不同的(1)直线段(2)三角形(3)四边形?‎ 初级点拨:首先观察是组合问题还是排列问题,那就要看你取的点是否与顺序有关?‎ 深度提示:很明显,你要画的三个图形都与取出点的顺序无关,所以三个问题都应该是组合问题。由于10个点都在圆周上,因此任意三点都不共线,故只要在10个点中任取2点,就可画出一条线段;在10个点中任取3个点,就可画出一个三角形;在10个点中任取4个点就可画出一个四边形。‎ 全解过程:由组合数公式:‎ ‎ (1),可画出45条线段;‎ ‎ (2),可画120个三角形; ‎ ‎ (3),画210个四边形。‎ 答:可以画出45条线段,120个三角形,210个四边形。‎ ‎8.七个人排成一排照相,其中甲、乙、丙三人必须排在一起,有多少种不同的排法?‎ 初级点拨:首先看是排列还是组合?这道题明显是排列问题,然后你再看所要排列的各个元素之间的关系,利用排列公式就可以了。‎ 深度提示:甲乙丙三人必须排在一起,可以用分类的方法,考虑三人在七个位置中的不同情况,如:‎ 甲 乙 丙 此时甲乙丙占了头三个位置,然后再排其他四个人,最后再考虑甲乙丙三人的顺序,这种方法比较复杂,我们可以换一种方法来考虑这个问题,由于甲乙丙要排在一起,因此我们可以先将这三个人看作一个元素,将这个元素与其他四个元素进行排序,最后将这三个元素排序,用这种方法大大简化了思维过程。‎ 第一步:甲、乙、丙看作一个元素与其他四个元素排列,即五个元素进行排序:‎ ‎;第二步:甲乙丙三个元素排序:‎ 全解过程:不同的排法数有:×=5×4×3×2×1×3×2×1=720。‎ 答,有720种不同的排法。‎ ‎9.(1)把八本书排在上下两格书架上,每格四本,有多少种不同的排法?‎ ‎(2)把八本书放在书架上,上格一本,中格三本,下格四本,有多少种排法?‎ 初级点拨:书放在上层和下层是否相同?弄清楚是排列还是组合?‎ 深度提示:很明显,书放在上层和下层不相同,应该用乘法原理,但每层书的摆放要用排列原理,(1)八本书中先选四本排在第一格,有种排法,再将剩下4本书排在第二格,有种排法;‎ ‎(2)八本书选一本放在上格有种排法,再从剩下的7本中选三本放中格,有种排法,最后四本书放下格,有种方法。‎ 全解过程:根据乘法原理:‎ ‎(1)不同的排法数是×=8×7×6×5×4×3×2×1=40320种;‎ ‎(2)不同的排法是××=8×7×6×5×4×3×2×1=40320种。‎ 答:把八本书排在上下两格书架上,每格四本,有40320种不同的排法;‎ 把八本书放在书架上,上格一本,中格三本,下格四本,有40320种不同的排法。‎ 注意:从上两小题发现,无论放几层,分几本放,结果都是一样的,都是40320,若分成4层呢?是不是还是40320?‎ ‎10.学校乒乓球队有10名男生,8名女生,现要选8人参加区里比赛,在下列的条件下,分别有多少种排法?‎ ‎(1)恰有3名女生入选;‎ ‎(2)至少有2名女生入选;‎ ‎(3)最多有3名女生入选;‎ ‎(4)某2名女生,某2名男生必须入选。‎ 初级点拨:此题是个典型的组合问题,元素之间没有顺序,第⑵、⑶小题中涉及至少至多的问题,一般可分类来解决,而至少有2名女生入选的情况有:2名,3名,4名,5名……8名女生入选,情况较多,因此考虑从全部选法中除去没有女生的选法和恰有1名女生的选法,这种方法称为间接法。‎ 深度提示:(1)先选3名女生:,再从10名男生中选5人:;‎ ‎(2)从全部选法中除去没有女生的选法和恰有1名女生的选法。‎ 全部选法数;‎ 恰有1名女生入选的选法:×。‎ 没有女生入选的选法:‎ ‎(3)分四类,第一类没有女生入选,;‎ 第二类,恰有一女生入选,;×;‎ 第三类,恰有二女生入选;‎ 第四类,恰有三名女生入选。‎ ‎(4)某2名女生,某2名男生必须入选,说明有4人已选定,只须从剩下的14人中再选4人。‎ 全解过程:(1)恰有3名女生入选共有:‎ ‎×=14112(种);‎ ‎(2)至少有2名女生入选的方法:-×-=42753(种);‎ ‎(3)最多有三名女生入选的选法数:‎ ‎+×++=45+960+5880+14112=20997(种);‎ ‎(4)从剩下的14人中再选4人,共有选法=1001(种)。‎ 答:(1)恰有3名女生入选,有14112种选法;‎ ‎(2)至少有2名女生入选,有42753种选法;‎ ‎(3)最多有3名女生入选,有20997种选法;‎ ‎(4)某2名女生,某2名男生必须入选,有1001种选法。‎ 第21讲 容斥原理 一, 知识地图 ‎ 二,基础知识 趣题导引:‎ 有一次,学而思小升初培训部进行数学和英语模拟测试,全体学员的考试成绩统计出来后,周老师在班上向同学报告所有学员的考试情况。‎ 周老师说:“这次考试成绩比上一次有了很大的提高,说明同学们在这一段时间内非常认真地学习了学而思的课程,有我们老师的功劳,但更重要的是你们的努力,希望下一次考试可以更上一层楼。我们全体六年级学员有1106人,其中数学成绩90分以上的有542人,英语成绩90分以上的有479人,数学和英语成绩都考90分以上的有256人,数学和英语成绩都在90分以下的有350人,希望这部分同学可以奋起直追,加倍努力,争取在下一次考试中也都可以拿到90分以上的好成绩。”‎ 周老师的话刚说完,其中一个同学小明就举手说:“老师,您的统计数据有问题,至少有一个人数是不对的。”周老师很从容的回答说:“没错,小明同学说得很对,确实有一个数据我故意说错的,就看大家能不能反应出来,你们知道是为什么吗?”于是大家都热烈讨论了起来,同学们,你们知道小明是如何很快又肯定的说有一个数据老师说错了吗?要想知道答案,先学好下面的内容了!‎ ‎(一)容斥原理介绍 本章节的主要内容是解决涉及包含与排除关系的计算题与应用题,运用到的一个基本原理称为容斥原理,下面我们将容斥原理的内容介绍给大家,由于容斥原理中涉及的各部分之间的关系非常的微妙,希望同学可以仔细学习,细心体会。‎ ‎1、两者容斥原理图形与公式 首先讨论两者之间的包含与排除关系,我们先来看一个例子,‎ 一个班级上有同学数学及格,有同学语文及格,有同学两门功 课都及格,还有同学两门功课都不及格,那么它们的人数关系 可以用右图表示:‎ 从图中我们可以看出它们之间的关系,即:‎ ‎(数学及格人数+语文及格人数-都及格人数)+都不及格人数=全班人数 大家知道为什么要减都及格人数吗?‎ 这是因为数学及格人数和语文及格人数里面都包含了两门都及格 的人数,这样两门都及格人数就重复计算两次,所以要减去一次。‎ 总结为一般规律,我们可以用公式来表示:‎ S=(A+B-A∩B)+D=A∪B +D ‎(字母含义:A∩B-既属于A、又属于B的元素;‎ A∪B-属于A,B中任何一个的元素;‎ D -既不属于A、又不属于B的元素。)‎ 这就是二者容斥原理的一般表达形式,图形表达(我们称之为韦恩图)如图所示。‎ ‎ ‎ ‎2、三者容斥原理图形与公式 三者的容斥原理韦恩图如图所示,我们可以总结出三者容斥原理 的一般公式形式为:‎ S=(A+B+C-A∩B-B∩C-C∩A+A∩B∩C)+D=A∪B∪C+D ‎(字母含义: S-全部元素 ‎ A∩B-既属于A、又属于B的元素;‎ ‎ B∩C-既属于B、又属于C的元素;‎ ‎ C∩A-既属于C、又属于A的元素;‎ ‎ A∩B∩C-既属于A、又属于B、又属于C的元素;‎ ‎ D -既不属于A、又不属于B、也不属于C的元素;‎ A∪B∪C-属于A,B,C的全部元素。)‎ 讨论:A∩B,B∩C,C∩A同时包含于两个集合里面,各多加一次,所以应该各减去一次,那么为什么还要再加上A∩B∩C呢?‎ 原来这是因为A,B,C,A∩B,B∩C,C∩A六个集合里面都包含了A∩B∩C,所以A∩B∩C被加了三次,又减了三次,这样就相当于没加没减,所以应该再加上一次A∩B∩C。‎ 上面两个韦恩图与两个公式就是容斥原理的全部内容,希望同学们可以结合图形与公式进行记忆。‎ ‎(二)容斥原理考察题型的一般形式 ‎1、基本计算题型:‎ ‎(1)求全部元素S,直接套用公式。‎ ‎(2)求属于三集合元素项:A∪B∪C = A+B+C-A∩B-B∩C-C∩A+ A∩B∩C ‎ ‎(3)求不属于三集合元素项D,可以利用两种公式变形:‎ D=S-(A+B-A∩B)(两者)‎ D=S-(A+B+C-A∩B-B∩C-C∩A+A∩B∩C)(三者)‎ ‎(4)求一集合元素或二集合元素中的一项(如A,A∩B等),利用公式变形比较麻烦,我们采用方程法:设所求部分为x,利用公式原型列出方程,解出未知数。‎ ‎(5)多项未知,则必须用方程法,设其中一项为x,表示出其他各项,利用公式原型列出方程,解出未知数。‎ ‎(6)求其他间接项,一般先求出相关部分项,再进行组合与排除。一般可能有以下几种情况:‎ ‎1,只属于某集合元素。如:只属于A元素=A-A∩B-C∩A+A∩B∩C;‎ ‎2,只属于一集合全部元素=A+B+C-A∩B×2-B∩C×2-C∩A×2+A∩B∩C×3;‎ ‎3,只属于某二集合元素:如只属于A,B元素=A∩B-A∩B∩C;‎ ‎4,只属于二集合全部元素=A∩B+B∩C+C∩A-A∩B∩C×3;‎ ‎5,至少属于二集合全部元素=A∩B+B∩C+C∩A-A∩B∩C×2 等。‎ 同学们可以自己结合韦恩图进行总结。‎ ‎2、与其他知识点相结合题型 ‎(1)与图形相结合求总面积:逐一求出公式中对应各部分面积,套用公式或列出方程。‎ ‎(2)与数论中倍数知识相结合:详见例题5。‎ ‎(3)与分数知识相结合,可转化为(2)中题型,详见例题6。‎ ‎(4)与数论中平方,立方数知识相结合,利用估算法,详见拓展训练2。‎ ‎(5)与最值问题相结合:公式中某项的最大最小问题,采用方程分析法和极端假设法,详见例题8,9。‎ ‎3、应用题型 ‎(1)关于电灯开关题型:电灯被拉奇数次改变开关状态,被拉偶数次不改变开关状态。弄清楚所要求项的具体含义及所对应韦恩图部分,先求出公式中相关项,利用间接法求出,请结合题型1(6)中公式进行。详见例题7。‎ ‎(2)关于报数转身题型:转身奇数次改变方向,转身偶数次不改变方向。与电灯开关题型思路一致。‎ ‎4、其他题型 当题目中不涉及重复部分的计算或者已知条件不是容斥原理中相关项时,可能不需要利用容斥原理公式进行计算,但必须画出韦恩图或者自己设计表格进行分析,必须让各部分关系直观清晰,详见拓展训练9。如一种常用的的表格形式为:‎ 男生 女生 总数 一班 ‎ 一班男生 一班女生 一班总数 二班 二班男生 二班女生 二班总数 三班 三班男生 三班女生 三班总数 总数 ‎ 男生总数 ‎ 女生总数 ‎ 所有总数 趣题解析:‎ 学完容斥原理的知识后,同学们是不是可以解释开篇趣题中小明的问题了呢?原来小明在周老师报完四个人数后很快的心算了一下,发现不符合容斥原理的数量关系,所以才说数据有问题。根据容斥原理的公式,全体学员人数应该是(542+479-256)+350=1115,而周老师又说全体学员人数为1106人,所以前后矛盾,其中肯定至少有一个数据是不对的,同学们,你们学会了吗?如果觉得很有意思,就继续往下做题吧!‎ 三、经典透析 ‎【例1】(☆☆☆)志诚中学5年级有200名学生踊跃申报学而思各学科培训班,已知申报奥数班的学生有140人,申报英语班的学生有120人,申报科技班的学生有60人,参加奥数和英语班的学生有60人,申报奥数和科技班的学生有40人,申报英语班和科技班的学生有30人,那么有多少人三个班都报了?‎ 审题要点:‎ 此题为涉及三者关系的容斥原理典型题型,大家先复习一下三者关系容斥原理的“韦恩图”与计算公式,根据条件对应逐一填入,然后直接运用公式将未知求出。‎ 详解过程:‎ 解:画出韦恩图,将相应人数填入,只有三个班都报的同学未知,设 为x人,根据容斥原理公式列出方程:140+120+60-60-40-30+x=200,‎ 解出x=10,所以共有10人三个班都报了。‎ 专家点评:‎ 此题中由于是三个班都报的同学未知,所以也可以不列方程,将公式变形用算术方法直接算出,200-(140+120+60-60-40-30)=10(人)。差别不大,同学们注意体会其中关系。‎ ‎【例2】(☆☆☆)火星小学四年级有45人参加了慰问坚守在青年宫、防洪纪念塔、九站三个地段抗洪的解放军叔叔的活动,去过青年宫慰问的有19人,去过防洪纪念塔的有18人,去过九站的有16人;去过青年宫、防洪纪念塔两处的有7人,去过青年宫、九站两处的有6人,去过防洪纪念塔、九站两处的有5人;有3人三处都去过;其余的在校准备慰问品,请问准备慰问品的有多少人?‎ 审题要点:‎ 此题也为涉及三者关系的容斥原理典型题型,题目中未知数为没有参加任何一项慰问活动的同学,可以列方程算出,也可以直接对公式变形用算术法算出。‎ 详解过程:‎ 解:方程法:画出韦恩图,将相应人数填入,设在校准备慰问品的人数 为x,根据公式直接列出方程为(19+18+‎16-7-6‎-5+3)+x=45,解出x=7,‎ 所以准备慰问品的人数为7人。‎ 算术法:直接将公式变形,可求得准备慰问品人数为 ‎45-(19+18+‎16-7-6‎-5+3)=7(人)。‎ 专家点评:‎ 对于求三者都参加或者三者都不参加的部分,用方程法与算术法差别不大,但是对于求只参加一项或参加两项的部分,建议最好用方程,因为在公式变形中很容易出现符号的变形错误,请看例题3。‎ ‎【例3】(☆☆☆☆)某校五年级有120名学生,订《故事大王》的有85人,订《儿童漫画》的有90人,订《优秀作文选》的有70人,同时订《故事大王》和《优秀作文选》的有62人,同时订《儿童漫画》和《优秀作文选》的有46人,同时订这三种杂志的有21人,此外,还有5名学生没有订任何杂志,问:恰好只订了《故事大王》和《儿童漫画》的有多少人? ‎ 审题要点:‎ 此题与前两题类型相同,也为涉及三者关系的容斥原理典型题型,题目中未知数为只订《故事大王》和《儿童漫画》的人数,所以建议使用方程法解出。注意题目中要求的是只订《故事大王》和《儿童漫画》的人数,而不是订了《故事大王》和《儿童漫画》的人数,所以应该先求出订了《故事大王》和《儿童漫画》的人数,然后再减去三项都订的人数即可。‎ 详解过程:‎ 解:设同时订《故事大王》和《儿童漫画》的有x人,根据 公式原型列出方程为:‎ ‎120-85-90-70+62+46+x-21=5,解得x=43‎ 所以,只订《故事大王》和《儿童漫画》的人数为43-21=22人。‎ 专家点评:‎ ‎1,此题中未知的是参加两项的人数,所以不易用将公式变形用算术方法求出,而应该列出方程,这样数据关系更清晰。‎ ‎2,注意题目要求的是只订《故事大王》和《儿童漫画》的人数,这不是容斥原理公式中涉及的部分,所以不能直接求出,而应该根据韦恩图中的关系间接求出。‎ ‎【例4】(☆☆☆☆‎ ‎)五年级三班有46名学生参加三项课外活动,其中24人参加了绘画小组,20人参加了合唱小组,参加朗诵小组的人数是既参加绘画小组又参加朗诵小组人数的3.5倍,又是三项活动都参加人数的7倍,既参加朗诵小组又参加合唱小组的人数相当于三项都参加人数的2倍,既参加绘画小组又参加合唱小组的有10人,求参加朗诵小组的人数。‎ 审题要点:‎ 此题与前三题类型相同,也为涉及三者关系的容斥原理典型题型,题目中不止一项未知数,但是各项未知数之间具有倍数关系,所以应该用方程法解答。‎ 详解过程:‎ 解:观察发现三项都参加的人数最少,所以设其为x,那么参加朗诵小组的 人数为7x,既参加绘画小组又参加朗诵小组人数为2x,既参加朗诵小组又 参加合唱小组的人数也为2x,画出韦恩图,根据公式列出方程:‎ ‎46= 24+20+7x-2x-2x-10+x解出x=3‎ 所以参加朗诵小组的人数为7x=21人。‎ 专家点评:‎ ‎1,此题不能直接利用公式变形算术方法算出,必须列出方程。‎ ‎2,列方程解应用题时x的选择并不一定是题目所求,而应该是所有未知量中较小值,这样其他未知量就比较好表示出,否则方程中出现分数或除法,不利于方程的解答。‎ ‎【例5】(☆☆☆☆)在1到2004的所有自然数中,既不是2的倍数,也不是3、5的倍数的数有多少个?‎ 审题要点:‎ 此题为容斥原理与数论知识相结合的一类典型题型,首先要纠正一种非常错误的方法:即用全部数分别减去2,3和5的倍数个数即得到答案。错误原因是2,3,5的倍数之中有很多重复的部分,所以应该利用容斥原理的基本原理,画出韦恩图,根据公式计算。‎ 详解过程:‎ 解:1,画出韦恩图,首先计算各集合部分的个数:‎ ‎2的倍数个数:2004÷2=1002,‎ ‎3的倍数个数:2004÷3=668,‎ ‎5的倍数个数:2004÷5=400…4,‎ ‎ 2,再计算各重复部分的个数:‎ 同时是2和3的倍数,即是6的倍数个数:2004÷6=334‎ ‎ 同时是2和5的倍数,即是10的倍数个数:2004÷10=200…4‎ 同时是3和5的倍数,即是15的倍数个数:2004÷15=133…9‎ 同时是2,3,5的倍数,即是30的倍数个数:2004÷30=66…24‎ ‎ 3,根据公式变形可得不是2、3、5的倍数个数为:2004-(1002+668+400-334-200-133+66)=535。‎ 专家点评:‎ ‎1,由于有些数可能同时是2,3,5的倍数,所以应该考虑联系到容斥原理的运用,因为容斥原理的主要功能就是解决有关重复内容的原理。‎ ‎2,同时是几个数的倍数,那么就是这几个数的最小公倍数的倍数。‎ ‎【例6】(☆☆☆☆)分母是385的最简真分数有多少个? ‎ 审题要点:‎ 由于分母385=5×7×11,要求分数为最简真分数,所以分子不能是5、7、11的倍数。此题即转化为求1~385中不是5、7、11的倍数的个数,和上一题完全类似,采取一样的解题思路与步骤。‎ 详解过程:‎ 解:1,画出韦恩图,首先计算各部分的个数:‎ ‎5的倍数个数:385÷5=77‎ ‎7的倍数个数:385÷7=55‎ ‎11的倍数个数:385÷11=35‎ ‎ 同时是5和7的倍数即是35的倍数个数:385÷35=11‎ ‎ 同时是5和11的倍数即是55的倍数个数:385÷55=7‎ 同时是7和11的倍数即是77的倍数个数:385÷77=5‎ 同时是5,7,11的倍数即是385的倍数个数:385÷385=1‎ ‎2,根据公式变形,可得不是5、7、11的倍数个数为:385-(77+55+‎35-11-7‎-5+1)=240。‎ 所以分母是385的最简真分数有240个。‎ 专家点评:‎ 此题也为容斥原理与数论知识相结合的一类典型题型,首先需要了解的知识点是最简真分数的特点,即分子与分母必须为互质关系,也即是分子不能是分母质因数的倍数,得出解题思路。‎ 另外,本题还可以用中国剩余定理和乘法原理来解决。‎ 需要寻找1385中不被5、7、11中任何一个数整除的个数。‎ 被5除的余数有1~4,共4种;‎ 被7除的余数有1~6,共6种;‎ 被11除的余数有1~10,共10种;‎ 根据中国剩余定理,对于任何一种余数组合,1~385中必存在唯一的数满足。所以,根据乘法原理,1-385中不被5、7、11中任何一个数整除的个数为4×6×10=240。‎ ‎【例7】(☆☆☆☆☆)有2000盏亮着的电灯,各有一个拉线开关控制着,现按其顺序编号为1,2,3,…,2000,然后将编号为2的倍数的灯线拉一下,再将编号为3的倍数的灯线拉一下,最后将编号为5的倍数的灯线拉一下,三次拉完后,亮着的灯有多少盏? ‎ 审题要点:‎ 此题与前两题一样,涉及数论中倍数关系的容斥原理应用题型,解题思路完全一样,但是要注意最后要求的部分不一样,必须利用间接计算法进行简单的分解与组合。‎ 详解过程:‎ 解:1、首先分析最后要求的部分含义是什么。‎ 电灯原来亮着,要求三次拉完之后还是亮着,则灯被拉的次数必须 为偶数,即可能是一次都没被拉,也可能是只被拉两次的。所以最 后的答案应该是两部分之和。‎ ‎2、进行各部分的计算:‎ ‎2的倍数个数:2000÷2=1000;‎ ‎3的倍数个数:2000÷3=666…2;‎ ‎5的倍数个数:2000÷5=400;‎ ‎6的倍数个数:2000÷6=333…2;‎ ‎10的倍数个数:2000÷10=200;‎ ‎15的倍数个数:2000÷15=133…5;‎ ‎30的倍数个数:2000÷30=66…20。‎ ‎3、根据公式变形,可得没被拉一次的电灯盏数(不是2、3、5的倍数)为:‎ ‎2000-(1000+666+400-333-200-133+66)=534。‎ ‎ 只被拉两次的电灯盏数(只是其中两者的倍数)为:333+200+133-66×3=468。‎ 所以最后亮着的灯的盏数为:534+468=1002。‎ 专家点评:‎ ‎1、此题首先要弄清楚是哪些灯最后还是亮着的,主要包括两部分的灯:一次都没拉和只拉了两次的。‎ ‎2、只拉了两次的灯数中不包括三次都拉了的灯数,所以计算时应该减去被拉三次灯数的三倍(因为多计算三次),请参考题型1(6)中的间接计算方法。‎ ‎【例8】(☆☆☆☆☆)图书室有100本书,借阅图书者需要在图书上签名。已知这100本书中有甲、乙、丙签名的分别有33、44和55本,其中同时有甲、乙签名的图书29本,同时有甲、丙签名的图书为25本,同时有乙、丙签名的图书为36本。问这批图书中最少有多少本没有被甲、乙、丙中的任何一人借阅过? ‎ 审题要点:‎ 此题属于容斥原理与最值问题相结合题型,公式中只有两项未知:没被任何人借阅过的和同时被三人借阅过的数目,一项的最值取决于另一项的取值,采用方程法分析。‎ 详解过程:‎ 解:题目中未知项有两项,没被任何人借阅过的和同时被三人借阅过的,分别设为x,y,根据公式列出不定方程为:‎ ‎100=(33+44+55-29-25-36+ y)+x,化简为:x + y=58‎ 要使x值取最少,那么y值应该尽量大,由韦恩图可看出y包含于三集合29,25,36中,所以y的最大值应该是25,此时x=33,即最少有33本没有被甲乙丙中的任何一人借阅过。‎ 专家点评:‎ ‎1,由于只有两项未知数,所以可以用方程法进行分析,如果未知数多于两个,则不宜用方程法,下一题即是此种情况。‎ ‎2,应该用包含的原理得出其中项的最大值或最小值。若A包含B,那么B的最大值为A,A的最小值为B,如:某班数学成绩满分人数为15,那么数学语文成绩均满分的人数最大为15,反之若数学语文成绩均满分的人数为5,那么语文成绩满分的人数最少为5人。‎ ‎【例9】(☆☆☆☆)甲、乙、丙同时给100盆花浇水。已知甲浇了78盆,乙浇了68盆,丙浇了58盆,那么3人都浇过的花最少有多少盆?‎ 审题要点:‎ 此题与上题类似,属于容斥原理与最值问题相结合题型,但是题目已知条件太少,公式中未知项有5项,所以不好直接用方程法分析出最后答案。采用极端假设法进行分析。‎ 详解过程:‎ 解:因为题目所求为3人都浇过的花最少为几盘,那么意思就是我们应该让3人浇过的花尽量分散,即每人尽量不要浇其他人浇过的花,采用极端假设法即假设每人都首先选择浇其他人没浇过的花。‎ 首先考虑甲和乙,甲浇了78盆,没浇100-78=22盆,那么 乙应该先浇甲没浇的22盆,剩下的只能选择甲已经浇过的 ‎68-22=46盆,这样两人都浇过的有46盆,只有一人浇过的 有100-46=54盆。‎ 再考虑丙,丙应该先选择浇只有一个人浇过的54盆,剩下的只能选择两人都浇过的58-54=4盆,这样三人都浇过的为4盆, ‎ 其他盆均为至多两人浇过的。所以,3人都浇过的花最少有4盆。‎ 专家点评:‎ ‎1,题目中所给条件太少,很难用二元方程的常规方法分析,所以选择用极端假设法。‎ ‎2,运用极端假设法时,必须随时满足题目要求的最值条件,这里应该要强调掌握从反面角度考虑问题的思路。不能怎么样,那么我们就应该怎么样;要怎么样,那么我们就不能怎么样。‎ ‎3,满足最值条件的假设结论即是我们要求的最值结论。‎ ‎4,此题也有另外的多元方程分析法。多元方程知识点基础比较好的同学可以参考使用:‎ 另解:如果从整体考虑,三个人一共浇了78+68+58=204(盆)花,如果设被浇次数为1、2、3次的花盆数分别为a、b、c,那么可以得到以下两条等式:‎ ‎②-①×2,得到:。‎ 因为,所以,‎ ‎ 所以被3个人都浇过的花至少有4盆。‎ 四、拓展训练 ‎1、边长为6、5、2的三个正方形,如图所示,求它们的盖住部分的面积。‎ ‎2、在1到1000的自然数中,既不是平方数也不是立方数的数有多少个?‎ ‎3、求在1~100的自然数中不是5的倍数也不是6的倍数的数有多少个?‎ ‎4、某校有28名学生参加市运动会,参加跑步类项目的有15人,参加跳类项目的有13人,参加投掷类项目的有14人,既参加跑又参加跳项目的有4人,既参加跑又参加投掷项目的有6人,既参加跳又参加投掷项目的有5人,三种项目都参加的有2人,试说明:这个报名表有误。‎ ‎5、以1001为分母的最简真分数共有多少个?‎ ‎6、学而思六年级竞赛班有50人,共有三个科技兴趣小组:天文、无线电和计算机,参加天文组的有38人,参加无线电组的有35人,参加计算机组的有31人,既参加天文组又参加无线电组的有29人,既参加天文组又参加计算机组的有28人,既参加无线电又参加计算机组的有26人,三个小组都参加的有24人,试求三个小组都没有参加的人数。‎ ‎7、不超过201的自然数中,至少有两个数字相同的奇数有多少个?‎ ‎8、某科室有12人,其中6人会英语,5人会俄语,5人会日语,有3人既会英语又会俄语,有2人既会俄语又会日语,有2人既会英语又会日语,有1人英、日、俄这三种语言全会,只会一种外语的人比一种外语也不会的人多多少人?‎ ‎9、(1)48人中无弟弟的有38人,有弟弟无妹妹的有8人,无弟弟有妹妹的人数是有弟弟有妹妹人数的2倍,试问:这48人当中是独生子女的有几个?‎ ‎(2)学而思举行各年级学生画展,其中18幅不是六年级的,20幅不是五年级的,现在知道五、六年级共展出22幅画,问:其他年级共展出多少幅画?‎ 8、 如图,5条同样长的线段拼成了一个五角形。如果每条线段上恰有1994个 8、 点被染成红色,那么在这个五角形上红色点最少有多少个?‎ 初级点拨:‎ ‎1、直接利用三者容斥原理的公式进行计算,首先应该分别算出各相应部分面积。‎ ‎2、直接利用二者容斥原理的公式进行计算,首先应该分别算出各相应部分个数。‎ ‎3、此题为两者关系的容斥原理与数论倍数相结合题型,求出各部分个数,利用公式计算。‎ ‎4、可以根据其他数据计算学生总人数,看是否等于已知条件,利用容斥原理公式。‎ ‎5、此题为容斥原理与数论知识的结合考察,先分解质因数1001=7×11×13,所以分子不能是7,11,13的倍数。‎ ‎6、容斥原理基本题型,直接利用公式,可列出方程,也可进行公式变形。‎ ‎7、此题较为复杂,是容斥原理与排列组合知识的综合题型。首先按数位分两大类:两位数与三位数。两位数中:只有5个(11、33、55、77、99)符合条件。三位数的个数必须利用容斥原理公式计算。‎ ‎8、此题为基本计算题,一种外语也不会的人可以直接利用公式变形算出,只会一种外语的人数应该要用间接法求出。‎ 有妹妹 无妹妹 总和 有弟弟 无弟弟 总和 ‎9、(1)此题可以设计表格进行分析。‎ ‎(2)此题属于容斥原理里面的特殊题型,含有否定的已知部分,应该转化为肯定的部分数据,画出韦恩图,然后列出方程进行解答。‎ ‎10、此题属于容斥原理与最值问题相结合题型,由于未知条件仅为红色点总数和重复点总数,应该用方程法分析。‎ 深度提示:‎ ‎1、三正方形面积分别为5×5=25,6×6=36,2×2=4,两两重复部分面积分别为3×3=9,1×2=2,1×2=2,三正方形均重复的部分面积为1×1=1,然后直接套用公式。‎ ‎2、⑴、1-1000中,312<1000<322,所以平方数个数为31。‎ ‎⑵、103=1000,所以立方数个数为10。‎ ‎⑶、即是平方数也是立方数则应该是六次方数,36<1000<46,所以个数为3。‎ 然后直接套用公式。‎ ‎3、5的倍数个数:100÷5=20;6的倍数个数:100÷6=16…4;‎ ‎30的倍数个数:100÷30=3…10。‎ 要求不是5,6的倍数,可利用公式变形。‎ ‎4、直接计算总人数为15+13+‎14-4-6‎-5+2=29人。‎ ‎5、计算各部分个数:7的倍数有143个,11的倍数有91个,13的倍数有77个,7×11的倍数有13个,13×7的倍数有11个,11×13的倍数有7个,只有1001是7、11、13的倍数,套用公式。‎ ‎6、方程法:(38+35+31-29-28-26+24)+x=50‎ ‎7、三位数中的个数计算:‎ 至少两个数字相同的反面是三个数字都不相同,奇数的反面是偶数,所以设计韦恩图为:‎ 可看出外围部分即为所求至少有两个数字相同的奇数。‎ 根据乘法原理:三个数字都不相同的数字个数为9×8=72,偶数有 ‎50个,三个数字都不相同的偶数为5×8=40个,套用公式。‎ ‎8,根据题型1(6)中公式介绍可知只会一种外语的人数为:‎ ‎6+5+5-(3+2+2)×2+1×3=5。‎ 有妹妹 无妹妹 总和 有弟弟 x ‎8‎ 无弟弟 ‎2x ‎38‎ 总和 ‎48‎ ‎9,(1)根据条件将数据填入表格:‎ ⑵ 根据韦恩图,18幅不是六年级的,即A+C=18,20幅不是五 年级的,即B+C=20,又五六年级共22幅图,即A+B=22。联立三方程,‎ 解出未知数。‎ ‎10、设总点数为x,重复点数为y,根据公式有x=1994×5-y,化简为x=9970-y,‎ 要求x为最少,对y进行分析。‎ 全解过程:‎ ‎1、根据三者容斥原理公式,覆盖总面积为25+36+‎4-9-2‎-2+1=53。‎ ‎2、根据公式得不是平方数也不是立方数的个数是:‎ ‎1000-(31+10-3)=962。‎ ‎ 3、公式变形为:100-(20+16-3)=67 即不是5,6倍数的数有67个。‎ ‎4、因为已知条件为28人,所以相互矛盾,报表有误。‎ ‎5、根据公式计算:最简真分数个数为1001-(143+77+‎91-7-13‎-11+1)=720。‎ ‎ 或者:(7-1)×(11-1)×(13-1)=720。‎ ‎6、初级点拨:容斥原理基本题型,可列出方程,也可进行公式变形。‎ 深度提示:方程法:(38+35+31-29-28-26+24)+x=50‎ 全解:解方程得x=5,也可利用公式变形:50-(38+35+31-29-28-26+24)=5人 ‎7、初级点拨:此题较为复杂,是容斥原理与排列组合知识的综合题型。‎ 首先按数位分两大类:两位数与三位数。两位数中,只有5个(11、33、‎ ‎55、77、99)符合要求。‎ 深度提示:三位数中的个数计算:‎ 至少两个数字相同的反面是三个数字都不相同,奇数的反面是偶数,‎ 所以设计韦恩图为:‎ 可看出外围部分即为所求至少有两个数字相同的奇数。‎ 根据乘法原理:三个数字都不相同的数字个数为9×8=72,偶数有50个,三个数字都不相同的偶数为5×8=40个。‎ 全解:根据公式,可知三位数中至少有两个数字相同的奇数为100-(72+50-40)=18。‎ ‎ 所以不超过201的自然数中,至少有两个数字相同的奇数有18+5=23个。‎ ‎8、初级点拨:此题为基本计算题,一种外语也不会的人可以直接利用公式变形算出,只会一种外语的人数应该要用间接法求出:‎ 深度提示:根据题型1(6)中公式介绍可知只会一种外语的人数为:6+5+5-(3+2+2)×2+1×3=5‎ 全解:根据公式变形可知一种外语也不会的人数为12-(6+5+‎5-3-2‎-2+1)=2 ‎ 所以只会一种外语的人比一种外语也不会的人多5-2=3(人)。‎ 另解:用分块计数法如下图可知,只会一种外语的有2+2+1=5人,一种也不会的有2人。所求答案是5-2=3人。‎ 有妹妹 无妹妹 总和 有弟弟 无弟弟 总和 ‎9、(1)初级点拨:此题可以设计表格进行分析。‎ ‎ ‎ 有妹妹 无妹妹 总和 有弟弟 x ‎8‎ 无弟弟 ‎2x ‎38‎ 总和 ‎48‎ 深度提示:根据条件将数据填入表格:‎ 有妹妹 无妹妹 总和 有弟弟 x=2‎ ‎8‎ A=10‎ 无弟弟 ‎2x=4‎ y=34‎ ‎38‎ 总和 ‎6‎ ‎ 42‎ ‎48‎ 全解:根据数据关系将表格补充完整:‎ ‎⑴、A=48-38=10 ⑵、x=A-8=2 ⑶、y=38-2x=34 ‎ y为无弟弟无妹妹即是独生子女,所以独生子女有34个。‎ ‎⑵初级点拨:此题属于容斥原理里面的特殊题型,含有否定的已知部 分,应该转化为肯定的部分数据,画出韦恩图,然后列出方程进行解答。‎ 深度提示:根据韦恩图,18幅不是六年级的,即A+C=18,‎ ‎20幅不是五年级的,即B+C=20,又五六年级共22幅图,‎ 即A+B=22,解出未知数。 ‎ 全解:三式全部相加可得:A+B+C=(18+20+22)÷2=30,所以其他年级的画数为30-22=8。‎ ‎10、初级点拨:此题属于容斥原理与最值问题相结合题型,由于未知条件仅为红色点总数和重复点总数,应该用方程法分析。‎ 深度提示:设总点数为x,重复点数为y,根据公式有x=1994×5-y,化简为x=9970-y,‎ 要求x为最少,对y进行分析。‎ 全解过程:要使x最少,y应该尽量大,五角星中线段与线段之间一共十个点,所以重复点数最多为10个,所以x最少为9970-10=9960,即红色点最少有9960个。‎ 第22讲 抽屉原理 抽屉原理有时也被称为鸽巢原理,它是德国数学家狄利克雷首先明确提出来并用以证明一些数论中的问题,因此,也称为狄利克雷原则。它是组合数学中一个重要而又基本的数学原理,应用它可以解决很多有趣的问题,并且常常能够起到令人惊奇的作用,因为许多看起来相当复杂,甚至无从下手的问题,在利用抽屉原理后,能很快使问题得到解决。那么,这一讲我们就来学习抽屉原理以及它的典型应用。‎ 抽屉原理推广到一般情形有以下两种表现形式。‎ 抽屉原理1:将多于n件的物品任意放到n个抽屉中,那么必有一个抽屉中至少有2件物品。‎ 例:有5只鸽子飞进4个鸽笼里,那么一定有一个鸽笼至少飞进了2只鸽子。‎ 抽屉原理2:将多于m×n件的物品任意放到n个抽屉中,那么必有一个抽屉中至少有m+1件物品。‎ ‎ 注意,要“保证”至少有m+1件物品放在n个抽屉里,物品数最少是m×n+1。‎ 例:如果将13只鸽子放进6只鸽笼里,那么至少有一只笼子要放3只或更多的鸽子。道理很简单。如果每只鸽笼里只放2只鸽子,6只鸽笼共放12只鸽子。剩下的一只鸽子无论放入哪只鸽笼里,总有一只鸽笼放了3只鸽子。‎ Ⅰ、抽屉原理的典型应用 解题思路:做抽屉问题关键是确定 “抽屉”和“苹果”,当题目中出现多个对象时,通常数量较多者为“苹果”,数量较少者为“抽屉”。确定“苹果”和“抽屉”之后,我们可以得出如下这个公式: 苹果÷抽屉=商……余数,得到的结论为:有一个抽屉里至少有(商+1)个苹果。注意,如果没有余数,那么求至少有多少个苹果时,商就不用+1了。这个公式对于求“至少”和“苹果”都有用。‎ 【例1】 证明:(1)任意28个人中,至少有3个人的属相相同。(2)要想保证至少4个人的属相相同,至少有几个人?(3)要想保证至少5个人的属相相同,但不能保证有6个人的属相相同,那么总人数应该在什么范围内?‎ 分析:(1)首先我们来确定苹果和抽屉,苹果比较容易确定,28个人就是28个苹果,抽 屉呢?题中提到了属相,由生活常识我们可知一共有12种属相,也就是有12个抽屉。两 者确定之后,我们套用公式就能解答了。‎ 把12种属相看作12个抽屉,28=2×12+4,根据抽屉原理,至少有2+1=3个人的属相 相同。‎ ‎(2)这道题我们知道了“至少”是4,因此每个抽屉先放3个苹果,要想总人数最少,也就是苹果最少,商和除数已经定下来了,只能让余数最少,也就是余数为1。要保证有至少4个人的属相相同,总人数最少为:3×12+1=37(人)‎ ‎(3)和第(2)小题一样,这道题是已知“至少”要求苹果,而且是要求苹果个数的范围。要 保证有5个人的属相相同,总人数最少为:4×12+1=49(人),不能保证有6个人属相相 同的最多人数为:5×12=60(人),所以总人数应该在49人到60人的范围内。‎ 点评:抽屉原理中最典型的两种类型的题就是求“至少”和求“苹果”,这就要求我们要熟练运用公式:m×n+1,无论顺推反推都运用自如。我们可以尝试下面这道题:‎ 学而思学校有55个同学参加数学竞赛,已知将参赛人任意分成四组,则必有一组的女生至少有3人,又知参赛者中任何10人中必有男生,则参赛男生的人数为多少人?‎ 分析:因为分成四组,必有一组的女生至少有3人,所以女生至少有4×2+1=9(人),因为任意10人中必有男生,所以女生人数也至多9人,所以女生有9人,则男生有55-9=46(人)。‎ Ⅱ、最不利原则 解题思路:有些题目中没有明显的“苹果”与“抽屉”,在解决问题时,需要要从问题的最差状态着手,也就是要考虑最不利的情况,才能满足题目的要求。尤其要注意这类题一般会提到两个词“至少”和“保证”,例如,教室里面现在有6个男生和4个女生共10人,问:至少出去多少人才能保证出去的人中有女生?经常会有人回答:“你不是问至少出去几个人么?既然要至少,那当然一个就行了,正好这个人是女生。这就是最少的答案。”这个答案是不对的,因为他只是考虑了“至少”,却没有考虑“保证”,万一出去的那个人是男生呢?有同学可能又要说了,既然你要“保证”,那么所有的人都出去不就可以保证了么?但是这又没考虑“至少”,所以在“至少”和“保证”中,我们应该优先考虑“保证”,再考虑“至少”,只有在保证的基础上,才去考虑最少的情况。这道题要想保证有女生,应该用最不利原则来考虑,也就是说出去一个不是女生,又出去一个还不是女生……直到所有的男生都出去了,再出去一个就是女生了,所以至少出去6+1=7个人,才能保证里面肯定有女生。‎ ‎【例2】一副扑克牌,共54张,问:至少从中摸出多少张牌才能保证:(1)至少有5张牌的花色相同;(2)四种花色的牌都有;(3)至少有3张牌是红桃。(4)至少从中取出几张牌,才能保证至少有2张梅花牌和3张红桃。‎ 分析:首先一定要弄清楚一副扑克牌有四种花色,分别是黑桃、红桃、梅花、方块,每种花色各13张,另外还有两张王牌,共54张。‎ ‎(1)为了“保证”5张牌花色相同,我们应从最“坏”的情况去分析,即先摸出了两张王牌。把四种花色看作4个抽屉,要想有5张牌属于同一抽屉,只需再摸出4×4+1=17(张),也就是共摸出19张牌。即至少摸出19张牌,才能保证其中有5张牌的花色相同。‎ ‎(2)因为每种花色有13张牌,若考虑最“坏”的情况,即摸出了2张王牌和三种花色的所有牌共计13×3+2=41(张),这时,只需再摸一张即一共42张牌,就保证四种花色的牌都有了。即至少摸出42张牌才能保证四种花色的牌都有。‎ ‎(3)最坏的情形是先摸出了2张王牌和方块、黑桃、梅花三种花色所有牌共计13×3+2=41张,只剩红桃牌。这时只需再摸3张,就保证有3张牌是红桃了。即至少摸出44张牌,才能保证其中至少有3张红桃牌。‎ ‎(4)因为每种花色有13张牌。,若考虑最“坏”的情况,即摸出2张王牌、方块和黑桃两种花色的所有牌共计:13×2+2=28,然后是摸出所有的梅花和3张红桃(想想若摸出所有的红桃和2张梅花,是最坏的情况么?请注意,这种想法是想当然的,因为,“按照这个数字”去凑最“坏”的情况,可能是摸出了所有的梅花,再摸到2张红桃。),共计:28+13+3=44张;‎ 点评:最不利原则中,扑克牌这种类型的题考的比较多,但是有一些同学没认真玩过,不了解扑克牌的常识数据,这样就没法做题了,也有些题目会考到象棋,国际象棋的常识以及围棋的棋盘,如果不了解的话这种题就没法做了,所以要注意生活中的积累,棋类常识知道一些是有必要的。‎ ‎【例3】有10把钥匙,去开10扇门,但是不知道哪把钥匙和哪个门是匹配的,问:至少要开多少次,才能把所有的钥匙和门匹配上?‎ 分析:要保证所有的钥匙和门都能匹配上,我们用最不利原则来考虑,最不利的情况是,第一把钥匙开一个门,打不开,又开了一个,还是打不开……那么开多少次才能保证打开一扇门呢?有同学说那不就是10次么?其实只要9次就可以了,因为如果9次都没打开,那么第10次就不用试了,肯定就是第10扇门了。那么第2把钥匙呢?第3把呢?‎ 通过分析,我们可以知道,至少要试9+8+7+6+5+4+3+2+1=45次。‎ 点评:这道题很容易错成只开10次就行了,原因有两个,一是没有优先考虑保证,只考虑了最少,另一个就是当前九次都打不开的时候,第10次就不用试了,我们在后面的题中要注意这些问题。‎ ‎【例4】口袋中有三种颜色的筷子各10根,问:‎ ‎  (1)至少取多少根才能保证三种颜色都取到?‎ ‎  (2)至少取多少根才能保证有2双颜色不同的筷子?‎ ‎  (3)至少取多少根才能保证有2双颜色相同的筷子?‎ 分析:这种题也是比较典型的最不利原则的题,主要难度是不知道什么情况是最不利的情况,比如第1小题,要保证三种颜色都取到,那么最不利的情况是什么呢?应该是尽量多取颜色相同的,这样,我们就可以把某一种颜色全部取完,然后再把某一种颜色全部取完,这样只需要再取一根筷子就可以保证有三种颜色了。‎ ‎(1)最坏的情况就是两种颜色的筷子都取掉了,还没有取到第三种颜色的,这时只要再取一根就能凑足3种颜色,所以至少取20+1=21(根)筷子。‎ ‎(2)最坏的情况是其中一种颜色的筷子都取到了,此外其它两种颜色的筷子各取了1根,这时只要再取一根,所以至少应该取10+2+1=13(根)筷子。‎ ‎(3)最坏的情况是每种颜色的筷子都取了3根,这时只要再取一根就能保证有2双颜色相同的筷子。‎ 至少要取3×3+1=10(根)筷子。‎ 点评:在最不利原则中,有一个难点就是不知道什么是最不利的情况,或者你考虑的情况并不是“最”不利的,这需要我们全面的考虑,尽量考虑“最不利”的情况。‎ ‎【例5】两个布袋各有12个大小一样的小球,且都是红、白、蓝各4个。从第一袋中拿出尽可能少的球,但至少有两种颜色一样的放入第二袋中;再从第二袋中拿出尽可能少的球放入第一袋中,使第一袋中每种颜色的球不少于3个。这时,两袋中各有多少个球?‎ 分析:这道题如果我们从正面分析,无论是第一次还是第二次的情况都很复杂,不容易考虑。那么我们可不可以反向考虑一下呢?也就是说我们从后往前推,当第二次取完之后,第一袋的球都保证了至少有三个,那么这个时候的最不利的情况应该是什么呢?是每种球都有3个么?不是,最不利的情况是,某两种球全都拿到第一袋中,而第三种还只有2个,这时候如果再拿一个,就能保证每种球都至少有3个了。‎ 第一次取完后,只需知道第一袋中有某种颜色的球不足3个即可(取了多少个球,怎样取的都可以不考虑)。第二次取后,要保证第一袋中每种颜色的球不少于3个,最不利的情况是两种颜色的球各有8个,另一种颜色的球有3个。所以,第一袋中有球8+8+3=19(个),第二袋中有球4×3×2-19=5(个)。‎ ‎ ‎ 点评:有的时候如果正面考虑情况太复杂,我们不妨从反面考虑一下,也许有意想不到的效果。‎ Ⅲ、构造抽屉解决问题 抽屉原理中比较难的题目一般都不会直接告诉我们什么是抽屉,或者是抽屉的数量,抽屉不太容易被发现,这个时候需要我们来构造抽屉,常见的方法有利用数论、几何等知识来构造抽屉,当然对于具体的题目还要灵活考虑。‎ ‎【例6】在任意的五个自然数中,是否其中必有三个数的和是3的倍数?‎ 分析:这道题看起来好象和抽屉原理没什么关系,因为没有明显的抽屉。因此我们需要自己来构造抽屉。我们先从余数的角度来考虑一下:任何整数除以3的余数只能是0,1,2。这三种情况能不能看成是三个抽屉呢?对于任意的五个自然数,根据抽屉原理,至少有一个抽屉里有两个或两个以上的数,于是可分下面两种情形来加以讨论。‎ ‎  第一种情形:有三个数在同一个抽屉里,即这三个数除以3后具有相同的余数,设为A。因为这三个数的余数之和是‎3A,故能被3整除,所以这三个数之和能被3整除。‎ ‎  第二种情形:至多有两个数在同一个抽屉里,那么每个抽屉里都有数,在每个抽屉里各取一个数,这三个数被3除的余数分别为0,1,2。因此这三个数之和能被3整除。‎ 综上所述,在任意的五个自然数中,其中必有三个数的和是3的倍数。‎ 点评:用余数来构造抽屉是一个常用的构造抽屉的方式。我们再看看下面这道题。‎ ‎【例7】证明:任意12个两位数,其中必有两个数的差是个位与十位数字相同的两位数。‎ 分析:什么叫个位与十位数字相同的两位数?比如22,55,99,很明显,这些数都是11的倍数,那么我们可以把这道题理解为:证明任意12个两位数,其中必有两个数的差是11的倍数。‎ 什么样的两位数的差是11的倍数?我们来举几个例子:35-13=22,86-31=55,93-60=33。好,现在我们再研究一下这些数除以11的余数,35和13除以11都余2,86和31除以11都余8,93和60除以11都余5。我们应该发现了,差是11的倍数的两个数除以11的余数是相同的,那我们能不能把这道题理解为:证明任意12个两位数,其中必有两个数除以11的余数相同?‎ 说到这里,这道题就很容易了,一个数除以11的余数只有0~10共11种情况,我们把12个数看成12个苹果,余数的11种情况看成11个抽屉,由抽屉原理得必有两个数除以11的余数相同,则这两个数的差是个位与十位数字相同的两位数。‎ ‎【例8】能否在10行10列的方格表的每个空格中分别填上1,2,3这3个数之一,而使大正方形的每行,每列及对角线上的各个数字和互不相同?对你的结论加以说明。‎ 分析:很多同学拿起这道题就开始在方格表里填数,填来填去发现填不出来,然后就说不能办到,至于为什么不能办到,也说不出个所以然来,更有甚者居然说填出来了,然后才发现是自己的计算错误。确实,10行10列计算起来计算量很大,而且变化很多,比较容易出错,所以如果是想填出一种方案来是很困难的。那我们试着用抽屉原理来解决一下。‎ 首先是要构造抽屉,10个数的和最小的情况是每个方格均填“1”,则十个数字和最小是10;‎ ‎ 10个数的和最大的情况是每个方格均填“‎3”‎,则十个数字和最大是30。‎ ‎ 因为从10到30之间只有21个互不相同的整数值,把这21个互不相同的值作为21个“抽屉”,而10行、10列及两条对角线上的各个数字和共有22个整数值,可以看作22个苹果,这样的苹果的个数比抽屉的个数多1个,根据抽屉原理可知,至少有两个数值同属于一个抽屉,即要使大正方形的每行、每列及对角线上的各个数字和互不相同是不可能的。‎ 【例3】 在边长为1的正方形内,任意放入9个点,则其中必有3个点,它们构成 的三角形面积不大于。‎ 分析:这是一道平面几何与抽屉原理相结合的题,一般对于这种题,我们可以用反推法来构造抽屉。“9个点”可以看成是9个苹果,“3个点”可以看成是“至少”,这样我们可以求出抽屉的个数为4,也就是要把大正方形分成四块。有因为最后三角形的面积不大于,则与它等底等高的四边形的面积不大于,因此我们正好可以把大正方形分成4块面积为的小正方形。这4个小正方形就是4个抽屉。‎ 如图将正方形分割为相同的四块,由抽屉原理得必然有三个点在同一个小正方形中,则这三个点构成一个面积小于的三角形。‎ 点评:在几何与抽屉原理相结合的题中,构造抽屉很困难,我们可以从题中的结论反推,求出抽屉的个数,然后有目的来构造抽屉。‎ 拓展训练 ‎1.用红、蓝两种颜色将一个2×5方格图中的小方格随意涂色(见右图),每个小方格涂一种颜色。是否存在两列,它们的小方格中涂的颜色完全相同?‎ ‎2.一副扑克牌,共54张,问:至少从中摸出多少张牌才能保证 ‎(1)至少从中取出几张牌,才能保证至少有5张梅花牌3张红桃和2张黑桃。‎ ‎(2)至少从中取出几张牌,才能保证至少有2张牌的数码(或字母)相同。‎ ‎3.要把61个乒乓球分装在若干个乒乓球盒中每个盒子最多可以装5个乒乓球,问:至少有多少个盒子中的乒乓球数目相同?‎ ‎4.在边长为‎3米的正方形中,任意放入28个点,求证:必定有四个点,以它们为顶点的四边形的面积不超过‎1平方米。‎ ‎5.用数字1,2,3,4,5,6填满一个6×6的方格表,如右图所示,每个 小方格只填其中一个数字,将每个2×2正方格内的四个数字的和称为这 个2×2正方格的“标示数”,问:能否给出一种填法,使得任意两个“标 示数”均不相同?如果能,请举出一例;如果不能,请说明理由。‎ 6. 奥数网竞赛班选拔考试,共有1123名同学参加,小明说:“至少有10名同学 来自同一个学校。”如果他的说法是正确的,那么最多有多少所学校参加了这次入学考试?‎ ‎7.任意取多少个自然数,才能保证至少有两个数的差是7的倍数?为什么?‎ 8. 口袋中装有10种不同颜色的珠子,每种都是100个。要想保证从袋中摸出3种不同颜色的珠子,并且每种至少10个,那么至少要摸出多少个珠子?‎ 9. 在1、4、7、10、…、100中任选20个数,其中至少有不同的两组数,其和都等于104,试证明这一结论。‎ 10. 平面上有17个点,两两连线,每条线段染红、黄、蓝三种颜色中的一种,这些线段能构成若干个三角形。证明:一定有一个三角形三边的颜色相同。‎ 初级点拨:1、什么样的情况叫两列的颜色相同?‎ ‎2、这是一道最不利原则的题,要考虑最不利的情况。‎ ‎3、用最不利原则来考虑。‎ ‎4、这是一道抽屉原理应用于平面几何的题。‎ ‎5、抽屉原理中有一类填数字的题,注意构造抽屉。‎ ‎6、这是一道已知苹果和“至少”,求抽屉的题。‎ ‎7、什么样的数差是7的倍数?观察可以发现,除以7的余数相同的数差是7的倍数。‎ ‎8、请用最不利原则来考虑。‎ ‎9、关键是怎样来构造抽屉,注意是任选20个数,所以抽屉的个数应该少于20。‎ ‎10、本题我们用到的思想——图论(解题并没有涉及图论知识),图论主要就是指这种平面上点与点连线的问题。‎ 深度提示:1、用红、蓝两种颜色给每列中两个小方格随意涂色,只有下面四种情形:‎ ‎2、什么样的情况是最不利的呢?以第1小题为例,最不利的情况应该是摸出2张王牌、所有的方块,然后摸出所有的黑桃与红桃,最后再摸梅花。‎ ‎3、最不利的情况是什么呢?要盒子中的乒乓球数目相同,我们应尽量让它们不同。‎ ‎4、我们知道苹果是28,“至少”是4,由抽屉原理的公式可以推出抽屉应该为9个。‎ ‎5、每个正方格内的数字和从最小到最大共有21种情况,可以视之为21个抽屉。‎ ‎6、1123个苹果,商是10-1=9,那么抽屉该是多少呢?请用公式自己推一下。‎ ‎7、除以7的余数只有7种情况,可以看作是7个抽屉。‎ ‎8、最不利的情况是什么呢?一方面要颜色尽量相同,另一方面还不能超过10个。‎ ‎9、和为104,所以尽量把和是104的数分为一组,以此作为抽屉。‎ ‎10、首先要强调一点,线段构成的三角形是形如左下图,而不是右下图 全解过程:1、用红、蓝两种颜色给每列中两个小方格随意涂色,只有下面四种情形:‎ 将上面的四种情形看成四个“抽屉”。将需要涂色的五列看作苹果,根据抽屉原理,将五列放入四个抽屉,至少有一个抽屉中有不少于两列,这两列的小方格中涂的颜色完全相同。‎ ‎2、(1)因为每种花色有13张牌。若考虑最“坏”的情况,即摸出2张王牌、所有的方块共计:13+2=15,然后是摸出所有的黑桃和红桃(想想若摸出所有的红桃和梅花,是最坏的情况么?)最后再摸出五张,共计:15+13+13+5=46张;‎ ‎(2)最不利的情况是先摸出一张王牌,然后A~K每样一张,再摸出一张就可以保证至少有2张牌的数码(或字母)相同,因此要摸出15张。‎ ‎3、每个盒子超不超过5个球,最“坏”的情况是每个盒子球数:1、2、3、4、5这5种各不相同的个数,共有:1+2+3+4+5=15,61÷15=4……1,最不利的分法是:得1、2、3、4、5个球的各4个,还剩1个球,要使每个盒子不超过5个,无论发给那个盒子,都会使至少有5个盒子的球数相同。‎ ‎4、将大正方形分成9个边长为‎1米的小正方形,则9个小正方形为“抽屉”有:28÷9=3……1,则必有一个小正方形里(上)至少有3+1=4(个)点,若这四个点恰好落在这个小正方形的四个顶点,那么以这4个点为顶点的四边形的面积最大为‎1平方米;若有一个点落在正方形的内部,则面积将小于‎1平方米 ‎,综上所述,不论怎么放,必定有四个点,以它们为顶点的四边形的面积不超过‎1平方米。‎ ‎5、先计算出每个2×2正方格内的四个数字的和最小为4,最大为24,从4到24共有21个不同的值,即有21个“抽屉”;再找出在6×6的方格表最多有:5×5=25(个)2×2正方格的“标示数”,即有25个“苹果”。根据抽屉原理:25÷21=1……4,必有两个“标示数”相同。‎ ‎6、本题需要求抽屉的数量,反用抽屉原理和最“坏”情况的结合,最坏的情况是只有10个同学来自同一所学校,而其他学校都只有9同学参加,则人数最多为:(1123-10)÷9=123……6,因此这个班最多有:123+1=124(所)学校(处理余数很关键,如果有125所学校则不能保证至少有10名同学来自同一个学校)。‎ ‎7、因为任何整数除以7,其余数只可能是0、1、2、3、4、5、6七种情形。我们将余数的这七种情形看成七个“抽屉”。一个整数除以7的余数属于哪种情形,就将此整数放在那个“抽屉”里。要想两个数的差是7的倍数,这两个数必同余,所以至少有一个抽屉里有两个数,故最少任取7+1=8个自然数才能保证至少有两个数的差是7的倍数。‎ ‎8、最坏的情况是摸了2种颜色各100个,其它颜色各9个,这时再摸一个就能摸出条件所要求的。‎ 所以至少要摸2×100+8×9+1=273个。‎ ‎9、1、4、7、10、…、100共有34个数,将其分为{4,100},{7,97},…,{49,55},{1},{52}共有18个抽屉。 从这18个抽屉里面任意抽取20个数,若取到{1},{52},则剩下的18个数取自前16个抽屉,至少有4个数取自某两个抽屉中。 若不全取1和52,则有多于18个数取自前16个抽屉,同样至少有4个数取自某两个抽屉中,而属于同一“抽屉”的两个数,其和是104。‎ 8、 我们再来看题目,因为17个点两两连线都画出来是136条线段,再找 三角形无从下手,我们不妨先拿一个点出来研究一下:如图(以A点为例),从 A点出发可以连16条线,根据抽屉原理,同色的线段至少有6条,我们设为红色:‎ ① B、C、D、E、F、G6个点之间只要有1条红线,就有1个三角形符合条件,得证;‎ ② B、C、D、E、F、G 6个点之间没有1条红线,也就是全为黄、蓝色,那么BC、BD、BE、BF、BG中至少3条同色,我们设BC、BD、、BE黄色;‎ ③ 再看C、D、E,三点之间有1条黄线,就有黄色三角形,得证;或者三点之 间全为蓝线,就有蓝色三角形,得证。‎ 综上所述,一定存在一个三角形满足题目要求。‎ 第23讲 逻辑推理 ‎ 一, 知识地图 ‎ 二,基础知识 趣题导引:‎ 古时候有一个国王非常的残忍,喜欢拿国民的生命开玩笑。有一天,他又想到一个残酷的游戏,他每天都随便抓一个国民跟他说一句话,预测一下明天将要发生的事情。如果预测错了,那么这个国民就将被处死;如果预测对了,那么这个国民就将得到很多的奖赏。所以每一个被抓去问话的国民都非常的害怕,因为谁也无法保证自己的预言会实现。有一次,一个非常聪明的年轻人被抓到问话,他灵机一动,想到一个办法为难一下这个国王。他到国王面前就说了一句话:“明天我将被处死。”同学想一想,为什么这句话可以把国王给难住?‎ ‎(一)逻辑推理介绍 逻辑推理题型是历届杯赛与小升初考试中比较常见的题型,因为对同学的逻辑思维能力要求比较高,要求同学同时具备分别从局部细节与整体综合方面考察问题的多面性,同时强调信息的挖掘,对应的推理,真假的判断等,‎ 这章节的知识整体来说属于中等偏上,但趣味性很强,希望同学们能够愉快的学完本章节的内容。‎ 逻辑推理是一门科学,称为逻辑学,同学们以后可能会陆续接触到一些相关方面的知识。这里我们将比较基本的几个知识点介绍给大家。‎ ‎(二)逻辑两大基本原理介绍 ‎1、排他性原理 排他性原理:一个事物如果确定是属于一种类型的东西,那么它就不可能属于另外一种和此种类型相矛盾的类型。例如,一个帽子是红色的,那么就可以确定这个帽子不是黑色或白色的,当然也不可能是其他任何颜色的。在逻辑推理里面这种原理的应用可以称为排除法。‎ ‎2、互补性原理 互补性原理:如果一个事物确定属于几个类型里面的一种,又可以知道这个事物不属于其中一种类型之外的其他所有类型,那么就可以确定这个事物就是这种类型。例如:已知一个帽子可能有四种颜色:红色,黑色,白色与蓝色。又知道这个帽子不是蓝色,不是黑色,也不是红色,那么就可以确定这个帽子是白色的,这个原理经常和排除法一起使用。‎ 当互补性原理里面的事物只有两种可能性时,称这两种类型为完全矛盾型,那么就可以知道这个类型肯定为一真一假,这在逻辑推理里面可以称之为矛盾分析法。例如:一个帽子是白色或红色的。小明说:这个帽子是白色的,小强说:这个帽子是红色的,那么我们就可以确定小明和小强中肯定有一个人说真话,一个人说假话。‎ ‎(三)逻辑两大常用方法介绍 ‎1、假设法 当有好几种可能的情况,但是从已有条件又无法直接判断时,可以考虑用假设法进行推理。利用假设法时注意两个原则:‎ ‎(1) 假设的情况尽量不要太多。例如:已知A、B、C、D四人中有两人说真话,那么我们假设的话就有六种可能性:(1)A、B (2)A、C (3)A、D (4)B、C (5)B、D (6)C、D。可能性太多,所以就应该换另外一种假设的方法。一般来说假设的可能性不应该超过四种。‎ ‎(2) 假设的条件最好能够直接推导出其他有用信息,所以一般不假设否定的结论,而应该假设肯定的结论。例如:应该假设“小明最喜欢足球”,而不应该假设“小明最喜欢的不是篮球”,因为前者可以直接推理出小明最喜欢的不是其他的运动,而后者不能推理出其他有效的结论。‎ ‎2、列表法 当涉及多者之间的对应关系时,应该设计表格将所有的信息表示出来,下面是常用的表格类型设计:‎ ‎(1)只有两种关系对应时,应用的表格一般是:‎ ‎ 足球 篮球 乒乓球 羽毛球 小明 ‎×可以确定 小华 ‎√ 已知 ‎×可以确定 ‎×可以确定 ‎×可以确定 小强 ‎×可以确定 小刚 ‎×可以确定 ‎(2)有三种关系相对应时,一般应用的表格是:‎ 北京 上海 广州 南京 足球 篮球 乒乓球 羽毛球 小明 ‎×‎ 已知 ‎×‎ 已知 ‎√‎ 可以确定 ‎×‎ 已知 小华 小强 ‎×‎ 已知 ‎√‎ 可以确定 ‎×‎ 已知 ‎√‎ 可以确定 小刚 运用列表法时,应该注意随时运用两个定理::一,排他性原理,;二,互补性原理。具体来说如下:‎ ‎ 一,排他性原理: 当表格中出现一个√时,那么它所在的同行和同列都应该打×。如果一个人同时对应着两项爱好,那么就还必须等另一项爱好也确定了才能运用排他性原理。‎ ‎ 二,互补性原理:当同一行或同一列中除一个表格外,其他全部为×时,那么就可以在这个表格中打√。同样,如果一人对应着两项爱好,那么剩两个空格时就可以全部打上√。‎ ‎(四)体育比赛中的逻辑推理知识点介绍 体育比赛中的逻辑推理,除了要运用上面介绍的几个原理与方法外,还有一些其他重要的原则要利用,主要包括以下几个方面的内容:‎ ‎1、确定比赛的队伍总数,比赛场次总数,得分总数 ‎(1)单循环赛时,比赛场次总数=队数×(队数-1)÷2。‎ ‎(2)双循环赛时,比赛场次总数=队数×(队数-1)。‎ ‎(3)当只有胜负局时,所有队得分总数为确定的,等于比赛场次数×胜局得分(如3分)。‎ ‎(4)当有平局,而平局两队得分总数和胜局一样时,所有队总分也是确定的,等于比赛场次数×胜局得分(例如:胜一局2分,平一局得1分,负一局得0分)。‎ ‎(5)当有平局,但平局两队得分总数和胜局不同时,所有队总分不是确定的。但是有一个最大值和最小值,最大值=比赛场次数×胜局得分,最小值=比赛场次数×平局两队得分总数,具体得分总数取决于平局的场数(例如:胜一局3分,平一局得1分,负一局得0分)。‎ ‎2、确定每队最高得分,最低得分,和所有可能的得分数 ‎(1)最高分=每队比赛场数×胜局得分 ‎(2)最低分=每队比赛场数×负局得分 ‎(3)可能得分数介于最高分与最低分之间,注意题目给出的特殊条件,如奇偶性等。‎ ‎3、注意胜负局与平局的对应关系 ‎(1)所有队的胜局数=所有队的负局数 ‎(2)所有队的平局数=平局场次数×2,所以总平局数必须为偶数 ‎(3)总场次数=所有队的胜局数(或负局数)+所有队的平局数÷2‎ ‎(五)信息挖掘的几种基本形式 ‎1、一个条件中的隐藏信息。如:“王老师经常和足球爱好者与数学老师一起讨论问题”,比较明显的信息可以得知:王老师不是足球爱好者也不是数学老师,但其实还可以推理出一个比较隐藏的信息:足球爱好者和数学老师不是同一个人。‎ ‎2、多个条件中的隐藏信息。‎ 如:条件一:王老师是最年长的;条件二:体育老师比数学老师年龄大,那么就可以推理出一个有效信息:王老师不是数学老师。要挖掘出这类信息,需要把条件中涉及到相关事件的几个放到一起,综合考虑进行推理。‎ ‎3、利用已推理出的信息进行再推理,即循环推理。如:条件说:“王老师不认识体育老师”,“赵老师常和数学老师一起下棋”。本来是两个不相关的条件,但是如果已经确定了赵老师是体育老师后,就可以综合三个条件推理出:王老师不是数学老师。‎ 趣题解析:‎ 学习了逻辑推理的知识之后,同学们应该对开篇的趣题有自己的理解了吧。这个趣题实质上就是一个真假推理问题,因为只有两种可能性:预测对错之分。采用的是假设法。假设年轻人预测对了,那么他明天就应该要被处死,但是根据游戏规则,被处死的应该是预测错误的国民,所以矛盾了。又假设年轻人预测错误,那么他明天就不应该被处死,但是同样根据游戏规则,预测正确才能不被处死,所以也矛盾。也就是说两种假设都不成立,那么就是国王无法判断年轻人的预测,所以就既不能把年轻人处死,又不能给年轻人奖赏,陷入两难。这实际上是一个有关“悖论”的名题,还有很多类似的“悖论”非常的有意思,同学们可以自己去思考一下。‎ 我们从这个例子里面应该学习到逻辑推理题型的一种分析方法,就是假设法。下面我们就来学习其他奥数里面很多的逻辑推理题型。‎ 三、经典透析 ‎【例1】(☆☆☆)某地质学院的三名同学对一种矿石进行分析。‎ ‎ 甲判断:不是铁,不是铜。‎ ‎ 乙判断:不是铁,而是锡。‎ ‎ 丙判断:不是锡,而是铁。‎ ‎ 经化验证明,有一个人判断完全正确,有一个人只说对了一半,而另一个人则完全说错误了。你知道三人中谁是对的,谁是错的,谁是只对了一半的吗?‎ 审题要点:‎ 此题属于真假推断题型,应该用假设法,但要注意按照什么来假设。这里有两种假设思路,一种是根据矿石的成分来假设,有四种可能性:铁、铜、锡以及其它。一种是根据甲、乙、丙的判断来假设,三种可能性:完全正确、一半正确一半错误、完全错误。但是由第二种假设不能直接推断矿石的成分,不符合假设的两个原则之一,所以这里选择第一种假设。‎ 详解过程:‎ 解: 方法一:四种假设:矿石为铁、铜、锡以及其它。‎ 假设一:矿石是铁 假设二:矿石为铜 假设三:矿石为锡 假设四:矿石为其它 甲 ‎×‎ ‎√‎ ‎√‎ ‎×‎ ‎√‎ ‎√‎ ‎√‎ ‎√‎ 乙 ‎×‎ ‎×‎ ‎√‎ ‎×‎ ‎√‎ ‎√‎ ‎√‎ ‎×‎ 丙 ‎√‎ ‎√‎ ‎√‎ ‎×‎ ‎×‎ ‎×‎ ‎√‎ ‎×‎ ‎ ‎ ‎ 从表格中可看出只有假设一符合甲一半正确,乙全错误,丙全正确的已知条件,其它均不合题意。‎ 注意:如何假设是解决真假问题的前提,假设的条件选择原则 是:假设后可以直接得出其他相关结论,此题中假设矿石的成分可直接推出甲、乙、丙的正确与否,是最好的选择。‎ 补充解法二:此题还有一种更简便的方法,可供同学学习与理解。我们称之为“矛盾分析法”。通过观察发现乙,丙两个人说的话完全相反,所以通过矛盾分析法可知:乙,丙两个人肯定一个全对,一个全错,这样可推断出矿石为锡或铁,再根据甲一半对一半错可知矿石为铁是正确的答案,这样即甲一对一错,乙全错误,丙全正确。‎ 专家点评:‎ 此题的解法一为经典的假设法解题。同学们应该认真掌握假设法的解题思路,以及得出矛盾的方法。解法二更为巧妙,是一种思维更发散的思路,对同学们的逻辑推理能力要求更高,可以作为一个重点补充知识点来学习。‎ ‎【例2】(☆☆☆)一次全校数学竞赛,A、B、C、D、E五位同学取得了前五名,发奖后有人问他们的名次,回答是:‎ A说:“B是第三名,C是第五名。”‎ B说:“D是第二名,E是第四名。”‎ C说:“A是第一名,E是第四名。”‎ D说:“C是第一名,B是第二名。”‎ E说:“D是第二名,A是第三名。”‎ 最后,他们都补充说:“我们的话半真半假。”请你判断一下他们每个人的名次。‎ 审题要点:‎ A:‎ B-3‎ C-5‎ B:‎ D-2‎ E-4‎ C:‎ A-1‎ E-4‎ D:‎ C-1‎ B-2‎ E:‎ D-2‎ A-3‎ 首先我们可以确定这是一个关于名次排序的真假判断逻辑推理题型,由于A、B、C、D、E五人每人两句话中一真一假,只有两种可能性,所以运用假设法。‎ 详解过程:‎ 解:1、首先将题目已知条件简化表示,便于我们推理:‎ A:‎ B-3√‎ C-5×‎ B:‎ D-2‎ E-4‎ C:‎ A-1‎ E-4‎ D:‎ C-1√‎ B-2×‎ E:‎ D-2√‎ A-3×‎ ‎2、开始假设:我们选择假设A中的B-3是真的,那么与其矛盾的所有对应为假,有D中的B-2,E中的A-3,图形变为:‎ 注意:这里选择假设A中B-3为真是任意的,同学们可以选择其它的进行解答,方法一样。‎ ‎ ‎ A:‎ B-3√‎ C-5×‎ B:‎ D-2√‎ E-4×‎ C:‎ A-1×‎ E-4√‎ D:‎ C-1√‎ B-2×‎ E:‎ D-2√‎ A-3×‎ ‎3、根据推断出的条件继续推理,E中D-2√推出B中的D-2√,E-4×;D中的C-1为真推出C中的A-1×,E-4√。则图表化为:‎ ‎4、观察表中结论是否一致,即是否出现矛盾。很明显B中的E-4×,C中的E-4√,出现矛盾,即可判断原假设为错误。‎ A:‎ B-3× ①‎ C-5√②‎ B:‎ D-2× ⑤‎ E-4√⑥‎ C:‎ A-1× ⑦‎ E-4√⑧‎ D:‎ C-1× ③‎ B-2√④‎ E:‎ D-2× ⑤‎ A-3√⑥‎ ‎5、将原假设纠正,即假设A中的B-3×,C-5√,依次推断,图表如下:‎ ‎ ‎ 注意:后面的序号代表推理的顺序,相同的序号代表同时推出,不分先后。同学们可以在稿纸上演练。‎ ‎6、观察检验是否出现矛盾,并列出正确名次为:A-3,B-2,C-5,D-1,E-4。‎ ‎【例3】(☆☆☆☆)小东、小兰、小英读书的学校是一中、二中、三中,他们各自爱好游泳、篮球,排球中的一项体育活动,但是究竟谁爱哪一项运动,在哪个学校读书还不清楚,只知道:‎ (1) 小东不在一中;‎ (2) 小兰不在二中;‎ (3) 爱好排球的不在二中;‎ (4) 爱好游泳的在一中;‎ (5) 爱好游泳的不是小兰。‎ 你能根据上面的条件弄清楚他们各自读书的学校和爱好的运动项目吗?‎ 审题要点:‎ 审题确定这是一个涉及姓名、学校、体育运动三种关系的对应推理,运用列表法进行推理。‎ 小东 小兰 小英 游泳 篮球 排球 一中 二中 三中 详解过程:‎ 解:1,首先,我们必须设计一种表格能将姓名,学校,体育运动三者的关系联系起来,图表设计如图:‎ 注意:设计表格时中间一列的选择对象很重要,一般将题目已知条件中涉及最多的项目放在中间,因为它能体现的关系最多。例题中5个条件里四个涉及学校,所以中间一列应该为学校,旁边各列为姓名和体育运动。‎ 小东 小兰 小英 游泳 篮球 排球 ‎×‎ 一中 ‎√‎ ‎×‎ 二中 ‎×‎ 三中 ‎2、根据条件进行判断,对应的表格内划入√和 ‎×。根据(1)~(4)得出结论表格为:‎ 注意:条件(5)为姓名和体育运动的关系,在 表格中无法直接表示,先放一边,后面再用。‎ 小东 小兰 小英 游泳 篮球 排球 ‎×‎ 一中 ‎√‎ ‎×‎ ‎×‎ ‎×‎ 二中 ‎×‎ ‎×‎ 三中 ‎×‎ ‎3、根据已知结论进行推断,打√的行列中其它均应打×(排他性原理)。‎ 注意:此次推理很重要,同学们务必记牢。‎ 小东 小兰 小英 游泳 篮球 排球 ‎×‎ 一中 ‎√‎ ‎×‎ ‎×‎ ‎×‎ 二中 ‎×‎ ‎√‎ ‎×‎ 三中 ‎×‎ ‎×‎ ‎√‎ ‎4、根据推出结论继续推理,一行或一列中除一个空格外其它均为×,则可确定此空格打√。(互补性原理)‎ 注意:此次推理也很重要,同学们务必记牢。‎ 小东 小兰 小英 游泳 篮球 排球 ‎×‎ ‎×‎ ‎√‎ 一中 ‎√‎ ‎×‎ ‎×‎ ‎√‎ ‎×‎ ‎×‎ 二中 ‎×‎ ‎√‎ ‎×‎ ‎×‎ ‎√‎ ‎×‎ 三中 ‎×‎ ‎×‎ ‎√‎ ‎5、学校与体育运动的关系已经确定,爱好游泳的是一中学校。结合条件(5)可推断出小兰不是在一中。重复(3)与(4)中两个原理的运用,即可得出最后表格为:‎ 注意:确定了学校与体育运动的关系后,条件(5)就很好用了。‎ ‎【例4】(☆☆☆☆)四名运动员分别来自北京、上海、浙江和吉林,在游泳、田径、乒乓球和足球四项运动中,每人只参加了一项,且四人的运动项目各不相同,除此之外,只知道一些零碎情况:‎ (1) 张明是球类运动员,不是南方人。‎ (2) 胡老纯是南方人,不是球类运动员。‎ (3) 李勇和北京运动员、乒乓球运动员三人同住一个房间。‎ (4) 郑永禄不是北京运动员,年龄比吉林运动员和游泳运动员两人的年龄小。‎ (5) 浙江运动员没有参加游泳比赛。‎ 根据这些条件,请你分析一下:这四名运动员各来自什么地方?各参加什么运动?‎ 审题要点:‎ 本题为一道较难的涉及四个人姓名、城市、运动三项的对应推理,条件比较多,注意挖掘条件中的一些隐藏信息。‎ 详解过程:‎ 北京 上海 浙江 吉林 游泳 田径 乒乓 足球 张明 胡老纯 李勇 郑永禄 解:1、设计表格,条件中涉及最多的姓名放置中间。 ‎ ‎ ‎ 北京 上海 浙江 吉林 游泳 田径 乒乓 足球 ‎√‎ ‎×‎ ‎×‎ ‎×‎ 张明 ‎×‎ ‎×‎ ‎1‎ ‎×‎ ‎3‎ ‎4‎ ‎×‎ 胡老纯 ‎×‎ ‎×‎ ‎×‎ ‎×‎ ‎×‎ ‎√‎ 李勇 ‎2‎ ‎×‎ ‎×‎ ‎5‎ ‎6‎ ‎×‎ 郑永禄 ‎×‎ ‎2、将题目条件直接输入表格,并根据排他性原理与互补性原理补充表格。‎ ‎3、挖掘(3)(4)中的隐藏信息:(3)中可以知道李勇不是北京人,不是乒乓运动员。但很多同学没有意识到北京运动员不是乒乓运动员,这就是隐藏信息。同理(4)中可知吉林运动员不是游泳运动员,可以确定1,2格均应填×,并补充完整表格。‎ 注意:此处是许多同学无法往下做的一个关键。‎ 北京 上海 浙江 吉林 游泳 田径 乒乓 足球 ‎√‎ ‎×‎ ‎×‎ ‎×‎ 张明 ‎×‎ ‎×‎ ‎×‎ ‎√‎ ‎×‎ ‎3√‎ ‎4×‎ ‎×‎ 胡老纯 ‎√‎ ‎×‎ ‎×‎ ‎×‎ ‎×‎ ‎×‎ ‎×‎ ‎√‎ 李勇 ‎×‎ ‎√‎ ‎×‎ ‎×‎ ‎×‎ ‎5×‎ ‎6√‎ ‎×‎ 郑永禄 ‎×‎ ‎×‎ ‎√‎ ‎×‎ ‎4、确定了体育运动员的对应,结合条件(5)可以确定4为×,补充完整3,4,5,6。最终确定结果为:‎ 张明-北京-足球,‎ 胡老纯-上海-游泳,‎ 李勇-吉林-田径,‎ 郑永禄-浙江-乒乓。‎ ‎【例5】(☆☆)甲、乙、丙、丁与小强一起比赛象棋,每两人都要比赛一盘。到现在为止,甲已经赛了4盘,乙赛了3盘,丙赛了2盘,丁赛了1盘,问小强已经赛了几盘?‎ 审题要点:‎ 此题为体育比赛中经常出现的基本题型,不涉及比赛结果,只涉及比赛场次,所以直接用连线法来解答。‎ 详解过程:‎ 解:1、先将五人排成一个圆的形状,每赛一场则用直线将两人连接起来。‎ ‎2、先考虑甲赛了四场,即表示与乙、丙、丁及小强各赛了一场;乙赛了 3场,不能确定与哪三人,但同时考虑到丁只赛了一场,所以和乙赛的三人为 甲、丙、小强,注意此时丙已经赛了2场,所以无其它场次,所以小强共赛了2场 。‎ 专家点评:‎ 此题中的连线法可以使比赛关系更直观,建议同学们在其它的题型中要常用。另外连线时必须确定和谁比赛才能连线。这就要综合考虑所有人的比赛场次。‎ ‎【例6】(☆☆☆)A、B、C、D、E五位选手进行乒乓球循环赛,每两人都只赛一盘。规定胜者得2分,负者不得分。现在知道的比赛结果是:A与B并列第一名(有两个并列第一名,就不再设第二名,下一个名次规定为第三名),D比C的名次高,每个人都至少胜了一盘。试求每人的得分。‎ 审题要点:‎ 因为乒乓球比赛没有平局,所以求胜的盘数与得分是一回事。此题是体育比赛中的推理题。‎ 涉及分数,胜负场次,所以必须联系基本知识点,确定一些必须满足的条件,包括总分、每队最高分、每队最低分、胜场与负场是否相等、平局总数是否为偶数等。‎ 详解过程:‎ 解:1、确定全部选手的总分与每队最高分。‎ 五人之间单循环场数为4+3+2+1=10(场),所以总分是102=20(分)。‎ 每人赛4场,所以最高分为42=8(分),最低分为0分。‎ 联系题目条件两个并列第一名,所以A、B不可能全胜,最高分为32=6(分)。每人至少胜一场,最低分为2分。由于无平局,所以得分均为偶数:2、4、6。‎ ‎2、假设最高分≤4,则总分最多为4+4+2+2+2=14(分),与总分20分矛盾;‎ 设最低分≥4,则总分最少为6+6+4+4+4=24(分),与总分20分矛盾。‎ 所以最高分即为6分,最低分即为2分。‎ 则其他分为20-6-6-2=6(分),所以D得分4分,C得分2分。‎ ‎3、得出答案A、B各6分,D得4分,C、E各得2分。‎ 专家点评:‎ 体育比赛推断必须确定总分,每队最高分与最低分及所有可能分数,此题中无平局,若有平局,可能分数则为2,3,4,5,6,讨论起来更复杂。另外有的比赛胜一局得3分,平局各得1分,则总分也将不确定,但有一个范围:总场次×2~总场次×3。‎ 题号 学生 ‎1‎ ‎2‎ ‎3‎ ‎4‎ ‎5‎ ‎6‎ ‎7‎ ‎8‎ ‎9‎ ‎10‎ A ‎×‎ ‎√‎ ‎√‎ ‎√‎ ‎×‎ ‎√‎ ‎×‎ ‎×‎ ‎√‎ ‎×‎ B ‎×‎ ‎×‎ ‎√‎ ‎√‎ ‎√‎ ‎×‎ ‎√‎ ‎√‎ ‎×‎ ‎×‎ C ‎√‎ ‎×‎ ‎√‎ ‎×‎ ‎√‎ ‎√‎ ‎√‎ ‎×‎ ‎√‎ ‎√‎ ‎【例7】(☆☆☆☆)A、B、C三名同学参加了一次标准化考试,试题共10道,都是正误题,每道题10分,满分为100分,正确画√,错误画×,他们的答卷如下表:‎ 考试成绩公布后,三人都得70分。请你给出各题的正确答案。‎ 审题要点:‎ 此题为推理题中的一个特殊题型,即涉及考试中判断题的得分情况,此题的难点是并不知道10道判断题的标准答案。应该从三个人的得分来推断正确答案,此题要用二分数进行比较,从整体上考虑两个人的分数。‎ 详解过程:‎ 解:1、先分析A与B的答案,发现有四道相同:1、3、4、10。六道不同:2、5、6、7、8,9。两人的总分为70+70=140分,那么我们可以这样来分析:六道不同答案的题目无论标准答案是什么,两人得到的总分都是60,这样说明两人的其他4道相同答案题总分为140-60=80分,即说明这4道题两人都答对了,由此得出其标准答案为1×、3√、4√、10×。‎ ‎2、同理,比较A,C可知:四道答案相同题目的标准答案为3√、6√、8×、9√,比较B、C可知:2×、3√、5√、7√。‎ 题号 ‎1‎ ‎2‎ ‎3‎ ‎4‎ ‎5‎ ‎6‎ ‎7‎ ‎8‎ ‎9‎ ‎10‎ 答案 ‎×‎ ‎×‎ ‎√‎ ‎√‎ ‎√‎ ‎√‎ ‎√‎ ‎×‎ ‎√‎ ‎×‎ ‎3、综合可知,标准答案为:‎ ‎1‎ ‎3‎ ‎?‎ 图1‎ ‎【例8】(☆☆☆)每个正方体的六个面上分别写着1~6这6‎ 个数字,并且任意两个相对的面上所写的数字之和都等于7,‎ 把这样的四个正方体连在一起,并且让紧接着的两个面上的 数字之和都等于8,想一 想,图中“?”对面的数字是什么?‎ 审题要点:‎ 此题为有关图形与逻辑推理相结合的题型,可直接根据条件推理出相关结论,必要时采用假设法进行分析,利用数字不能超过6这个条件进行排除,得出结论。‎ 详解过程:‎ 解:1、为了便于分析,我们将每个正方体的六个面按方向 分为前后面,上下面,左右面。‎ ‎2、首先,从正方体A前面的3开始分析,根据两对面数字 之和为7,可知A后面为4,又根据相邻的两面数字和为8,‎ 可知B前面为4,依次类推,B后面为3,C前面为5,后面为2,又C上面为1,则下面为6,所以可推理出C正方体左右面分别为3和4,但是左右还不能确定。‎ ‎3、利用假设法分析,若C左面为4,右面为3,则根据条件可继续推理出D左面为5,右面为2,E左面为6,右面为1,此时F左面必须为7才能满足相加为8,无法满足,排除。‎ ‎4、假设另一种情况:若C左面为3,右面为4,则根据条件可继续推理出D左面为4,右面为3,E左面为5,右面为2,F左面为6,右面为1,无矛盾,满足条件。所以?的对面数字为6。‎ ‎【例9】(☆☆☆☆☆)阅读下面的段落,回答题1~4:‎ ‎ 中秋节快要到了,食品厂职工正在紧张地为生产的月饼装盒,某牌子的月饼盒中准备装入5块月饼,要从(A)莲蓉双黄、(B)椰蓉双黄、(C)火腿蛋黄,这3种中选出2种,从(D)豆沙月饼、(E)枣泥月饼、(F)凤梨月饼、(G)山楂月饼、(H)椒盐月饼,这5种月饼中选出3种,同时还要求满足下列条件:‎ (1) B不能与H同时选出;‎ (2) D不能与G同时选出;‎ (3) E不能与H同时选出。‎ a、如果B被选出来放入盒子,那么下列哪种月饼也一定会被选中?‎ ‎(A)G (B)E (C)D (D)C (E)A b、下面哪一种月饼一定会在每一盒中都被选中?‎ ‎(A)A (B)B (C)C (D)D (E)F c、如果H被选中放入月饼盒,那么这个月饼盒中一定还有下面的哪两种月饼?‎ ‎(A)A和B (B)B和C (C)C和A ‎ ‎(D)D和F (E)F和G d、对于符合要求的一盒月饼,下面哪种替换方法保证替换后的月饼仍然满足所有的要求?‎ ‎(A)把F换成E (B)把D换成H (C)把F换成G ‎(D)把G换成D (E)把E换成H ‎ 审题要点:‎ 此题型比较新颖,我们称之为逻辑推理组题的题型,题目的已知条件分为两部分,题干中条件是所有小题都可以使用的条件,小题中的条件在其他小题中不能使用,所以应该首先从题干条件中 挖掘出确切的有效信息,这里的挖掘信息是综合两个甚至多个条件得出,所以应该多从肯定与否定的转换中获取信息。两大原理的运用也十分的必要。‎ 详解过程:‎ 解:1,题目条件复杂,可以适当设计表格将条件形象表示出来:‎ ‎(两者之间有连线表示不能同时入选)‎ ‎2、先可仅从题干条件挖掘信息,题干已知条件集中于第二组中,‎ 综合起来看:五种选三种,又有两组不能同时选择,所以另外一 种即F必须入选,而且两组中分别必须至少入选一种,综合起来即两组中必须且只能入选一种。其他暂时没有十分明显的信息,但是结合选项中条件可能出现新信息。‎ ‎3、结合选项条件解答:‎ a,B被选出,根据B、H不能同时入选,可知H不能入选;又之前E,H中有且只有一人入选,所以E必须入选。B选项正确。‎ b、直接根据题干条件,由上面分析得知F一定会在每一盒中都被选中。E选项正确。‎ c、H被选中,根据B,H不能同时入选,可知B不入选;又第一组A,B,C中必须入选两种,所以A和C一定入选。选项C正确。‎ d、利用排除法:F必须入选,所以将F替换将肯定不能满足要求,选项A,C不正确;由于H同时满足两个条件,不和B,E同时入选,所以当B入选时,其他替换为H将不能满足条件,选项B、E不正确。选项D中G、D中只要保证一个入选即可,又没有其他限制条件,所以相互替换将依旧满足要求。D选项正确。‎ 所以答案为:a-B,b-E,c-C,d-D。‎ 专家点评:‎ ‎1,用图形表示已知关系更直观清晰,利于分析与挖掘信息,同学可以讨论自己设计更好的表示形式。‎ ‎2,首先应该从条件集中的地方进行分析,条件越多,信息量也越大,注意两大原理的利用,否定与肯定之间的转换应该灵活自如。此题中由两组不能同时入选的否定性结论,推理出F必须入选的肯定结论,又由此肯定结论推理出两组中都不能同时不入选。‎ ‎3,d小题中,应该从确定的结论和限制条件最多的项入手进行排除,如F已确定必须入选,而H满足的条件最多,所以很容易排除相关答案,此处必须利用排除法将正确答案选出,当然最后可以进行简单的验证。‎ 四、拓展训练 ‎1、老吴、老周、老杨分别是工程师、会计师和农艺师,还分别是业余作家、画家和音乐家,但不知道每人的职业及业余爱好,只知道:‎ ‎(1)业余音乐家、作家常和老吴一起看电影;‎ ‎(2)画家常请会计师讲经济学的道理;‎ ‎(3)老周一点也不爱好文学;‎ ‎(4)工程师埋怨自己对绘画、音乐一窍不通。‎ 请你指出每个人的职业和爱好。‎ 题号 学生 ‎1‎ ‎2‎ ‎3‎ ‎4‎ ‎5‎ ‎6‎ 得分 甲 ‎√‎ ‎√‎ ‎√‎ ‎√‎ ‎√‎ ‎7‎ 乙 ‎×‎ ‎×‎ ‎×‎ ‎×‎ ‎9‎ 丙 ‎×‎ ‎×‎ ‎×‎ ‎×‎ ‎√‎ ‎7‎ 丁 ‎×‎ ‎√‎ ‎√‎ ‎×‎ ‎×‎ ‎√‎ ‎?‎ ‎2、某次数学测验,共有6道试题,均是是非题,正确地画√,错误的画×,每题答对得2分,不答得1分,答错得0分。甲,乙,丙,丁的答案及前三人的得分如下表,求丁得了多少分?‎ ‎3、在一次数学竞赛中,A、B、C、D。E五位同学分别得了前5名(没有并列同一名次的),关于各人的名次,大家作出下面的猜测:‎ A说:“第二名是D,第三名是B”‎ B说:“第二名是C,第四名是E”‎ C说:“第一名是E,第五名是A”‎ D说:“第三名是C,第四名是A”‎ E说:“第二名是B,第五名是D”‎ 结果每人都只猜对了一半,他们的名次如何?‎ ‎4、徐、王、陈、赵四位师傅分别是工厂的木工、车工、电工和钳工,他们都是象棋迷。‎ ‎(1)电工只与车工下棋,电工也只与车工下棋;‎ ‎(2)王师傅经常与木工下棋;‎ ‎(3)徐师傅与电工下棋,互有胜负;‎ ‎(4)赵师傅不认识木工。‎ 问:徐、王、陈、赵四位师傅各从事什么工作?‎ ‎5、小亮、小红、小娟分别在一小、二小、三小读书,各自爱好围棋、体操、足球中的一项,现知道:‎ ‎(1)小亮不在一小;‎ ‎(2)小红不在二小;‎ ‎(3)爱好足球的不在三小;‎ ‎(4)爱好围棋的在一小,但不是小红。‎ 问:小亮、小红、小娟各在哪个学校读书,和各自的爱好是什么?‎ ‎6、A、B,C、D,四人中只有一人体育未达标,当有人问他们是谁体育未达标时,A说:是B,B说:是D,C说:不是我,D说:B说错了。如果这四句话中只有一句是对的,那么体育未达标的是谁?‎ ‎7、下图是标有1,2,3,4,5,6数字的正方形的三种不同摆法,问三个正方体朝左那一面的数字之和是多少?‎ ‎8、A、B、C三个足球队进行一次比赛,每两个队赛一场,按规则每胜一场得2分,平一场得1分,负一场得到0分。现在已知:‎ ‎(1)B队一球未进,结果得1分;‎ ‎(2)C队进一球,失2球,并且胜了一场;求A队结果得几分,并写出每场比赛的具体比分。‎ ‎9、在三只盒子里,一只装有两个黑球,一只装有两个白球,还有一只装有黑球和白球个一个。现在三只盒子上的标签全贴错了。你能否仅从一只盒子里拿出一个球来,就确定这三只盒子里各装的是什么球?‎ ‎10、北京小学与希望小学结成手拉手友好学校,两校决定成立一个委员会研究合作问题。委员会由3位教师组成,2位来自北京小学,1位来自希望小学,并且不能包括教同样课程的教师,北京小学提出的候选人有:英语教师J,数学教师K,自然教师L;希望小学的候选人有:数学教师M,音乐教师N,英语教师O和P。‎ ‎1.能够组成符合要求的委员会的3位教师是:‎ ‎(A)J、O、N (B)K、L、M (C)J、K、L ‎ ‎(D)K、L、N (E)J、K、M ‎2.如果委员会中有P,那么另外的2个人必定是:‎ ‎(A)K和M (B)K和L (C)K和O ‎ ‎(D)J和K (E)M和N ‎3.如果L因故不能加入委员会,那么必须在委员会中的教师是:‎ ‎(A)M和J (B)O和J (C)N和J ‎ ‎(D)N和O (E)P和J ‎4.在下面的这3个判断中:‎ ‎(1)N和O都在委员会中;‎ ‎(2)M和O不可能同时在委员会中;‎ ‎(3)若M在委员会中,则L必须在委员会中 ‎(A)只有(1)正确 (B)只有(2)正确 (C)只有(3)正确 ‎(D)只有(1)和(2)正确 (E)只有(2)和(3)正确 初级点拨:‎ ‎1、涉及三者的对应推理,应该先设计好表格。注意观察题目中哪个项目是涉及最多的,对应例题3。‎ ‎2、此题与例题7相似,思路与技巧一致。综合比较甲,乙,丙每两人的分数,进行相加,得出某些题的标准答案。‎ ‎3、与例2题型完全一样,采用假设法,可选择假设A所说“第二名是D”是正确或错误。‎ 木工 车工 电工 钳工 徐 ‎×‎ 王 ‎×‎ 陈 赵 ‎×‎ ‎4、属于两者的列表推断题,设计好表格。‎ 小亮 小红 小娟 围棋 体操 足球 一小 二小 三小 ‎5、此题是三者关系对应题,利用列表法先设计好表格:‎ ‎6、此题是属于逻辑推理里面的真假推理题型,与例1属于同一题型,两种解题思路:假设法与矛盾分析法。‎ ‎7、此题属于对应推理类型,不能直接推理,只能利用排除法与互补性原理。‎ A B C 得分 ‎ A ‎—‎ B ‎—‎ C ‎—‎ ‎8、此题为体育比赛中的推理题型,涉及得失球数与得分情况,情况复杂,需要综合分析,先设计好表格。‎ ‎9、此题属于推理题里面的问题解决题型。根据题目意思,关键是确定拿出的球是来自哪只盒子才能确定出三只盒子各装什么。可以采用假设法。‎ ‎10、题目条件比较复杂,将条件表格化,更直观,设计表格为:‎ 北京小学 希望小学 英语 ‎ J O,P 数学 ‎ K ‎ M 自然 L 音乐 ‎ N ‎ 2‎ ‎1‎ ‎ ‎ 深度提示:‎ 老吴 老周 老杨 工程师 会计师 农艺师 ‎×‎ ‎×‎ 作家 画家 ‎×‎ ‎×‎ ‎×‎ 音乐家 ‎×‎ ‎1、设计好表格后,先将已知表示出来:‎ ‎ ‎ ‎2、先比较甲与乙,不同的答案题有三道①④⑥,相同的答案题一道⑤,②③道各有一人不答,总分7+9=16,不同的答案题分数为2×3=6分,说明其他题得分为16-6=10分,若相同答案题⑤不得分,则分数不够,说明⑤题两人都答对,标准答案为√。②③也两人都答对,标准答案为√,×。‎ A D2√‎ B3‎ B C2×‎ E4√‎ C E1‎ A5‎ D C3‎ A4‎ E B2×‎ D5√‎ ‎3、假设A所说“第二名是D”正确,得出表格:‎ 明显D5和D2同时对,矛盾。所以假设不对。‎ 木工 车工 电工 钳工 徐 ‎×‎ ‎×‎ 王 ‎×‎ ‎×‎ ‎×‎ ‎√‎ 陈 ‎×‎ 赵 ‎×‎ ‎×‎ ‎4、将已知条件填入表格:‎ 挖掘信息:联系(1)(2)可知王师傅不是电工也不是车工,又由(2)知王师傅不是木工,所以王师傅是钳工。‎ 小亮 小红 小娟 围棋 体操 足球 ‎×‎ ‎×‎ 一小 ‎√‎ ‎×‎ 二小 三小 ‎×‎ ‎5、将四个条件依次填入可推出:‎ ‎6、假设法:可以分别假设A、B、C、D为体育未达标,然后分别判断四人所说的话是对是错,看是否是符合题目条件中的只有一句话是对的。‎ 矛盾分析法:可以看出B和D所说的话是完全矛盾的,所以必须有一个是对的,则其他两个A和C是错误的。‎ ‎7、由(1)(2)(3)知3出现次数最多,先考虑3,3的对面不是1、2、4、6,所以只能是5。‎ A B C 得分 ‎ A ‎—‎ ‎2:0‎ B ‎—‎ ‎0:1‎ C ‎—‎ ‎8、由条件(2)可知,C胜一场为1:0,负一场为0:2,又根据B队一球未进可知C胜场对B,负场对A。‎ ‎ ‎ ‎9、假设从标有两黑的盒子里取出球,若取出白球,则不能确定真正装的是什么球,假设从标有两白的盒子中取,也是如此。‎ ‎10、结合表格看,主要为两个限制条件:‎ A、同一行中只能入选一个。B、北京小学一列必须入选两个,希望小学必须入选一个。‎ 利用两大原理及肯定否定间的转换进行分析。‎ 全解过程:‎ ‎1、解:(1)涉及三者的对应推理,应该先设计好表格。注意观察题目中哪个项目是涉及最多的,对应例题3。‎ 老吴 老周 老杨 工程师 会计师 农艺师 ‎×‎ ‎×‎ 作家 画家 ‎×‎ ‎×‎ ‎×‎ 音乐家 ‎×‎ ‎(2)设计好表格后,先将已知表示出来: ‎ ‎ ‎ 老吴 老周 老杨 工程师 会计师 农艺师 ‎×‎ ‎×‎ ‎√‎ 作家 ‎√‎ ‎×‎ ‎×‎ ‎√‎ ‎×‎ ‎×‎ 画家 ‎×‎ ‎×‎ ‎√‎ ‎×‎ ‎√‎ ‎×‎ 音乐家 ‎×‎ ‎√‎ ‎×‎ ‎(3)根据排他性原理与互补性原理将表格补充完整。‎ ‎2、解:(1)此题与例题7相似,思路与技巧一致。综合比较甲,乙,丙每两人的分数,进行相加,得出某些题的标准答案。‎ ‎(2)先比较甲与乙,不同的答案题有三道①④⑥,相同的答案题一道⑤,②③道各有一人不答,总分7+9=16,不同的答案题分数为2×3=6分,说明其他题得分为16-6=10分,若相同答案题⑤不得分,则分数不够,说明⑤题两人都答对,标准答案为√。②③也两人都答对,标准答案为√,×。‎ ‎(3)②③⑤答案确定的基础上比较乙,丙,可得①④⑥的标准答案为×,×,√。再对照丁所给答案,可知丁的得分为8分。‎ ‎3、解:(1)与例2题型完全一样,采用假设法,可选择假设A所说“第二名是D”是正确或错误。‎ A D2√‎ B3‎ B C2×‎ E4√‎ C E1‎ A5‎ D C3‎ A4‎ E B2×‎ D5√‎ ‎(2)假设A所说“第二名是D”正确,得出表格:‎ 明显D5和D2同时对,矛盾。所以假设不对。‎ A D2×‎ B3√‎ B C2×‎ E4×‎ C E1√‎ A5×‎ D C3×‎ A4√‎ E B2×‎ D5√‎ ‎(3)假设A所说:D2错,而B3对。则得表格:‎ 所以名次依次为:1-E,2-C,3-B,4-A,5-D。‎ 木工 车工 电工 钳工 徐 王 陈 赵 ‎4、解:(1)属于两者的列表推断题,设计好表格。‎ 木工 车工 电工 钳工 徐 ‎×‎ ‎×‎ 王 ‎×‎ ‎×‎ ‎×‎ ‎√‎ 陈 ‎×‎ 赵 ‎×‎ ‎×‎ ‎(2)将已知条件填入表格:‎ 挖掘信息:联系(1)(2)可知王师傅不是电工也不是车工,又由(2)知王师傅不是木工,所以王师傅是钳工。‎ 木工 车工 电工 钳工 徐 ‎×‎ ‎√‎ ‎×‎ ‎×‎ 王 ‎×‎ ‎×‎ ‎×‎ ‎√‎ 陈 ‎√‎ ‎×‎ ‎×‎ ‎×‎ 赵 ‎×‎ ‎×‎ ‎√‎ ‎×‎ ‎(3)继续挖掘信息:联系(1)(2)可确定徐师傅是车工,然后再根据排他性原理与互补性原理把表格补充完整:‎ 小亮 小红 小娟 围棋 体操 足球 一小 二小 三小 ‎5、(1)此题是按者关系对应题,利用列表法先设计好表格:‎ 小亮 小红 小娟 围棋 体操 足球 ‎×‎ ‎×‎ 一小 ‎√‎ ‎×‎ 二小 三小 ‎×‎ ‎(2)将四个条件依次填入可推出:‎ ‎(3)根据排他性原理与互补性原理将表格填完整。‎ 小亮 小红 小娟 围棋 体操 足球 ‎×‎ ‎×‎ ‎√‎ 一小 ‎√‎ ‎×‎ ‎×‎ ‎√‎ ‎×‎ ‎×‎ 二小 ‎×‎ ‎×‎ ‎√‎ ‎×‎ ‎√‎ ‎×‎ 三小 ‎×‎ ‎√‎ ‎×‎ ‎6、(1)此题是属于逻辑推理里面的真假推理题型,与例1属于同一题型,两种解题思路:假设法与矛盾分析法。‎ ‎(2)假设法:可以分别假设A、B、C、D为体育未达标,然后分别判断四人所说的话是对是错,看是否是符合题目条件中的只有一句话是对的。‎ 矛盾分析法:可以看出B和D所说的话是完全矛盾的,所以必须有一个是对的,则其他两个A和C是错误的。‎ ‎(3)假设法:经过逐一假设与推理,推知当假设C体育未达标时,只有D所说话为对,其他都错误,所以正确答案是:C未达标,D说话正确。‎ 矛盾分析法:由矛盾分析法可知:A、C说话错误,由C说话错误可知就是C未达标,再推理出D说话正确。‎ ‎7、(1)此题属于对应推理类型,不能直接推理,只能利用排除法与互补性原理。‎ ‎(2)由(1)(2)(3)知3出现次数最多,先考虑3。3的对面不是1,2,4,6,所以只能是5。‎ ‎(3)3-5为对面,再考虑2。2的对面不能是1,4,所以只能是6。‎ 得出最终对应结果为:3-5,2-6,1-4,所以朝左那面的数字和为1+5+4=10。‎ A B C 得分 ‎ A ‎—‎ B ‎—‎ C ‎—‎ ‎8、(1)此题为体育比赛中的推理题型,涉及得失球数与得分情况,情况复杂,需要综合分析,先设计好表格。‎ A B C 得分 ‎ A ‎—‎ ‎2:0‎ B ‎—‎ ‎0:1‎ C ‎—‎ ‎(2)由条件(2)可知,C胜一场为1:0,负一场为0:2,又根据B队一球未进可知C胜场对B,负场对A。 ‎ A B C 得分 ‎ A ‎—‎ ‎0:0‎ ‎2:0‎ ‎3‎ B ‎0:0‎ ‎—‎ ‎0:1‎ ‎1‎ C ‎0:2‎ ‎1:0‎ ‎—‎ ‎2‎ ‎(3)由条件(1),结合B输C,可知:C与A比分为0:0,补充完整表格:‎ ‎9、(1)此题属于推理题里面的问题解决题型。根据题目意思,关键是确定拿出的球是来自哪只盒子才能确定出三只盒子各装什么。可以采用假设法。‎ ‎(2)假设从标有两黑的盒子里取出球,若取出白球,则不能确定真正装的是什么球,假设从标有两白的盒子中取,也是如此。‎ ‎(3)假设从标有一黑一白的盒子中取出球,则若取出白球则可以确定真正装有两白球,即标两白的盒子装的就是两黑(否则标两黑装两黑与题目已知矛盾)。即可以确定三只盒子各装什么球。同理可知,若取出黑球,也可确定三只盒子各装什么球 。‎ ‎ 所以答案就是从标有一黑一白的盒子中取球,则可以判断三个盒子真正所装的球。‎ ‎10、(1)题目条件比较复杂,将条件表格化,更直观,设计表格为:‎ ‎(2)结合表格看,主要为两个限制条件:‎ A,同一行中只能入选一个。B,北京小学一列必须入选两个,希望 小学必须入选一个。‎ ‎(3)1。(A)J、O同一行,排除。(B)K、M同一行,排除。(C)J、K、‎ L北京小学三个,排除。(D)符合要求。(E)K、M同一行,排除。‎ 答案:(D)。‎ ‎2.有P,同一行中J、O不能入选,排除(C)、(D);希望小学一列中M、N不能入选,排除(A)、(E);2个人必定是K、L。答案:(B)。‎ ‎3.L不能入选,北京小学一列中J、K必须入选,则同一行中O、P、M不能入选,希望小学一列中N必须入选,答案:(C)。‎ ‎4.利用排除法与反证法进行确认。(1)N入选,O不入选,错误。‎ ‎(2)根据希望小学一列中只能入选一个,判断M、O不可能同时入选,正确。‎ ‎(3)M在,K不能入选, J、L必须入选,所以正确。答案:(E)。‎ 第24讲 统筹与策略 一.知识地图 二.基础知识 ‎(一).统筹 运筹学是利用数学来研究人力、物力的运用和筹划,使它们能发挥最大效率的科学。它包含的内容非常广泛,例如物资调运、场地设置、工作分配、排队、对策、实验最优等等,每类问题都有特定的解法。运筹学作为一门科学,要运用各种初等的和高等的数学知识及方法,但是其中分析问题的某些朴素的思想方法,如高效率优先的原则、调整比较的思想、尝试探索的方法等,都是我们小学生能够掌握的。这些来源于生活实际的问题,正是启发同学们学数学、用数学最好的思维锻炼题目。‎ 比如,想泡壶茶喝。当时的情况是:开水没有;水壶要洗,茶壶茶杯要洗;火生了,茶叶也有了。怎么办?办法甲:洗好水壶,灌上凉水,放在火上;在等待水开的时间里,洗茶壶、洗茶杯、拿茶叶;等水开了,泡茶喝。办法乙:先做好一些准备工作,洗水壶,洗茶壶茶杯,拿茶叶;一切就绪,灌水烧水;坐待水开了泡茶喝。办法丙:洗净水壶,灌上凉水,放在火上,坐待水开;水开了之后,急急忙忙找茶叶,洗茶壶茶杯,泡茶喝。‎ 哪一种办法省时间?我们能一眼看出第一种办法好,后两种办法都窝了工。在实际生活中,我们科学利用统筹安排的方法可以大大节省时间、人力、物力以及资源,提高做事的效率。下面我们就来看看几种统筹类型题目的具体应用。‎ 三.经典透析 ‎【例1】(☆☆)6个人各拿一只水桶到水龙头接水,水龙头注满6个人的水桶所需时间分别是5分钟、4分钟、3分钟、10分钟、7分钟、6分钟。现在如果有甲、乙两个水龙头可用,怎么安排这6个人打水,才能使他们等候的总时间最短,最短的时间是多少?‎ 分析:首先我们来看看如果只有一个水龙头时候的情况。要想等候的时间最短,我们应该怎样安排这些人呢?能把10分钟那个人放在第一个么?显然不行,如果那样的话,后边每一个人都会等他10分钟。说到这里大家应该明白该把什么样的人安排在前面了吧?对,应该把时间短的人尽量安排在前边,例如应该最先安排3分钟那个人,这样后边的5个人外加他自己都等了3分钟,依此类推,以下依次安排4,5,6,7,10,很容易计算出等候的总时间是3×6+4×5+5×4+6×3+7×2+10=100(分)。‎ 详解:现在看看有两个水龙头的情况,由刚才的分析我们可以看出为使总的等待时间尽量短,应让打水所需时间少的人先打。安排需3分钟的,然后5分钟的,最后7分钟的在甲水龙头打;安排需4分钟的,然后6分钟的,最后10分钟的在乙水龙头打;在甲水龙头3分钟的人打时,有2人等待,占用三人的时间和为(3×3)分;然后,需 5分钟的人打水,有1人等待,占用两人的时间和为(5×2)分;最后,需7分钟的人打水,无人等待。甲水龙头打水的三个人,共用(3×3+5×2+7)分,乙水龙头的三人,共用(4×3+6×2+10)分。总的占用时间为(3×3+5×2+7)+(4×3+6×2+10)=60(分)。‎ 点评:这道题的关键在于弄清楚安排的顺序,尽量将时间短的人往前安排,以使后面的人等待的时间短。‎ ‎【例2】(☆☆☆)(2000年《小学生数学报》数学邀请赛)烙饼需要烙它的正、反面,如果烙熟一块饼的正、反面,各用去3分钟,那么用一次可容下2块饼的锅来烙2007块饼,至少需要多少分钟? ‎ 分析:首先很容易想到这样一种烙饼的方法,每次烙两张饼,正反两面共用6分钟,2007÷2=1003余1最后那张饼只能再用6分钟烙好,这样总共就用了6×1003+6=6024分钟。但是这是用时最少的方法么?可以看出烙最后一张饼的时候对锅的利用不够充分,所以很有可能浪费了一部分时间,那么有没有更好的方法呢?‎ 详解:先将两块饼同时放人锅内一起烙,3分钟后两块饼都熟了一面,这时取出一块,第二块翻个身,再放人第三块,又烙了3分钟,第二块已烙熟取出,第三块翻个身,再将第一块放入烙另一面,再烙3分钟,锅内两块饼均烙熟。这样烙3块饼,用去9分钟,所以烙2007块饼,至少用2007÷3×9=6021(分钟)。这种方法保证了所有时间段都能充分的利用锅,不造成浪费。 因此这种方法是用时最短的。‎ 点评:如果是有2的倍数张饼,可以直接用第一种方式来操作,如果是一个3的倍数,可以直接用第二种方式,但是如果既不是2,又不是3的倍数(比如7张饼)该怎么操作呢?我们可以把这样的数拆成一个2的倍数与一个3的倍数的和来操作,如7可以拆成4+3,4这一部分用第一种方式操作,3用第二种,总时间为6×2+9=21分钟。‎ ‎【例3】(☆☆☆)如下图,在街道上有A、B、C、D、E、F六栋居民楼,每栋楼里每天都有20个人要坐车,现在想设立一个公交站,使居民到达车站的距离之和最短,应该设在何处?‎ 分析:先看下面这道题,如下图,在街道上有A、B、C、D、E五栋居民楼,现在设立一个邮筒,为使五栋楼的居民到邮筒的距离之和最短,邮筒应设于何处?‎ 条件中只有五个楼的名字和排列顺序,楼与楼的距离也不确定。那么我们先来分析一下A、E两个点,不论这个邮筒放在AE之间的那一点,A到邮筒的距离加上E到邮筒的距离就是AE的长度,也就是说邮筒放在哪儿不会影响这两个点到邮筒的距离之和;那么我们就使其他的3个点到邮筒的距离之和最短,再看为了使B、D两个到邮筒的距离之和小,应把邮筒放在BD之间。同理,只要是在BD之间,B、D到邮筒的距离之和也是不变的,等于BD。最后,只需要考虑C点到邮筒的距离最近就行了。那么当然也就是把邮筒放在C点了。这里就体现了一个“向中心靠拢的思想”。‎ 详解:再看我们这道题,因为每栋楼的人数相同所以数量不影响选择,找最中间的那栋楼,可这时最中间的楼有两个,这该怎么办呢?其实经过研究发现,建在这两个楼都一样,路程和最短,所以可以建在C或D。如果我们只要求建在这条道路上的一点即可,那么CD之间及点C、D均可。‎ 点评:由上面两道题可以看出有“奇数”个点时就选最中间的那个点,有“偶数”个点的时候就选择最中间的两个点之间的任意一点。‎ ‎【例4】(☆☆☆☆)甲,乙两人要到沙漠中探险,他们每天向沙漠深处走‎20千米,已知每人最多可以携带一个人24天的食物和水,如果不准将部分食物存放于途中,那么其中一人最远可以深入沙漠多少千米(要求最后二人返回出发点)?‎ 分析:题目的要求是要其中一人走的更远,假设这个人是甲,怎样才能让甲走的更远呢?他必须获得乙的帮助,也就是说乙的食物和水当中的一部分得分给甲,由此,我们可以设计出如下的一个方案:‎ 由线段图可知,两人先从A走到C地,然后乙将自己的一部分水和食物给甲,将甲的补给加满,只留下够自己回去的食物和水就行,然后甲继续往前走到B地然后返回A。有同学会问,那乙为什么不一开始就把自己的所有食物和水都给甲呢?原因是每人最多只能带24天的食物和水,所以太多了他也带不下。‎ 详解:现在问题的关键是C地究竟在什么位置?我们来分析一下乙的食物和水分成了几个部分,他自己用去了两个部分(A到C,C回A),还把甲给加满了,也就是甲从A到C耗费的那部分,总共三部分,而且还是相等的。24÷3=8,即从A到C用去了8天,那么甲总共用去了24+8=32天的水和食物,因为是往返,所以从A到B总共用去了32÷2=16天,每天走‎20千米,所以A,B两地相距20×16=320(千米)。‎ 点评:题目说不能将食物和水放在途中,如果可以将食物和水放在途中呢?会不会有不同的答案呢?我们再来研究一下,可以假设有以下这种情况:两人走到C地,乙给甲加满了,还在C地放了一部分食物,然后才回去的,而甲走到B地返回到C地时,正好用完了所有的食物和水,然后将乙留给他的食物和水取走,顺利返回。如果是这样,乙的食物和水应该分成了4部分,24÷4=6天,也就是从A到C地用去了6天,甲总共获得了24+12=36天的补给,因此他走了36÷2×20=‎360千米。‎ ‎【例5】(☆☆☆☆)某商店汽水做促销活动,规定每5个空瓶能换1瓶汽水。小强家买了80瓶汽水,喝完后再按规定用空瓶去换汽水,那么他们家前后最多能喝到多少瓶汽水?‎ ‎ ‎ 分析:很多人会考虑到用80个空瓶换16瓶汽水,然后用16个空瓶换3瓶汽水,然后就不会往下想了,事实上只是到此为止了么?那么我们手上的4个空瓶还能不能再发挥点作用呢?‎ 方法一:我们按照实际换汽水过程分析:‎ ‎  喝掉80瓶汽水,用80个空瓶换回16瓶汽水;‎ ‎  喝掉16瓶汽水,用16个空瓶换回3瓶汽水余1个空瓶;‎ ‎  喝掉3瓶汽水,连上次余下的1个空瓶还剩4个空瓶。此时,再借1个空瓶,与剩下的4个空瓶一起又可换回1瓶汽水,喝完后将空瓶还了。‎ ‎  所以,他们家前后最多能喝到汽水:80+16+3+1=100(瓶)。‎ ‎  以上方法正确运用“5个空瓶可换1瓶汽水”这个条件,特别是最后一次换瓶的技巧,你不充分利用可就“吃亏了”!但如果一开始瓶数很多,那么这个换的过程就会很长。有没有简便的算法呢?‎ 方法二:每4个空瓶就能换到1瓶汽水(不带瓶),即买4瓶就可以喝到5瓶,喝的瓶数是买的瓶数的倍。买了80瓶,那么可以喝80×=100(瓶)‎ 点评:对于这种空瓶换汽水的题,主要有两种类型,一种是已知买了多少瓶,求最多能喝多少瓶(以下简称买推喝),另一种是已知喝了多少瓶,求至少要买多少瓶(以下简称喝推买)。无论是那种题,我们首先都要弄清楚喝的瓶数与买的瓶数之间的倍数关系。例如4瓶换1瓶,则倍数为,7瓶换1瓶,倍数为,然后喝推买的时候用喝的瓶数除以这个倍数,买退喝的时候就乘以这个倍数,这里要注意,喝的瓶数一定会比买的瓶数多,这个可以用来检验我们是否做对的标准。但是这里又会出现一个问题,万一得数不是一个整数该怎么办呢?这里我们要站在卖汽水的人的角度来看问题,他一定是希望我们买得多,喝得少,也就是说买推喝的时候,如果得数是100.9这样的数,那么我们只能喝到100瓶,而喝推买的时候,如果得数是100.1这样的数,我们必须买101瓶才行,这里一定是买推喝全舍,喝推买全入,而不是四舍五入。‎ ‎(二)策略 在这里介绍几种数学游戏的必胜策略,希望同学们学习完本讲的内容后能够感受到兴趣,并且从中获益!‎ ‎【例6】(☆☆☆)小明与弟弟在玩一种“抢报‎30”‎的游戏。从1开始到30,两人轮流报数,每人每次最多报两个数,不能不报,谁先抢到“‎30”‎算赢。请问,在他们先报与后报的人中,谁有必胜的策略。‎ 分析:每次能够报1个或2个,当对手报1个时,自己就报2个,对手报2个,自己就报一个,这种方法可以称之为“凑3”。因此可以以3个数为一组(以下简称周期),对手报的是3个数中的前一部分,自己报的是后一部分。‎ 详解:因为30÷3=10,没有余数,所以应该让对手先报,后报的人有必胜策略,就是“凑3”‎ 如果是抢报31呢?31÷3余1,这个时候先报的人有必胜策略,先报这个余数,然后再与对手“凑3”。‎ 我们还可以考虑一下下面这个问题。对于抢报50,最多可取三个数的情况,谁取到“50”为输。请问必胜的策略是什么呢?‎ 要想让对手报50,自己必须抢到49这个数,这次可以报1、2、3个数,周期为4,所以策略应该为“凑4”,因为49÷4余1,所以应该先报,报1,然后与对手“凑4”。‎ 点评:谁报最后一个谁赢的情况下,用总数÷‎ 周期,如果有余数,则先报的人赢,必胜策略是先报余数,然后与对手凑周期;如果没有余数,则后报的人赢,必胜策略是直接与对手凑周期。谁报最后一个谁输的情况下,先用总数-1,然后用报最后一个赢的方式操作就可以了。‎ 可以试试下面这道题:两人作移火柴棍的游戏,游戏的规则如下:两人从一堆火柴棍中可轮流移走1~7根,直到移尽为止。挨到谁移最后一根就算谁输。如果开始时有1000根火柴,则先移的人第一次应该移动几根火柴棍,才能保证在游戏中获胜?‎ 分析:其实这道题和上一道题是差不多的,因此它们的必胜策略相同。因为是谁拿到最后一根谁输,所以先用1000-1=999,又因为周期是8,所以用999÷8余7,因此先拿的人必须拿走7根火柴,然后与对手“凑8”,就一定能获胜。‎ ‎【例7】(☆☆☆)甲、乙两人轮流往一个圆桌面上放同样大小的硬币。规则是:每人每次只能放一枚,硬币不许重叠,谁放完最后一枚硬币而使对方再也无处可放,谁就获胜。如果甲先放,那么他怎样放才能取胜?‎ 分析:这个问题初看太抽象,既不知道圆桌的大小,又不知道硬币的大小,谁知道该怎样放呀!那我们先看看这道题:有两堆火柴,一堆35根,另一堆24根。两人轮流在其中任一堆中拿取,取的根数不限,但不能不取。规定谁取得最后一根火柴谁胜。先取者有何获胜的策略?‎ 由于火柴的堆数多于一堆,所以获胜的策略与一堆火柴的情形完全不同。‎ ‎ 先取者在35根一堆的火柴中取11根火柴,使得取后剩下两堆的火柴数相同。以后无论对手在某一堆取几根火柴,你只需在另一堆也取同样多根火柴。只要对手有火柴可取,你就有火柴可取,也就是说,最后一根火柴总会被你拿到。这样先取者总可获胜。在上面的玩法中,利用了对称的思想,两堆火柴数相同就是一种对称。‎ 详解:由上一题我们可以看出,可以用对称的思想来解决一些问题,那么这道题可不可以也用对称的思想来考虑呢,我们来分析一下。圆是关于圆心对称的图形,若A是圆内除圆心外的任意一点,则圆内一定有一点B与A关于圆心对称(见上图,其中AO=OB)。也就是说,圆内除圆心外,任意一点都有一个(关于圆心的)对称点。由此可以想到,只要甲把第一枚硬币放在圆桌面的圆心处,以后无论乙将硬币放在何处,甲一定能找到与之对称的点放置硬币,只要乙能放,甲就一定能放。最后无处可放硬币的必是乙,这就是必胜策略。‎ 点评:对称的思想在很多数学题中都发挥了重要的作用。例如有一道题,1~11这十一个自然数,每次任意取走一个或两个连续的自然数,谁取走最后一个谁赢,必胜策略是什么?‎ 先取的人赢,先拿走6,然后对手取什么,他就取对手关于6对称的数,则最后一个数肯定被先取的人拿走了。‎ ‎【例8】(☆☆☆☆)‎ 如图,在一个2004×16的长方形棋盘左上角的方格中有一个棋子(用★表示)。‎ ‎★‎ ‎……‎ 小兵和小燕按如下规则下棋: (1)小兵先走,以后两人轮流移动棋子;‎ ‎(2)棋子纵向或横向(斜向不可)走几个方格都可以,但至少要走1个方格;‎ ‎(3)每个方格允许棋子通过或停留一次(即已经走过的格子不能再走了);‎ ‎(4)轮到哪一方没方格可走时,哪一方就算失败。‎ 两人都在为取胜尽力,其中必有一胜。‎ 请问:谁有必胜的把握?简述取胜的策略。‎ 分析: 小兵有必胜的把握,可按下面策略走棋:‎ 第一步,小兵向右走至最后一格,小燕只能向下走若干格;‎ 第二步,小兵向左走至最后一格,小燕只能向上或向下走若干格;‎ 第三步,小兵向右走至最后一格,小燕只能向上或向下走若干格;‎ 就这样,小兵始终横向走,使得小燕只能纵向走。最后小兵必胜。‎ 点评:为什么这样小兵必胜呢?我们可以看出,这道题中行比列多,小兵横向走,每次都废掉了一行,而小燕纵向走,每次都废掉了一列,又因为行比列多,所以小兵的选择比小燕多,也就坚持的时间比较长了。如果把这道题中的棋盘竖过来,很显然小兵不能再选择横向走了,而只能纵向走,也就是说先走的人永远必胜,他只要选择去废掉行或列中较多一种就可以保证自己能赢。也许有同学要问了,如果行和列同样多呢?还是先走的人赢,这个时候他不论选择横向或者纵向都可以获胜。‎ ‎【例9】(☆☆☆☆☆)在一个3×3的方格中(下图),甲、乙两人轮流(甲先)往方格中写1、3、4、5、6、7、8、9、10九个数中的一个,数字不能重复。最后甲的得分是上、下两行六个数之和,乙的得分是左、右两列六个数之和,得分多者为胜。请你为甲找出一种必胜的方法。‎ 分析:把题中的九个格标上字母:a、b、c、d、e、f、g、h、i。甲的得 分为:a+b+c+g+h+i=(a+c+g+i)+(b+h);‎ 乙的得分为:a+d+g+c+f+i=(a+c+g+i)+(d+f)‎ 要想使甲的得分高于乙的得分,必须且只需使b+h>d+f。要想使b+h>d+f,‎ 甲有两种策略:一是增强自己的实力——使b、h格内填的数尽可能地大;二是削弱对方的实力——使d、f格内填的数尽可能地小。取胜的总策略是“增强自己,削弱对方”两者兼顾。‎ 详解:如果优先考虑增强自己,则先在b或h中填10,这时如果对手在b或h中填1,则无论自己在d或f中填什么,都不能保证d+f小于11,所以很可能就输了,因此要优先考虑削弱对方,先在d或f中填入1,这时候就算对手给他自己加一个10还是给我一个最小的3,都可以保证我们的b+h>d+f。‎ 专家点评:应该注意到这道题中没有2这个数字,如果将2这个数字加进去,则有可能出现b,f和d,h分别为10,1,或9,2,这样和都是11,就不能用上面的方法取胜了。‎ 记九个数从大到小依次为A到I,把b和h两格称为“甲格”,因为它们决定甲获胜,把d和f两格称为“乙格”,其余五格称为“无关格”。‎ 如果A+IB+H,对称地,甲使用增强自己的方法,先将A填入甲格,然后乙只能将I填入另一个甲格,此时甲可以将H填入乙格,这样,乙即使将B填入乙格,总和B+H还是小于A+I。‎ 如果A+I=B+H,用上面的两种方法将会导致平局。但是甲占据先手之利,用其他方法仍然可以获胜!不妨假设甲优先使用“增强自己”的方法,将A填入甲格,然后乙将I填入另一个甲格。甲如果将H填入乙格,则平局,但是甲可以将B填入无关格。此时如果乙将C填入乙格,甲再将H填入另一个乙格,甲就获胜了。除此之外,乙只能也把H填入无关格。问题转化为C+G和A+I的大小。如果C+GA+I,也不要紧,一开始甲可以采用削弱对方的方法,将I填入乙格,然后乙将A填入乙格,甲将H填入无关格,乙将B填入无关格,甲将C填入甲格而获胜。‎ 如果C+G=A+I,则继续考虑D+F,可类似分析。‎ 如果一直到D+F还相等,甲仍然必胜,此时无论使用增强自己还是削弱对方都可以获胜。例如,甲一开始将A填入甲格,乙将I填入甲格;甲将B填入无关格,乙将H填入无关格;甲将C填入无关格,乙将G填入无关格;甲将D填入无关格,此时乙格只能填E和F,总和小于A+I。用削弱对方的方法也同样可以获胜。‎ 拓展训练:‎ ‎1.用一只平底锅煎饼,每次能同时放两个饼。如果煎1个饼需要2分钟(假定正、反面各需1分钟),问煎1993个饼至少需要几分钟?问煎1994个饼至少需要几分钟?‎ ‎2.理发室里有甲、乙两位理发师,同时来了五位顾客,根据他们所要理的发型,分别需要10,12,15,20和24分钟。怎样安排他们的理发顺序,才能使这五人理发的等候所用时间的总和最少?最少要用多少时间?‎ ‎3.车间里有五台车床同时出现故障,已知第一台到第五台修复时间依次为18,30,17,25,20分钟,每台车床停产一分钟造成经济损失5元。现有两名工作效率相同的修理工,‎ (1) 怎样安排才能使得经济损失最少?‎ (2) 怎样安排才能使从开始维修到维修结束历时最短?‎ 4. ‎(三帆中学分班考试)有七个村庄A1、A2、…、A7分布在公路两侧(见右图),由一些小路与公路相连,要在公路上设一个汽车站,要使汽车站到各村庄的距离和最小,车站应设在哪里?‎ ‎5.有5位探险家计划横穿沙漠。他们每人驾驶一辆吉普车,每辆车最多能携带可供一辆车行驶‎315千米的汽油。显然,5个人不可能共同穿越500千米以上的沙漠。于是,他们计划在保证其余车安全返回出发点的前提下,让一辆车穿越沙漠。当然,实现这一计划需要几辆车相互借用汽油(但不允许将汽油放在途中)。问:穿越沙漠的那辆车最多能穿越多宽的沙漠?‎ ‎6、桌子上放着55根火柴,甲、乙二人轮流每次取走1~3根。规定谁取走最后一根火柴谁获胜。如果双方都采用最佳方法,甲先取,那么谁将获胜?如果谁取走最后一根谁输,那么谁将获胜?‎ ‎7、99张卡片上分别写着1~99。甲先从中抽走一张,然后乙再从中抽走一张,如此轮流下去。若最后的两张上的数是互质数,则甲胜;若最后剩下的两个数不是互质数,则乙胜。问:甲要想获胜应该怎样抽取卡片? ‎ ‎8.有大、中、小3个瓶子,最多分别可以装入水‎1000克,‎700克和‎300克。现在大瓶中装满水,希望通过水在3个瓶子间的流动使得中瓶和小瓶上标出‎100克水的刻度线,问最少要倒几次水 ‎9.甲、乙两人玩下面的游戏:有三堆玻璃球,A堆有29个,B堆有16个,C堆有16个,甲、乙两人依次从中拿取,每次只许从同一堆中拿,至少拿一个,多拿不限,规定拿最后一个者获胜。问如果甲先拿,他有无必胜的策略?‎ ‎10. 某人从金坛出发去扬州、常州、苏州、杭州各一次,最后返回金坛 。已知各城市之间的路费如下表所示,请为他设计一条路费最省的路线 。(表中单位:元)‎ 金 坛 常州 扬州 苏州 杭州 金坛 ‎0‎ ‎30‎ ‎40‎ ‎50‎ ‎60‎ 常州 ‎30‎ ‎0‎ ‎15‎ ‎25‎ ‎30‎ 扬州 ‎40‎ ‎15‎ ‎0‎ ‎15‎ ‎25‎ 苏州 ‎50‎ ‎25‎ ‎15‎ ‎0‎ ‎15‎ 杭州 ‎60‎ ‎30‎ ‎25‎ ‎15‎ ‎0‎ ‎(第5届“华罗庚金杯”少年数学邀请赛口试题)‎ 初级点拨:‎ 1. 这道题和我们的例2是同一类型的题,参照例2看看能不能自己做出来。‎ 2. 一人理发时,其他人需等待,为使总的等待时间尽量短,应让理发所需时间少的人先理。 ‎ 3. 这道题和例1是同一种类型的题。参照例1的分析考虑一下。‎ 4. 这道题的图有些复杂,我们能不能把这道题的图简化一下呢?‎ 5. 这是一道多人过沙漠的题,各人之间一定要协作,关键就在于每个人在什么地方将汽油给别人然后返回,我们要分析每个人的汽油分成了几个部分。‎ 6. 这是一道火柴棍游戏的题。我们在例6中已经对这种类型的题的解题方法进行了概括,同学们自己分析一下。‎ 7. 什么样的数互质?最容易想到的应该是相邻的数互质。从这个角度去考虑一下。‎ 8. 首先要弄清楚1000,700和300怎么才能凑出100?‎ 9. 这道题和我们的例7的前铺有些类似,想想那道题是怎么处理的?‎ 10. 我们能不能用一个图来把那个表改一下呢?‎ 深度提示:‎ ‎1.关键是1993个饼该怎么分拆,应该拆成一个2的倍数和一个3的倍数 ‎2.甲先给需10分钟的人理发,然后15分钟的,最后24分钟的;乙先给需12分钟的人理发,然后20分钟的。 ‎ ‎3.怎样安排才能使经济损失最少,必须等待的时间少,那么我们应该把时间短的放在前面,并且使两名修理工的工作时间尽量接近。‎ ‎4. 本题可简化为“B,C,D,E,F处分别站着1,1,2,2,1个人(见右图),求一点,使所有人走到这一点的距离和最小”。‎ ‎5. 如右图所示,5辆车从A点一起出发,到B点时第1辆车留下够自己返回A点的汽油,剩下的汽油全部转给其余4辆车,注意,B点的最佳选择应满足刚好使这4辆车全部加满汽油;剩下的4辆车继续前进,到C 点时第2辆车留下够自己返回A点的汽油,剩下的汽油全部转给其余3辆车,使它们刚好加满汽油;剩下的3辆车继续前进……到E点时,第4辆车留下返回A点的汽油,剩下的汽油转给第5辆车。此时,第5辆车是加满汽油的,还能向前行驶315千米。‎ 以这种方式,第5辆车能走多远呢?同学们可以自己来算算。‎ ‎6. 解题的关键首先是要弄清楚是取到最后一根赢还是取到最后一根输,然后是要找到周期,并且注意有没有余数。‎ ‎7. 我们可以把相邻的数分为一组,多余的那个数就是我们第一次应该取出的数。‎ ‎8. 弄清楚了之后再想想怎么凑才能使次数最少?‎ ‎9. 当只有两堆球,且两堆球的个数相同且个数不等于1时,先拿的必败。‎ ‎10. 将两个城市之间的路费标在它们连线上,如图 可以看到有三对城市之间路费最低,都是15元 . ‎ 把它们连接起来,‎ 常州扬州苏州杭州一线 共用45元,这时可以组成方案(1) ‎ ‎ 金坛常州扬州苏州杭州金坛 ‎ 共需路费135元 。这是运费最少的方案么?‎ 全解过程:‎ ‎11993=1990+3,前1990张饼两张为一组,每组用2分钟,后三张饼为一组,用三分钟,因此总共用1990÷2×2+3=1993分钟,1994张饼每两个一组,每组用两分钟,共用1994÷2×2=1994分钟.‎ ‎2甲给需10分钟的人理发时,有2人等待,占用三人的时间和为(10×3)分;然后,甲给需 15分钟的人理发,有1人等待,占用两人的时间和为(15×2)分;最后,甲给需 24分钟的人理发,无人等待。甲理发的三个人,共用(10×3+15×2+24)分,乙理发的两个人,共用(12×2+20)分。总的占用时间为(10×3+15×2+24)+(12×2+20)=128(分)‎ ‎3. (1)一人修17、20、30,另一人修18、25 ;最少的经济损失为:5×(17×3+20×2+30+18×2+25)=910(元)。‎ ‎(2)因为(18+30+17+25+20)÷2=55(分),一人修需18,17和20分钟的三台,另一人修需30和25分钟的两台,修复时间最短为55分钟。‎ ‎4. 把人尽量靠拢,显然把人聚到D、E最合适,靠拢完的结果变成了D=4,E=3,最好是移动3个人而不要移动4个人。所以车站设在D点。‎ ‎5. 5辆车到达B点时,第1辆车要把另外4辆车消耗掉的汽油补上,加上自己往返AB的汽油,所以应把行驶‎315千米的汽油分成6份,2份供自己往返AB,4份给另外4辆车每辆加1份,刚好使这4辆车都加满汽油。AB长为:315÷6=52.5(千米);‎ ‎ 4辆车从B点继续前进,到达C点时,4辆车共消耗掉4份(BC)汽油,再加上第2辆车从C经B返回A,所以第2辆车是把汽油分成:5份BC+1份AB=315(千米),由上可知6份AB=315(千米),所以AB=BC,也就是说第2辆车仍是把汽油分成6份,3份供自己从B到C,再从C返回A,3份给另外3辆车加满汽油,由此知BC长也是52.‎5千米。‎ ‎ 同理,CD=DE=52.5(千米)。‎ 所以第5辆车最远能行驶52.5×4+315=525(千米)。‎ ‎6. 如果取走最后一根获胜,55÷(1+3)=13……3,因为有余数,所以先取的人获胜,即甲获胜,必胜策略为先取三根然后与乙“凑‎4”‎。如果取走最后一根输,(55-1) ÷(1+3)=13……2,甲先取走两根, 然后与乙“凑‎4”‎,还是甲获胜。‎ ‎7. (法1):甲抽1 ,把剩下的数两两分组为(2,3)(4,5)…(98,99),无论乙抽何数,甲都抽同组中的另一个数。这样最后将剩下同一组中的两个数,这两数相邻必互质,甲胜。‎ ‎(法2):甲抽99,把剩下的数两两分组为(1,2)(3,4)…(97,98),无论乙抽何数,甲都抽同组中的另一个数。这样最后将剩下同一组中的两个数,这两数相邻必互质,甲胜。‎ ‎8. 通过对三个数字的分析,我们发现700-300-300=100,是计算步数最少的得到100的方法。而由于我们每计算一步就相当于倒一次水,所以倒水最少的方案应该是:‎ ‎1.大瓶往中瓶中倒满水。‎ ‎ 2.中瓶往小瓶中倒满水,这时中瓶中还剩下‎400克水。‎ ‎3.小瓶中水倒回大瓶。‎ ‎4.中瓶再往小瓶中倒满水,这时中瓶中只剩下‎100克水,标记。‎ ‎5.小瓶中水倒回大瓶。‎ ‎ 6.中瓶中100水倒入小瓶,标记。所以最少要倒6次水。‎ ‎9. 甲先取时,甲把A堆中的29个球全部取走,这时留给乙的是两堆球数相同且个数不等于1的局面。然后按照两堆球游戏的策略,甲就能获胜。‎ ‎10. 方案(2)‎ ‎ 金坛常州杭州苏州扬州金坛 共需路费130元 。‎ ‎(2)比(1)小,因此,要路费最省,15元的路段至多用2条 。当有2条15元的路段时,次低25元的路段至多有1条,因此5段路费不会少于 ‎15 + 15 + 25 + 30 + 40 = 125(元)‎ 下面来说明,总路费125元是不能达到的 ‎ 。事实上,由金坛一进一出的30+40=70元不能变动,如果变动,总路费便会增加 。这样,在选定 金坛常州,金坛扬州 的前提下,两个15元的路段就不能取 扬州常州,而只能取扬州苏州,苏州杭州 。这时,再选扬州杭州,或苏州常州, 哪一条都不能形成起于金坛最后又止于金坛的经过其余每个城市各一次的回路 。所以,总路费125元是不能实现的 。由于路费差价至少是5元,所以方案(2)的总路费130元是最省的 。‎ 答:路费最省的路线为 ‎ 金坛常州杭州苏州扬州金坛 或 金坛扬州苏州杭州常州金坛 共需路费130元 。‎
查看更多